Sie sind auf Seite 1von 448

ROBERT ROWLAND

4T H EDITION

MATHS QUEST
HSC Mathematics
General 2

Fourth edition published 2013 by


John Wiley & Sons Australia, Ltd
42 McDougall Street, Milton, Qld 4064
First edition published 2001
Second edition published 2007
Third edition published 2010
Typeset in 10/12pt Times LT Std
John Wiley & Sons Australia, Ltd 2001, 2007, 2010, 2013
The moral rights of the author have been asserted.
National Library of Australia
Cataloguing-in-Publication data
Author:
Rowland, Robert, 1963 author.
Title: Maths quest HSC mathematics general 2 /
Robert Rowland.
Edition:
4th ed.
ISBN:
978 1 118 60659 9 (paperback)

978 1 118 60649 0 (loose-leaf)

978 1 118 60646 9 (ebook)
Includes index.
Notes:
Target Audience:
For secondary school students.
Subjects: Mathematics New South Wales
Textbooks.
Mathematics Problems, exercises,etc.
Mathematics Study and teaching
(Secondary) New South Wales.
Dewey number:
510
Reproduction and communication for educational purposes
The Australian Copyright Act 1968 (the Act) allows a maximum
of one chapter or 10% of the pages of this work, whichever
is the greater, to be reproduced and/or communicated by any
educational institution for its educational purposes provided that
the educational institution (or the body that administers it) has
given a remuneration notice to Copyright Agency Limited (CAL).
Reproduction and communication for other purposes
Except as permitted under the Act (for example, a fair dealing
for the purposes of study, research, criticism or review), no part
of this book may be reproduced, stored in a retrieval system,
communicated or transmitted in any form or by any means
without prior written permission. All inquiries should be made to
the publisher.
Cover and internal design images: vic&dd/Shutterstock.com.
Cartography by MAPgraphics Pty Ltd Brisbane
Illustrated by diacriTech and Wiley Composition Services
Typeset in India by diacriTech
Printed in China by
Printplus Ltd
1098765432

Contents
Introductionvi
About eBookPLUS viii
Acknowledgementsix
CHAPTER 1

Credit and borrowing 1


1A Credit cards 1

CHAPTER 4

The normal distribution 95


4A z-scores95

Exercise 4A 98
4B Comparison of scores 100

Exercise 4B 101
4C Distribution of scores 103

Exercise 4C 105

Exercise 1A 5
1B Flat rate interest 7

Exercise 1B 9
1C Loan repayments 11

Exercise 1C 14
1D Present value and future

value16
Exercise 1D 18

Summary20
Chapter review 21
ICT activities 24
Answers25

CHAPTER 2

Annuities and loan repayments 27


2A Future value of an annuity 27

Exercise 2A 31
2B Present value of an annuity 32

Exercise 2B 35
2C Loan repayments 36
Exercise 2C 38
2D Loan repayment graphs 40
Exercise 2D 40

Summary43
Chapter review 44
ICT activities 46
Answers47

CHAPTER 3

Interpreting sets of data 49


3A Grouped data 49

CHAPTER 5

Sampling and populations 115


5A Sampling115

Exercise 5A 116
5B Samples and means 118

Exercise 5B 118
5C Estimating populations 120
Exercise 5C 121

Exercise 3B 57
3C Analysis of data sets 59
Exercise 3C 65
3D Comparison of data sets 69
Exercise 3D 72
3E Skewness76
Exercise 3E 77

Summary81
Chapter review 82
ICT activities 88
Answers89

Summary123
Chapter review 124
ICT activities 126
Answers127

CHAPTER 6

Further applications of area


and volume 129
6A Relative error 129

Exercise 6A 131
6B Area of parts of the circle 133

Exercise 6B 134
6C Area of composite shapes 136

Exercise 6C 138
6D Simpsons rule 141

Exercise 6D 143
6E Surface area of some prisms 146

Exercise 3A 52
3B Measures of location and spread 54

Summary108
Chapter review 109
ICT activities 113
Answers114

Exercise 6E 147
6F Surface area of cylinders and spheres 149

Exercise 6F 151
6G Volume of pyramids, cones and spheres 153

Exercise 6G 155
6H Volume of composite solids 157

Exercise 6H 159
6I Error in measurement 163

Exercise 6I 164

Summary167
Chapter review 168
ICT activities 173
Answers174

CHAPTER 7

Applications of trigonometry 177


7A Review of right-angled triangles 177

Exercise 7A 182
7B Using the sine rule to find side lengths 183

Exercise 7B 187
7C Using the sine rule to find angles 189
Exercise 7C 191
7D Using the cosine rule to find side lengths 193
Exercise 7D 196
7E Using the cosine rule to find angles 197
Exercise 7E 200
7F Area of a triangle 202
Exercise 7F 204
7G Bearings206
Exercise 7G 210
7H Radial surveys 213
Exercise 7H 216

Summary219
Chapter review 221
ICT activities 225
Answers226

CHAPTER 8

Spherical geometry 229


8A Arc lengths 229

Exercise 8A 230
8B Great circles and small circles 233
Exercise 8B 234
8C Latitude and longitude 237
Exercise 8C 239
8D Distances on the Earths surface 240
Exercise 8D 242
8E Time zones 243
Exercise 8E 245

Summary248
Chapter review 249
ICT activities 253
Answers254

CHAPTER 9

Multi-stage events and applications


ofprobability257
9A Counting techniques 257

Exercise 9A 260
9B Tree diagrams 261
Exercise 9B 263
9C Probability and counting techniques 265
Exercise 9C 266
9D Expected outcomes 268
Exercise 9D 269

Summary272
Chapter review 273
ICT activities 276
Answers277

iv Contents

CHAPTER 10

Further algebraic skills and


techniques279
10A Addition and subtraction of algebraic

expressions279
Exercise 10A 280
10B The index laws 281
Exercise 10B 283
10C Expanding and simplifying 285
Exercise 10C 285
10D Solving equations 286
Exercise 10D 287
10E Equations and formulas 290
Exercise 10E 291

Summary294
Chapter review 295
ICT activities 297
Answers298

CHAPTER 11

Modelling linear
relationships301
11A Linear functions 301

Exercise 11A 305


11B Direct variation 307

Exercise 11B 308


11C Linear modelling 310

Exercise 11C 311


11D Problem solving 313

Exercise 11D 314


11E Simultaneous equations 315

Exercise 11E 318

Summary321
Chapter review 322
ICT activities 324
Answers325

CHAPTER 12

Modelling non-linear
relationships329
12A Quadratic functions 329

Exercise 12A 332


12B Cubics, hyperbolas and exponential

functions334
Exercise 12B 336
12C Direct variation 337
Exercise 12C 340
12D Inverse variation 341
Exercise 12D 343
12E Graphing physical phenomena 345
Exercise 12E 347

Summary350
Chapter review 351
ICT activities 354
Answers355

CHAPTER 13

Mathematics and
health361
Body measurements 361
13A Drawing scatterplots 361
Exercise 13A 366
13B Correlation370
Exercise 13B 374
13C Fitting a regression line 377
Exercise 13C 379
Medication382
13D Units of measurement 382
Exercise 13D 383
13E Dosage rate 384
Exercise 13E 385
Life expectancy 386
13F Life expectancy 386
Exercise 13F 387

Summary391
Chapter review 393
ICT activities 396
Answers397

ICT activities Projectsplus

pro-0152 Life expectancy 402


CHAPTER 14

Mathematics and resources 405


14A Interpreting information about water usage 405

Exercise 14A 409


14B Collecting and using water 412

Exercise 14B 414


14C Dams, land and catchment areas 415

Exercise 14C 417


14D Energy and sustainability 421
Exercise 14D 423

Summary426
Chapter review 427
ICT activities 430
Answers431

ICT ACTIVITIES PROJECTSPLUS

pro-0152 Water catchment 432

Glossary435
Index437

Contents v

Introduction
Maths Quest HSC Mathematics General 2 Fourth Edition is specifically designed for the Mathematics
General Stage 6 Syllabus 2012.
There are five strands and two Focus studies:
Strand: Financial Mathematics
Strand: Data and Statistics
Strand: Measurement
Strand: Probability
Strand: Algebra and Modelling
Focus study: Mathematics and Health
Focus study: Mathematics and Resources
There is a suite of resources available:
a student textbook with accompanying eBookPLUS
a teacher edition named eGuidePLUS
flexi-saver versions of all print products
a Solutions Manual containing fully worked solutions to every question.

Student textbook
Full colour is used throughout to produce clearer graphs and diagrams, to
provide bright, stimulating photos and to make navigation through the text
easier.

Worked examples in a Think/Write format provide a clear explanation of


key steps and suggest a process for solutions. Technology is incorporated
into worked examples to demonstrate judicious use.
Exercises contain many carefully graded skills and application problems,
including multiple-choice questions. Cross-references to relevant worked
examples appear beside the first matching question throughout the
exercises. Each exercise also contains new further development questions.

MATHS QUEST
HSC Mathematics
General 2

4TH E D ITION

Clear, concise theory sections contain worked examples and highlighted


important text.

ROBERT ROWLAND

Investigations, including spreadsheet investigations, provide further learning opportunities through


discovery.
A glossary of mathematical terms is provided to assist students understanding of the terminology
introduced in each unit of the course. Words in bold type in the theory sections of each chapter are
defined in the glossary at the back of the book.
Each chapter concludes with a summary and chapter review exercise, containing questions in a variety
of forms (multiple-choice, short answer and analysis) that help consolidate students learning of new
concepts.
Technology is fully integrated, in line with Board of Studies recommendations. As well as graphics
calculators, Maths Quest features spreadsheets, interactivities, eLessons and tutorials.

Student website eBookPLUS


The accompanying eBookPLUS contains the entire student textbook in HTML plus additional exercises.
Students may use the eBookPLUS on any device, and cut and paste material for revision or the creation
of notes for exams, tablets.
WorkSHEET icons link to editable Word documents that may be completed on screen or printed and
completed by hand.
Interactivity icons link to dynamic animations which help students to understand difficult concepts.
Video icons link to videos that expand mathematical ideas.
vi Introduction

Tutorial icons link to tutorials that provide students with a step-by-step guide through selected worked
examples.
Test yourself tests are also available and answers are provided for students to receive instant feedback.

Teacher edition eGuidePLUS


The eGuidePLUS contains everything in the eBookPLUS and more. Two tests per chapter, fully worked
solutions to WorkSHEETs, the work program and other curriculum advice in editable Word format are
provided. Maths Quest is a rich collection of teaching and learning resources within one package.
Maths Quest HSC Mathematics General 2 provides ample material, such as exercises, analysis
questions, investigations, worksheets and technology files, from which teachers may set assessment
tasks.

Maths Quest HSC Mathematics


General 2 Solutions Manual

MATHS QUEST
HSC Mathematics
General 2
SOLUTIONS MANUAL

NILGN SAFAK

4TH E D ITION

The Solutions Manual contains fully worked solutions to every question


in the student textbook. Students are provided with explanations as well
asthe solution process. The PDFs of the Solutions Manual are available on
eBookPLUS.

LEE TOLL

Introduction vii

About eBookPLUS
Next generation teaching and learning
This book features eBookPLUS:
an electronic version of the
entire textbook and supporting
multimedia resources. It is
available for you online at the
JacarandaPLUS website
( www.jacplus.com.au ).

Using the JacarandaPLUS


website
To access your eBookPLUS
resources, simply log on to
www.jacplus.com.au using your
existing JacarandaPLUS login and enter the registration
code. If you are new to JacarandaPLUS, follow the three
easy steps below.
Step 1. Create a user account
The first time you use the JacarandaPLUS system, you will
need to create a user account. Go to the JacarandaPLUS
home page ( www.jacplus.com.au ), click on the button
to create a new account and follow the instructions on
screen. You can then use your nominated email address
and password to log in to the JacarandaPLUS system.
Step 2. Enter your registration code
Once you have logged in, enter your unique registration
code for this book, which is printed on the inside front
cover of your textbook. The title of your textbook will
appear in your bookshelf. Click on the link to open your
eBookPLUS.
Step 3. View or download eBookPLUS
resources
Your eBookPLUS and supporting resources are provided
in a chapter-by-chapter format. Simply select the desired
chapter from the drop-down list. Your eBookPLUS contains
the entire textbooks content in easy-to-use HTML. The
student resources panel contains supporting multimedia
resources for each chapter.
Once you have created your account, you can use the same
email address and password in the future to register any
JacarandaPLUS titles you own.

viii About eBookPLUS

Using eBookPLUS references


eBookPLUS logos are used throughout
the printed books to inform you that a
multimedia resource is available for the
content you are studying.
Searchlight IDs (e.g. INT-0001) give
you instant access to multimedia
resources. Once you are logged in,
simply enter the searchlight ID for that
resource and it will open immediately.

Minimum requirements
JacarandaPLUS requires you to use a supported
internet browser and version, otherwise you will
not be able to access your resources or view all
features and upgrades. Please view the complete
list of JacPLUS minimum system requirements
at http://jacplus.desk.com/customer/portal/
articles/463717.
Troubleshooting
Go to the JacarandaPLUS help page at
www.jacplus.com.au/jsp/help.jsp.
Contact John Wiley & Sons Australia, Ltd.
Email: support@jacplus.com.au
Phone: 1800 JAC PLUS (1800 522 7587)

Acknowledgements
The authors and publisher would like to thank the following copyright holders, organisations and
individuals for their assistance and for permission to reproduce copyright material in this book.

Images
Comstock, p. 78/ Comstock Corbis Royalty Free: p. 15/ Corbis; pp. 145, 162, 165, 192, 223/
Corbis Corporation; p. 171/ Corbis Royalty Free Digital Stock: pp. 240, 266, 293/ Digital Stock/
Corbis Corporation Digital Vision, p. 12, 79 (bottom), 84, 148, 232, 236, 417 (bottom)/ Digital Vision
Google Maps: p. 349/Map Data: Google, Sinclair Knight Merz; pp. 417 (top), 418, 419, 420, 428 (top),
428 (bottom)/Map Data: Google, Cnes/Spot Image Image Addict: p. 14/ Image Addict Image Disk
Photography: p. 152/Copyright 2002 Image Disk Photography iStockphoto: p. 5/iStockphoto.com/
serts; p. 8/ iStockphoto.com/Michal Rozanski IT StockFree: pp. 161, 258/ IT StockFree John
Wiley & Sons Australia: p. 262/ John Wiley & Sons, Australia/Photo by Werner Langer Microsoft
Corporation: pp. 12, 29, 34, 69, 153, 271/Screenshot reprinted by permission from Microsoft Corporation
MAPgraphics: p. 238/MAPgraphics Pty Ltd, Brisbane Photodisc: pp. 10 (centre), (bottom),
74(2images), 107 (2 images), 135, 142 (4 images), 143, 149, 157, 164, 201, p. 212, 222, 260, 263, 264,
268, 269, 273, 332, 340, 373/ Photodisc Shriro Australia Pty Ltd: pp. 3, 4, 97, 178, 179, 185, 189,
194, 198, 290, 302, 304, 305, 362/ CASIO Shutterstock: p. 7/ Vasaleks/2010 Used under license
from Shutterstock.com; p. 10 (bottom)/Poznyakov/Shutterstock.com; p. 39 (bottom)/Shutterstock.
com/Eugene Buchko; p. 39 (centre)/Shutterstock.com/archideaphoto; p. 39 (top)/Shutterstock.com/
Igor Klimov; p. 44/Shutterstock.com/rj lerich; p. 60/Cheryl Ann Quigley/Shutterstock.com; p. 71/
Shestakoff/Shutterstock.com; p. 79/Fer Gregory/Shutterstock.com; p. 98/auremar/Shutterstock.com;
p. 102/Lichtmeister/Shutterstock.com; p. 110 (bottom)/ VanHart, 2010 Used under license from
Shutterstock.com; p. 110 (top)/ Benis Arapovic, 2010 Used under license from Shutterstock.com;
p.111 (bottom)/ STILLFX, 2010 Used under license from Shutterstock.com; p. 111 (top)/Laurence
Gough, 2010 Used under license from Shutterstock.com; p. 116/Mr Doomits/Shutterstock.com;
p.117 (centre)/Zurijeta/Shutterstock.com; p. 117 (top)/Eduard Kyslynskyy/Shutterstock.com;
p.118/Oleksiy Mark/Shutterstock.com; p. 119 (bottom)/Neale Cousland/Shutterstock.com; p. 119/
(centre)/Thorsten Rust/Shutterstock.com; p. 119 (top)/Umberto Shtanzman/Shutterstock.com; p. 120/
evronphoto/Shutterstock.com; p. 121/Shutterstock.com/Kjuuurs; p. 124/schankz/Shutterstock.com;
p.125(centre)/Linali/Shutterstock.com; p. 125 (top)/Natursports/Shutterstock.com; p. 131 (bottom)/
Pete Niesen/2010 Used under license from Shutterstock.com; p. 205/CandyBox Images/Shutterstock.
com; p. 214/Mircea BEZERGHEANU/Shutterstock.com; p. 239/Ehrman Photographic/Shutterstock.
com; p. 246 (top)/Shutterstock.com/lakov Kalinin; p. 246 (bottom)/Shutterstock.com/EpicStockMedia;
p. 247/Shutterstock.com/AHMAD FAIZAL YAHYA; p.250 (bottom)/ david harding, 2010 Used
under license from Shutterstock.com; p. 250 (centre)/James Steidl, 2010 Used under license from
Shutterstock.com; p. 251 (bottom)/ dragon_fang, 2010 Used under license from Shutterstock.com;
p.251 (top)/ Norman Pogson, 2010 Used under license from Shutterstock.com; p. 252 (bottom)/
Stas Volik, 2010 Used under license from Shutterstock.com; p.252 (centre)/ Luke Schmidt,
2010Used under license from Shutterstock.com; p. 252 (top)/Lars Christensen, 2010 Used under
license from Shutterstock.com; p. 261/ Herbert Kratky/2010 Used under license from Shutterstock.com;
p. 267/ Stephen Coburn/2010 Used under license from Shutterstock.com; p. 274 (bottom)/
Used under license from Shutterstock.com; p. 274 (top)/Corepics VOF/Shutterstock.com; p. 313/
sculpies/Shutterstock.com; p. 314/senkaya/Shutterstock.com; p. 315/Hywit Dimyadi/Shutterstock.
com; p. 317/stefanolunardi/Shutterstock.com; p. 319 (centre)/moneymaker11/Shutterstock.com;
p.319 (top left)/coka/Shutterstock.com; p. 319 (top right)/Bertl123/Shutterstock.com; p. 320/Brian
A Jackson/Shutterstock.com; p. 322 (bottom)/Germanskydiver/Shutterstock.com; p. 322 (bottom
left)/Valentina_S/Shutterstock.com; p. 319 (bottom)/Elena Schweitzer/Shutterstock.com; p. 323/
djdarkflower/Shutterstock.com; p. 344 (bottom)/Vorobyeva/Shutterstock.com; p. 344 (centre)/
Charlie Hutton/Shutterstock.com; p. 345 (centre)/Ant Clausen/Shutterstock.com; p. 345 (top)/
Ivan Cholakov/Shutterstock.com; p. 348 (centre)/Taras Vyshnya/Shutterstock.com; p. 348 (top)/
Hannamariah/Shutterstock.com; p. 364/Nattika/Shutterstock.com; p. 365/Evgeny Vorobiev/Shutterstock.
com; p. 366 (bottom)/s_bukley/Shutterstock.com; p. 366 (centre)/Dan Kosmayer/Shutterstock.com;
p. 367 (top)/suravid/Shutterstock.com; p. 367 (bottom)/puhhha/Shutterstock.com; p.372/Neale
Cousland/Shutterstock.com; p.373 (top)/oksana2010/Shutterstock.com; p.374/Deklofenak/
Acknowledgements ix

Shutterstock.com; p. 375/Robyn Mackenzie/Shutterstock.com; p. 376/mangostock/Shutterstock.


com; p. 380 (centre)/Olegro/Shutterstock.com; p. 380 (top)/fixer00/Shutterstock.com; p. 381/
Trevor Allen/Shutterstock.com; p. 382/Stephen Mcsweeny/Shutterstock.com; p. 383 (bottom)/
Tatik22/Shutterstock.com; p. 383 (top)/Geo Martinez/Shutterstock.com; p.384 (bottom)/
Jemny/Shutterstock.com; p. 384 (centre)/Monkey Business Images/Shutterstock.com; p. 384
(top)/Djomas/Shutterstock.com; p. 385 (bottom)/greenland/Shutterstock.com; p. 385 (centre)/
Nathalie Speliers Ufermann/Shutterstock.com; p. 386 (bottom)/giorgiomtb/Shutterstock.com;
p. 386 (centre)/sippakorn/Shutterstock.com; p. 386 (top)/matka_Wariatka/Shutterstock.com;
p. 390/Goran Shutterstock/Shutterstock.com; p.395 (centre)/Jaren Jai Wicklund/Shutterstock.
com; p. 395 (top)/Fenton/Shutterstock.com; p.402 (torso)/Bayanova Svetlana/Shutterstock.com;
p. 402 (soldiers)/iurii/Shutterstock.com; p. 397 (apple)/Kiselev Andrey Valerevich/Shutterstock.
com; p. 402 (beach)/iko/Shutterstock.com; p. 405/Aperture51/Shutterstock.com; p. 407/Diana
Valujeva/Shutterstock.com; p. 408/BMJ/Shutterstock.com; p. 409 (bottom)/Jaroslav74/Shutterstock.
com; p. 409 (top)/motodan/Shutterstock.com; p.410 (bottom)/tab62/Shutterstock.com; p. 410
(centre)/Jodie Johnson/Shutterstock.com; p. 410 (top)/Repina Valeriya/Shutterstock.com; p. 412
(bottom)/zstock/Shutterstock.com; p. 412 (top)/Taras Vyshnya/Shutterstock.com; p. 413 (bottom)/
Steven Frame/Shutterstock.com; p. 413 (top)/Cameramannz/Shutterstock.com; p. 414/Susan Law
Cain/Shutterstock.com; p. 415 (centre)/Steve Scott/Shutterstock.com; p. 415/Steve Scott/Shutterstock.
com; p. 416/ andres balcazar/iStockphoto; p. 415 (bottom)/Tupungato/Shutterstock.com; p. 422/
Oleksiy Mark/Shutterstock.com; p. 423/F AnatolyM/Shutterstock.com; p. 424 (centre)/Designs
Stock/Shutterstock.com; p. 424 (bottom)/ppart/Shutterstock.com; p. 425/gualtiero boffi/Shutterstock.
com; p. 424 (top)/Sarunyu_foto/Shutterstock.com; p. 425 (bottom)/VooDoo13/Shutterstock.com; p. 429
(bottom)/Shcherbakov Ilya/Shutterstock.com; p. 421/Venus Angel/Shutterstock.com; p. 432 (centre left)/
Kathie Nichols/Shutterstock.com; p. 432 (top left)/Jelle vd Wolf/Shutterstock.com; p. 432 (bottom left)
maps/zstock/Shutterstock.com; p. 432 (top right)/Markus Gebauer/Shutterstock.com; p. 432 (bottom
right)/keantian/Shutterstock.com Stockbyte: pp. 211, 333/ Stockbyte.
Every effort has been made to trace the ownership of copyright material. Information that will enable the
publisher to rectify any error or omission in subsequent editions will be welcome. In such cases, please
contact the Permissions Section of John Wiley & Sons Australia, Ltd.

x Acknowledgements

Chapter 1

Credit and borrowing


CHAPTER CONTENTS
1A Credit cards
1B Flat rate interest
1C Loan repayments
1D Present value and future value

1A

Credit cards

Credit cards are the most common line of day-to-day credit that most people use. A credit card works as
a pre-approved loan up to an amount agreed upon by the customer and the bank. The card can then be
used until the amount of the debt reaches this limit.
As with other types of loan, the bank charges interest upon the amount that is owed on the card, and
repayments must be made monthly. The way in which the interest is calculated varies with different
types of credit cards.
Some cards have interest charged from the day on which the purchase was made. Others have what is
called an interest-free period. This means that a purchase that is made will appear on the next monthly
statement. Provided that this amount is paid by the due date, no interest is charged. Hence, the customer
can repay the loan within a maximum of 55 days and be charged no interest.
Generally, credit cards without an interest-free period have a lower interest rate than those with an
interest-free period. These cards, however, generally attract an annual fee. This annual fee can in some
cases be waived if a certain amount is spent on the card over the year.
The minimum monthly repayment on most credit cards is 5% of the outstanding balance, or $10,
whichever is greater.

Annual rates
The annual percentage rates for some standard credit cards are:
16.90% for MasterCard/Visa (up to 55 interest-free days with an annual fee)
15.25% for MasterCard/Visa (no interest-free days with no annual fee).

Minimum payments
For the examples and exercises in this chapter, minimum credit card account payments are calculated as
follows.

Balance less than $25


If the closing balance of the statement is less than $25, then the minimum payment is the same as the
closing balance.

Balance more than $25


Where the closing balance is greater than $25, the payment to be made is the greater of:
1. $25, or
2. 1.5% of the closing balance (rounded down to the nearest $1 if the balance exceeds $1700).
Chapter 1 Credit and borrowing 1

Note: If the closing balance is greater than the credit limit of the card, then the minimum payment must
also include the excess of the balance over the credit limit.
WORKED EXAMPLE 1

Tutorial
int-0900
Worked example 1

Find the minimum payment due for each of the following balances using the information
supplied previously.
a $23.40
b $1836.25
c $280.10
d $1960 with the credit limit being $1900
THINK

WRITE

a Since the closing balance is under $25, it should

be paid in full.

a As $23.40 < $25, the amount due = $23.40.


b Amount due = 1.5% of $1836.25

b 1 Since the closing balance is over $1700,

calculate 1.5% of it.

1.5
1836.25
100

= $27.54
2

Round down to the nearest dollar.

Rounded down to the nearest dollar, the


amount due is $27.
c $25 < $280.10 < $1700

c Since the closing balance is above $25 but

Therefore payment due = $25

below $1700, the minimum payment is $25.

1.5
1960
100
= 29.40
= 29

d 1.5% of $1960 =

d 1 Since the closing balance is above $1700,

calculate 1.5% of it and round down to the


nearest dollar.
2

Calculate the excess of the closing balance


above the credit limit.

The excess of the closing balance above the


credit limit = $1960 $1900
= $60

Add the two amounts.

29 + 60 = 89. The amount due is $89.

To calculate the interest on a credit card the simple interest formula is used.
I = Prn
where P = initial quantity (Principal)
r = interest rate per period expressed as a decimal
n = number of interest periods.
Credit cards, as with most loans, have interest calculated more frequently than annually. In the following
examples you will see that interest is calculated at least monthly but often daily.
WORKED EXAMPLE 2

The outstanding balance on a credit card is $2563.75. If the full balance is not paid by the due
date, one months interest will be added at a rate of 18% p.a. Calculate the amount of interest
that will be added to the credit card.
THINK

Use the simple interest formula to calculate one


months interest.

WRITE

I = Prn
1
= $2563.75 0.18 12
= $38.46

In practice, most credit cards calculate interest on the outstanding balance at a daily rate and then add
the interest monthly. If a credit card advertises its interest rate as 18% p.a., the daily rate is 0.049315%.
2 Maths Quest HSC Mathematics General 2

WORKED EXAMPLE 3

An extract from a credit card statement is shown below.


Interest rate = 15% p.a.
Daily rate = 0.041096%
Date
1 June
10 June
15 June
22 June
1 July

Credit

Debit

Tutorial
int-2410
Worked example 3

Balance
$900
$500
$850
$990

$400 repayment
$350 purchase
$140 purchase
??? interest

Calculate the interest that will be due for the month of June given that there is no interest-free
period.
THINK

WRITE

Method 1: Technology-free
1

For 1 June 9 June inclusive (9 days), the


balance owing is $900. Calculate the interest.

I = 0.041096% of $900 9
= $3.33

For 10 June 14 June inclusive (5 days), the


balance owing is $500. Calculate the interest.

I = 0.041096% of $500 5
= $1.03

For 15 June 21 June inclusive (7 days), the


balance owing is $850. Calculate the interest.

I = 0.041096% of $850 7
= $2.45

For 22 June 30 June inclusive (9 days), the


balance owing is $990. Calculate the interest.

I = 0.041096% of $990 9
= $3.66

Add each amount of interest to calculate the


total interest for the month.

Total interest = $3.33 + $1.03 + $2.45 + $3.66


= $10.47

Method 2: Technology-enabled
1

When doing this type of question where we


need to consider interest calculated on a daily
basis the TVM mode of your calculator is very
useful. Consider the method shown below for
Worked example 3.
From the MENU of your calculator select TVM.

Press 1 to select Simple Interest.

For 9 days the balance is $900, so enter:


n=9
I% = 15
PV = 900

Press 1 (SI) to get the interest for these 9 days.

Interest = $3.33

Chapter 1 Credit and borrowing 3

For 5 days the balance is $500. Press J to


return to the previous screen; change the values
of n and PV.
n=5
I% = 15
PV = 500
Then press 1 for the simple interest.
For 7 days the balance is $850. Press J to
return to the previous screen; change the values
of n and PV.
n=7
I% = 15
PV = 850
Then again press 1 for the simple interest.
For 9 days the balance is $990. Press J to
return to the previous screen; change the values
of n and PV.
n=9
I% = 15
PV = 990
Then again press 1 for the simple interest.
Add each amount of interest to find the total
amount of interest for the month.

Interest = $1.03

Interest = $2.45

Interest = $3.66
Total interest
= $3.33 + $1.03 + $2.45 + $3.66
= $10.47

When deciding which credit card is most suitable for your needs, consider if you will generally be able
to pay most items off before the interest-free period expires. The total cost in interest over a year will
vary according to the repayment pattern.

WORKED EXAMPLE 4

Kerry pays a $1200 bill for her council rates on her credit card. Kerry has a credit card
with aninterest-free period and interest is then charged on the outstanding balance at a rate
of18%p.a.
ai Kerry pays $600 by the due date. What is the outstanding balance on the card?
ii Calculate the interest Kerry must then pay for the second month.
bi
An alternative credit card charges 12% p.a. interest with no interest-free period. Calculate
the interest that Kerry would have been charged on the first month.
ii
Calculate the balance owing after Kerry pays $600 then calculate the interest for the second
month.
c Which credit card would be the cheapest to use for this bill?
THINK

a i Subtract the repayment from the balance.


ii Use the simple interest formula to calculate

one months interest (i.e. interest for the


second month).
b i Use the simple interest formula to calculate

the first months interest.

4 Maths Quest HSC Mathematics General 2

WRITE

a i Balance owing = $1200 $600

= $600

ii I = Prn

= $600 0.18 12
= $9.00

b i I = Prn

1
= $1200 0.12 12
= $12.00

ii Balance owing = $1200 + $12 $600

ii Add the interest to the amount of the bill and

= $612
I = Prn
1
= $612 0.12 12
= $6.12

subtract the repayment.


Use the simple interest formula to calculate
the second months interest.
c Add the two months of interest together for the

c The interest on the second card is $18.12 in

second card and compare with the interest for the


first card.

Exercise 1A

total and therefore the card with the interest-free


period is cheaper in this case.

Credit cards

1 WE1 Roy has a credit card with an outstanding

balance of $2730. Calculate the minimum payment


if he must pay 1.5% of the balance, or $25,
whichever is greater.
2 The minimum monthly repayment on a credit card
is 1.5% of the balance, or $25, whichever is greater.
Calculate the minimum monthly repayment on a
balance of:
a $3500
b $1194.50
c $492.76
d $150
e $9205.20.
3 WE2 Leonie has a credit card with an outstanding
balance of $1850. If the interest rate is 18% p.a., calculate
the amount of interest that Leonie will be charged for one
month if the balance is not paid by the due date.
4 Hassim buys a refrigerator for $1450 with his credit card. The card has no interest-free period and
interest is charged at a rate of 15% p.a. Calculate one months interest on this purchase.
5 Michelle has a $2000 outstanding balance on her credit card. The interest rate charged is 21% p.a. on
the balance unpaid by the due date.
a If Michelle pays $200 by the due date, calculate the balance owing.
b Calculate the interest that Michelle will owe for the next month.
c What will be the balance owing on Michelles next credit card statement?
d What will be the total amount owing on the credit card after another months interest is added?
6 Chandra has a credit card which charges interest at a rate of 12% p.a. but has no interest-free period.
He makes a purchase of $1750 on the credit card.
a After one month Chandras credit card statement arrives. What will be the out standing balance on
the statement?
b The minimum repayment will be 2% of the outstanding balance. Calculate the amount that
Chandra will owe if he makes only the minimum payment.
c In the next month Chandra makes purchases totalling $347.30. Calculate the interest charged and
the balance owing for the next months statement.
7 WE3 An extract of a credit card statement is shown below. Take 1 year = 365.25 days.
Interest rate = 18% p.a.
Daily rate = 0.04928%
Date
1 July
10 July
20 July
1 August

Credit ($)

Debit ($)

Balance ($)
256.40

40 purchase
40 repayment
??? interest

a Complete the balance column. Calculate the balance owing on 10 July and 20 July.
b Calculate the interest due on 1 August and the balance on that date.
Chapter 1 Credit and borrowing 5

8 Study the credit card statement below.

Interest rate = 16.5% p.a.


Daily rate = ______
Date
1 Jan.
8 Jan.
15 Jan.
1 Feb.
8 Feb.
1 March
a
b
c
d

Credit ($)

Debit ($)

Balance ($)
1548.50

500 repayment
399 purchase
??? interest
??? repayment
??? interest

Calculate the daily rate of interest, correct to 4 decimal places (take 1 year = 365.25 days).
Calculate the interest added to the account on 1 February.
On 8 February the minimum repayment of 5% is made. Calculate the amount of this repayment.
Calculate the outstanding balance on the account on 1 March.

9 WE4 Kai has two credit cards. One has an interest-free period and interest is then charged on the

outstanding balance at a rate of 18% p.a. The other has no interest-free period with interest added from
the date of purchase at a rate of 14% p.a. Kai has $1500 worth of bills to pay in the coming month and
intends to use one of the cards to pay them, then pay the balance off in monthly instalments of $500.
a If Kai uses the card with the interest-free period and pays $500 by the due date, what is the
outstanding balance on the card?
b Calculate the interest Kai must then pay for the second month.
c Calculate the balance owing at the end of the second month and the balance owing at the end
ofthe third month, at which time Kai pays off the entire balance.
d Calculate the interest payable in the first month if Kai uses the card without the interest-free period.
e Calculate the balance owing after Kai pays $500 then calculate the interest for the second
month.
f Calculate the balance owing at the end of the second month and the balance owing at the end
ofthe third month, at which time Kai pays off the entire balance.
g Which card should Kai use for these bills?

Further development
10 A credit card has a minimum monthly repayment of the greater of $10 or 2.5% of the outstanding

balance. Calculate the largest outstanding balance for which the minimum monthly repayment
wouldbe $10.
11 Dan and Paul each have a credit card that charges interest at 18% p.a. calculated daily.
a Dan makes a $1000 purchase on 5 July. His credit card does not begin to charge interest until

the beginning of the month following the purchase. Calculate the interest that appears on Dans
statement at the end of July.
b Pauls credit card charges interest from the date of purchase. How much more interest must Paul
pay on the same purchase on the same date?
12 Mark has an outstanding balance of $1425 on his credit card. Mark is charged $6.83 for 10 days on

this balance. Calculate the annual rate of interest charged.


13 Kylie has an outstanding balance of $1280 on her credit card as of 1 August. Kylie has an interest-

free period on her credit card and as such she will not have to pay interest until 1September.
Theinterest rate on the card is 21% p.a., and Kylie will make a $400 repayment on 1 September.
a Calculate the interest that will be charged in September.
b Kylie receives an offer from a rival credit card offering her an interest rate of 12% p.a. interest but
no interest-free period. Should Kylie accept the offer and transfer the balance of her current credit
card to the new card?
Digital doc
Investigation
doc-2746
Researching credit
cards

14 Rob has an average credit card balance of $1000 and the choice of two credit cards. Each month Rob

pays off $500 of the balance but averages $500 in purchases.


Card A 18% p.a. with a one month interest-free period.
Card B 12% p.a. with no interest-free period.
Which credit card should Rob choose?

6 Maths Quest HSC Mathematics General 2

1B

Flat rate interest

Flat rate interest is the borrowing equivalent of simple interest. Flat rate interest applies to many small
loans and hire purchase agreements.
When money is borrowed from a lending institution such as a bank at a flat rate of interest, the total
amount of interest is calculated as a percentage of the initial amount borrowed and then this is multiplied by
the term of the loan. The term of the loan is the length of time over which the loan is agreed to be repaid, n.
As we have just seen, the formula for calculating the amount of flat interest to be paid on a loan is the
same formula as for simple interest (I):
I = Prn
where P = initial quantity
r = percentage interest rate per period expressed as a decimal
n = number of periods.

interactivities
int-0745
Effects of P, r, I
and t
int-2770
Simple interest

As you work through the financial mathematics strand there are several formulas that use the same
pronumerals.
While the initial quantity (P) will be the principal in an investing scenario, it will represent the
amountborrowed in a loan situation.
All of these formulas use the same pronumerals and all of them require r to be expressed as a
decimal. It should be part of your normal practice when doing such questions to convert the interestrate,
expressed as a percentage, to a decimal. In simple or flat rate interest, r will always be a rate per annum
or per year and there will be no variation on this regardless of how often interest ispaid.
WORKED EXAMPLE 5

Calculate the flat interest to be paid on a loan of $20000 at 7.5% p.a. if the loan is to be repaid
over 5 years.
THINK

WRITE

Convert the interest rate to a decimal.

r = 7.5 100
= 0.075

Write the formula.

I = Prn

Substitute the values of P, r (as a decimal) and n.

= $20000 0.075 5

Calculate.

= $7500

Once the interest has been calculated, we can calculate the total amount that must be repaid in a loan.
This is calculated by adding the principal and the interest.
WORKED EXAMPLE 6

Alvin borrows $8000 to buy a car at a flat rate of 9% p.a.


interest. Alvin is to repay the loan, plus interest, over
4 years. Calculate the total amount that Alvin is
to repay on this loan.

THINK

WRITE

Convert the interest rate to a decimal.

r = 9 100
= 0.09

Write the interest formula.

I = Prn

Chapter 1 Credit and borrowing 7

Substitute the values of P, r and n.

= $8000 0.09 4

Calculate the interest.

= $2880

Calculate the total repayments by adding the


interest and principal.

Total repayments = $8000 + $2880


= $10880

Most loans are repaid on a monthly basis. Once the total amount to be repaid has been calculated, this
can be divided into equal monthly, fortnightly or weekly instalments.
WORKED EXAMPLE 7

Tutorial
int-2408
Worked example 7

Narelle buys a computer on hire purchase. The cash price of the


computer is $3000, but Narelle must pay a 10% deposit with
the balance paid at 8% p.a. flat rate interest in equal monthly
instalments over 3 years.
a Calculate the deposit.
b Calculate the balance owing.
c Calculate the interest on the loan.
d Calculate the total amount to be repaid.
e Calculate the amount of each monthly instalment.

THINK

WRITE

a Find 10% of $3000.

a Deposit = 10% of $3000

b Subtract the deposit from the cash price to find the

b Balance = $3000 $300

amount borrowed.
c

= $300

= $2700

c I = Prn

Write the interest formula.

Substitute for P, r and n.

= $2700 0.08 3

Calculate the interest.

= $648

d Add the interest to the amount borrowed.

d Total repayments = $2700 + $648

e Divide the total repayments by 36 (the

e Monthly repayments = $3348 36

number of monthly instalments in 3 years).

= $3348

= $93.00

If given the amount to be repaid each month, we can calculate the interest rate. The interest on the loan
is the difference between the total repaid and the amount borrowed. This is then calculated as a yearly
amount and written as a percentage of the amount borrowed.
WORKED EXAMPLE 8

Theresa borrows $12000 to buy a car. This is to be repaid over 5 years at $320 per month.
Calculate the flat rate of interest that Theresa has been charged.
THINK
1

Calculate the total amount that is repaid.

8 Maths Quest HSC Mathematics General 2

WRITE

Total repayments = $320 60


= $19200

Subtract the principal from the total


repayments to find the interest.

Interest = $19200 $12000


= $7200

Calculate the interest paid each year.

Interest per year = $7200 5


= $1440

Write the annual interest as a percentage of the


$1440
Interest rate =
100%
amount borrowed.
$12 000
= 12%

Exercise 1B

Flat rate interest

1 WE5 Calculate the amount of flat rate interest paid on each of the following loans.
a $5000 at 7% p.a. for 2 years
b $8000 at 5% p.a. for 3 years
c $15000 at 10% p.a. for 5 years
d $9500 at 7.5% p.a. for 4 years
e $2500 at 10.4% p.a. for 18 months
2 Roula buys a used car that has a cash price of $7500. She has saved a deposit of $2000 and borrows

Digital doc
SkillSHEET 1.1
doc-1284
Converting a
percentage to a
decimal

the balance at 9.6% p.a. flat rate to be repaid over 3 years. Calculate the amount of interest that
Roula must pay.
3 WE6 Ben borrows $4000 for a holiday. The loan is to be repaid over 2 years at 12.5% p.a. flat rate

interest. Calculate the total repayments that Ben must make.


4 Calculate the total amount to be paid on each of the following flat rate interest loans.
a $3500 at 8% p.a. over 2 years
b $13500 at 11.6% p.a. over 5 years
c $1500 at 13.5% p.a. over 18 months
d $300 at 33% p.a. over 1 month
e $100000 at 7% p.a. over 25 years
5 Mr and Mrs French purchase a new lounge suite, which has a cash price of $5500. They purchase the

lounge on the following terms: 30% deposit with the balance to be repaid at 9% p.a. flat interest over
2 years. Calculate:
a the deposit
b the balance owing
c the interest to be paid
d the total amount that they pay for the lounge.
6 WE7 Yasmin borrows $5000 from a credit union at a flat interest rate of 8% p.a. to be repaid over

4years in equal monthly instalments. Calculate:


a the interest that Yasmin must pay on the loan
b the total amount that Yasmin must repay
c the amount of each monthly repayment.

Digital doc
SkillSHEET 1.2
doc-1285
Finding a
percentage of a
quantity (money)

Digital doc
SkillSHEET 1.3
doc-1286
Calculating simple
interest

Digital doc
EXCEL Spreadsheet
doc-1287
Simple interest

7 Ian borrows $2000 from a pawnbroker at 40% p.a. interest. The loan is to be paid over 1 year in

equal weekly payments.


a Calculate the interest on the loan.
b Calculate the total that Ian must repay.
c Calculate Ians weekly payment.
8 The Richards family purchase an entertainment system for their home. The total cost of the system is

$8000. They buy the system on the following terms: 25% deposit with the balance repaid over
3years at 12% p.a. flat interest in equal monthly instalments. Calculate:
a the deposit
b the balance owing
c the interest on the loan
d the total repayments
e the amount of each monthly repayment.

Digital doc
GC program
Casio
doc-1288
Interest

Digital doc
GC program TI
doc-1289
Interest

Chapter 1 Credit and borrowing 9

9 Sam buys an electric guitar with a cash price of $1200. He buys the

guitar on the following terms: one-third deposit, with the balance at


15% p.a. flat interest over 2 years in equal monthly instalments.
Calculate the amount of each monthly repayment.
10 MC The amount of flat rate interest on a loan of$10000 at 10% p.a.

for 2 years is:


A $1000
B $2000
C $11000
D $12000
11 MC A refrigerator with a cash price of $1800 is boughton the

following terms: 20% deposit with the balancepaid in 12 equal monthly


instalments at 12% p.a. flatinterest. The total cost of the refrigerator
whenpurchased on these terms is:
A $172.80
B $216.00
C $1972.80
D $2016.00
12 WE8 Andy borrows $4000, which is to be repaid over 4 years at

$110 per month. Calculate the flat rate of interest that Andy has been
charged.
13 Sandra buys a used car with a cash price of $12000 on the

following terms: 20% deposit with the balance paid at


$89.23 per week for 3 years. Calculate:
a the deposit
b the balance owing
c the total cost of the car
d the flat rate of interest charged.
14 Calculate the flat rate of interest charged on a lounge

suite with a cash price of $5000 if it is purchased on the


following terms: 15% deposit with the balance paid at
$230.21permonth for 2 years.

Further development
15 Lennie and his sister are in a band. Lennie borrowed money to buy a new guitar and his sister, Lisa,

borrowed money to buy a drum kit.


a Lennie paid $576 in interest on a flat rate loan that charges interest at a rate of 9.5%p.a. over
4 years. How much did Lennie borrow initially?
b Lennies sister Lisa was also charged $576 in interest on her loan which was at 9% p.a. over
3years. How much did Lisa initially borrow?

10 Maths Quest HSC Mathematics General 2

16 James borrowed $2500 to buy a new laptop computer. He repaid a total of $2725. The flat interest rate

charged was 4.5% p.a. charged monthly. Calculate the length of time that it took James to repay the loan.
17 MC A loan of $1000 is taken over 5 years. The simple interest is calculated monthly. The
totalamount repaid for this loan is $1800. The simple interest rate per year on this loan is closest to:
A 8.9%
B 16%
C 36%
D 5%
18 MC Emily borrowed $5000 with the intention of repaying it in 3 years. The terms of the loan
indicated that Emily was to pay 9% p.a. interest. The interest Emily paid on the loan was:
A $446.25
B $1350
C $121.88
D $1211.88
19 Ange and Jack decide to borrow money to improve their boat but cannot agree which loan is the
better value. They would like to borrow $2550. Ange goes to the Big Bank and finds that they will
lend her the money at 11.33% p.a. simple interest for 3 years. Jack finds that the Friendly Building
Society will lend the $2550 to them at 1% per month simple interest for the 3 years.
a Which institution offers the best rates over the 3 years?
b Explain why.
20 Michael borrows $40000 to buy a car. He is repaying the loan at 9% p.a. flat rate interest over
5years. One year into paying off the loan Michael wins $10000 in the lottery. Should Michael invest
the money or use the $10000 to pay off part of his loan?

1C

Loan repayments

When money is borrowed from a financial institution such as a bank, interest is charged.
A reducing balance loan is one where the interest is added monthly and repayments are made
monthly. Each month the balance owing on the loan is lower than the previous month and hence the
amount of interest owing each month is less.
Consider the case of a $2000 loan that is repaid at 15% p.a. (1.25% per month) over 1 year with equal
monthly repayments of $180.52.
The table below shows the interest and the balance remaining on the loan at the end of each month.
Atthe end of the 12 months the loan has a balance of $0.
Month

Opening balance

Interest

Closing balance

$2000.00

$25.00

$1844.48

$1844.48

$23.06

$1687.02

$1687.02

$21.09

$1527.59

$1527.59

$19.09

$1366.17

$1366.17

$17.08

$1202.73

$1202.73

$15.03

$1037.25

$1037.25

$12.97

$869.70

$869.70

$10.87

$700.05

$700.05

$8.75

$528.29

10

$528.29

$6.60

$354.37

11

$354.37

$4.43

$178.29

12

$178.29

$2.23

$0.00

Digital doc
WorkSHEET 1.1
doc-11019

interactivity
int-0194
Reducing balance
depreciation

Home loans
Home loans usually operate over a long period of time.
Consider a home loan of $250000 at 9% p.a. reducible interest. The monthly repayment on this loan
is $2500 per month. The interest rate of 9% p.a. converts to 0.75% per month.
First months interest =
=
Balancing owing =
=

0.75% of $250 000


$1875
$250 000 + $1875$2500
$249 375
Chapter 1 Credit and borrowing 11

In the second month the interest is calculated on the balance owing at the end of the first month.
Second months interest = 0.75% of $249 375
= $1870.31
Balancing owing = $249 375 + $1870.31$2500
= $248 745.31
The progress of this loan can be followed in the following computer application.

Computer Application 1
The actual calculation of the amount to be repaid each month to pay off the loan plus interest in the
given period of time is beyond this course. The most practical way to find the amount of each monthly
repayment is to use a table of repayments.

The progress of this loan can be followed in the following computer application.

Computer application 1: Home loan calculator


Digital doc
EXCEL Spreadsheet
doc-1291
Home loan

Access the spreadsheet Home Loan from the Maths Quest HSC Mathematics General 2 eBookPLUS.
This spreadsheet will allow you to follow the progress of a home loan as it is paid off.

12 Maths Quest HSC Mathematics General 2

WORKED EXAMPLE 9

Mr and Mrs Grimes take out a $500000 home loan at 8% p.a. reducible interest over 25 years.
Interest is calculated and added on the first of each month. They make a payment of $3875 each
month. Calculate:
a the interest added after one month
b the balance owing after one month.
THINK

WRITE

a 1 Convert 8% p.a. to a monthly rate.

a 8% p.a. = 3% per month

Calculate 23% of $500000 to find the


interest for one month.

Interest = 23% of $500000


= $3333.33

State the answer.

The interest added after one month is $3333.33.

b 1 Add the interest to the principal and

subtractthe repayment.
2

b Balance owing = $500000 + $3333.33 $3875

State the answer.

= $499458.33

The balance owing after one month is $499458.33

The table below shows the monthly repayment on a $1000 loan at various interest rates over various
terms. To calculate the repayment on a loan, we simply multiply the repay ment on $1000 by the number
of thousands of dollars of the loan.
Monthly repayment per $1000 borrowed
Year
1
2
3
4
5
6
7
8
9
10
11
12
13
14
15
16
17
18
19
20
21
22
23
24
25

5%
$85.61
$43.87
$29.97
$23.03
$18.87
$16.10
$14.13
$12.66
$11.52
$10.61
$9.86
$9.25
$8.73
$8.29
$7.91
$7.58
$7.29
$7.03
$6.80
$6.60
$6.42
$6.25
$6.10
$5.97
$5.85

6%
$86.07
$44.32
$30.42
$23.49
$19.33
$16.57
$14.61
$13.14
$12.01
$11.10
$10.37
$9.76
$9.25
$8.81
$8.44
$8.11
$7.83
$7.58
$7.36
$7.16
$6.99
$6.83
$6.69
$6.56
$6.44

7%
$86.53
$44.77
$30.88
$23.95
$19.80
$17.05
$15.09
$13.63
$12.51
$11.61
$10.88
$10.28
$9.78
$9.35
$8.99
$8.67
$8.40
$8.16
$7.94
$7.75
$7.58
$7.43
$7.30
$7.18
$7.07

8%
$86.99
$45.23
$31.34
$24.41
$20.28
$17.53
$15.59
$14.14
$13.02
$12.13
$11.42
$10.82
$10.33
$9.91
$9.56
$9.25
$8.98
$8.75
$8.55
$8.36
$8.20
$8.06
$7.93
$7.82
$7.72

Interest rate
9%
10%
11%
$87.45 $87.92 $88.38
$45.68 $46.14 $46.61
$31.80 $32.27 $32.74
$24.89 $25.36 $25.85
$20.76 $21.25 $21.74
$18.03 $18.53 $19.03
$16.09 $16.60 $17.12
$14.65 $15.17 $15.71
$13.54 $14.08 $14.63
$12.67 $13.22 $13.78
$11.96 $12.52 $13.09
$11.38 $11.95 $12.54
$10.90 $11.48 $12.08
$10.49 $11.08 $11.69
$10.14 $10.75 $11.37
$9.85 $10.46 $11.09
$9.59 $10.21 $10.85
$9.36 $10.00 $10.65
$9.17 $9.81 $10.47
$9.00 $9.65 $10.32
$8.85 $9.51 $10.19
$8.71 $9.38 $10.07
$8.59 $9.27 $9.97
$8.49 $9.17 $9.88
$8.39 $9.09 $9.80

12%
$88.85
$47.07
$33.21
$26.33
$22.24
$19.55
$17.65
$16.25
$15.18
$14.35
$13.68
$13.13
$12.69
$12.31
$12.00
$11.74
$11.51
$11.32
$11.15
$11.01
$10.89
$10.78
$10.69
$10.60
$10.53

13%
$89.32
$47.54
$33.69
$26.83
$22.75
$20.07
$18.19
$16.81
$15.75
$14.93
$14.28
$13.75
$13.31
$12.95
$12.65
$12.40
$12.19
$12.00
$11.85
$11.72
$11.60
$11.50
$11.42
$11.34
$11.28

14%
$89.79
$48.01
$34.18
$27.33
$23.27
$20.61
$18.74
$17.37
$16.33
$15.53
$14.89
$14.37
$13.95
$13.60
$13.32
$13.08
$12.87
$12.70
$12.56
$12.44
$12.33
$12.24
$12.16
$12.10
$12.04

15%
$90.26
$48.49
$34.67
$27.83
$23.79
$21.15
$19.30
$17.95
$16.92
$16.13
$15.51
$15.01
$14.60
$14.27
$14.00
$13.77
$13.58
$13.42
$13.28
$13.17
$13.07
$12.99
$12.92
$12.86
$12.81

Chapter 1 Credit and borrowing 13

WORKED EXAMPLE 10

Calculate the monthly repayment on a loan of $85000 at 11% p.a. over a 25-year term.
THINK

WRITE

Look up the table to find the monthly


repayment on $1000 at 11% p.a. for 25years.

Multiply this amount by 85.

Monthly repayment = $9.80 85



= $833

This table can also be used to make calculations such as the effect that interest rate rises will have on a
home loan.
WORKED EXAMPLE 11

The Radley family borrow $360000 for a home at 8% p.a. over a 20-year term. They repay
the loan at $3009.60 per month. If the interest rate rises to 9%, will they need to increase their
repayment and, if so, by how much?
THINK

WRITE

Look up the table to find the monthly


repayment on $1000 at 9% p.a. for 20 years.

Multiply this amount by 360.

If this amount is greater than $3009.60, state the The Radley family will need to increase their
amount by which the repayment needs to rise.
monthly repayments by $230.40.

Exercise 1C

Monthly repayment = $9.00 360



= $3240

Loan repayments

1 WE9 Mr and Mrs Devcich borrow


Digital doc
SkillSHEET 1.4
doc-1292
Finding values of n
and r in financial
formulas

$80000 to buy a home. The interest rate is


12% p.a. and their monthly payment is
$850permonth.
a Calculate the interest for the first month
of the loan.
b Calculate the balance owing at the end
of the first month.
2 The repayment on a loan of $180000 at

Digital doc
SkillSHEET 1.5
doc-1293
Calculating
compound interest

7.5% p.a. over a 15-year term is $1668.62


per month.
a Calculate the interest for the first month
of the loan and the balance owing at the
end of the first month.
b Calculate the amount by which the balance has reduced in the first month.
c Calculate the interest for the second month of the loan and the balance at the end of the second
month.
d By how much has the balance of the loan reduced during the second month?
3 WE10 Use the table of repayments on page 13 to calculate the monthly repayment on a $75000

Digital doc
SkillSHEET 1.6
doc-11020
Reading tables

loan at 7% p.a. over a 15-year term.


4 Use the table of repayments to calculate the monthly repayment on each of the following loans.
a $2000 at 8% p.a. over a 2-year term
b $15000 at 13% p.a. over a 5-year term

14 Maths Quest HSC Mathematics General 2

c $64000 at 15% p.a. over a 25-year term


d $100000 at 12% p.a. over a 20-year term
e $174000 at 9% p.a. over a 22-year term
5 Jenny buys a computer for $4000 on the following terms: 10% deposit with the balance paid in equal

monthly instalments over 3 years at an interest rate of 14% p.a.


a Calculate Jennys deposit.
b Calculate the balance owing on the computer.
c Use the table of repayments to calculate the amount of each monthly repayment.

Digital doc
EXCEL Spreadsheet
doc-1299
Reducing balance
loans

6 WE11 Mr and Mrs Dubois borrow $125000

over 20 years at 10% p.a. to purchase a house.


They repay the loan at a rate of $1500 per
month. If the interest rate rises to 12% p.a.,
will MrandMrsDubois need to increase
the sizeoftheir repayments and, if so,
byhowmuch?
7 Mr and Mrs Munro take out a $180000 home

loan at 9% p.a. over a 25-year term.


a Calculate the amount of each monthly
repayment.
b After 5 years the balance on the loan has been reduced to $167890. The interest rate then rises
to10% p.a. Calculate the new monthly repayment required to complete the loan within the
existing term.
8 A bank will lend customers money only if they believe the customer can afford the repayments.

Todetermine this, the bank has a rule that the maximum monthly repayment a customer can afford
is 25% of his or her gross monthly pay. Darren applies to the bank for a loan of $62000 at 12% p.a.
over 15 years. Darren has a gross annual salary of $36000. Will Darrens loan be approved? Use
calculations to justify your answer.
9 Tracey and Barry have a combined gross income of $84000.
a Calculate Tracey and Barrys gross monthly income.
b Using the rule applied in the previous question, what is the maximum monthly repayment on a

loan that they can afford?


c If interest rates are 11% p.a., calculate the maximum amount (in thousands) that they could

borrow over a 25-year term.


10 Mr and Mrs Yousef borrow $95000 over 25 years at 8% p.a. interest.
a Calculate the amount of each monthly repayment on the loan.
b Mr and Mrs Yousef hope to pay the loan off in a much shorter period of time. By how

muchwill they need to increase the monthly repayment to pay the loan off in
15 years?
11 Mr and Mrs Bath borrow $375000 at 8% p.a. reducible over a 25-year term, with repayments to be

made monthly.
a Calculate the amount of each monthly repayment.
b Calculate the total amount that Mr and Mrs Bath will repay over the term of the loan.
c What is the total amount of interest that Mr and Mrs Bath will pay on the loan?
d Calculate the average amount of interest that Mr and Mrs Bath will pay each year.
e Calculate the equivalent flat rate of interest by expressing your answer to part d as a percentage of
the amount borrowed.
12 A loan of $240000 is taken out over a 25-year term at an interest rate of 7% p.a. reducible.
a Calculate the amount of each monthly repayment.
b Calculate the total repayments made on the loan.
c Calculate the amount of interest paid on the loan.
d Find the equivalent flat rate of interest.
e By following steps a to d above calculate the equivalent flat rate of interest if the term of the

loanis:
i 20 years
ii 15 years.
Chapter 1 Credit and borrowing 15

Further development
13 The average home loan in Australia at present is approximately $300000.
a Use the table of monthly home loan repayments to calculate the monthly repayment on a loan of

$300000 taken out at 7% p.a. over a 25-year term.


b During difficult economic times interest rates are increased by 1% p.a. Calculate the new monthly

repayment on the average $300000 loan.


c Calculate the amount by which loan repayments will have increased over a one-year period.
d Jim has a $300000 loan. How much extra per week should Jim budget to cover his higher loan

repayments after the interest rate rise?


14 Use the table of monthly home loan repayments to calculate the monthly increase in the repayment

of a $400000 loan taken out over 25 years if interest rates are increased from 8% p.a. to9% p.a.
15 At a time when interest rates are 7% p.a. Chloe applies for a loan. To ensure that customers can cope

with higher rates the bank will only allow borrowers to borrow an amount where the repaymentis 13
of the borrowers gross monthly income if interest rates were 2% higher than their current level.
a Chloe earns $72000 per year. What is the maximum repayment that Chloe can afford
eachmonth?
b If interest rates were 2% higher, what would be the maximum amount that Chloe could borrow
over a 25-year term (in thousands)?
16 Use the table on page 13 to estimate the monthly repayment on the average $300000 loan over

20years if interest rates are:


a 7% p.a.
b 8% p.a.

c 7.5% p.a.

d 7.25 p.a.

17 Rebecca borrows $300000 for a home loan at 9% p.a. over 25 years.


a Use the table of monthly home loan repayments to find the amount of each monthly repayment.
b Shortly after Rebecca begins to repay the loan the interest rate drops to 8% p.a. Rebecca

maintains her current repayment with the intention of paying the loan off quicker. Use the table
toestimate the number of years by which the loan will be shortened.
c Estimate the amount that will be saved on this loan.
18 Marcus borrows $300000 at an interest rate of 8% p.a. over a 20-year period.
a Use the table of monthly home loan repayments to calculate the amount of each monthly repayment.
b Calculate the total amount to be repaid over the period of the loan.
c Marcus receives a lump sum payment of $20000 and uses it to reduce the amount of his loan.

Marcus hopes this will shorten his loan by two years if he maintains the current repayment.
Determine if Marcus will achieve this goal.

1D

Present value and future value

In the Preliminary Course you learned about compound interest. The compound interest formula is:
interactivities
int-0810
Compound interest
int-2791
Compound interest

A = P(1 + r) n
where A = final amount,
P = initial amount (or principal),
r = interest rate per period expressed as a decimal
n = number of compounding periods.
In the financial world the initial amount P is called the present value, PV and the final amount A is
called the future value, FV. As such, the compound interest formula can be written as:
FV = PV(1 + r) n
Consider the case of $2000 invested today for 3 years at 8% p.a. with interest compounded annually.
FV = PV(1 + r) n
= $2000(1.08) 3
= $2519.42
This investment has a present value of $2000 as this is the value at the present time, the beginning of
the investment.
The future value of the investment is $2519.42 as this is what the investment will be worth at the end
of the investment period.

16 Maths Quest HSC Mathematics General 2

WORKED EXAMPLE 12

Rick has an investment with a present value of $5000. The terms of the investment are 5 years
with an interest rate of 6.5% p.a. with interest compounded annually. Calculate the future value
of Ricks investment.
THINK

WRITE

Write the compound interest formula.

FV = PV(1 + r) n

Write the values of PV, r and n.

PV = $5000, r = 0.065, n = 5

Substitute these values into the formula.

FV = 5000(1.065)5

Calculate the future value.

= $6850.43

When doing any compound interest problem it is important to remember that:


r = interest rate per interest period expressed as a decimal
n = number of interest periods.
Interest rates are almost always given as a rate per annum and as such if interest is compounded more
frequently than annually the values of r and n must be adapted accordingly.
WORKED EXAMPLE 13

Michonne has $3000 to invest. She plans to invest this money for 4 years at an interest rateof
6.4%p.a. with interest compounded quarterly. Calculate the future value of Michonnes investment.
THINK

WRITE

Write the compound interest formula.

FV = PV(1 + r) n

Write the values of PV, r and n.

PV = $3000, r = 0.064 4, n = 4 4
= 0.016
= 16

Substitute the values into the formula.

FV = 3000(1.016)16

Calculate the future value.

= $3867.41

Rearranging this formula will allow you to calculate the present value of an investment required to
generate a pre-determined future value. That is, the amount that needs to be invested now if you want a
certain amount of money at the end of the given period of time.
Given that FV = PV(1 + r) n, making PV the subject of the formula gives
FV
.
PV =
(1 + r) n
WORKED EXAMPLE 14

Herchelle wants a holiday in 2 years time that will cost $10000. Given that he can invest his
money at 7.4% p.a. with interest compounded six monthly, calculate the amount that Herchelle
needs to invest so that he will have $10000 at the end of the investment.
THINK

WRITE

FV
(1 + r) n

Write the compound interest formula with


PV as the subject.

PV =

Write the values of FV, r and n.

FV = $10000, r = 0.074 2, n = 2 2
= 0.037= 4

Substitute the values into the formula.

PV =

Calculate the present value.

10000
(1.037) 4

= $8647.39

Chapter 1 Credit and borrowing 17

Exercise 1D
interactivity
int-2400
Simple and
compound interest

Present value and future value

1 WE12 Daryl has $2000 to invest. He invests the money at 6% p.a. for 6 years with interest

compounded annually. Calculate the future value of Daryls investment.


2 Simon has $4000 to invest. He invests the money at 5% p.a. for 3 years with interest compounded

annually.
a Use the formula FV = PV(1+ r)n to calculate the amount to which this investment
will grow.
b Calculate the compound interest earned on the investment.
3 Use the formula FV = PV(1+ r)n to calculate the amount to which each investment will grow with

interest compounded annually.


a $3000 at 4% p.a. for 2 years
c $16000 at 9% p.a. for 5 years
e $9750 at 7.25% p.a. for 6 years

b $9000 at 5% p.a. for 4 years


d $12500 at 5.5% p.a. for 3 years
f $100000 at 3.75% p.a. for 7 years

4 WE13 Calculate the future value of an investment of $14000 at 7% p.a. for 3 years with interest

compounded quarterly.
5 A passbook savings account pays interest of 0.3% p.a. Jill has $600 in such an account. Calculate the

amount in Jills account after 3 years, if interest is compounded quarterly.


6 Damien is to invest $35000 at 7.2% p.a. for 6 years with interest compounded six-monthly.

Calculate the compound interest earned on the investment.


7 Sam invests $40000 in a one-year fixed deposit at an interest rate of 7% p.a. with interest

compounding monthly.
a Convert the interest rate of 7% p.a. to a rate per month.
b Calculate the future value of the investment.
8 Calculate the compounded value of each of the following investments.
a $870 for 2 years at 3.50% p.a. with interest compounded six-monthly
b $9500 for 2 years at 4.6% p.a. with interest compounded quarterly
c $148000 for 3 years at 9.2% p.a. with interest compounded six-monthly
d $16000 for 6 years at 8% p.a. with interest compounded monthly
e $130000 for 25 years at 12.95% p.a. with interest compounded quarterly.
9 MC A sum of $7000 is invested for 3 years at the rate of 5.75% p.a., compounded quarterly.

The interest paid on this investment, to the nearest dollar, is:


a $1208
B $1308
C $8208

D $8308

10 MC After selling their house, Mr and Mrs Fernhill have $73600. They plan to invest it at 7% p.a.

with interest compounded annually. The future value of their investment will first exceed $110000
after:
A 5 years
B 6 years
C 8 years
D 10 years
11 MC Maureen wishes to invest $15000 for a period of 7 years. The following investment alternatives

are suggested to her. The best investment would be:


A simple interest at 8% p.a.
B compound interest at 6.7% p.a. with interest compounded annually
C compound interest at 6.6% p.a. with interest compounded six-monthly
D compound interest at 6.5 % p.a. with interest compounded quarterly
12 MC An amount is to be invested for 5 years and compounded semi-annually at 7% p.a. Which of

the following investments will have a future value closest to $10000?


A $700
B $6500
C $7400
D $9000

Further development
13 Daniel has $15500 to invest. An investment over a 2-year term will pay interest of 7% p.a.
a Calculate the compounded value of Daniels investment if the compounding period is:
i 1 year ii 6 months
iii 3 months
iv monthly.
b Explain why it is advantageous to have interest compounded on a more frequent basis.
18 Maths Quest HSC Mathematics General 2

14 Jake invests $120000 at 9% p.a. for a 1-year term. For such large investments interest is

compounded daily.
a Calculate the daily percentage interest rate, correct to 4 decimal places. Use 1 year = 365 days.
b Calculate the future value of Jakes investment on maturity.
c Calculate the amount of interest paid on this investment.
d Calculate the extra amount of interest earned compared with the case where the interest is
calculated only at the end of the year.
15 Lori invests $6000 for 4 years at 8% p.a. simple interest. Shane also invests $6000 for 4 years, but
his interest rate is 7.6% p.a. with interest compounded quarterly.
a Calculate the future value of Loris investment.
b Show that the future value of Shanes investment is greater than Loris investment.
c Explain why Shanes investment is worth more than Loris investment despite receiving a lower
rate of interest.

Digital doc
WorkSHEET 1.2
doc-11021

Chapter 1 Credit and borrowing 19

Summary
Credit cards

A credit card is a pre-approved loan up to a certain amount called the credit limit.
There are many kinds of credit cards and the most important difference is that some cards have an
interest-free period while others attract interest from the date of making a purchase.
Credit cards without an interest-free period generally have a lower rate of interest than those with
an interest-free period.
Each credit card will have a monthly statement and will require a minimum payment each month.
When evaluating the best credit card for your circumstances, you need to consider if you will be
able to pay most bills by the due date and consider any fees attached to the card.

Flat rate interest

A flat rate loan is one where interest is calculated based on the amount initially borrowed.
Flat rate loans have the interest calculated using the simple interest formula:
I = Prn
The total repayments on a flat rate loan are calculated by adding the interest to the amount
borrowed.
The monthly or weekly repayments on a flat rate loan are calculated by dividing the total
repayments by the number of weeks or months in the term of the loan.

Loan repayments

The interest on home loans is calculated at a reducible rate. This means that the interest is
calculated on the outstanding balance at the time and not on the initial amount borrowed.
The interest on home loans is usually calculated and added monthly while repayments are
calculated on a monthly basis.
To calculate the total cost of a home loan, we multiply the amount of each monthly payment by the
number of payments.
The amount of each monthly repayment is best calculated using a table of monthly repayments.
The monthly repayment on a $1000 loan at the given rate over the given term is multiplied by the
number of thousands of dollars in the loan to find the size of each repayment.

Present and future


values

The present value of an investment is the amount that is invested at the beginning of the term.
The future value of a compound interest investment is the value the investment will have at the end
of the term.
The compound interest formula can be written as FV = PV(1 + r)n where
FV = future value
PV = present value
r = interest rate per period expressed as a decimal
n = number of interest periods.
Interest rates are usually expressed as a percentage per annum, however if interest is calculated
more frequently than annually the values of r and n need to be adjusted accordingly.

20 Maths Quest HSC Mathematics General 2

Chapter review
1 The total repayments for a $3400 loan on a flat rate interest of 8.5% p.a. over a 3-year period are:
A $867
B $942.78
C $4267
D 4342.78

M U LT IP L E
C H O IC E

2 A $115000 loan is repaid over a 25-year term at the rate of $1211.21 per month. The total amount of

interest that is paid on this loan is:


A $30280.25
C $248363.00

B $145280.25
D $363363.00

3 A $150000 loan is to be taken out. Which of the following loans will have the lowest total cost?
A
B
C
D

4% p.a. flat rate interest to be repaid over 20 years


8% p.a. reducible interest to be repaid over 20 years at $1254.66 per month
9% p.a. reducible interest to be repaid over 15 years at $1521.40 per month
8.5% p.a. reducible interest to be repaid over 15 years at $1512.49 per month with a $900 loan
application fee and $12 per month account management fee

4 Look at the table of loan repayments per $1000 shown below.

Term

9%

10
15
20
25

$12.67
$10.14
$9.00
$8.39

Interest rate (p.a.)


10%
11%
$13.22
$10.75
$9.65
$9.09

$13.78
$11.37
$10.32
$9.80

12%
$14.35
$12.00
$11.01
$10.53


Daniel has an $80000 mortgage at 10% p.a. over 10 years. After interest rates rise to 12% Daniel

extends the term of his loan to 15 years. What is the change in Daniels monthly repayments?
A They increase by $1.13 per month.
B They decrease by $1.22 per month.
C They increase by $90.40 per month.
D They decrease by $97.60 per month.
1 Stephanie has a credit card with an outstanding balance of $423. Calculate the minimum payment to

be made if she must pay 5% of the balance, or $10, whichever is greater.

S ho rt
a nsw er

2 Lorenzo has a credit card with an outstanding balance of $850. If the interest rate is 24% p.a.,

calculate the amount of interest that Lorenzo will be charged if the balance is not paid by the due
date.
3 Jessica pays for her car repairs, which total $256.50, using her credit card. The credit card has an

interest rate of 15% p.a. and interest is charged daily from the date of purchase. Calculate the amount
of interest charged after one month on this card.
4 Study the extract from the credit card statement below.

Interest rate = 19.5% p.a.


Date
1 Jan.
6 Jan.
15 Jan.
1 Feb.
8 Feb.
15 Feb.
1 March

Credit ($)

Debit ($)

Balance ($)
2584.75

600 repayment
39.99 purchase
??? interest
??? repayment
425.85 purchase
??? interest

a Calculate the daily rate of interest. (Take 1 year = 365.25 days and answer correct to 4 decimal

places.)

b Calculate the interest due for January.


Chapter 1 Credit and borrowing 21

c If the minimum monthly payment of 5% of the outstanding balance from January is made on

8 February, calculate the amount of this repayment.


d Calculate the interest for February.
5 Calculate the amount of flat rate interest that will be paid on each of the following loans.
a $8000 at 7% p.a. for 2 years
b $12500 at 11.5% p.a. for 5 years
c $2400 at 17.8% p.a. for 3 years
d $800 at 9.9% p.a. over 6 months
e $23400 at 8.75% p.a. over 6 years
6 Calculate the total repayments made on a loan of $4000 at 23% p.a. flat rate interest to be repaid

over 3 years.
7 Noel borrows $5600 at 7.6% p.a. flat rate interest to be repaid in monthly instalments over 3 years.

Calculate the amount of each monthly instalment.


8 Shane borrows $9500 to purchase a new car. He repays the loan over 4 years at a rate of $246.60 per

month. Calculate the flat rate of interest charged on the loan.


9 Mr and Mrs Smith borrow $125000 to purchase a home. The interest rate is 12% p.a. and the

monthly repayments are $1376.36. Calculate:


a the first months interest on the loan

b the balance of the loan after the first month.

10 Mr and Mrs Buckley borrow $130000 to purchase a home. The interest rate is 8% p.a. and over a

20-year term the monthly repayment is $1087.37.


a Copy and complete the table below.
Month
1
2
3
4
5
6
7
8
9
10
11
12

Principal ($)
130000.00
129779.29

Interest ($)
866.67

Balance ($)
129779.29

b Mr and Mrs Buckley decide to increase their monthly payment to $1500. Complete the table

below.
Month
1
2
3
4
5
6
7
8
9
10
11
12

22 Maths Quest HSC Mathematics General 2

Principal ($)
130000.00
129366.67

Interest ($)
866.67

Balance ($)
129366.67

c How much less do Mr and Mrs Buckley owe at the end of one year by increasing their monthly

repayment?
11 Use the table of repayments on page 13 to calculate the monthly repayment on each of the

followingloans.
a $25000 at 9% p.a. over a 10-year term
b $45000 at 14% p.a. over a 15-year term
c $164750 at 15% p.a. over a 25-year term
d $425000 at 12% p.a. over a 15-year term
12 Mr and Mrs Rowe take out a $233000 home loan at 12% p.a. over a 25-year term.
a Use the table of repayments to calculate the amount of each monthly repayment.
b After 3 years the balance on the loan has been reduced to $227657. The interest rate then rises

to13% p.a. Calculate the new monthly repayment required to complete the loan within the
existing term.
13 Daniela is to invest $16000 for 2 years at 9% p.a. with interest compounded annually.
a Calculate the future value of this investment.
b Calculate the amount of interest that Daniela earned.
14 Natasha invests $7000 at 6% p.a. for 4 years with interest compounded six monthly. Calculate the

future value of Natashas investment.


1 David buys a computer that has a cash price of $4600. David pays 10% deposit with the balance in

weekly instalments at 13% p.a. flat rate interest over a period of 4 years.
a Calculate the balance owing after David has paid the deposit.
b Calculate the total repayments that David must make on this loan.
c Calculate the amount of each weekly instalment.
(1 + r) n 1
d Use the formula E =
to calculate the equivalent reducible interest rate on this loan.
n
2 Mr and Mrs Tarrant borrow $186500 to purchase a home. The interest rate is 9% p.a. and the loan is
over a 20-year term.
a Use the table below to calculate the amount of each monthly repayment.
Term

9%

10
15
20
25

$12.67
$10.14
$9.00
$8.39

Interest rate (p.a.)


10%
11%
$13.22
$10.75
$9.65
$9.09

$13.78
$11.37
$10.32
$9.80

Ex tended
R es p ons e

12%
$14.35
$12.00
$11.01
$10.53

b Calculate the total amount that they can expect to make in repayments.
c After 10 years the outstanding balance is $132463 and the interest rate is increased to 11%.

Calculate the amount of the monthly repayment they will need to make to complete the loan
within the term.
d The loan has a $5 per month account management fee. The Tarrants also had a $400 loan
application fee and $132.75 in stamp duty to pay in establishing the loan. Calculate the total cost
of the loan after 20 years.
3 Paul has a credit card that has an interest-free period. The interest rate is 21% p.a.
a If Paul has an outstanding balance of $275.50, calculate the minimum payment he must make by

the due date if it is 5% of the balance, or $10, whichever is greater.


b If Paul pays only the minimum balance by the due date, calculate the balance owing for the next

month.
c Calculate the interest that Paul will be charged on his next months statement.
d If Paul pays the whole balance off next month, is this card cheaper than a card without an interest-

Digital doc
Test Yourself
doc-11022
Chapter 1

free period but an interest rate of 15% p.a.? Use calculations to justify your answer.
Chapter 1 Credit and borrowing 23

ICT activities
1A Credit cards

1CLoan repayments

Tutorial
WE1 int-0900: Evaluate credit card repayments. (page 2)
WE3 int-2410: Calculate interest payments from a credit card
statement. (page 3)

interactivity
int-0914: Reducing balance depreciation. (page 11)

Digital docs
Investigation (doc-2746): Researching credit cards. (page 6)

1B Flat rate interest


Tutorial
WE7 int-2408: Learn how to calculate repayments on hire
purchase items. (page 8)
interactivities
int-0745: Effects of P, r, I and t. (page 7)
int-2770: Simple interest. (page 7)
Digital docs
SkillSHEET 1.1 (doc-1284): Converting a percentage to a decimal.
(page 9)
SkillSHEET 1.2 (doc-1285): Finding a percentage of a quantity
(money). (page 9)
SkillSHEET 1.3 (doc-1286): Calculating simple interest. (page 9)
Spreadsheet (doc-1287): Simple interest. (page 9)
GC program Casio (doc-1288): Interest. (page 9)
GC program TI (doc-1289): Interest. (page 9)
WorkSHEET 1.1 (doc-11019): Credit cards and flat rate interest.
(page 11)

24 Maths Quest HSC Mathematics General 2

Digital docs
Spreadsheet (doc-1291): Home loan. (page 12)
SkillSHEET 1.4 (doc-1292): Finding values of n and r financial
formulas. (page 14)
SkillSHEET 1.5 (doc-1293): Calculating compound interest. (page 14)
SkillSHEET 1.6 (doc-11020): Reading tables. (page 14)
Spreadsheet (doc-1299): Reducing balance loans. (page 15)

1d Present value and future value


Interactivities
int-0810: Compound interest. (page 16)
int-2791: Compound interest. (page 16)
int-2400: Simple and compound interest. (page 18)
Digital doc
WorkSHEET 1.2 (doc-11021): Loan repayments and present and
future values. (page 19)

Chapter review
Digital doc
Test Yourself (doc-11022): Take the end-of-chapter test to test your
progress. (page 23)

To access eBookPLUS activities, log on to www.jacplus.com.au

Answers chapter 1
Credit and borrowing
Exercise 1A

Credit cards

1
3

b The Big Bank charges 11 % p.a. for a

loan while the Friendly Building Society

10 B
11 C
12 C
13 a i $17745.95
ii $17786.61
iii $17807.67
iv $17821.99
b The more frequently the interest is paid

charges 12%.
1 $40.95
20 It would be to Michaels financial
2 a $52.50
b $25.00
advantage to invest his winnings. As the
c $25.00 d
$25.00
loan is a flat rate loan, making a lump
e $138.08
sum payment will not reduce the amount
3 27.75
of interest that he must pay. If Michael
4 $18.13
the interest itself earns more interest.
invests, he will earn some interest, which
5 a $1800
b $31.50
14 a 0.0247%
would be to his financial advantage.
c $1831.50
d $1863.55
b $131319.81
6 a $1767.50 b
$1732.15
Exercise 1C Loan repayments
c $11319.81
c Interest = $20.79,
1 a $800 b
$79950
d $519.81
Balance owing = $2100.24
2 a $1125, $179456.38
15 a $7920
7 a $296.40, $256.40
b $543.62
b $8108.46
b $4.12, $260.52
c $1121.60, $178909.36
c Shanes interest is compounded meaning
8 a 0.0452% b
$19.34
d $547.02
his interest earns interest while Loris
c $73.34 d
$1411.36
3 $674.25
interest is paid on maturity and earns
9 a $1000 b
$15
4 a $90.46 b
$341.25
nothing.
c $530.23 d
$17.50
c $819.84 d
$1101.00
e $11.87 f
$535.55
e $1515.54
Chapter Review
g The card with the interest-free period
5 a $400 b
$3600
Multiple choice
10 $400
c $123.05
1 C
11 a $0 b
$13.31
6 They will not need to increase their
2 C
12 17.51% p.a.
repayments.
3 A
13 a $15.68
7 a $1510.20 b
$1620.14
4 D
b Kylie should not accept the offer as she
8 Yes. The repayment is $744 and the most
will be charged $21.71 in interest over
2 months compared to $15.68 with her
current card.
14 Card A charges 18% p.a. on $500 unpaid
balance, which equals $7.50 per month
interest. Card B charges 12% p.a. on
$1000 each month, which equals $10 per
month interest. Rob should choose Card A.

he can afford is $750.

Short answer

9 a $7000 b
$1750
1 $21.15
c $178000
2 $17.00
10 a $733.40 b
$174.80
3 $3.21
11 a $2895 b
$868500
4 a 0.0534%
c $493500 d
$19740
b $34.82
e 5.264%
c $102.99
12 a $1696.80 b
$509040
d $32.65
c $269040 d
4.484%
Exercise 1B Flat rate interest
5 a $1120
e i4.3%
ii4.12%
1 a $700 b
$1200
b $7187.50
13 a $2121 b
$2316
c $7500 d
$2850
c $1281.60
c $2340 d
$45
e $390
d $39.60
14 $268
2 $1584
e $12285.00
15 a $2000 b
$238000
3 $5000
6 $6760
16 a $2325 b
$2508
4 a $4060 b
$21330
7 $191.02
c $2416.50 d
$2370.75
c $1803.75 d
$308.25
8 6.15%
17 a $2517 b
5 years
e $275000
9 a $1250
c $151020
5 a $1650 b
$3850
b $124873.64
18 a $2508 b
$601920
c $693 d
$6193
10 a
c Marcus would only need to pay $2450
6 a $1600 b
$6600

to pay the loan off in 18 years, so he will


c $137.50
Month
achieve this.
7 a $800 b
$2800
1
c $53.85
Exercise 1D Present value and future
8 a $2000 b
$6000
2
value
c $2160 d
$8160
1 $2837.04
3
e $226.67
2 a $4630.50 b
$630.50
4
9 $43.33
3 a $3244.80 b
$10939.56
10 B
5
c $24617.98 d
$14678.02
11 C
e $ 14838.45
6
12 8%
f $129394.77
7
13 a $2400 b
$9600
4 $17240.15
8
c $16319.88 d
15%
5 $605.42
14 15%
6 $18503.86
9
15 a $1515.79 b
$2133.33
7 a 0.583% b
$42891.60
10
16 2 years
8 a $932.52 b
$10410.00
11
17 B
c $193843.76 d
25816.04
18 B
12
e $3 145511
19 a The Big Bank offers the best rates.
9 B

Principal
($)

Interest
($)

Balance
($)

130000.00
129779.30
129557.12
129333.47
129108.32
128881.67
128653.51
128423.83
128192.62
127959.87
127725.56
127489.70

866.67
865.20
863.71
862.22
860.72
859.21
857.69
856.16
854.62
853.07
851.50
849.93

129779.30
129557.12
129333.47
129108.32
128881.67
128653.51
128423.83
128192.62
127959.87
127725.56
127489.70
127252.26

Chapter 1 Credit and borrowing 25

Month

Principal Interest Balance


($)
($)
($)

130000.00 866.67 129366.67

129366.67 862.44 128729.11

128729.11 858.19 128087.31

128087.31 853.92 127441.22

127441.22 849.61 126790.83

126790.83 845.27 126136.10

126136.10 840.91 125477.01

125477.01 836.51 124813.52

124813.52 832.09 124145.61

10

124145.61 827.64 123473.25

11

123473.25 823.16 122796.40

12

122796.40 818.64 122115.05

11 a $316.75 b
$599.40
c $2110.45 d
$5100
12 a $2453.49 b
$2618.06
13 a $19009.60 b
$3009.60
14 $8867.39
Extended response

1 a $4140 b
$6292.80
c $30.25 d
15.76%
$402840
2 a $1678.50 b
c $1825.34 d
$422193.55
3 a $13.78
b $261.72
c $4.58
d The card with this interest-free period
is cheaper as the card without the
interest-free period would have charged
$6.75interest.

c $5137.21

26 Maths Quest HSC Mathematics General 2

Chapter 2

Annuities and loan repayments


CHAPTER CONTENTS
2A Future value of an annuity
2B Present value of an annuity
2C Loan repayments
2D Loan repayment graphs

2A

Future value of an annuity

An annuity is a form of investment involving regular periodic contributions to an account. On such an


investment, interest compounds at the end of each period and the next contribution to the account is
made.
Superannuation is a common example of an annuity. With superannuation, people invest in a fund
on a regular basis, the interest on the investment compounds while each subsequent period a further
contribution is added to the principal.
In Chapter 1 we used the compound interest formula and introduced the terms present value and
future value.
The compound interest formula is:
FV = PV(1 + r) n
FV = the future value of the investment, that is, the value that the investment will have at the end of
the investment period.
PV = the present value of the annuity, that is, the single investment required at the current time to
produce the future value.
r = interest rate per compounding period expressed as a decimal.
n = number of compounding periods.

interactivity
int-0193
Comparing
investment options

WORKED EXAMPLE 1

Calculate the value of a $5000 investment made at 8% p.a. for 4 years.


THINK

WRITE

Write the values of P, r and n.

P = $5000, r = 0.08, n = 4

Write the formula.

A = P(1 + r)n

Substitute values for P, r and n.

= $5000 (1.08)4

Calculate the value of A.

= $6802.44

An annuity takes the form of a sum of compound interest investments. Consider the case of a person
who invests $1000 at 10% p.a. at the end of each year for 5 years.
To calculate the total value of this investment we would need to calculate the value of:
The first $1000 invested for 4 years at 10% p.a.
The second $1000 invested for 3 years at 10% p.a.
Chapter 2 Annuities and loan repayments 27

The third $1000 invested for 2 years at 10% p.a.


The fourth $1000 invested for 1 year at 10% p.a.
Each of these investments would be added together with the final $1000, which is added at the end of
the investment period.
1st contribution
FV = PV (1 + r)n
= $1000(1.1)4
= $1464.10

2nd contribution
FV = PV (1 + r)n
= $1000(1.1)3
= $1331.00

3rd contribution
FV = PV (1 + r)n
= $1000(1.1)2
= $1210.00

4th contribution
FV = PV (1 + r)n
= $1000(1.1)
= $1100.00

The future value of this annuity (FVA) is therefore


FVA = $1464.10 + $1331.00 + $1210.00 + $1100.00 + $1000.00
= $6105.10
In the case of most annuities it is not practical to find the total value of the annuity using this method.
The future value of an annuity can be calculated using the formula
FVA = a e

(1 + r) n 1
f
r

where FVA = future value of the annuity


a= the contribution per period paid at the end of the period
r= interest rate per compounding period expressed as a decimal
n= number of compounding periods.
Consider Worked example 1.
(1 + r) n 1
FVA = a e
f
r
FVA = $1000 e

(1.1) 5 1
0.1

= $6105.10

In this course you are not required to calculate the future value of an annuity using this formula. In
practice the future value of an annuity is calculated using a table of values that show the future value of
an annuity where $1 is contributed each period.
Suppose that an annuity is created where $1 is paid at the end of each year for a period of 20 years.
The interest rate is 8% p.a. with interest compounded annually.
Using the formula above
FVA = a e

FVA = 1 e

(1 + r) n 1
f
r
(1.08) 20 1
0.08

= 45.7620

This value can be used to calculate the future value of any similar annuity. For example consider an
annuity where a contribution of $2500 is made at the end of each year for 20 years at an interest rate of
8% p.a. with interest compounded annually.
FVA = 2500 45.7620
= $114 405
We can create a table of future values using the above formula and an Excel spreadsheet.

Computer application 1: Future value of $1


Consider $1 invested into an annuity interest period. The table we are going construct on a spreadsheet
shows the future value of that $1.
1. Open a new spreadsheet.
2. Type in the following information as shown in step 3.
28 Maths Quest HSC Mathematics General 2

3. In cell B4 enter the formula =((1+B$3)^$A41)/B$3. (This is the future value formula from
Exercise2A with the value of M omitted, as it is equal to 1.) Format the cell, correct to
4 decimalplaces.
4. Highlight the range of cells B3 to M23. From the Edit menu, use Fill Down and Fill Right functions
to copy the formula to all other cells in this range.

This completes the table. The table shows the future value of an annuity of $1 invested for up to
10interest periods at up to 10% per interest period. You can extend the spreadsheet further for other
interest rates and longer investment periods.
The table overleaf is the set of future values of $1 invested into an annuity. This is the table you
should have obtained in Computer application 1.
A table such as this can be used to find the value of an annuity by multiplying the amount of the
annuity by the future value of $1.

WORKED EXAMPLE 2

Use the table on page 30 to find the future value of an annuity when $1500 is invested at the end of
each year at 7% p.a. with interest compounded annually for 9 years.
THINK

WRITE

Look up the future value of a $1 annuity at


7%p.a. for 9 years.

Future value of $1 = 11.9780

Multiply this value by the amount of each


contribution, $1500.

Future value = $1500 11.9780


= $17967

Chapter 2 Annuities and loan repayments 29

Future value of $1
Interest rate per period
Period

1%

2%

3%

4%

5%

6%

7%

8%

9%

10%

11%

12%

1.0000

1.0000

1.0000

1.0000

1.0000

1.0000

1.0000

1.0000

1.0000

1.0000

1.0000

1.0000

2.0100

2.0200

2.0300

2.0400

2.0500

2.0600

2.0700

2.0800

2.0900

2.1000

2.1100

2.1200

3.0301

3.0604

3.0909

3.1216

3.1525

3.1836

3.2149

3.2464

3.2781

3.3100

3.3421

3.3744

4.0604

4.1216

4.1836

4.2465

4.3101

4.3746

4.4399

4.5061

4.5731

4.6410

4.7097

4.7793

5.1010

5.2040

5.3091

5.4163

5.5256

5.6371

5.7507

5.8666

5.9847

6.1051

6.2278

6.3528

6.1520

6.3081

6.4684

6.6330

6.8019

6.9753

7.1533

7.3359

7.5233

7.7156

7.9129

8.1152

7.2135

7.4343

7.6625

7.8983

8.1420

8.3938

8.6540

8.9228

9.2004

9.4872

9.7833 10.0890

8.2857

8.5830

8.8923

9.2142

9.5491

9.8975 10.2598 10.6366 11.0285 11.4359 11.8594 12.2997

9.3685

9.7546 10.1591 10.5828 11.0266 11.4913 11.9780 12.4876 13.0210 13.5795 14.1640 14.7757

10

10.4622 10.9497 11.4639 12.0061 12.5779 13.1808 13.8164 14.4866 15.1929 15.9374 16.7220 17.5487

11

11.5668 12.1687 12.8078 13.4864 14.2068 14.9716 15.7836 16.6455 17.5603 18.5312 19.5614 20.6546

12

12.6825 13.4121 14.1920 15.0258 15.9171 16.8699 17.8885 18.9771 20.1407 21.3843 22.7132 24.1331

13

13.8093 14.6803 15.6178 16.6268 17.7130 18.8821 20.1406 21.4953 22.9534 24.5227 26.2116 28.0291

14

14.9474 15.9739 17.0863 18.2919 19.5986 21.0151 22.5505 24.2149 26.0192 27.9750 30.0949 32.3926

15

16.0969 17.2934 18.5989 20.0236 21.5786 23.2760 25.1290 27.1521 29.3609 31.7725 34.4054 37.2797

16

17.2579 18.6393 20.1569 21.8245 23.6575 25.6725 27.8881 30.3243 33.0034 35.9497 39.1899 42.7533

17

18.4304 20.0121 21.7616 23.6975 25.8404 28.2129 30.8402 33.7502 36.9737 40.5447 44.5008 48.8837

18

19.6147 21.4123 23.4144 25.6454 28.1324 30.9057 33.9990 37.4502 41.3013 45.5992 50.3959 55.7497

19

20.8109 22.8406 25.1169 27.6712 30.5390 33.7600 37.3790 41.4463 46.0185 51.1591 56.9395 63.4397

20

22.0190 24.2974 26.8704 29.7781 33.0660 36.7856 40.9955 45.7620 51.1601 57.2750 64.2028 72.0524

As shown in Chapter 1, when considering compound interest we need to consider the compounding
period and adjust the values of r and n accordingly. The same applies when calculating annuities.

WORKED EXAMPLE 3

Andrea invests $500 every six months into an annuity that pays 6% p.a. interest compounded
six-monthly. Calculate the future value of this annuity after 5 years.
THINK

WRITE

Write down the values of r and n.

r = 6% 2, n = 5 2
= 3%= 10

Look up the future value of a $1 at 3% per


period for 10 periods.

Future value of $1 = 11.4639

Calculate the future value by multiplying this


value by the amount of each contribution.

FVA = $500 11.4639

= $5713.95

30 Maths Quest HSC Mathematics General 2

Exercise 2A

Future value of an annuity

1 WE1 Calculate the value after 5 years of an investment of $4000 at 12% p.a., with interest

compounded annually.
2 Calculate the value to which each of the following compound interest investments will grow.
a $5000 at 6% p.a. for 5 years, with interest calculated annually
b $12000 at 12% p.a. for 3 years, with interest calculated annually
c $4500 at 8% p.a. for 4 years, with interest compounded six-monthly
d $3000 at 9.6% p.a. for 3 years, with interest compounded six-monthly
e $15000 at 8.4% p.a. for 2 years, with interest compounded quarterly
f $2950 at 6% p.a. for 3 years, with interest compounded monthly
3 At the end of each year for four years Rodney invests $1000 in an investment fund that pays

7.5%p.a. interest, compounded annually. By calculating each investment of $1000 separately,


usethe compound interest formula to calculate the future value of Rodneys investment after
fouryears.
4 Caitlin is saving for a holiday in two years and so every six months she invests $2000 in an account

that pays 7% p.a. interest, with the interest compounding every six months.
a Use the compound interest formula to calculate the amount to which the:
i first investment of $2000 will grow
ii second investment of $2000 will grow
iii third investment of $2000 will grow
iv fourth investment of $2000 will grow.
b If Caitlin then adds a final deposit of $2000 to her account immediately before her holiday, what
is the total value of her annuity?
5 WE2 Use the table of future values on page 30 to determine the future value of an annuity of $800

invested per year for 5 years at 9% p.a., with interest compounded annually.
6 Use the table of future values to determine the future value of each of the following annuities.
a $400 invested per year for 3 years at 10% p.a., with interest compounded annually
b $2250 invested per year for 8 years at 8% p.a., with interest compounded annually
c $625 invested per year for 10 years at 4% p.a., with interest compounded annually
d $7500 invested per year for 7 years at 6% p.a., with interest compounded annually
7 WE3 Samantha invests $500 every 6 months for 5 years in an annuity at 8% p.a., with interest

compounded every 6 months.


a What is the interest rate per interest period?
b How many interest periods are there in Samanthas annuity?
c Use the table to calculate the future value of Samanthas annuity.
8 Use the table to calculate the future value of each of the following annuities.
a $400 invested every 6 months for 4 years at 14% p.a., with interest compounded six-monthly
b $600 invested every 3 months for 2 years at 12% p.a., with interest compounded quarterly
c $100 invested every month for 5 years at 10% p.a., with interest compounded six-monthly.
9 Use the table of future values to determine whether an annuity at 5% p.a. for 6 years or an annuity at

6% p.a. for 5 years will produce the greatest financial outcome. Explain your answer.
10 MC Use the table of future values to determine which of the following annuities will have the

greatest financial outcome.


A 6% p.a. for 8 years, with interest compounded annually
B 8% p.a. for 6 years, with interest compounded annually
C 7% p.a. for 7 years, with interest compounded annually
D 10% p.a. for 5 years, with interest compounded six-monthly

Further development
11 Barbara currently has $60000 in an investment account that is averaging an interest rate of

6%p.a.,compounded annually. She wants to calculate the amount that she will receive after
20years.
a Use the compound interest formula to find the value of $60000 investment at maturity.
b If she deposits $9000 each year, use the table on page 30 to find the future value of this annuity.
Chapter 2 Annuities and loan repayments 31

c If she places her $60000 into a different savings account that can offer 8% p.a. compounded

annually and increases her deposits to $10000 each year, find the amount available to her after
20years.
d Over the 20-year period, calculate the extra amount saved by investing $60000 in an investment
account and $10000 each year at 8% p.a. compared with $9000 each year at 6% p.a.
12 Find the final value of the following annuity investments by using the compound interest formula

together with the table on page 30.


a An initial amount of $10000 earning 6% p.a. with annual contributions of $7000 for the next
20years.
b An initial amount of $400000 earning 10% p.a. with annual contributions of $12000 for the next
5 years.
c An initial amount of $100000 earning 8% p.a. compounded six-monthly with six-monthly
contributions of $1200 for the next 9 years.
d An initial amount of $50000 earning 12% p.a. compounded quarterly with quarterly contributions
of $1200 for the next 5 years.
13 Merle makes an investment of $1000 into an annuity at the end of each year. He has the option of

investing at 8% p.a. with interest compounded annually or six-monthly.


a Calculate the future value of the annuity where interest is compounded annually.
b If Merle chooses the option where interest is compounded six-monthly he will need to
contribute$500 to the annuity at the end of each interest period. Calculate the future value of this
investment.
c Calculate the percentage increase in the future value that Merle will receive if he invests with
interest compounded six-monthly.

2B

Present value of an annuity

In Chapter 1 we learned about the present value of an investment under compound interest. In the
formula
elesson
eles-0117
Small business

FV = PV(1 + r) n
PV is the present value of the annuity, that is, the single investment required at the current time to
produce the future value.
The same idea applies to an annuity. The present value of an annuity is the single sum investment
thatcan be made under compound interest that will produce the same future value as the annuity in
question.
As we learned in Chapter 1 the present value of an investment under compound interest can be
calculated using the formula
PV =

FV
(1 + r) n

Now let us consider the annuity in Worked example 2. In this example $1500 was invested at the end of
each year at 7% p.a., with interest compounded annually for 9 years. This investment has a future value
of $17967.
If we are to consider the present value of this investment we want to find the single sum that needs to
be invested at 7% p.a., with interest compounded annually for 9 years, that will produce this financial
outcome.
PV =
=

FV
(1 + r) n
17967
(1.07) 9

= $9772.86
This means that making a single investment of $9772.86 at the beginning of the term is equivalent to
investing $1500 each year for 9 years.
32 Maths Quest HSC Mathematics General 2

Comparing the two investments further:


Annuity

Single sum investment

Future value

$17967

Investors contribution

$1500 9 = $13500

$9772.86

Interest

$4467

$8194.14

$17967

As you can see, a single-sum contribution generates more interest from a smaller contribution
than anannuity, however for the single-sum contribution the entire contribution is required at
thebeginningofthe investment, whereas for the annuity a much smaller annual contribution
is made.
FV
The formula PV =
is used to calculate the present value of an annuity when we know the
(1
+ r) n
future value.
(1 + r) n 1
f can be used to calculate the present value of an annuity when
r(1 + r) n
given the amount of each contribution. In this formula:
FVA = future value of the annuity
a = the contribution per period paid at the end of the period
r = interest rate per compounding period expressed as a decimal
n = number of compounding periods.
Now consider the annuity in Worked example 3. In this example a $500 contribution was being made
every six months. The interest rate is 6% p.a. compounded six-monthly for a 5-year term.
Therefore a = 500, r = 0.03 and n = 10
The formula PVA = a e

PVA = a e

(1 + r) n 1
r(1 + r) n

PVA = 500 e

(1.03) 10 1
0.03 (1.03) 10

= $4265.10

Also, as with future values, you will not be required to calculate present values using this formula.
The formula is used to also create a table of present values of $1 under various investments.
Consider the present value of an annuity where $1 is invested each interest period at 4% per interest
period for 20 interest periods.
PVA = a e
PVA = 1 e

(1 + r) n 1
r(1 + r) n

(1.04) 20 1

0.04 (1.04) 20

= 13.5903

This means that a $1 annuity at 4% per interest period for 20 interest periods is equivalent to a single
investment of $13.59

Computer application 2: Present value table


The table we are about to make on a spreadsheet shows the present value of an annuity of $1 invested
per interest period.
1. Open a new spreadsheet.
2. Enter the following information.
3. In cell B4 type the formula =((1+B$3)^$A41)/(B$3*(1+B$3)^$A4).
4. Drag from cell B4 to K13, and then from the Edit menu use the Fill Down and Fill Right functions
to copy this formula to the remaining cells in your table.
Chapter 2 Annuities and loan repayments 33

The table created in Computer application 2 shows the present value of an annuity $1 per interest
period for up to 12% per interest period and for up to 20 interest periods.
The table below shows the results that you should obtain for the above computer application and
should be used to complete the following exercise.
Present value of $1
Interest rate per period
Period

1%

2%

3%

4%

5%

6%

0.9901

0.9804

0.9709

0.9615

0.9524

0.9434

1.9704

1.9416

1.9135

1.8861

1.8594

2.9410

2.8839

2.8286

2.7751

3.9020

3.8077

3.7171

4.8534

4.7135

5.7955

9%

10%

11%

12%

0.9346 0.9259

0.9174

0.9091

0.9009

0.8929

1.8334

1.8080 1.7833

1.7591

1.7355

1.7125

1.6901

2.7232

2.6730

2.6243 2.5771

2.5313

2.4869

2.4437

2.4018

3.6299

3.5460

3.4651

3.3872 3.3121

3.2397

3.1699

3.1024

3.0373

4.5797

4.4518

4.3295

4.2124

4.1002 3.9927

3.8897

3.7908

3.6959

3.6048

5.6014

5.4172

5.2421

5.0757

4.9173

4.7665 4.6229

4.4859

4.3553

4.2305

4.1114

6.7282

6.4720

6.2303

6.0021

5.7864

5.5824

5.3893 5.2064

5.0330

4.8684

4.7122

4.5638

7.6517

7.3255

7.0197

6.7327

6.4632

6.2098

5.9713 5.7466

5.5348

5.3349

5.1461

4.9676

8.5660

8.1622

7.7861

7.4353

7.1078

6.8017

6.5152 6.2469

5.9952

5.7590

5.5370

5.3282

10

9.4713

8.9826

8.5302

8.1109

7.7217

7.3601

7.0236 6.7101

6.4177

6.1446

5.8892

5.6502

11

10.3676

9.7868

9.2526

8.7605

8.3064

7.8869

7.4987 7.1390

6.8052

6.4951

6.2065

5.9377

12

11.2551 10.5753

9.9540

9.3851

8.8633

8.3838

7.9427 7.5361

7.1607

6.8137

6.4924

6.1944

34 Maths Quest HSC Mathematics General 2

7%

8%

Present value of $1 (continued)


Interest rate per period
Period

1%

2%

3%

4%

5%

6%

7%

8%

9%

10%

11%

12%

13

12.1337 11.3484 10.6350

9.9856

9.3936

8.8527

8.3577 7.9038

7.4869

7.1034

6.7499

6.4235

14

13.0037 12.1062 11.2961 10.5631

9.8986

9.2950

8.7455 8.2442

7.7862

7.3667

6.9819

6.6282

15

13.8651 12.8493 11.9379 11.1184 10.3797

9.7122

9.1079 8.5595

8.0607

7.6061

7.1909

6.8109

16

14.7179 13.5777 12.5611 11.6523 10.8378 10.1059

9.4466 8.8514

8.3126

7.8237

7.3792

6.9740

17

15.5623 14.2919 13.1661 12.1657 11.2741 10.4773

9.7632 9.1216

8.5436

8.0216

7.5488

7.1196

18

16.3983 14.9920 13.7535 12.6593 11.6896 10.8276 10.0591 9.3719

8.7556

8.2014

7.7016

7.2497

19

17.2260 15.6785 14.3238 13.1339 12.0853 11.1581 10.3356 9.6036

8.9501

8.3649

7.8393

7.3658

20

18.0456 16.3514 14.8775 13.5903 12.4622 11.4699 10.5940

9.1285

8.5136

7.9633

7.4694

9.8181

This table can be used in the same way as the future values table.
WORKED EXAMPLE 4

Liam invests $750 per year in an annuity at 6% per annum for 8 years, with interest compounded
annually. Use the table to calculate the present value of Liams annuity.
THINK

WRITE

Use the table to find the present value of


$1annuity at 6% for 8 interest periods.

Present value of $1 = 6.2098

Multiply this value by 750.

Present value = $750 6.2098


= $4657.35

Exercise 2B

Present value of an annuity

1 WE4 Use the table of present values on page 34 to determine the present value of an annuity of

$1250 per year for 8 years invested at 9% p.a.


2 Use the table of present values to determine the present value of each of the following annuities.
a $450 per year for 5 years at 76% p.a., with interest compounded annually
b $2000 per year for 10 years at 10% p.a., with interest compounded annually
c $850 per year for 6 years at 4% p.a., with interest compounded annually
d $3000 per year for 8 years at 9.5 p.a., with interest compounded annually
3 Use the table of present values on page 34 to determine the present value of an investment of $500 at

the end of each quarter at 12% p.a., with interest compounded quarterly for 3 years.
4 Use the table of present values to find the present value of each of the following annuities.
a $400 invested every 6 months for 4 years at 14% p.a., with interest compounded six-monthly
b $600 invested every 3 months for 2 years at 12% p.a., with interest compounded quarterly
c $100 invested every month for 1 year at 12% p.a., with interest compounded monthly
5 Calculate the present value of each of the following annuities.
a $1000 invested at the end of each year at 8% p.a. for 5 years with interest compounded annually
b $500 invested at the end of June and December for 5 years with interest compounded every six

months
c $250 invested at the end of each quarter for 5 years with interest compounded quarterly
6 Jonah invests $2500 each year into his employee superannuation fund. The find earns 6% p.a., with

interest compounded annually. Given that his employer matches this investment calculate the present
value of his superannuation assuming that Jonah will work for a further 20 years.
Chapter 2 Annuities and loan repayments 35

7 Bernice invests $2000 per year into a superannuation fund that pays 8% p.a., with interest

compounded annually for 10 years.


a Calculate the present value of the annuity.
b If the annuity is split into two $1000 payments and interest is paid six-monthly, calculate the
present value.
c Find the percentage change in the present value of the annuity under the two conditions. (Give
your answer correct to two decimal places.)

Further development
8 Justin is aged 42 and plans to retire at age 60. He estimates that he needs $680000 to provide for

his retirement. His current superannuation fund has a balance of $40000 and is returning 12% p.a.
compounded annually.
a Use the compound interest formula to find the future value of Justins current
superannuation.
b Use the future value table on page 30 to find the annual contribution needed to reach his target.
9 Johnny has an annuity of $1500 per year at 5.5% p.a., with interest compounded annually over a

20-year period.
a Use the table on page 30 to estimate the future value of a $1 annuity over a 20 year period.
b Use your answer to part a to find the future value of Johnnys annuity.
c Use the table on page 34 to estimate the present value of this annuity.
10 Jodie has an annuity into which she pays $1800 every quarter. The interest rate is 10% p.a., with
Digital doc
WorkSHEET 2.1
doc-11023

interest compounded quarterly. The annuity is to run for 4 years.


a Use the table to estimate the future value of Jodies annuity.
b Jodies fianc Paul wants to make a single investment compounded annually at 10% interest with
a future value equal to Jodies annuity. Find the amount of the investment that Paul must make.

2C

Loan repayments

We studied loans and the way in which they are repaid in Chapter 1. We learned that most loans have
interest calculated and added each month and at the same time a monthly repayment is made. This monthly
payment is calculated so that the outstanding balance on the loan at the end of the term is exactly zero.
A loan can be considered much like an annuity where we need to calculate the amount of each
contribution (the monthly repayment) to generate a certain future value (amount of the loan).
To begin let us consider a simple example. $5000 is to be borrowed over a three-year term at 9% p.a.
interest, which is added annually. The loan is to be repaid in three equal annual instalments.
The present value interest factor for 9% per interest period over three interest periods is 2.5313.
From our previous work we know that PVA = a PVIF so considering PFA to be the amount of the loan
and a the amount of each monthly repayment we form the equation.
Therefore $5000 = a 2.5313
a = $5000 2.5313
= $1975.26
We will now examine the progress of this loan with an annual repayment of $1975.26.
Opening balance

Interest

Closing balance

Year 1

$5000

9% of $5000
= $450

$5000 + $450 $1975.26


= $3474.74

Year 2

$3474.74

9% of $3474.74
= $312.73

$3474.74 + $312.73 $1975.26


= $1812.21

Year 3

$1812.21

9% of $1812.21
= $163.10

$1812.21 + $163.10 $1975.26


= $0.05

At the end of the third year the balance of the loan is zero (well almost!). The 5 cents that is still
owing on the loan is due to rounding to the nearest cent in each calculation. And yes, the bank will make
you pay the extra five cents in the last repayment.
36 Maths Quest HSC Mathematics General 2

In practice, because the interest and repayments on loans are made monthly the interest rates that are
used are much smaller and the interest periods much greater than those used in the table of future value
interest factors on page 30.
The table below shows the present value interest factors for common monthly interest rates and for
loans up to 25 years (300 months).
Table of present value interest factors
0.005

0.0055

0.006

0.0065

0.007

0.0075

0.008

0.0085

0.009

0.0095

0.01

12

11.6189

11.5818

11.5448

11.5080

11.4714

11.4349

11.3986

11.3625

11.3265

11.2907

11.2551

24

22.5629

22.4258

22.2899

22.1552

22.0216

21.8891

21.7578

21.6276

21.4984

21.3704

21.2434

36

32.8710

32.5791

32.2907

32.0060

31.7247

31.4468

31.1723

30.9012

30.6334

30.3689

30.1075

48

42.5803

42.0856

41.5988

41.1199

40.6486

40.1848

39.7284

39.2792

38.8372

38.4022

37.9740

60

51.7256

50.9865

50.2621

49.5520

48.8559

48.1734

47.5042

46.8481

46.2047

45.5738

44.9550

72

60.3395

59.3205

58.3253

57.3534

56.4041

55.4768

54.5710

53.6859

52.8212

51.9762

51.1504

84

68.4530

67.1236

65.8300

64.5713

63.3463

62.1540

60.9933

59.8634

58.7632

57.6918

56.6485

96

76.0952

74.4296

72.8149

71.2492

69.7310

68.2584

66.8300

65.4442

64.0995

62.7944

61.5277

108

83.2934

81.2702

79.3159

77.4277

75.6030

73.8394

72.1345

70.4860

68.8918

67.3497

65.8578

120

90.0735

87.6751

85.3666

83.1439

81.0035

78.9417

76.9552

75.0409

73.1955

71.4164

69.7005

132

96.4596

93.6720

90.9981

88.4326

85.9703

83.6064

81.3364

79.1558

77.0606

75.0468

73.1108

144 102.4747

99.2869

96.2396

93.3257

90.5383

87.8711

85.3180

82.8733

80.5317

78.2879

76.1372

156 108.1404 104.5441 101.1180

97.8527

94.7395

91.7700

88.9366

86.2318

83.6489

81.1813

78.8229

168 113.4770 109.4665 105.6585 102.0411

98.6033

95.3346

92.2252

89.2659

86.4484

83.7644

81.2064

180 118.5035 114.0753 109.8845 105.9163 102.1569

98.5934

95.2139

92.0070

88.9624

86.0704

83.3217

192 123.2380 118.3905 113.8177 109.5015 105.4251 101.5728

97.9300

94.4834

91.2203

88.1290

85.1988

204 127.6975 122.4308 117.4785 112.8186 108.4309 104.2966 100.3985

96.7205

93.2479

89.9669

86.8647

216 131.8979 126.2138 120.8857 115.8875 111.1953 106.7869 102.6419

98.7416

95.0688

91.6076

88.3431

228 135.8542 129.7558 124.0569 118.7269 113.7377 109.0635 104.6807 100.5676

96.7042

93.0723

89.6551

240 139.5808 133.0721 127.0084 121.3539 116.0760 111.1450 106.5336 102.2171

98.1728

94.3799

90.8194

252 143.0908 136.1773 129.7555 123.7844 118.2265 113.0479 108.2175 103.7074

99.4917

95.5473

91.8527

264 146.3969 139.0846 132.3123 126.0331 120.2044 114.7876 109.7479 105.0537 100.6762

96.5894

92.7697

276 149.5110 141.8067 134.6920 128.1136 122.0234 116.3781 111.1387 106.2699 101.7399

97.5198

93.5835

288 152.4441 144.3554 136.9068 130.0384 123.6963 117.8322 112.4027 107.3688 102.6952

98.3503

94.3056

300 155.2069 146.7418 138.9683 131.8193 125.2349 119.1616 113.5515 108.3615 103.5531

99.0918

94.9466

r
N

WORKED EXAMPLE 5

Use the table of present value interest factors to calculate the monthly repayment of a loan of
$250000 at 0.5% per month over 20 years.
THINK

WRITE

Write down the value of r and N.

r = 0.005, N = 240

Write down the present value interest factor.

PVIF = 139.5808

Write an equation using PVA = a PVIF.

250000 = a 139.5808

Solve the equation for a to find the amount of


each repayment.

a = $1791.08

Chapter 2 Annuities and loan repayments 37

It is important, however, to remember that unlike in Worked example 6, interest rates are usually
expressed as a percentage per annum (p.a.). These interest rates need to first be converted to a monthly
rate and then to a decimal before we are able to use the table of present value interest factors.
WORKED EXAMPLE 6

Mr. and Mrs Grimes borrow $500000 to buy a home. The interest rate is 9% p.a. and is
calculated monthly. The loan is to be repaid over 25 years. Use the table of present value interest
factors to calculate the amount of each repayment.
THINK

WRITE

Write down the value of r and N.

r = 0.09 12, N = 25 12
= 0.0075
= 300

Write down the present value interest factor.

PVIF = 119.1616

Write an equation using PVA = a PVIF.

500000 = a 119.1616

Solve the equation for a to find the amount of


each repayment.

a = $4195.98

The table can also be used to calculate the approximate length of time that it takes to repay a loan.
This is done using the same equation (PVA = a PVIF) but solving for the interest factor. The nearest
interest factor in the table will give the approximate time taken to repay the loan.
WORKED EXAMPLE 7

Use the table on page 37 to calculate the length of time that it will take to repay a $20000 loan at
an interest rate of 6% p.a. with a repayment of $900 per month.
THINK

WRITE

Write down the value of r.

r = 0.005

Write an equation using PVA = a PVIF,


with PVA = 20000 and a= 900.

20000 = 900 PVIF

Solve the equation for PVIF.

PVIF = 22.2222

Look in the 0.005 column for the interest


factor closest to 22.2222 (22.5629). Read off
the corresponding value of N.

N = 24, the loan will take 2 years to pay off.

Exercise 2C

Loan repayments

To calculate monthly repayments in this exercise use the table of present value interest factors on
page 37.
1 WE5 Calculate the monthly repayment on a loanof$15000 over 5 years at 0.65% per month.
2 WE6 Yiannis takes out a $10000 loan over 5 years at 10.2% p.a. reducible interest

withequalmonthly repayments to be made. Calculate the amount of each monthly repayment.


3 Calculate the amount of each monthly repayment on a loan of $8000 to be repaid over 4 years at

12% p.a.
4 Calculate the amount of each monthly repayment on each of the following loans.
a $2000 at 12% p.a. over 2 years
b $15000 at 9% p.a. over 5 years
c $120000 at 6% p.a. over 20 years
d $23000 at 9.6% p.a. over 5 years
e $210000 at 7.2% p.a. over 25 years
38 Maths Quest HSC Mathematics General 2

5 Jack and Diane take out a $275000 home loan. If the interest rate on the loan is 8.4% p.a. reducible

and the term of the loan is 25 years, calculate the amount of each monthly repayment.
6 Jiro purchases a computer on terms. The cash price of the computer is $3750. The terms are

a deposit of 10.00% with the balance paid in equal monthly instalments at 9% p.a.
reducible interest over 3years.
a Calculate Jiros deposit on the computer.
b What is the balance owing on the computer?
c Calculate the amount of each monthly repayment.
7 Jeremy and Patricia spend $15000 on new furnishings for their home. They pay a 15%

deposit on the furnishings with the balance paid in equal monthly instalments at 11.4% p.a.
interest over 4 years. Calculate the amount of each monthly repayment.

8 Thanh is purchasing a car on terms. The cash price of the car is $35000 and he pays a $7000 deposit.
a What is the balance owing on the car?
b If the car is to be repaid in equal monthly instalments over 5 years at an interest rate of 9% p.a.

reducible interest, calculate the amount of each monthly payment.

9 Ron borrows $33500 to purchase a car. The loan is to be repaid in equal monthly instalments over a

3-year term at an interest rate of 12% p.a. Calculate the total repayments made on the loan.
10 Calculate the total repayments on each of the following loans.
a $4000 at 8.4% p.a. reducible interest to be repaid over 2 years in equal monthly repayments
b $20000 at 6% p.a. reducible interest to be repaid over 6 years in equal monthly instalments
c $60000 at 7.2% p.a. reducible interest to be repaid over 15 years in equal monthly instalments
d $150000 at 10.8% p.a. reducible interest to be repaid over 20 years in equal monthly instalments
11 MC A loan of $5000 is taken out at 9% p.a. reducible interest over 4 years. Which of the following

equations will be used to find the amount of each monthly repayment?


A 5000 = a 40.1848
B 5000 = a 38.8372
C 5000 = a 11.4349
D 5000 = a 11.3265
12 MC A loan of $12000 is taken out at 12% p.a. reducible interest in equal monthly instalments over

5 years. The total amount of interest paid on the loan is:


B $4016.02
C $7200

A $266.93

D $16016.02

13 WE7 Use the table on page 37 to calculate the length of time that it will take to repay a $50000

loan at an interest rate of 9% p.a. with a $900 per month repayment.


14 A loan of $255000 is taken out over 15 years at 9% p.a. reducible interest.
a Calculate the amount of the minimum monthly payment.
b If the payment is increased to $2900 per month, calculate the length of time that it will take to

repay the loan.


Chapter 2 Annuities and loan repayments 39

15 A $150000 loan is taken out over a 25-year term. The interest rate is 9.6% p.a.
a Calculate the minimum monthly repayment.
b Calculate the total repayments on the loan.
c Calculate the length of time that it will take to repay the loan at $1625 per month.
d Calculate the total saving on the loan by repaying the loan at $1625 per month.
16 Link borrows $148000 taken out over 10 years. The loan is to be repaid in monthly instalments.

Calculate the amount of each monthly repayment at each of the following rates.
a 6% p.a.
b 7.2% p.a.
c 9% p.a.
d 12% p.a.
17 A loan of $200000 over 20 years has interest charged monthly at a rate of 9% p.a.
a Calculate the monthly repayment.
b After 3 years the balance owing is $187676.80. The interest rate then rises to 9.6%. What will be

the new repayment required to pay the loan off in the remaining 17 years?
18 Ben took out of loan for $20000 to buy a new car. The contract required that he repay the loan over

5 years with monthly instalments of $420.00. Use a trial-and-error method together with the PVIF
table to find the interest rate that Ben is paying. Give your answer as a percentage p.a. as close as the
table will allow.

2D

Loan repayment graphs

A reducing balance loan is structured in such as way that in the first month the majority of the
repayment is interest and only a small amount is reducing the principal. In each subsequent month the
amount of interest is slightly less and the amount coming off the balance slightly more.
Hence the graph of a reducing balance loan will show the bulk of the loan paid off in the final few years.
A graph that shows the outstanding balance on a loan will have time on the horizontal axis, the
balance on the vertical axis while the graph itself will be a smooth curve. The following exercise will
provide opportunities for you to examine loan repayment graphs and you will be able to see how the
balance declines over time and some of the strategies that can be used to pay them off more quickly.

Exercise 2D

Loan repayment graphs

1 The figure below tracks the balance owing on a loan of $250000. The interest on the loan is charged

at 9% p.a. reducible and the monthly repayment is $2098.


Balance owing

$250 000.00

Balance ($)

$200 000.00
$150 000.00
$100 000.00

0
12
24
36
48
60
72
84
96
108
120
132
144
156
168
180
192
204
216
228
240
252
264
276
288
300

$50 000.00

Number of months
a
b
c
d

What is the term of the loan?


Approximately how much is still owing on the loan after 5 years?
What is the outstanding balance on the loan half way through the term?
How long does it take for half the loan to be paid off?

2 The Andersons take out a $300000 home loan. The terms of the loan are that interest is charged at a

rate of 8.4% p.a., which is calculated and charged monthly with a minimum monthly repayment of
$2396. The Andersons decide to increase their repayment to $2600 per month.
The effect of this extra payment is shown in the following graph.
40 Maths Quest HSC Mathematics General 2

Repayment of $2396 per month


Repayment of $2600 per month

$300 000.00

Balance ($)

$250 000.00
$200 000.00
$150 000.00
$100 000.00

0
14
28
42
56
70
84
98
112
126
140
154
168
182
196
210
224
238
252
266
280
294

$50 000.00

Number of months
a Approximately how many months does it take to repay the loan:
i at $2396 per month
ii at $2600 per month?
b What is the approximate total cost of repaying the loan:
i at $2396 per month
ii at $2600 per month?
c What is the saving made by paying the loan off at the higher monthly repayment?
3 The Shorrocks take out a $425000 home loan. The loan is over a 25-year term at an interest rate of

7.2% p.a., with interest calculated monthly. The minimum monthly repayment is $3060.
a What is the total cost of repaying this loan?
b What is the balance owing after 1 year?
c After one year a one-off $10000 repayment is made on the loan.
The effect of this extra payment is shown by the graph below.
Minimum repayment
Minimum repayment with one off
$10 000 payment after 1 year.
$400 000.00
$350 000.00

Balance ($)

$300 000.00
$250 000.00
$200 000.00
$150 000.00
$100 000.00

0
14
28
42
56
70
84
98
112
126
140
154
168
182
196
210
224
238
252
266
280
294

$50 000.00

Number of months

By how many months is the term of the loan reduced?


d Approximately how much is saved by making the one-off repayment?
Chapter 2 Annuities and loan repayments 41

Further development

Digital doc
EXCEL Spreadsheet
doc-1301
Loan repayments

Making increased repayments, one-off extra payments or more frequent payments are methods that can
be used to repay a loan more quickly. The remaining questions require you to use the spreadsheet Loan
from eBookPLUS. Formulas in the spreadsheet work for the first 12 months of the loan. Use the Fill
Down function to complete these formulas for the length of each loan.
4 From the eBookPLUS open the spreadsheet Loan. The sheet titled Basic models a Loan. Use the
spreadsheet to draw graphs to model the following loans. In each case you will need to use the table
on page 37 to calculate what the minimum monthly repayment should be.
a A $20000 loan at 6% p.a. over 5 years
b A $500000 loan at 9% p.a. over 20 years
c A $1000000 loan at 8.4% p.a. over 25 years
5 From the eBook open the spreadsheet Loan. The sheet titled Higher payment allows you to
explore the effects of increasing the amount of the monthly repayment.
Consider a $275000 loan to be taken out at 9.6% p.a. over 20 years.
Draw a graph showing the path of the loan under the minimum monthly repayment and compare that
to paying an extra $400 per month.
6 From the eBook open the spreadsheet Loan. The sheet titled Extra payment allows you to explore
the effects of making a one-off extra payment.
Consider a $395000 loan to be taken out at 7.2% p.a. over 25 years.
Draw a graph showing the path of the loan under the minimum monthly repayment and compare that
to making a one-off $20000 repayment after 2 years. (In the balance owing extra payment column in
the cell for the 24th month include a subtraction of $20000 in the formula.)
Investigation: Types of loan arrangements

Digital doc
WorkSHEET 2.2
doc-11024

Research each of the following types of loans.


A. Hire purchase agreement
This is the type of loan where a major item such as a computer, wide screen television or household
appliance is purchased on terms. Usually a deposit is paid then the balance plus interest is repaid over
a period of time.
B. Personal loan
This is a loan taken from a bank or financial institution. The loan is unsecured which means the bank
has no item of property that can be claimed by the bank and sold to recoup the amount of the loan if
repayments are not made. This may mean that you need a guarantor in order to take out the loan.
C. Home loan
This is a secured loan. This means that if repayments are not made the bank can claim the property
and sell it to recoup the amount outstanding on the loan.
For each of the above loans answer the following questions.
1. What is the interest rate? Is the interest calculated as a flat rate or reducible rate of interest?
2. What is the term of the loan?
3. How regularly must repayments be made?
4. Can addition repayments be made to shorten the term of the loan?
5. Can the interest rate be altered after repayments have begun to be made?
6. What other fees and charges apply to borrowing the money?
7. Are there any early exit fees? That is, additional charges made if the loan is paid out before the
term of the loan has expired.

42 Maths Quest HSC Mathematics General 2

Summary
Future value of an
annuity

An annuity is where regular equal contributions are made to an


investment. The interest on each contribution compounds as additions are
made to the annuity.
The future value of an annuity is the value that the annuity will have at the
end of a fixed period of time.
A table that shows the present value of $1 in an annuity is the method used
to find the future value of an annuity.
The future value of $1 must be multiplied by the contribution per period to
calculate its present or future value.

Present value of an
annuity

The present value of an annuity is the single sum that would need to be
invested at the present time to give the same financial outcome at the end
of the term.
A table that shows the present value of $1 under an annuity is the method
used to find the present of an annuity.
The present of $1 must be multiplied by the contribution per period to
calculate its present or future value.

Loan repayments

The table of present values of an annuity can be used to calculate the


amount of each periodical repayment in a reducing balance loan.
This is done by considering the present value of an annuity as the amount
borrowed and solving the equation PVA = a PVIF, for a which is the
amount of each contribution or loan repayment.
The total amount to be repaid during a loan is calculated by multiplying
the amount of each monthly repayment by the number of repayments
to be made.

Loan graphs

A loan graph shows the declining balance of a loan over the term of the
loan.
Reading these graphs can demonstrate the effect that making extra
payments or increasing the amount of each payment has on how quickly
aloan can be paid off.

Chapter 2 Annuities and loan repayments 43

Chapter review
MULTIPL E
C HOICE

1 Jenny invests $1000 per year for 20 years in an annuity. The interest rate is 6% p.a. and interest is

compounded annually. The future value of the annuity is:


A $3678.50
B $16785.60
C $35785.60

D $36785.60

2 Madeline invests $1000 per year for 20 years in an annuity. The interest rate is 7% p.a. and interest is

compounded annually. The present value of the annuity is:


A $10594.00
B $20594.00
C $20995.20

D $40995.50

3 Which of the following investments has the greatest future value after 5 years?
A
B
C
D

An annuity of $500 per year at 8% p.a., with interest compounded annually


An annuity of $250 per six months at 8% p.a., with interest compounded six-monthly
An annuity of $125 per quarter at 8% p.a., with interest compounded quarterly
A single investment of $2000 at 8% p.a., with interest compounded annually

4 A loan of $80000 is taken out over a 20-year term at an interest rate of 9% p.a. The monthly repayment

is $719.78. What would the approximate total saving be if the term were reduced to 15years?
A $91.63
B $16493.40
C $21991.20
D $26693.40
Sh ort
a nswer

1 Calculate the amount to which each of the following investments will grow.
a $3500 at 12% p.a. for 3 years, with interest compounded annually
b $2000 at 8% p.a. for 5 years, with interest compounded six-monthly
c $15000 at 9.2% p.a. for 8 years, with interest compounded quarterly
d $4200 at 13.2% p.a. for 2 years, with interest compounded monthly
2 $400 per year is invested into an annuity at 7% p.a., with interest compounded annually. Use the

table on page 30 to calculate the value of the annuity after 20 years.


3 Use the table on page 37 to calculate the future value of each of the following annuities.
a $500 invested per year for 20 years at 12% p.a., with interest compounded annually
b $1000 invested every 6 months for 10 years at 10% p.a., with interest compounded six-monthly
c $600 invested every 3 months for 5 years at 8% p.a., with interest compounded quarterly
d $2500 invested per month for 1 years at 12% p.a., with interest compounded monthly
4 Jessica is saving for a car. She starts an annuity consisting of $500

deposits every month for 15months. The interest rate is 12% p.a. and
interest is compounded monthly. Find the future value of the annuity.
5 Use the compound interest formula FV = PV(1 + r) n and solve the

equation for PV to find the present value of an annuity with a future


value of:
a $10000 after 10 years at 5% p.a., with interest compounded annually
b $400000 after 40 years at 12% p.a., with interest compounded annually
c $5000 after 5 years at 9% p.a., with interest compounded six-monthly
d $120000 after 8 years at 15% p.a., with interest compounded quarterly.

6 Phuong wants to purchase a car in 3 years. He feels that he will need

$15000. The best investment he can find is at 8.5% p.a., interest


compounded quarterly. What is the present value of this investment?
7 Ruth invests $5000 per year in a retirement fund. The investment is at 9% p.a., with interest

compounded annually.
a What is the future value of the annuity in 15 years?
b What is the present value of this annuity?
8 Use the table future values of $1 to calculate the future value of an annuity of $4000 deposited per

year at 7% p.a. for 8 years, with interest compounded annually.


9 Use the table of future values of $1 to calculate the future value of the following annuities.
a $750 invested per year for 5 years at 8% p.a., with interest compounded annually
b $3500 invested every six months for 4 years at 12% p.a., with interest compounded six-monthly
c $200 invested every 3 months for 2 years at 16% p.a., with interest compounded quarterly
d $1250 invested every month for 3 years at 10% p.a., with interest compounded six-monthly.
44 Maths Quest HSC Mathematics General 2

10 Use the table present values of $1 to calculate the present value of an annuity of $500 invested per

year for 6 years at 9% p.a., with interest compounded annually.


11 Use the table of present values to calculate the present value of each of the following annuities.
a $400 invested per year for 5 years at 10% p.a., with interest compounded annually
b $2000 invested every six months for 5 years at 14% p.a., with interest compounded six-monthly
c $500 invested every three months for 2 years at 16% p.a., with interest compounded quarterly
d $300 invested every month for 4 years at 12% p.a., with interest compounded half-yearly.
12 A home loan of $210000 is to be repaid in equal monthly repayments over 25 years at an interest

rate of 8.4% p.a.


a Use the table on page 37 to calculate the amount of each monthly repayment.
b Calculate the total cost of repaying this loan.
13 The graph below shows a $400000 loan that is being paid off over 20 years. The interest rate is

7.2% p.a. and interest is calculated monthly.


$400 000.00

Balance owing

$350 000.00

Balance ($)

$300 000.00
$250 000.00
$200 000.00
$150 000.00
$100 000.00

0
11
22
33
44
55
66
77
88
99
110
121
132
143
154
165
176
187
198
209
220
231

$50 000.00

Number of months
a
b
c
d

Use the table on page 37 to calculate the amount of each monthly repayment.
Calculate the total cost of repaying the loan.
Use the graph to find an approximation for the balance still owing after 10 years.
Use the graph to approximate when the loan is half paid.

1 Lien invests $2000 per year in an annuity. The term of the annuity is 10 years and the interest rate is

8% p.a., with interest compounding annually.


a Calculate the future value of this annuity.
b Calculate the present value of this annuity.
c By how much will the future value of the annuity increase if Lien deposits $1000 per six months
and interest is compounded every six months?

Ex tended
R es p ons e

2 Eddie has the goal of saving $1000000 in the next 20 years. Eddie expects to be able to obtain an

average 8% p.a. in interest with interest compounded annually.


a Calculate the present value of this annuity.
b Use the table on page 37 to calculate the annual contribution to the annuity.
c For the first 10 years of the annuity Eddie makes no contributions, preferring to direct all his

money into paying off a mortgage. At that time he makes a single contribution to catch up on the
annuity. What amount must Eddie deposit?
3 Jim and Catherine take out a $350000 loan. The interest rate on the loan is 12% p.a. and the loan is

to be repaid in equal monthly repayments over a 20-year term.


a Use the table on page 37 to calculate the amount of each monthly repayment.
b Calculate the total amount of interest that Jim and Catherine will need pay on this loan.
c Calculate the saving that Jim and Catherine will make by repaying the loan over a 12-year term.

Digital doc
doc-11025
Test yourself
Chapter 2

Chapter 2 Annuities and loan repayments 45

ICT activities
2A Future value of an annuity

2d Loan repayment graphs

Interactivity
int-0193: Comparing investment options. (page 27)

Digital docs
Spreadsheet (doc-1301): Loan repayments. (page 42)
WorkSHEET 2.2 (doc-11024): Apply your knowledge of loan
repayments and graphs. (page 42)

2BPresent value of an annuity


elesson
eles-0117: Small business. (page 32)
Digital doc
WorkSHEET 2.1 (doc-11023): Apply your knowledge of future and
present values of annuities. (page 36)

2C Loan repayments

46 Maths Quest HSC Mathematics General 2

Chapter review
Test Yourself (doc-11025): Take the end of chapter practice test to test your
progress. (page 45)

To access eBookPLUS activities, log on to www.jacplus.com.au

Answers chapter 2
Future value of an annuity

1 $7049.37
2 a $6691.13
b $6859.14
c $6158.56
d $3974.56
e $17713.21
f $3530.21
3 $4472.93
4 a i$2217.44
ii
$2142.45
iii
$2070
iv
$2000
b $10429.89
5 $4787.76
6 a $1324.00
b $23932.35
c $7503.81
d $62953.50
7 a 4%
b 10
c $6003.05
8 a $4103.92
b $5335.38
c $7546.74
9 5% for 6 years. $1 will grow to $6.8019

but at 6% for 5 years it will grow to $5.6371.

10 D
11 a $192428.13
c $737277.43
12 a $289570.55
c $233356.13
13 a $1000
c 2%

b $4498.63

b $4211.82
b $4055.45

b $27180.60
b $6679.88
b $52395

Balance owing

$500 000.00
$450 000.00
$400 000.00
$350 000.00
$300 000.00
$250 000.00
$200 000.00
$150 000.00
$100 000.00
$50 000.00

0
11
22
33
44
55
66
77
88
99
110
121
132
143
154
165
176
187
198
209
220
231
240

b $23520.09

Loan repayments

1 $302.71
2 $213.46
3 $210.67
4 a $94.15
c $859.72
e $1511.14
5 $2195.87
6 a $375
c $107.32
7 $332.01
8 a $28000
9 $40056.46
10 a $4359.36
c $98285.02
11 A
12 B
13 6 years
14 a $2586.38

Balance owing

$20 000.00
$18 000.00
$16 000.00
$14 000.00
$12 000.00
$10 000.00
$8 000.00
$6 000.00
$4 000.00
$2 000.00

Number of months

c $7984.99
b $311.38
d $484.17

b $3375

b $581.23
b $23864.96
d $366700.35

$1 000 000.00
$900 000.00
$800 000.00
$700 000.00
$600 000.00
$500 000.00
$400 000.00
$300 000.00
$200 000.00
$100 000.00

Balance owing

0
14
28
42
56
70
84
98
112
126
140
154
168
182
196
210
224
238
252
266
280
294
300

Exercise 2C

b $12289.20
d $16604.40

b $419000
d $45080

0 3 6 9 12 15 18 21 24 27 30 33 36 39 42 45 48 51 54 57 60
Number of months

Present value of an annuity

1 $6918.50
2 a $1845.09
c $4455.79
3 $4997
4 a $2388.52
c $1125.51
5 a $3992.70
c $4087.85
6 $57349.50
7 a $13420.20
c 1.27%
8 a $307598.63
9 a $34.93
c $17955
10 a $34916.58

b $396297
d $123294
b $1733.70
d 2123.37
b $1869.32

Loan repayment graphs


b $235000
d 220 months
ii 237 months
ii $616200

1 a 25 years
c $190000
2 a i 300 months
b i $718800
c $102600
3 a $918000
c 18 months
4 a $386.66

Balance ($)

Exercise 2B

b $331070.40
d $213778.90
b $717465.20
d $122550.04
b $1020

Exercise 2D

Balance ($)

Exercise 2A

15 a $1320.98
c 14 years
16 a $1643.10
c $1874.80
17 a 1799.45
18 9.6% p.a.

Balance ($)

Annuities and loan repayments

b 12 years

Number of months

Chapter 2 Annuities and loan repayments 47

2 $16398.20

3 a $36026.20
$250 000.00

b $33066.00
c $14578.44

Balance ($)

$200 000.00

d $49036.75
4 $8048.45

$150 000.00

5 a $6139.13
b $4298.72

$100 000.00

c $3219.64
d $36945.53

$50 000.00

6 $11654.84

0
11
22
33
44
55
66
77
88
99
110
121
132
143
154
165
176
187
198
209
220
231
240

7 a $146804.50
Number of months

8 $41039.20
9 a 4399.95

$400 000.00

b $34641.25

$350 000.00

c $1842.84
d $51014.25

$300 000.00

10 $2242.95

$250 000.00

11 a $1516.32

$200 000.00

b $14047.20
c $4055.45

$150 000.00

d $11177.64

$100 000.00

12 a $1676.85

$50 000.00

b $503055
13 a $3149.40
1
15
29
43
57
71
85
99
113
127
141
155
169
183
197
211
225
239
253
267
281
295
300

Balance ($)

b $40303.42

Number of months

Chapter Review
Multiple Choice

1 D
2 A
3 C
4 D
Short answer

1 a $4917.25
b $2960.49
c $31053.57
d $5461.06

48 Maths Quest HSC Mathematics General 2

b $755856
c $270000
d 160 months
Extended Response

1 a $28973.20
b $13420.20
c $804.90
2 a $214548.21
b $21852.19
c $316563.94
3 a $3853.80
b $574912
c $262949.76

Chapter 3

Interpreting sets of data


CHAPTER CONTENTS
3A Grouped data
3B Measures of location and spread
3C Analysis of data sets
3D Comparison of data sets
3E Skewness

3A

Grouped data

Organising data
Previously you will have studied frequency tables, frequency histograms and frequency polygons, also
called ogives.
A frequency table is used to organise large data sets in a way that the nature of the distribution can be
seen easily and summary data can be easily calculated.

WORKED EXAMPLE 1

The scores below represent the number of wickets that Mitchell has taken in his last 20 cricket
matches.
2

a Put the data in a frequency table.


b Show the data in a bar graph.
THINK

a 1 Draw up a frequency table with five

columns headed, Score, Tally, and


Frequency.
2

In the frequency column list all


scores from lowest to highest.

Read the scores adding a tally mark


against each score as it is read.

Count the tally marks to calculate


each frequency.

WRITE

Score
Frequency
(x) Tally
(f)
0
5
||||
1
5
||||
2
||||
4
3
|||
3
4
|
1
5
||
2

Chapter 3 Interpreting sets of data 49

b 1 Draw axes with scores on the

horizontal axis and frequencies


on the vertical axis.

Draw 1-cm columns with gaps


to the height of each frequency.

Frequency

6
4
3
2
1
0

Score

The data in Worked example 1 is called ungrouped data. This is because the data is not organised into
groups but rather each score is counted individually. In most cases this is not practical and the data is
organised into groups of scores. This is referred to as grouped data.
The data should be grouped in such a way as to create typically between 5 and 10 classes or
groups. On the horizontal axis of the frequency table an extra value needs to be added for the class
centre. The middle of each grouping, the class centre, is found by averaging the first and last number in
each group.
When drawing the histogram and cumulative frequency histogram the class centres are graphed on
the horizontal axis.
WORKED EXAMPLE 2

A sample of 40 people was surveyed regarding the number of hours per week spent watching
television. The results are listed below.
12
7
23
16

18
16
19
20

9
26
29
17

17
15
12
10

20
7
19
24

7
28
12
21

24
11
16
5

16
20
21
13

9
9
8
29

27
11
6
25

a
Organise the data into 5 class intervals and use these to create a frequency distribution table

that displays the class intervals, class centres and frequencies.

b Construct a combined histogram and frequency polygon to represent the tabulated data.
THINK

a 1 Determine the size of the class intervals.

WRITE

a Class interval of 5 hours will create 5 groups.

Note: Intervals of 5 hours would create


5groups, the required number.
Class intervals have been recorded as
5<10, 10<15; and so on, since the data
are continuous.
2

Draw a table with three columns, headed


Hours of television watched (class
interval), Class centre (midpoint of a
class interval) and Frequency. Note: The
midpoint of a class interval is calculated
by taking the average of the two extreme
values of that class interval. For example
the midpoint of the 5<10 class
interval is 5 + 10 = 7.5
2

50 Maths Quest HSC Mathematics General 2

Hours of
television
watched
5<10

Class centre
7.5

Frequency
9

10<15

12.5

15<20

17.5

10

20<25

22.5

25<30

27.5

6
f = 40

Systematically go through the list,


determine how many times each score
occurs and enter the information into the
frequency column.

Calculate the total of the frequency column.

b 1 Draw a set of axes on graph paper.

Frequency polygon of hours


of television watched

Add a title to the graph.


Label the horizontal axis Number of hours
of television watched and the vertical axis
Frequency.

Leaving a unit or interval space, draw in


the first column so that it starts and finishes
halfway between class intervals and
reaches a vertical height of 9 people.

8
Frequency

10

6
4

Draw in the columns for each of the other


scores.

Mark the midpoints of the tops of the


columns obtained in the histogram.

Join the midpoints by straight line


intervals.

Close the polygon by drawing lines at each


end down to the class interval (x-) axis.

7.5

12.5 17.5 22.5 27.5


Number of hours of television
watched

The ogive
From an ogive we are able to make estimates of important summary statistics such as the median, the
quartile data and the deciles.
WORKED EXAMPLE 3

The following frequency distribution table


gives the number of customers who order
different volumes of concrete from a
Readymix concrete company during
the course of a day. Find the interquartile
range of the data.

THINK
1

An ogive can be used to estimate the


25th and 75th percentiles. First add a
class centre column and a cumulative
frequency column to the frequency
distribution table and fill them in.

Volume (m3)
0.0<0.5
0.5<1.0
1.0<1.5
1.5<2.0
2.0<2.5
2.5<3.0

Frequency
15
12
10
8
2
3

WRITE

Volume
0.0<0.5

Class centre
0.25

f
15

cf
15

0.5<1.0

0.75

12

27

1.0<1.5

1.25

10

37

1.5<2.0

1.75

45

2.0<2.5

2.25

47

2.5<3.0

2.75

50

Chapter 3 Interpreting sets of data 51

Draw the ogive. A percentage axis


will be useful.

100%
90%
80%
70%
60%
50%
40%
30%
20%
10%
0%

Cumulative frequency

50
45
40
35
30
25
20
15
10
5
0

0.25

0.75

1.25

1.75

2.25

2.75

Volume

QU 1.6QL 0.4

Find the upper quartile (75th


percentile) and lower quartile (25th
percentile) from the ogive.

The interquartile range is the difference IQR 1.6 0.4


between the upper and lower quartiles.
1.2

Exercise 3A

Grouped data

1 WE2 In a survey, 40 people were asked about the number of hours a week they spent watching

television. The results are listed below.


10, 13, 7, 12, 16, 11, 6, 14, 6, 11, 5, 14, 12, 8, 27, 17, 13, 8, 14, 10,
13, 7, 15, 10, 16, 8, 18, 14, 21, 28, 9, 12, 11, 13, 9, 13, 29, 5, 24, 11
a Organise the data into 5 class intervals and use these to create a frequency distribution table that
displays the class intervals, class centres and frequencies.
b Construct a combined histogram and frequency polygon to suit the table.
2 Copy and complete this grouped frequency distribution table.
Class interval
04
59

Class centre
2
7
12

1519
2024

Tally
|||| |
|||

Frequency

12
||

22

9
32

3 The number of phone calls made on average per week in a sample of 56 people are listed below.

21,
56,
52,
12,

50,
17,
17,
72,

8,
59,
54,
36,

64,
23,
78,
66,

33,
34,
69,
15,

58,
57,
53,
28,

35,
49,
2,
67,

61,
2,
42,
13,

3,
24,
52,
23,

51,
50,
28,
10,

5,
27,
67,
72,

62,
33,
25,
72,

16,
55,
48,
89,

44,
7,
63,
80

a Organise the data into a grouped frequency distribution table using a suitable class interval.
b Display the data as a combined histogram and frequency polygon.
4 WE2 The height, in centimetres, of 30 students in Year 12 was recorded as follows.

146, 163, 156, 168, 159, 170, 152, 174, 156, 163, 157, 161, 178, 151, 148,
167, 162, 157, 166, 154, 150, 166, 160, 155, 164, 157, 171, 168, 158, 162
a Organise the data into 7 classes, starting at 145 cm and draw up a frequency distribution table.
b Display the data as a histogram.
c How many students stood at least 160 cm tall?
d Reorganise the data into class intervals of 4 cm; that is, 145<148 cm.
e Draw a new histogram and compare it to the previous one. Discuss any advantages or
disadvantages of having a smaller class interval.
52 Maths Quest HSC Mathematics General 2

5 Construct the following:


a a frequency distribution table using intervals 1215, 1619, etc.
b a combined histogram and frequency polygon for the following data.

Class sizes at Mathsville High


38

24

20

23

27

27

22

17

30

26

25

16

29

26

15

26

19

22

13

25

21

19

23

18

30

20

23

16

24

18

12

26

22

25

14

21

25

21

31

25

6 The following data give the length (in cm) of the forearm of 30 middleweight boxers.

24.0

26.5

31.4

30.8

25.7

26.9

27.2

29.9

25.2

26.3

28.4

26.3

29.2

30.5

26.0

27.2

26.0

24.9

25.8

24.7

28.3

26.3

25.7

26.9

27.8

25.7

25.0

29.3

25.8

28.0

a Construct a frequency distribution table for the data. Use a class size of 1 cm.
b Draw a histogram of the data.
7 The following data give the times (in seconds) taken for athletes to complete a 100-m sprint.

12.2

12.0

11.9

12.0

12.6

11.7

11.4

11.0

10.9

11.7

11.2

11.8

12.2

12.0

12.7

12.9

11.3

11.2

12.8

12.4

11.7

10.8

13.3

11.7

11.6

11.7

12.2

12.7

13.0

12.2

a Construct a frequency distribution table for the data. Use a class size of 0.5 seconds.
b Draw a histogram of the data.
c Add a polygon to the data.
8 The data below show the fat content (%) of 20 packs of bacon selected from a supermarkets shelves.

13

15

18

13

25

21

13

18

20

16

15

18

12

a Construct a frequency distribution table for the data using a class size of 4.
b Construct a frequency distribution table for the data using a class size of 10.
c Comment upon how changing the size of the group affects the appearance of the histogram.
9 Use the ogive shown to determine the interquartile range of the data.

Cumulative frequency

40
30

50%

20
10
100

120 140 160


Height (cm)

Cumulative frequency (%)

100%

50

180

10 WE3 The following frequency distribution table gives the amount of time spent by 50 people on

shopping for Christmas presents. Find the IQR of the data.


Time (h)
Frequency

0<0.5

0.5<1

1<1.5

1.5<2

2<2.5

2.5<3

3<3.5

3.5<4

15

13

Chapter 3 Interpreting sets of data 53

Further development
11 The following frequency distribution table shows the life expectancy of 40 household batteries.

Life (hours)
Frequency

50<55
4

55<60
10

60<65
12

65<70
8

70<75
5

75<80
1

a Draw an ogive which represents the data in the table above.


b Use the ogive to answer the following questions.
i What is the median score?
ii What are the upper and lower quartiles?
iii What is the interquartile range?
iv How many batteries lasted less than 60 hours?
v How many batteries lasted 70 hours or more?
12 Calculate the IQR for the following data.

3B
interactivities
int-0084
Measures of centre
int-2352
Measures of centre
int-2362
Measures of centre

Class interval

Frequency

120<130
130<140
140<150
150<160
160<170
170<180
180<190
190<200

2
3
9
14
10
8
6
3

Measures of location and spread

Consider the following set of scores that are the exam results for 10 students.
55, 57, 57, 58, 60, 60, 62, 63, 63, 65
To identify a score that is typical in this data set, we can use the mean or median.
The mean is calculated by adding all the scores and dividing by the number of scores in the set.
When the data is a small set of scores the mean is found using the formula
x=

ax

n
where x = mean
x = individual scores (Therefore, x represents the sum of individual scores.)
n = number of scores.
Where the data is presented in a frequency table we use the formula
x=

a fx
af

where x = mean
x = individual scores
f = frequency.
In this formula fx represents the sum of the frequency score column on the frequency table and f
represents the sum of frequency column.
The median is the middle score (odd number of scores) or the average of the two middle scores (even
number of scores).
For this set of scores:
Mean = 600 10
= 60
Median = 60
Both the mean and median are a measure of location within a data set.
54 Maths Quest HSC Mathematics General 2

WORKED EXAMPLE 4

For the set of scores 13, 19, 31, 40, 55, 65, 90, 92, 95, 100 calculate:
a the mean
b the median.
THINK

WRITE

a 1 Find the total of the scores.


2

Divide the total by the number of scores.

b Average the two middle scores.

a Total = 600

Mean = 600 10
= 60
b Median = (55 + 65) 2

= 60

We have now examined two data sets. Look at these data sets side by side.
Set A: 55, 57, 57, 58, 60, 60, 62, 63, 63, 65
Set B: 13, 19, 31, 40, 55, 65, 90, 92, 95, 100
Although both sets of scores have the same mean and median, they are very different sets of scores.
Clearly, in Set B the scores are more spread out than in Set A. To measure the spread of a set of scores,
we use one or all of the following.
Range: Highest score lowest score
Set A: Range = 65 55
= 10

Set B = 100 13
= 87

Interquartile range (IQR): The difference between the upper quartile and lower quartile.
Set A: Interquartile range = 63 57
Set B: Interquartile range = 92 31
= 6 = 61
Standard deviation: Found using the n (population) or sn (sample) functions on the calculator.
Set A: n = 3.07

Set B: n = 31.51

Each of these measures of spread show that in Set B the scores are more scattered than in Set A.
WORKED EXAMPLE 5

For the set of scores 45, 62, 75, 69, 50, 87, 92 calculate:
a the range
b the interquartile range
c the standard deviation.
THINK

WRITE

a Subtract the lowest score from the highest score.

a Range = 92 45

b 1 Write the scores in ascending order.

b 45, 50, 62, 69, 75, 87, 92

Tutorial
int-2417
Worked example 5

= 47

Divide the data in two halves, leaving the


middle score out of both sets.

45, 50, 62, 75, 87, 92

The lower quartile is the median of the lower


half; the upper quartile is the median of the
upper half.

Lower quartile = 50 Upper quartile = 87

Subtract the lower quartile from the upper


quartile.

Interquartile range = 87 50
= 37

Enter the set of scores into your calculator


using the statistics function.

c n = 16.36

Chapter 3 Interpreting sets of data 55

WORKED EXAMPLE 6

Nadia is a gymnast. For a routine she is given the following scores by 10 judges.
9.08.79.29.39.89.28.89.49.09.1
Use your graphics calculator to find
a the mean
b the median
c the mode
d the range
e the interquartile range
f the population standard deviation
g the sample standard deviation.
THINK
1

From the MENU select STAT.

Delete any existing data, and enter the scores


above in List 1.

Press 2 (CALC). You may need to first


press 6 for more options.

Press 6 (SET). Check that 1Var Xlist is


set to List 1 and 1Var Freq is set to 1.

Press w to return to the previous screen,


and then press 1 (1Var). All statistics will
now be on display using the scroll function.

WRITE

Mean
Population
standard deviation
Sample
standard deviation
Number of scores
Lowest score
Lower quartile
Median

Upper quartile
Highest score
Mode
a The mean is denoted by the symbol x.

a x = 9.15

b The median is denoted by Med.

b Median = 9.15

56 Maths Quest HSC Mathematics General 2

c Mode = 9.0 and 9.2

c The mode is displayed by Mod. Check the

scores for yourself as the data is bimodal only


the largest mode is displayed.

d Range = 9.8 8.7

d The range is the highest score (maxX) minus

= 1.1

the lowest score (minX).

e Interquartile range = 9.3 9

e The interquartile range is the upper quartile

= 0.3

(Q3) minus the lower quartile (Q1).


f The population standard deviation is denoted by

f n 0.297

g The sample standard deviation is denoted by

g n 1 0.314

xn.

xn1.

Exercise 3B

Measures of location and spread

1 WE4 The number of goals scored by a team in 10 games of soccer are:

2, 1, 3, 1, 0, 0, 1, 1, 6, 1.
a Calculate the mean number of goals scored.
b Calculate the median number of goals scored.
2 For each of the following sets of scores, calculate the mean, median and mode (if one exists).
a 56, 75, 88, 20, 37, 23, 44
b 2, 1, 7, 4, 6, 1, 1, 4, 5, 3
c 9.9, 9.4, 9.8, 9.6, 9.0, 9.2, 9.8, 9.9
Score
Frequency
d 13, 15, 16, 17, 10, 13, 15, 14, 19, 20
3 The table on the right shows the scores out of 10 by a class of

30students on a spelling test.


a Use the statistics function on your calculator to find the mean
score.
b Add a cumulative frequency column to the table and use it to
calculate the median score.
c State the mode.

4
5
6
7
8
9
10

2
6
7
9
3
2
1

Digital doc
SkillSHEET 3.1
doc-11026
Finding the mean
SkillSHEET 3.2
doc-11027
Finding the mode

Digital doc
SkillSHEET 3.3
doc-11028
Finding the median

4 The table below shows the scores achieved by a football team

over a season.
Score
15
610
1115
1620
2125
2630

Class centre

Frequency

Digital docs
EXCEL Spreadsheet
doc-1333
One variable
statistics
GC program
Casio
doc-1334
UV Stats
GC program TI
doc-1335
UV Stats

Cumulative
frequency

2
4
8
7
3
1

a Copy and complete the table.


b Calculate the mean.
c Draw a cumulative frequency histogram and curve and use them to estimate the median.
5 WE5 Below is the number of students in each class at a small primary school.

28, 29, 27, 28, 30, 28, 25, 27, 23, 28, 27, 28
a Calculate the range of the distribution.
b Calculate the interquartile range.
c Use the statistics function on your calculator to find the mean and standard deviation.

Digital doc
SkillSHEET 3.4
doc-11137
Finding the range

Chapter 3 Interpreting sets of data 57

6 The following table shows the number of cars sold in a car yard each
Digital docs
SkillSHEET 3.5
doc-11029
Finding the
interquartile range
SkillSHEET 3.6
doc-11030
Choosing the
appropriate
standard deviation
EXCEL Spreadsheet
doc-1339
Boxplots

week over one year.


a Calculate the range of the number of cars sold.
b Add a cumulative frequency column to the table and use the table
to calculate:
i the median
ii the upper and lower quartiles
iii the interquartile range.
c Use the statistics function on the calculator to find:
i the mean
ii the standard deviation.
d Draw a box-and-whisker plot of the data.

No. of cars

Frequency

6
7
8
9
10
11
12

3
5
9
15
11
8
1

7 The table below shows crowds at each match for a team during football season.

Crowd

Class centre

1000015000
1500020000
2000025000
2500030000
3000035000
a
b
c
d

Frequency

Cumulative
frequency

5
8
6
4
3

Copy and complete the table.


Draw a cumulative frequency histogram and curve.
Use the graph in part b to estimate the interquartile range.
Find the mean and standard deviation. (Give your answer correct to 2 significant figures.)

8 WE6 Below are the scores of two rugby league teams over a period of 10 matches.

Team A: 14, 16, 16, 20, 10, 12, 18, 16, 18, 20
Team B: 28, 12, 32, 2, 0, 8, 40, 10, 12, 16
a For each team calculate the mean score.
b For each team calculate:
i the range
ii the interquartile range
iii the standard deviation.
c Comment on the difference between the performance of the two teams over this 10-game period.
The information below is to be used for questions 9 to 12.
A basketball squad has eight players. The mean height of the eight players is 1.8 m, and the standard
deviation in the heights of the players is 0.1 m. In the first game the tallest player, who is 1.9 m tall, is
injured and replaced in the squad by a player who is 1.98 m tall.
9 MC The mean height of the basketball squad will now be:
A 1.8 m
B 1.81 m
C 1.86 m
10 MC As a result of the substitution:
A
B
C
D

the standard deviation will increase


the standard deviation will decrease
the standard deviation will be unchanged
the effect on the standard deviation cannot be calculated

11 MC As a result of the substitution:


A
B
C
D

the range will increase


the range will decrease
the range will be unchanged
the effect on the range cannot be calculated

12 MC As a result of the substitution:


A
B
C
D

the interquartile range will increase


the interquartile range will decrease
the interquartile range will be unchanged
the effect on the interquartile range cannot be calculated

58 Maths Quest HSC Mathematics General 2

D 1.96 m

Further development
13 a On a particular day, the number of cars that stopped at the drive-in area at a McFast restaurant

during each hour (from 8.00 am until 11.00 pm) is shown below.
15 10 4 13 12 28 25 15 19 29 24 22 25 30 15
Find the interquartile range of this set of data.
b At the nearby Kirbys Fried Chicken restaurant on the same day, the number of cars stopping
during each hour that the restaurant was open is shown below.
6 10 15 22 18 11 13 14 21 17 25 20 19 10 17
Find the interquartile range of these data.
c What do these values suggest about the two restaurants?
14 Consider the following two groups of people.
a Calculate the mean height,
Group A
median height and mode height
160 170 170 170 170 170 180

e
f

for each group. What do you


notice?
Are the groups really the same?
Which group would you expect
to show the greatest range in
heights?
Which group would you expect
to show the greatest interquartile
range in heights?
Which group would you expect to show the greatest standard deviation in heights?
Calculate these statistics to confirm your predictions.
Height (cm)

b
c

Group B
160 170 170 110 230 170 180

15 aWorkers are arguing for a pay rise but the management of the factory claims that workers are well

paid because the mean salary of the factory is $23100. Are they being honest?
Position
Machine operator
Machine mechanic
Floor steward
Manager
Chief executive officer

Salary ($)

Number of employees

19000
21000
25000
63000
81000

50
15
10
4
1

b Suppose that you were representing the factory workers and had to write a short submission

in support of the pay rise. How could you explain the managements claim? Quote some other
statistics in favour of your case.

3C

Analysis of data sets

In the previous section we studied the mean, median and mode as well as the range, interquartile range
and standard deviation of a data set. You need to be able to use this information to compare data sets as
well as to examine the effect that individual scores have on them.
Consider the case of a basketball team. There are five players on the team, and their heights are: 2.01 m,
2.05 m, 1.52 m, 2.03 m and 2.19 m.
The teams mean height is 1.96 m. Only one of the five players in the team is shorter than the mean
height. This is because there is one member of the data set whose height is much less than the others.
Ascore in a data set that is either much smaller or much greater than all others is called an outlier.
WORKED EXAMPLE 7

In a small street there are five houses. The values of these houses are:
$450000, $465000, $465000, $480000, $495000.
A new house is built and valued at $750000. Describe the effect that this outlier has on the:
a mean
b median
c mode.
Chapter 3 Interpreting sets of data 59

WRITE

THINK

a 1 Calculate the mean before the new house

a Before new house is built:

is built.

Total = $2355000
Mean = $2355000 5
= $471000

Calculate the mean after the new house is


built.

After new house is built:


Total = $3105000
Mean = $3105000 6
= $517500

Comment on the change in the mean


caused by the outlier.

The outlier has caused the mean to increase by


$46500. Only the new house is valued at more
than the mean and, as such, has made the mean
a poor measure of the typical price.

b 1 Calculate the median before the new

b Before new house is built:

house is built.

Median = $465000

Calculate the median after the new house


is built.

After new house is built:


Median = ($465000 + $480000) 2
= $472500

Comment on the change in the median


caused by the outlier.

The outlier has caused only a small increase in


the median and, as such, the median remains a
good measure of the typical score in this data set.

c 1 Calculate the mode before the new house

is built.

Before new house is built:


Mode = $465000

Calculate the mode after the new house


is built.

After new house is built:


Mode = $465000

Comment on the change in the mode


caused by the outlier.

The outlier has had no effect on the mode.

In the case of the five basketball players the outlier was fairly obvious. The player who was 1.52 m tall
was much shorter than the other four players. Worked example 7 was similar in that when the $750000
house is added to the data set it is much greater than all others. It is difficult to exactly define an outlier
for such a small data set, however.
The question has to be asked, however, how much above or below all the other scores does a single
score need to be to be defined as an outlier.
The definition of an outlier is:
For a score lower than all others the score must be lower than QL 1.5 IQR
For a score higher than all others the score must be greater than QU + 1.5 IQR
Consider the following data.
There are 10 horses in a race.

60 Maths Quest HSC Mathematics General 2

Shannon checks the form guide and notes the number of races each horse has previously won
0, 5, 5, 6, 7, 8, 9, 9, 11, 22.
There are two scores in the data set, 0 which is significantly below all others and 22 which is much
greater than all others. The question is, are they actually outliers?
For the data set QL = 5, QU = 9 and IQR = 4
The lower limit for an outlier will be QL 1.5 IQR
= 5 1.5 4
= 1
and so 0 is technically not an outlier.

The upper limit for an outlier will be QU + 1.5 IQR


= 9 + 1.5 4
= 15
and so 22 is an outlier.
It is important that we are able to analyse the impact that an outlier has on a data set. Consider this
data set with the outlier (22) included and then without.

Mean
Median
Mode
Range
Interquartile Range
Standard deviation

Data set including outlier


0, 5, 5, 6, 7, 8, 9, 9, 11, 22
8.2
7.5
5, 9
22
4
5.4

Data set not including outlier


0, 5, 5, 6, 7, 8, 9, 9, 11
6.7
7
5, 9
11
4
3.0

When considering the measures of location, removing the outlier had the greatest effect on the mean,
a small impact on the median and no impact on the modes.
With measures of spread, removing the outlier had a huge effect on the range, a significant effect on
the standard deviation and no impact on the interquartile range.
These are the effects that will generally occur in most cases although the addition or removal of an
outlier may have a small impact on the interquartile range in some cases.
WORKED EXAMPLE 8

The data below shows the marks achieved by a group of 10 students in an exam.
21, 45, 46, 48, 48, 52, 54, 59, 61, 75
a Identify any outliers in the data set.
b
Explain the effect that the removal of the outlier will have on the mean and standard deviation
without actually calculating their values.
THINK

a 1 Find the lower quartile, upper quartile and

WRITE

a 21, 45, 46, 48, 48, 52, 54, 59, 61, 75

interquartile range.

QL = 46 QU = 59
IQR = 13

Find the lower limit for any outlier.

Lower limit = QL 1.5 IQR


= 46 1.5 13
= 26.5

Find the upper limit for any outlier.

Upper limit = QU + 1.5 IQR


= 59 + 1.5 13
= 78.5

Identify any scores that are outside these


limits.

21 is the outlier in the data set.

b 1 Consider the effect of the outlier on the


2

b The outlier is below all other scores. Removing

mean.

this score will therefore increase the mean.

Consider the effect of the outlier on the


standard deviation.

The outlier is the further score from the mean


than any other score and so removing that score
will reduce the standard deviation.

Chapter 3 Interpreting sets of data 61

In this course the display of information is an important skill. In particular we need to be able to display
two data sets in a way that allows for comparisons to be made.
interactivity
int-2773
Back-to-back
stem-and-leaf
plots

Stem-and-leaf plots
Two sets of data can be displayed on the same stem-and-leaf plot. This is done by having the stem in
the centre of the plot, with both sets of data back to back.
WORKED EXAMPLE 9

The data shown below display the marks of 15 students in both English and Maths.
English: 45 67 81 59 66 61 78 71 74
Maths:
85 71 49 66 64 68 75 71 69
Display the data in a back-to-back stem-and-leaf plot.
THINK

91
60

60
63

49
80

58
87

62
54

70
59

WRITE

Key: 4 | 5 = 45

Write a key at the top of the stem-and-leaf plot.

Draw the stem showing categories of 10 in


the centre of the page.

English
95

Display the information for English on the


left of the stem.

98

49

76210

034689

Display the information for Maths on the


right of the stem.

8410

115

057

Maths

This stem-and-leaf plot allows for both distributions to be easily seen, and for a judgement on the
skewness of the distribution to be made.

Box-and-whisker plots
interactivity
int-0802
Box plot and
five-number
summary

From a box-and-whisker plot we are able to determine where certain percentages of the population lie.
There are 5 critical values in a box-and-whisker plot, they are:
The lowest score
The lower quartile
The median
The upper quartile
The highest score.
Between each of these critical points lies 25% of the population.
25%

25%
25%

Lowest score

Q1

25%
Median

Q3

Highest score

1.1 m
1.2 m

0.7 m

0.1 m

The box-and-whisker plot below shows the heights of a sample of plants in a nursery.
Find the percentage of plants that are
between:
a 1.1 m and 1.2 m
b 0.1 m and 1.1 m
c 0.7 m and 1.5 m.
Heights of plants (m)

62 Maths Quest HSC Mathematics General 2

1.5 m

WORKED EXAMPLE 10

THINK

a The median is 1.1 and the upper quartile is 1.2

WRITE

a 25% of plants are between 1.1 m and 1.2 m

therefore 25% of plants are in this height range.


b 0.1 is the lowest score and 1.1 is the median and so

in height.
b 50% of plants lie between 0.1 m and 1.1 m

50% of plants lie in this height range


c

0.7 m is the lower quartile and 1.5 m is the highest


score so 75% of plants lie in this height range.

in height.
75% of plants lie between 0.7 m and 1.5 m
in height.

A single axis can be used to draw two box-and-whisker plots that will allow the comparison of the
median, range and interquartile range of two distributions to be compared.
interactivity
int-2788
Parallel box plots

WORKED EXAMPLE 11

Use the back-to-back stem-and-leaf plot drawn in Worked example 9 to:


a calculate the median of each distribution
b calculate the range of each distribution
c calculate the interquartile range of each distribution
d draw a box-and-whisker plot of each distribution on the same scale.
THINK

WRITE

Method 1: Technology-free
a The median will be the eighth score in each

distribution.
b To calculate the range of each distribution, subtract

the lowest score from the highest score.

c 1 The lower quartile will be the fourth score.

a English median = 66

Maths median = 68

b English range = 91 45

= 46
Maths range = 87 49
= 38

c English lower quartile = 59

Maths lower quartile = 60

The upper quartile will be the twelfth score.

English upper quartile = 74


Maths upper quartile = 75

The interquartile range is the difference between


the quartiles.

English interquartile range = 74 59


= 15
Maths interquartile range = 75 60
= 15

d 1 Draw a scale.
2

Draw the English box-and-whisker plot.

Draw the Maths box-and-whisker plot.

English
Maths
0 10 20 30 40 50 60 70 80 90 100
Marks

Method 2: Technology-enabled
1

From the MENU select STAT.

Enter the data for English in List 1 and the data


for Maths in List 2.

Chapter 3 Interpreting sets of data 63

Press 1 (GRPH), and then 6 (SET). We


will set the English data as GPH1 and the Maths
data as GPH2. Press 1 (GPH1) and enter the
settings shown in the screen at right.

Press w once these settings have been entered.


Press 6 (SET) and 2 (GPH2), and again
enter the settings shown at right.

To show both graphs on the same screen press


w after entering the settings above, press
4 (SEL), and set both StatGraph1 and
StatGraph2 to DrawOn as shown.

Press 6 (DRAW) to draw both graphs.

Press SHIFT 1 (TRACE), and use the arrow


keys to move around to the five key points on
each graph. The screen at right displays the
median for the Maths data.

Examining exam results


Collect data on the most recent exam that has been done in your class.
1. Display the data in a stem-and-leaf plot.
2. Find all the information needed to display the data in a box-and-whisker plot.
3. Is there any skewness evident in the data?
4. Which measure of location best describes the typical score in this data set?

Radar and area charts


WORKED EXAMPLE 12

The table below shows the number of admissions to two hospitals, each month, over a one-year
period. Display both sets of data on a radar chart.
Month
Hospital A
Hospital B
January
3
15
February
6
12
March
7
9
April
9
10
May
10
8
June
15
7
July
14
9
August
16
6
September
10
8
October
5
5
November
3
9
December
7
2

64 Maths Quest HSC Mathematics General 2

THINK
1

WRITE

Draw the radar with a 30 angle between the


months.

Draw a scale around the radar.

Plot each set of points.

Hospital A

Jan
Dec 20
15
Nov
10
5
Oct
0

Hospital B

Feb
March
April

Sep

May
Aug

July

June

Area charts are another method of comparing information. In an area chart, line graphs are stacked on
top of each other, thus allowing the area between each line graph to serve as the comparison between the
data sets.
WORKED EXAMPLE 13

The table below shows the amount of rainfall, in millimetres, in Sydney, Melbourne and
Brisbane each month throughout a year.
January

February

March

April

May

June

Sydney

103

117.1

133.7

126.6

120.4

131.7

Melbourne

49

47.7

51.8

58.4

57.2

50.2

159.6

158.3

140.7

92.5

73.7

67.8

July

August

September

October

November

December

Sydney

98.2

79.8

69.9

77.5

83.1

79.6

Melbourne

48.7

50.6

59.4

67.7

60.2

59.9

Brisbane

56.5

45.9

45.7

75.4

97

133.3

Brisbane

Tutorial
int-2419
Worked
example13

Show this information in an area chart.


WRITE

Draw a pair of axes. The vertical axis will need


to be at least the rainfall total of all three cities in
the wettest month.

Draw a line graph of Sydneys rainfall, and shade


the area below it.

Add Melbournes rainfall to Sydneys rainfall,


and draw a line graph showing these figures.
Colour the area between the two graphs, as this
area represents Melbournes rainfall.

Add Brisbanes rainfall to the previous total.


Colour the area above the previous line, as this
area represents Brisbanes rainfall.

Exercise 3C

Brisbane
Rainfall (mm)

Melbourne

Sydney

350
300
250
200
150
100
50
0
Ja
n
Fe
b
M
a
Apr
Mr
ay
Ju
n
Ju
Au l
g
Se
p
Oc
No t
v
De
c

THINK

Analysis of data sets

1 WE7 A business records the average weekly takings for five weeks. The results are $50000,

$65000, $65000, $80000, $75000.


The following week the takings are $15000. Describe the effect that this outlier has on the:
a mean
b median
c mode.
Chapter 3 Interpreting sets of data 65

2 Ian is shopping for a new pair of shoes. His chosen shoe can be found in five stores in a shopping

centre for the following prices.


$125, $120, $124, $232, $135
a Which score in the data set could be described as an outlier?
b If the outlier is removed from the data set, explain whether this will have the greatest effect on the
mean, median or mode.
3 WE8 A television program shows the series Vengeance every Wednesday night at 8:30 pm. The

rating figure for the 13 weeks of the series is given below.


21, 25, 26, 28, 28, 22, 24, 29, 11, 25
a Identify any outliers in the data set.
b Explain the effect that the removal of the outlier will have on the mean and standard deviation
without actually calculating their values.
4 MC A cricket player makes the following scores in 8 innings.

23, 43, 28, 33, 45, 5, 19, 22


In his next innings he makes a score of 190.
What will be the effect on the mean and standard deviation.
A The mean will increase and the standard deviation will increase.
B The mean will increase and the standard deviation will decrease.
C The mean will decrease and the standard deviation will increase.
D The mean will decrease and the standard deviation will decrease.
5 WE9 In a class of 30 students there are 15 boys and 15 girls. Their heights are measured and are
Digital doc
SkillSHEET 3.7
doc-11031
Compiling a
stem-and-leaf plot

listed below.
Boys: 1.65, 1.71, 1.59, 1.74, 1.66, 1.69, 1.72, 1.66, 1.65, 1.64, 1.68, 1.74, 1.57, 1.59, 1.60
Girls: 1.66, 1.69, 1.58, 1.55, 1.51, 1.56, 1.64, 1.69, 1.70, 1.57, 1.52, 1.58, 1.64, 1.68, 1.67
Display this information in a back-to-back stem-and-leaf plot.
6 The number of points scored in each match by two rugby union teams are shown below.

Team 1: 34, 32, 24, 25, 8, 18, 17, 23, 29, 40, 19, 42
Team 2: 23, 20, 35, 21, 46, 7, 9, 24, 27, 38, 41, 30
Display these data in a back-to-back stem-and-leaf plot.
Digital doc
SkillSHEET 3.8
doc-11032
Finding the mean,
median, mode
from a stem-andleaf plot

7 WE11 The stem-and-leaf plot below is used to display the number of vehicles sold by the Ford and

Holden dealerships in a Sydney suburb each week for a three-month period.


Key: 1 | 5 = 15
Ford
74
952210
8544
0
a
b
c
d

Holden
39
111668
2279
5

0
1
2
3

State the median of both distributions.


Calculate the range of both distributions.
Calculate the interquartile range of both distributions.
Show both distributions on a box-and-whisker plot.

8 A motoring organisation tests two different brands of tyres. Twenty tyres of each brand are tested to
Digital doc
SkillSHEET 3.9
doc-11033
Drawing a boxand-whisker plot

find out the number of kilometres each tyre could travel before the tread had worn down. The results
are shown in the stem-and-leaf plot below.
Key: 1 | 2 = 12000 km 1* | 7 = 17000 km
Brand A
98
43110
777665
4431100

Brand B
0*
1
1*
2
2*

0011224
5678889
0134
55

Draw two box-and-whisker plots on the same scale to display this information.
66 Maths Quest HSC Mathematics General 2

9 The figures below show the ratings of two radio stations each week over a three-month period.

Station A: 9.2, 9.4, 9.2, 9.5, 9.7, 9.9, 10.1, 9.1, 8.8, 8.7, 9.0, 8.5, 9.3
Station B: 8.5, 8.1, 8.2, 8.9, 9.0, 9.2, 8.4, 8.7, 8.8, 10.5, 11.2, 11.4, 8.7
a Display the information in a back-to-back stem-and-leaf plot.
b Use the stem-and-leaf plot to display both sets of data on the same box-and-whisker plot.
10 WE10 The box-and-whisker plot drawn below displays statistical data for two AFL teams over a

season.
Team A
Team B
50 60 70 80 90 100 110120130140 150 160
Scores
a Which team had the higher median score?
b What was the range of scores for each team?
c For each team calculate the interquartile range.
11 The two five-number summaries below show the performance of Emad and Larry on their

Mathematics exams throughout the year.


Emad: 45, 64, 68, 76, 80
Larry: 51, 58, 65, 72, 75
a Compare the performance of Emad and Larry on a box-and-whisker plot.
b What is the range for both students?
c What is the interquartile range for both students?
12 MC The box-and-whisker plot drawn below shows Emmas performance in her Physics and

Chemistry exams. Which of the following statements is correct?


Physics
Chemistry
0 10 20 30 40 50 60 70 80 90 100
Marks
A The median of Emmas mark in Physics is greater than for Chemistry.
B The range of Emmas marks in Physics is greater than in Chemistry.
C The interquartile range of Emmas marks in Physics

is greater than in Chemistry.


D All of the above.

13 WE12 This radar chart shows the average daily maximum temperature

for both Sydney and Melbourne for each month of a year.


a Which month had the lowest temperature in Sydney?
b What was the range of temperatures in Melbourne?
c What was the average of the temperatures in Sydney?

Sydney temperature (C)


Melbourne temperature (C)
J
F
D 30
20
N
M
10
O
A
0
S

M
A

14 This radar chart shows the number of customers in two different supermarkets at two-hour intervals.
Supermarket Y
Supermarket X
Midnight
2 am
10 pm 120
100
80
60
8 pm
4 am
40
20
0
6 pm
6 am
4 pm

2 pm

Noon

10 am

8 am

a Find the range for each supermarket.


b Describe the general pattern at each supermarket.
Chapter 3 Interpreting sets of data 67

15 WE13 This area chart shows the average number of rainy days

Brisbane

Melbourne

Sydney

40
35
30
25
20
15
10
5
0

Jan
Fe
b
Ma
Apr
Ma r
Juny
J
Auul
g
Se
p
Oc
No t
Dev
c

Average no. of rainy days


per month

each month in Sydney, Melbourne and Brisbane. Display this


information as a table.

Further development
16 The stem-and-leaf plot drawn below shows the marks obtained by 20 students in both English and

Maths.
Key: 7 | 1 = 71
English
7410
9976653110
87752
2

4
5
6
7
8
9

Maths
17
24799
133466
4448
36
4

a Calculate the median mark for both English and Maths.


b Calculate the range of marks for both English and Maths.
c Comment on the distribution of marks in each of the subjects.
17 The box-and-whisker plot below shows the number of wet days in Sydney per year over a period

oftime.
State the percentage of years
in which the number of wet
days was between:
a 61 and 98
b 56 and 147
c 56 and 98.

32

56 61

98

147

Number of wet days in Sydney per year

18 The box-and-whisker plot below shows the crowds at various matches in the 2013 NRL season.
Digital doc
WorkSHEET 3.1
doc-11034

6500

16 000 19 000

35 000

80 000
Number of people

a
b
c
d
e
f

What was the highest number of people who attended a match of the season?
What was the range for the season?
What was the interquartile range for the season?
What was the median crowd attendance?
What percentage of matches had a crowd less than 16000?
What percentage of matches had a crowd greater than 16000?

Computer application 1: Displaying statistical data


Digital doc
EXCEL Spreadsheet
doc-1344
Fast Food Sales

1. From the Maths Quest HSC Mathematics General 2 eBookPLUS access the spreadsheet Fast
FoodSales.
2. In cell B12 use the spreadsheets inbuilt statistical function to find McDonalds average daily sales.
[=AVERAGE(B4:B10)]
3. In cell B13 use the spreadsheets inbuilt statistical function to find the standard deviation of
McDonalds daily sales. [=STDEV(B4:B10)]
4. Under Edit, use the Fill and Right functions to copy these formulas for KFC and Pizza Hut.
5. Use the charting facility to draw an area chart of the figures presented.

68 Maths Quest HSC Mathematics General 2

3D

Comparison of data sets

When multiple data displays are used to display similar sets of data, comparisons and conclusions can
then be drawn about the data.
Multiple displays such as stem-and-leaf plots and box-and-whisker plots allow for comparison of
statistics such as the median, range and interquartile range, while radar charts and area charts allow for
trends and overall quantities to be compared.
WORKED EXAMPLE 14

A bank surveys the average morning and


Key: 1|2 = 1.2 minutes
Morning
Afternoon
afternoon waiting time for customers. The figures
7 0 788
were taken each Monday to Friday in the morning
and afternoon for one month. The stem-and-leaf
86311 1 1124456667
plot at right shows the results.
9666554331 2 2558
a
Find the median morning waiting time and the
952 3 16
median afternoon waiting time.
5 4
b
Calculate the range for morning waiting times
5 7
and the range for afternoon waiting times.
cWhat conclusions can be made from the display about the average waiting time at the bank in
the morning compared with the afternoon?
THINK

a There are 20 scores in each set and so

the median will be the average of the


10th and 11th scores.

WRITE

a Morning: Median = (2.4 + 2.5) 2

= 2.45 minutes
Afternoon: Median = (1.6 + 1.6) 2
= 1.6 minutes
Chapter 3 Interpreting sets of data 69

b For each data set, subtract the lowest

score from the highest score.

c Conclude that waiting time in the

afternoon is generally less and more


consistent except for one outlier.

b Morning: Range = 4.5 0.7

= 3.8 minutes
Afternoon: Range = 5.7 0.7
= 5 minutes

c The waiting time is generally shorter in the afternoon.

There is one outlier in the afternoon scores which


causes the range to be larger. However, apart from this
outlier the afternoon scores are less spread.

Two-way tables can also be a meaningful way of displaying data. A two-way table allows for two
variables to be compared.
WORKED EXAMPLE 15

A survey of 25000 people is taken. The sex of each respondant is noted and whether they are a
smoker or non-smoker is also noted. The results are displayed in the two-way table below.
Smokers
Non-smokers
Totals

Males
4125
8436
12561

Females
4592
7847
12439

Totals
8717
16283
25000

a What percentage of the females surveyed were smokers?


b What percentage of the smokers surveyed were female?
THINK

a Write 4592 as a percentage of 12439.

WRITE

a Percentage of females who smoke

4592
100%
12 439
= 36.9%
=

b Write 4592 as a percentage of 8717.

b Percentage of smokers who are female

4592
100%
8717
= 52.7%
=

The most common method, however, for comparing data sets is to compare the summary statistics from
the data sets. The measures of location such as mean and median are used to compare the typical score
in a data set. Measures of spread such as range, interquartile range and standard deviation are used to
make assessments about the consistency of scores in the data set.
WORKED EXAMPLE 16

Tutorial
int-2420
Worked
example 16

Below are the scores for two students in eight Mathematics tests throughout the year.
Jane: 45, 62, 64, 55, 58, 51, 59, 62
Pierre: 84, 37, 45, 80, 74, 44, 46, 50
aUse the statistics function on the calculator to find the mean and standard deviation for
eachstudent.
b Which student had the better overall performance on the eight tests?
c Which student was more consistent over the eight tests?
THINK

a Enter the statistics into your calculator and use

the x function for the mean and the n function


for the standard deviation.

70 Maths Quest HSC Mathematics General 2

WRITE

a Jane: x = 57, n = 6

Pierre: x = 57.5, n = 17.4

b The student with the higher mean performed

better overall.

b Pierre performed slightly better overall,

ashismean mark was higher than Janes.

c The student with the lower standard deviation

was more consistent.

c Jane was the more consistent student,

asherstandard deviation was much lower


thanPierres.

Cross tabulation
Cross tabulation is used to compare the variables within a data set. Consider the case
of an ice-cream business that has stores in both Sydney and Melbourne.
To consider how the business is performing the owners needtoconsider both the city
and if the weather is hot or cool.
To cross tabulate a data set, each of the variables must be organised separately.

WORKED EXAMPLE 17

Over a one-week period the following results were obtained from an ice-cream business that has
a store in both Sydney and Melbourne.
Week
Monday
Tuesday
Wednesday
Thursday
Friday
Saturday
Sunday

City
Sydney
Melbourne
Sydney
Melbourne
Sydney
Melbourne
Sydney
Melbourne
Sydney
Melbourne
Sydney
Melbourne
Sydney
Melbourne

Temperature
Hot
Hot
Cool
Hot
Hot
Cool
Cool
Cool
Hot
Cool
Hot
Cool
Hot
Hot

Sales
178
201
152
186
212
98
125
101
284
111
214
147
201
222

Organise the data into a two-way table and interpret the results.
THINK
1

Reorganise the data in a twoway table to show the two


stores vertically and the weather
horizontally.

Calculate the mean and standard


deviation of each data set
separately.

WRITE

Hot
Cool
Sydney
178, 212, 284, 214, 201 152, 125
Melbourne 201, 186, 222
98, 101, 111, 147
Ice-cream sales
Data
Hot
Mean
217.8
Standard deviation
35.5
Melbourne Mean
203
Standard deviation
14.8

City
Sydney

Cool
138.5
13.5
114.25
19.5

Chapter 3 Interpreting sets of data 71

Explain what this information


tells the owner of the ice-cream
shops.

Exercise 3D

The owner of the ice-cream business can see that while the
Sydney business is doing slightly better than the Melbourne
business the weather is a much greater factor in the number of
ice-creams that each store is likely tosell.

Comparison of data sets

1 WE14 The stem-and-leaf plot drawn below shows the marks obtained by 20 students in both

English and Maths.


Key: 7 | 1 = 71
English
7410
9976653110
87752
2

4
5
6
7
8
9

Maths
17
24799
133466
4448
36
4

a Calculate the median mark for both English and Maths.


b Calculate the range of marks for both English and Maths.
c Comment on the distribution of marks in each of the subjects.
2 Tracey measures the heights of twenty Year 10 boys and twenty Year 10 girls and produces the

following five-number summaries for each data set.


Boys: 1.47, 1.58, 1.64, 1.72, 1.81 Girls: 1.55, 1.59, 1.62, 1.66, 1.73
a Draw a box-and-whisker plot for both sets of data and display them on the same scale.
b What is the median of each distribution?
c What is the range of each distribution?
d What is the interquartile range for each distribution?
e Comment on the spread of the heights among the boys and the girls.
3 The following box-and-whisker plots show the heights of
Year 7
a sample of Year 7 boys and a similar-sized sample of Year
Year 12
12 boys.
1.0 1.1 1.2 1.3 1.4 1.5 1.6 1.7 1.8 1.9 2.0
a Calculate the range of heights among both the Year 7
and Year 12 boys.
Height (m)
b Calculate the interquartile range of the heights among
both the Year 7 and Year 12 boys.
c Comment on the relationship between the two data sets, both in terms
Hardware
Software
of measures of location and measures of spread.
J
F
D 2.5
4 The values of hardware and software sales for a chain of computer stores
2
1.5
N
M
are shown for each month in the radar chart on the right. Comment on
1
any relationship observed in this chart between the sales of hardware
0.5
O
0
A
and the sales of software.
S

M
A

areas of New South Wales throughout the year.


a Which region has the greatest rainfall?
b In which region is the range of rainfall figures least?
c What relationship exists between the rainfall in each
of the areas?

1200
1000
Rainfall (mm)

5 The area chart on the right shows the rainfall in four

800
600
400
200

0
mer tumn Winter Spring
Sum Au

72 Maths Quest HSC Mathematics General 2

Western region
Southern region
North/Eastern
region
North/Western
region

6 WE15 The two-way table below shows the results of random breath testing by Sydney police over

one weekend. A driver is charged if they record a reading of 0.05% prescribed concentration of
alcohol (PCA).
Over 0.05 PCA
Below 0.05 PCA
Totals

Males

Females

Totals

26
962
988

7
743
750

33
1705
1738

What percentage of the drivers tested were female?


What percentage of the drivers tested had a PCA over 0.05?
What percentage of female drivers had a PCA over 0.05?
What percentage of male drivers had a PCA over 0.05?
Based on the above results, can any conclusion be drawn concerning the prevalence of drink
driving among males and females? Explain your answer.
7 Ashley is the star player of a football team. To analyse the importance of Ashley to the team, the
coach prepares the two-way table below, showing the results of games over three years both when
Ashley is playing and not playing.
a
b
c
d
e

Won

Lost

Totals

Ashley playing

38

42

Ashley not playing

10

18

Totals

48

12

60

a What percentage of games were won when Ashley played?


b What percentage of games were won when Ashley did not play?
c Do you think that Ashley has a significant impact on the performance of the team? Explain your

answer.
8 To compare the performance of city and country students in the HSC, the number of students
achieving a UAI of at least 90 is studied in six city and six country high schools.
City

Country

Totals

UAI 90

58

61

119

UAI < 90

551

569

1120

Totals

609

630

1239

a What percentage of city students achieved a UAI of at least 90?


b What percentage of country students achieved a UAI of at least 90?
c Of those students who achieved a UAI of at least 90, what percentage were from:
i the city?
ii the country?
d Based on the above results, could any conclusion be drawn about the performance of city and

country students in the HSC?


9 WE16 Calvin recorded his marks for each test that he did in Physics and Chemistry throughout
theyear.
Physics:
65, 74, 69, 66, 72, 64, 75, 60
Chemistry: 45, 85, 91, 42, 47, 72, 87, 85
a In which subject did Calvin achieve the better average mark?
b In which subject was Calvin more consistent? Explain your answer.
10 The police set up two radar speed checks in a country town. In both places the speed limit is
60km/h. The results of the first 10 cars that have their speed checked are given below.
Point A: 60, 62, 58, 55, 59, 56, 65, 70, 61, 64
Point B: 55, 58, 59, 50, 40, 90, 54, 62, 60, 60
a Calculate the mean and standard deviation of the readings taken at each point.
b At which point are drivers generally driving faster?
c At which point is the spread of the readings taken greater? Explain your answer.
Chapter 3 Interpreting sets of data 73

11 Aaron and Sunil open the batting for the local cricket team. The number of runs they have scored in

each innings this season are listed below.


Aaron: 45, 43, 33, 56, 21, 38, 0, 29, 76, 40
Sunil: 5, 70, 12, 54, 68, 11, 8, 64, 32, 69
a Calculate the mean number of runs scored for each player.

b What is the range of runs scored by each player?


c What is the interquartile range of runs scored by each player?
d Which player would you consider to be the more consistent player? Explain your answer.
12 MC Andrea surveys the age of people attending a concert

Band A

given by two bands. The box-and-whisker plots shown at right


Band B
shows the results.
0 10 20 30 40 50 60 70 80
Which of the following conclusions could be drawn based on
the above information?
Age (years)
A Band A attracts an older audience than Band B.
B Band A appeals to a wider age group than Band B.
C Band B attracts an older audience than Band A.
D None of the above.
13 MC Two drugs are tested to see which is more effective at fighting disease. The results are
displayed in the two-way table below.
Recovered
Not recovered
Totals

Drug 1

Drug 2

Totals

124
32
156

136
45
181

260
77
337

Of those patients who recovered, the percentage who were treated with drug 1 was:
A 46.3%
B 47.7%
C 69.0%
D 79.5%
14 MC The figures below show the ages of the mens and womens
champions at a tennis tournament.
Mens: 23, 24, 25, 26, 25, 25, 22, 23, 30, 24
Womens: 19, 27, 20, 26, 30, 18, 28, 25, 28, 22
Which of the following statements is correct?
A The mean age of the mens champions is greater than the
mean age of the womens champions.
B The range is greater among the mens champions than
among the womens champions.
C The interquartile range is greater among the mens champions
than among the womens champions.
D The standard deviation is greater among the mens champions
than among the womens champions.

74 Maths Quest HSC Mathematics General 2

15 WE17 When training Craig either runs quickly or jogs slowly over either 1 km or 2 km. After each

run he records his pulse rate. The results are shown in the table below.
Distance

Speed

Pulse rate

1 km

Quick

111

2 km

Slow

96

1 km

Slow

80

2 km

Quick

120

1 km

Quick

101

2 km

Slow

109

1 km

Slow

88

2 km

Quick

132

1 km

Quick

100

2 km

Slow

104

Cross-tabulate the data to show the mean and standard deviation of Craigs pulse rate for each
distance and speed.
16 A rugby league team plays some matches at night and others during daylight hours. According to

many people this team is very dependent on its star player named Joey. At the end of the season
the coach records the results of day and night games and whether or not Joey plays. The results are
shown below.
(A win by six points is shown as a +6, while a loss by six points is shown as 6)
Day/Night

Joey?

Result

Day/Night

Joey?

Result

Day/Night

Joey?

Result

Day

Yes

+6

Night

No

Night

Yes

+5

Night

Yes

+12

Night

No

+6

Day

No

+14

Night

Yes

Day

Yes

+12

Night

No

+6

Day

Yes

+14

Day

Yes

+20

Day

No

10

Night

No

+2

Night

Yes

+10

Night

Yes

+4

a Cross-tabulate these results to find the mean and standard deviation for day and night matches and

whether or not Joey plays.


b Comment on the difference in performance of the team in day and night matches and whether or

not it appears that Joey is important to the team.


17 A company producing matches advertises that there are 50 matches in each box. Two machines are

used to distribute the matches into the boxes. The results from a sample taken from each machine are
shown in the stem-and-leaf plot below.
Key: 5 | 1 = 51 5* | 6 = 56
Machine A

Machine B

4 4
9 9 8 7 7 6 6 5 4* 5 7 8 9 9 9 9 9 9 9 9
43222211100000 5

0000011111223

5 5 5* 9
a Display the data from both machines on a box-and-whisker plot.
b Calculate the mean and standard deviation of the number of matches distributed from both

machines.
c Which machine is the more dependable? Explain your answer.
Chapter 3 Interpreting sets of data 75

Further development
18 MC The two-way table has had one figure copied incorrectly.

Agree
Disagree
Total

Men
45
72
117

Women
92
93
195

Total
137
175

Which figure was copied incorrectly?


A The number of men who agreed.
C The number of women who agreed.

B The number of men who disagreed.


D The number of women who disagreed.

19 The two-way table shows the results of a study into whether men or women prefer to live alone.

Complete all missing information from the two-way table.


Live alone
Share with friends
Total

Women

21

39

Total
35

Frequency

By examining a graph, we can make judgements about the nature of a data set.
Consider the first graph shown on the right.
This graph is symmetrical and we can see that the mean, median and mode
areall equal to 3. The majority of scores are clustered around the mean. This is
anexample of a normal distribution.
We can compare the standard deviation of data sets by looking at such graphs.
The more clustered the data set, the smaller the standard deviation.
The second graph is still normally distributed with the mean, median and
mode still equal to 3. However, there are more scores which are further away
from the mean and, hence, the standard deviation of the data set is greater.
The third graph shows a data set where the scores are not clustered and there
are two modes at either end of the distribution.
In this example, although it is still symmetrical there are two modes, 1 and 5,
while the mean and median are still 3. The standard deviation in this distribution
is greater than either of the two previous examples as there are more scores
further away from the mean.
The mean and median can be seen from the graph only because it is
symmetrical.

Frequency

Skewness

Frequency

3E

Men
12

8
7
6
5
4
3
2
1
0

6
5
4
3
2
1
0
6
5
4
3
2
1
0

1 2 3 4 5 x

1 2 3 4 5

1 2 3 4 5

Tutorial
int-2418
Worked
example 18

The figure on the right shows the distribution of a set of scores on a


spelling test.
a Is the graph symmetrical?
b What is the mode(s)?
c Can the mean and median be seen from the graph?
THINK

Frequency

WORKED EXAMPLE 18

5
4
3
2
1
0

WRITE

6 7 8 9 10
Score

a The columns either side of the middle are equal.

a The graph is symmetrical.

b The scores that occur the most often are 7 and 9.

b Mode = 7 and 9

c The middle score will be the mean and median.

c Mean = 8, median = 8

76 Maths Quest HSC Mathematics General 2

8
7
6
5
4
3
2
1
0

Frequency

Frequency

When a graph is not symmetrical, the mean and median cannot be easily seen from the graph. Consider
the distribution in the graph below on the left.
The way in which the data are gathered to one end of the distribution is called the skewness. A greater
number of scores are distributed at the lower end of the distribution. In this case, the data are said to be
positively skewed. Similarly, when most of the scores are distributed at the upper end, the data are said
to be negatively skewed, as shown in the graph below on the right.

1 2 3 4 5 x

8
7
6
5
4
3
2
1
0

1 2 3 4 5 x

20
16
12
8
4
0
51
6
61 0
7
71 0
8
81 0
91 90
1
00

The distribution on the right shows the results of the Maths Trial
HSC at a certain school.
a What is the modal class?
b Describe the skewness of the data set shown on the right.

Frequency

WORKED EXAMPLE 19

Maths results

THINK

WRITE

a The class occurring the most often is the 8190 class.

a Modal class = 8190

b The majority of data are at the upper end of the distribution.

b The data are negatively skewed.

Skewness
Frequency

1 WE18 In the distribution shown on the right:


a is the graph symmetrical
b what is the modal class(es)
c can the mean and median be seen from the graph? and, if so, what are

2 For the distribution shown on the right:


a are the data symmetrical
b what is the modal class(es)
c can the mean and median be seen from the graph? and, if so, what

are their values?


3 The table on the right shows the number of goals scored by a hockey

team throughout a season.


a Show this information in a frequency histogram.
b Are the data symmetrical?
c What is the mode(s)?
d Can the mean and median be seen for this distribution? and, if so,
what are their values?

Frequency

their values?

12
10
8
6
4
2
0

1 2 3 4 5

7
6
5
4
3
2
1
0
0
4
5
10 9

15 14
1
20 9

25 24
2
9

Exercise 3E

Number of goals

Chapter 3 Interpreting sets of data 77

12
10
8
6
4
2
0

No. of goals

Frequency

0
1
2
3
4
5

6
4
4
4
4
6

0
1
1
2
2
3
3
4
4
5

Frequency

4 WE19 For the following distribution:


a what is the modal class(es)?
b describe the skewness of the distribution.

Number of goals
5 For each of the following dot plots describe the skewness of the distribution.
a

b

c
x
14 15 16 17 18 19 20
9.5 9.6 9.7 9.8 9.9 10 x
x
0 1 2 3 4 5
6 For the stem-and-leaf plots drawn below describe the distribution
a Key 3 | 5 = 35
b Key 4 | 3 = 4.3 4* | 6 = 4.6

Stem
2
3
4
5
6

Leaf
259
0012589
2289
09
0

Stem
2*
3
3*
4
4*

7 The table below shows the number of goals scored by a

basketball team throughout a season.


No. of goals

Frequency

1120
2130
3140
4150
5160

3
6
7
23
21

a Draw a frequency histogram of the data.


b Describe the data set in terms of its skewness.
8 MC Which of the distributions below has the smallest

standard deviation?

Frequency

6
5
4
3
2
1
0

Frequency

10
8
6
4
2
0

1 23 4 5 x

1 23 4 5 x

78 Maths Quest HSC Mathematics General 2

Frequency

Frequency

6
5
4
3
2
1
0
8
7
6
5
4
3
2
1
0

12345 x

1 2 3 4 5 x

Leaf
9
04
5588
00011344
555678899

9 MC The distribution represented by the graph on the right is:

positively skewed
negatively skewed
symmetrical
normally distributed

Frequency

A
B
C
D

16
14
12
10
8
6
4
2
0

1 2 3 4 5 x

10 A movie is shown at a cinema 30 times during the week.

The number of people attending each session of the


movie is shown in the table below.

a
b
c
d

No. of people

Frequency

150

51100

101150

151200

10

201250

10

Present the data in a frequency histogram.


Are the data symmetrical?
What is the modal class(es)?
Describe the skewness of the distribution.

11 Year 12 at Wallarwella High School sit exams in Chemistry and Maths. The results are shown in the

table below.

a
b
c
d

Mark

Chemistry

Maths

3140

4150

5160

6170

7180

8190

91100

Is either distribution symmetrical?


If either distribution is not symmetrical, state whether it is positively or negatively skewed.
State the mode of each distribution.
In which subject is the standard deviation greater? Explain your answer.

12 Draw an example of a graph which is:


a symmetrical
b positively skewed with one mode
c negatively skewed with two modes.
Chapter 3 Interpreting sets of data 79

Further development
13 MC Match the box plot with its most likely histogram.

B f

A f

C f

D f

14 For each of the following, write down whether the mean or the median would provide a better

Digital doc
WorkSHEET 3.2
doc-11035

indication of the centre of the distribution.


a A positively skewed distribution
b A symmetric distribution
c A distribution with an outlier
d A negatively skewed distribution
15 For each of the following, state whether you would expect the distribution to be positively skewed,
negatively skewed or symmetrical.
a The number of days absent had by students of a school in one term
b The height of Year 12 students
c The number of cars in each household
d The scores out of 10 by Year 12 students on a times tables quiz

80 Maths Quest HSC Mathematics General 2

Summary
Grouped data

Grouped data is used when scores spread across too large a range to remain ungrouped for the
frequency table.
Groups should be organised so as to create 5 to 10 classes.
A column for class centre needs to be created. The class centre is calculated by averaging the highest
and lowest score in each class. The class centre is then used in all calculations.
For grouped data the cumulative frequency histogram and ogive are used to estimate the median,
quartiles and deciles from the data set.

Measures of location Measures of location give the typical score in the data set. The mean, median and mode are measures
and spread
of location.

The mean of a small data set is found using:

x
n
where x = the mean, x = individual scores and n = number of scores.
Where data is in a table, the mean is found using:
x=

x=

fx
f

where x = the mean, x = individual scores and f = frequency.


Measures of spread describe how spread out the data are. The range, interquartile range and standard
deviation are measures of spread.
An outlier is a single score that is much greater or much less than most of the scores. The outlier
may have a great effect on the mean but has only a slight effect on the median and no effect on the
mode in a small data set. The larger the data set, the less the effect a single outlier will have.
Displaying multiple
data sets

Two sets of data can be displayed on a stem-and-leaf plot by displaying the data back to back.
The summary statistics from two data sets can be displayed by using the same scale and drawing two
box-and-whisker plots.
Two sets of data can be displayed on a radar chart to display related trends over a period of time.
An area chart can be drawn to display several sets of data. The area in each section of the graph then
displays the quantities for comparison.

Comparison of
datasets

The summary statistics from two data sets can be compared from a stem-and-leaf plot or box-andwhisker plot.
Two-way tables are used to compare data where there are two variables involved.
Data are most commonly compared using the mean and standard deviation.
Data where there are two variables can be analysed by cross tabulation. To cross tabulate data it is
broken into each subset and each subset analysed separately.

Skewness

When the data are symmetrical, they are said to be normally distributed.
The more clustered the data are around the mean, the smaller the standard deviation.
When most of the data are below the mean, the data are said to be positively skewed.
When most of the data are above the mean, the data are said to be negatively skewed.

Chapter 3 Interpreting sets of data 81

Chapter review
mult ip le
ch oice

1 The table below shows the number of patients seen each day by a local doctor.

No. of patients

Frequency

12
13
14
15
16
17

3
8
15
23
18
13

Which of the following statements are correct?


A The range of the data is 20.
C The standard deviation of the data is 1.34.

B The mean of the data is 15.05.


D The median of the data is 15.

2 The data below show the number of people that live in each house in a small street.

4, 4, 5, 3, 2, 5, 11, 2
The outlier in this data set has:
A the greatest effect on the mean
C the greatest effect on the mode

B the greatest effect on the median


D an equal effect on the mean, median and mode

3 The two data sets below show the number of goals scored in 15 matches by two soccer teams.

Frequency

Manchester: 0, 2, 1, 2, 1, 6, 0, 0, 1, 5, 0, 0, 1, 1, 1
Liverpool:
5
4
3
2
1
0

0 1 2 3 4 5 x
Number of goals

Which of the following statements is correct?


A The Manchester data are negatively skewed, while the Liverpool data are positively skewed.
B The Liverpool data are negatively skewed, while the Manchester data are positively skewed.
C Both sets of data are positively skewed.
D Both sets of data are negatively skewed.
4 The two-way table below shows the number of men and women who work in excess of 45 hours

perweek.
45 hours
> 45 hours
Totals

Men

Women

Totals

132
69
201

128
34
162

260
103
363

The percentage of men who work greater than 45 hours per week is closest to:
B 34%
C 51%
D 67%

A 28%

5 The figures below show the number of attempts that the boys and girls in a Year 12 class take to get

their drivers licence.


Boys: 1, 2, 4, 1, 1, 2, 1, 1, 2, 2, 3, 1
Girls: 2, 2, 4, 2, 1, 2, 2, 3, 1, 1, 1, 2
When comparing the performance of the boys and the girls, it is found that the boys have:
A a lower mean and a lower standard deviation
B a lower mean and a higher standard deviation
C a higher mean and a lower standard deviation
D a higher mean and a higher standard deviation
82 Maths Quest HSC Mathematics General 2

1 The figures below show the marks obtained out of 100 in an exam by 50 Year 12 students on their

half-yearly exam.
68
58
64
80
69
a
b
c
d
e

91
67
65
68
80

92
64
84
81
87

55
64
53
80
57

95
63
85
79
96

59
59
52
82
51

55
91
52
81
71

54
95
56
78
96

69
76
82
98
61

S ho rt
a nsw er

92
78
90
50
71

Display the results in a frequency table using a class size of 10.


Use the table to estimate the mean.
Reconstruct the frequency table using a class size of 5.
Use the new table to do another estimate of the mean.
Which table would give the most accurate estimate of the mean? Explain your answer.

2 Below are the ages of 15 players in a soccer squad.

23, 28, 25, 19, 17, 28, 29, 29, 22, 21, 35, 30, 22, 27, 26
a Calculate the mean age of the players in the squad.
b Find the median age of players in the squad.
3 The table below shows the number of house calls that a doctor has been required to make each day

over a 32-day period.


Number of house calls
0
1
2
3
4
5
a
b
c
d

Frequency
1
6
8
9
6
2

Copy the table into your workbook and add a cumulative frequency column.
Calculate the mean number of house calls per day.
Find the median number of house calls per day.
What is the modal number of house calls per day?

4 The set of figures shown below shows the number of pages in a daily newspaper every day for

twoweeks.
72, 68, 76, 80, 64, 60, 132, 72, 84, 88, 60, 56, 76, 140
a What is the mean number of pages in the newspaper?
b What is the range?
c What is the interquartile range?
d Use the statistics function on your calculator to find the standard deviation.
5 The table below shows the number of rescues that are made each weekend at a major beach.

Number of rescues

Frequency

8
9
10
11
12
13
14

2
5
12
3
0
1
3

Use the statistics function on your calculator to find the mean and the standard deviation of thesedata.
Chapter 3 Interpreting sets of data 83

6 The table below shows the customer waiting time at 10 am each morning at a bank over an 8-week

period.
Waiting time
01 minute
12 minutes
23 minutes
34 minutes
45 minutes
56 minutes
a
b
c
d

Class centre Frequency


1
4
10
13
9
3

Cumulative frequency

Copy and complete the table.


Use the statistics function on your calculator to find the mean and standard deviation.
Draw a cumulative frequency histogram and polygon.
Use the graph to estimate the interquartile range of the data.

7 The figures below show the marks obtained by 20 students in English and Maths.

English: 56, 45, 57, 56, 65, 82, 74, 80, 91, 84, 68, 52, 67, 64, 60, 66, 74, 77, 77, 66
Maths: 65, 66, 58, 60, 61, 70, 74, 66, 69, 68, 71, 55, 51, 49, 50, 71, 99, 85, 70, 66
a Display the data in a back-to-back stem-and-leaf plot.
b For each subject find the median.
c For each subject state the range.
d For each subject find the interquartile range.
8 Betty runs a surf and ski shop. The table below shows the monthly sales of both types of equipment.

Month
January
February
March
April
May
June
July
August
September
October
November
December

Surf sales ($)


20000
18000
12000
9000
6000
4000
5000
8000
10000
11000
15000
22000

Ski sales ($)


5000
6000
8000
10000
12000
12000
9000
8000
6000
3000
4000
9000

a Display both sets of data on the same radar chart.


b Use the chart to compare trends in the sales.

84 Maths Quest HSC Mathematics General 2

9 The data below give the cost per minute of a long-distance telephone call with three companies.

Telecomm

Omtus

Tel One

21c
18c
12c

25c
15c
12c

17.5c
17.5c
17.5c

Day
Economy
Night

Display this information in an area chart.


10 The stem-and-leaf plot below compares the crowds (correct to the nearest thousand) at a football

teams home and away matches.


Key: 2 | 5 = 25000
Home
Away
8 0 67
732 1 0116899
6632 2 45
552 3
a
b
c
d

Calculate the median of both data sets.


Calculate the range of both data sets.
Calculate the interquartile range of both data sets.
Display both sets of data on a box-and-whisker plot.

11 The figure below shows a box-and-whisker plot showing the average number of weekly car sales

made in 2003 and 2004.


2003
2004
0

a
b
c
d

5 6 7 8 9 10 11 12 13 14 15
Average weekly car sales

What was the median for each year?


In which year was the range of sales greatest?
In which year was the interquartile range of sales greatest?
In which year did the car yard perform better? Explain your answer.

12 The two-way table below compares the number of men and women who are right- and left-handed.

Right-handed
Left-handed
Totals

Men
158
17
175

Women
172
15
187

Totals
330
32
362

a What percentage of males are left-handed?


b What percentage of females are left-handed?
c Based on the above data, is there any significant difference between the percentage of male and

female left-handers?
13 Hsiang compares her marks in 10 English exams and 10 Maths exams.

English: 76, 74, 80, 77, 73, 70, 75, 37, 72, 76
Maths: 80, 56, 92, 84, 65, 58, 55, 62, 70, 71
a Calculate Hsiangs mean mark in each subject.
b Calculate the range of marks in each subject.
c Calculate the standard deviation of marks in each subject.
d Based on the above data, in which subject would you say that Hsiang performs more consistently?
Chapter 3 Interpreting sets of data 85

14 Kelly is interested in the surf at his local beach. He wants to reference the size of the swell against

the direction of the wind, and if the weather is wet or dry. The results are shown in the table below.
Weather

Wind

Swell (m)

Weather

Wind

Swell (m)

Wet

North

1.5

Wet

South

1.8

Dry

South

1.4

Wet

South

2.6

Dry

East

2.3

Wet

North

1.8

Dry

West

1.0

Dry

South

1.6

Dry

West

0.9

Dry

East

1.9

Dry

North

1.7

Wet

North

1.7

Wet

West

1.2

Dry

West

1.2

Wet

South

1.4

Dry

North

1.9

15 The figures below show the number of points scored by a basketball player in six matches of a

tournament.
36, 2, 38, 41, 27, 33

16 Consider the data set represented by the frequency histogram on the right.
a Are the data symmetrical?
b Can the mean and median of the data be seen?
c What is the mode of the data?

Frequency

a Calculate the mean number of points per game.


b Calculate the median number of points per game.
c Explain why there is such a large difference between the mean and the median.
8
7
6
5
4
3
2
1
0

15 16 17 18 19 20 x

17 The table below shows the number of attempts that 20 members of a Year 12 class took to obtain a

drivers licence.
Number of attempts

Frequency

11

a Show these data in a frequency histogram.


b Are the data positively or negatively skewed?
18 Draw an example of a frequency histogram for which the data are negatively skewed.
Ext end ed
R esp ons e

1 The data below show the weekly income among ten Year 12 boys and girls.

Boys: $80, $110, $75, $130, $90, $125, $100, $95, $115, $150
Girls: $50, $80, $75, $90, $90, $60, $250, $80, $100, $95
a Calculate the median of both sets of data.
b Calculate the range of both sets of data.
c Calculate the interquartile range of both sets of data.
d Display both sets of data on a box-and-whisker plot.

86 Maths Quest HSC Mathematics General 2

e Use the statistics function on the calculator to find the mean and standard deviation of both

setsofdata.
f Discuss whether the boys or girls have a more consistent average weekly income.
2 In the week leading up to the NRL grand final, Kylie records the number of points scored by both

teams in each game throughout the season and displays the information on the stem-and-leaf plot
below.
Key: 1 | 8 = 18
Sharks
8
84422
88644432200
886200
862
a
b
c
d

Bulldogs
0
1
2
3
4

5558889
0022226668889
000222

Find the median of both sets of data.


Which teams scores are the more consistent?
Describe the skewness of the Sharks scores.
Find the mean and standard deviation of the Bulldogs scores.

Digital doc
Test Yourself
doc-11036
Chapter 3

Chapter 3 Interpreting sets of data 87

ICT activities
3B Measures of location and speed
interactivities
int-0084: Measures of centre. (page 54)
int-2352: Measures of centre. (page 54)
int-2362: Measures of centre. (page 54)
Digital docs
SkillSHEET 3.1 (doc-11026): Finding the mean. (page 57)
SkillSHEET 3.2 (doc-11027): Finding the mode. (page 57)
SkillSHEET 3.3 (doc-11028): Finding the median. (page 57)
EXCEL Spreadsheet (doc-1333): One variable statistics. (page 57)
GC program Casio (doc-1334): UV Stats. (page 57)
GC program TI (doc-1335): UV Stats. (page 57)
SkillSHEET 3.4 (doc-11137): Finding the range. (page 57)
SkillSHEET 3.5 (doc-11029): Finding the interquartile range. (page 58)
SkillSHEET 3.6 (doc-11030): Choosing the appropriate standard
deviation. (page 58)
EXCEL Spreadsheet (doc-1339): Boxplots. (page 58)

3CAnalysis of data sets


interactivities
int-2773: Back-to-back stem-and-leaf plots. (page 62)
int-0802: Box plot and five-number summary. (page 62)
int-2788: Parallel box plots. (page 63)
Tutorial
WE13 int-2419: Learn to standardise scores. (page 65)
Digital docs
SkillSHEET 3.7 (doc-11031): Compiling a stem-and-leaf plot. (page 66)
SkillSHEET 3.8 (doc-11032): Finding the mean, median, mode from a
stem-and-leaf plot. (page 66)

88 Maths Quest HSC Mathematics General 2

SkillSHEET 3.9 (doc-11033): Drawing a box-and-whisker plot. (page 66)


WorkSHEET 3.1 (doc-11034): Apply your knowledge of statistics to
problems. (page 68)
EXCEL Spreadsheet (doc-1344): Fast Food Sales. (page 68)

3D Comparison of data sets


Tutorial
WE16 int-2420: Learn how to compare performance using
statistics. (page 70)

3ESkewness
Tutorial
WE18 int-2418: Learn how to determine skewness. (page 76)
Digital doc
WorkSHEET 3.2 (doc-11035): Apply your knowledge of statistics to
problems. (page 80)

Chapter review
Digital doc
Test Yourself (doc-11036): Take the end-of-chapter test to test your
progress. (page 87)

To access eBookPLUS activities, log on to www.jacplus.com.au

Answers chapter 3
Grouped data

Frequency

5<10

7.5

|||| |||| |

11

10<15

12.5

|||| |||| |||| ||||

19

15<20

17.5

||||

20<25

22.5

||

25<30

27.5

|||

Total

40

Hours of TV watched
per week

Frequency

Class
interval

Class
centre

Tally

145<150

147.5

||

1215

13.5

||||

1619

17.5

|||| ||

2023

21.5

|||| |||| |

11

2427

25.5

|||| |||| |||

13

2831

29.5

||||

7.5

17.5
27.5
12.5
22.5
Number of hours

Class
interval

Class
centre

Tally

157.5

|||| |||

160<165

162.5

|||| ||

165<170

167.5

||||

3235

33.5

170<175

172.5

|||

3639

37.5

175<180

177.5

Total

40

Total

30

c 16
d

146.5

||

148<151

149.5

||

|||| ||||

151<154

152.5

||

Total

32

154<157

155.5

||||

157<160

158.5

||||

160<163

161.5

||||

163<166

164.5

|||

166<169

167.5

||||

169<172

170.5

||

172<175

173.5

175<178

176.5

178<181

179.5

Total

30

59

|||

1014

12

|||| |||| |

1519

17

2024

22

3 a

Class
interval

Class
centre

Tally

09

4.5

|||| |

1019

14.5

|||| ||

2029

24.5

|||| |||

3039

34.5

||||

4049

44.5

||||

5059

54.5

|||| |||| ||

12

6069

64.5

|||| |||

7079

74.5

||||

4
2

Total

56

12
10
8
6
4
2
0

e
Frequency

||

Number of phone calls made per week


Frequency

Frequency

5
4
3
2
1
0

13.5 21.5 29.5 37.5


17.5 25.5 33.5
Class size

Forearm length of boxers

145<148

12
10
8
6
4
2
0

6 a Frequency column: 3, 7, 8, 3, 3, 3, 2, 1
b

147.5 157.5 167.5 177.5


152.5 162.5 172.5
Heights (cm)

12

|||| |

Mathsville class sizes

Heights of Year 9 students

Class
centre

84.5

8
7
6
5
4
3
2
1
0

Class
interval

04

8089

||||

Tally Frequency

8
7
6
5
4
3
2
1
24.5 25.5 26.5 27.5 28.5 29.5 30.5 31.5
Forearm length (cm)

7 a Frequency column: 2, 5, 8, 8, 5, 2
b, c
8

100 m sprint times

6
4
2
10.75 11.25 11.75 12.25 12.75 13.25
Time (s)

8 a Frequency

column:
1, 2, 4, 6,
4, 2, 1

Fat content of packs of bacon


6
4
2
0 4 8 12 16 20 24 28
Fat content (%)

b Frequency column:
Heights of Year 9 students

146.5 152.5 158.5 164.5 170.5 176.5


149.5 155.5 161.5 167.5 173.5 179.5
Heights (cm)

7, 10, 3

10

24.5 44.5 64.5 84.5


14.5 34.5 54.5 74.5
Number of phone calls

Fat content of
packs of bacon

8
6
4
2

Discuss in class.
4.5

Frequency

155<160

Frequency

152.5

150<155

Frequency

Frequency

20
18
16
14
12
10
8
6
4
2
0

Tally

Frequency

Tally

Class
centre

Frequency

Class
Class
interval centre

Class
interval

Frequency

1 a

5 a

Frequency

Exercise 3A

4 a

Frequency

Interpreting sets of data

c The data appear more

0 10 20 30
Fat content (%)

clustered if the group size is larger.

Chapter 3 Interpreting sets of data 89

Cumulative frequency

9 22
10 0.6
11 a 40

6 a 6
b i9
iiLower quartile = 8,

Exercise 3C

iii2
c iMean = 9.04
iin = 1.44
d

c The outlier will have no effect on the

upper quartile = 10

35
30
25
20
15
10
5
0

7 8

median
mode.

2 a $232
b The greatest effect will be on the mean

9 10 11 12

7 a

50 55 60 65 70 75 80

Battery life (h)

Cumulative frequency

b i62.5
iiQ1 = 58, Q3 = 67
iii9
iv14
v 6
12
55
50
45
40
35
30
25
20
15
10
5
0

500010000

1000015000 12500

1500020000 17500

13

2000025000 22500

19

2500030000 27500

23

3000035000 32500

26

15

610

1115

13

14

1620

18

21

2125

23

24

2630

28

25

Cumulative frequency

b 14.6
c

Median = 14

25
20
15
10
5
0

8 13 18 23 28
Class centre

5 a 7
b 1
c Mean = 27.3, n = 1.7

1256788

1.6

4467899

4421

1.7

6 Key: 1 | 8 = 18

Team 1

Team 2

0
1

9543

01347

42

058

20

16

Measures of location

Cumulative
Class
centre Frequency frequency

1.5

987

IQR = 24

Score

997
98665540

30
25
20
15
10
5
0

Class interval

andspread
1 a 1.6
b 1
2 a Mean = 49,median = 44,
mode = no mode
b Mean = 3.4,median = 3.5,
mode = 1
c Mean = 9.575,median = 9.7,
mode = 9.8, 9.9
d Mean = 15.2,median = 15,
mode = 13, 15
3 a 6.5
b 6.5
c 7
4 a

as the total of all wages will be greatly


reduced.
3 a 11
b Removing the outlier will increase the
mean and reduce the standard deviation.
4 A
5 Key: 1.5 | 5 = 1.55
Boys
Girls

b cf

120 130 140 150 160 170 180 190 200

Exercise 3B

Class
Cumulative
centre Frequency frequency

Crowd

Analysis of data sets

1 a The outlier will greatly reduce the mean.


b The outlier will have no effect on the

75%
50%
25%
0

10000 15000 20000 25000 30000 35000

Q1 Q2
Q3
Crowd numbers

c 9500
d x = 21000, n = 6300
8 a Team A = 16, Team B = 16
b iTeam A = 10, Team B = 40
iiTeam A = 4, Team B = 20
iiiTeam A = 3.1, Team B = 12.5
c Both teams had the same mean score.

However, Team A was more consistent


as shown by a lower reading in all three
measures of spread.

9 B
10 A
11 A
12 C
13 a 12
b 9
c The IQRs (middle 50%) are similar

for the two restaurants, but McFeast is


busier around lunch time.
14 a Group A: mean = median = mode =
170 cm
Group B: mean = median = mode =
170 cm
b No
c Group B
d Group B
e Group B
f Group A: range = 20, interquartile
range = 0, = 5.345
Group B: range = 120, interquartile
range = 20, = 32.51
15 a Yes
b Check with your teacher.

90 Maths Quest HSC Mathematics General 2

79

7 a Ford: median = 15, Holden: median = 16


b Ford: range = 26, Holden: range = 32
c Ford: interquartile range = 14,
d

Holden:interquartile range = 13.5


Ford
Holden

0 4 8 12 16 20 24 28 32 36 40 Scale

Brand A
Brand B

0 2 4 6 8 10 12 14 16 18 20 22 24 26 Scale

9 a Key: 8 | 5 = 8.5

Station A
875

Station B
8

12457789

975432210

02

10

11

24

Station A
Station B

7 7.5 8 8.5 9 9.5 10 10.5 11 11.5 12 Scale

10 a Team A
b Team A: range = 60, Team B: range = 90
c Team A: interquartile range = 13,
11 a

Team B: interquartile range = 11


Emad
Larry

0 10 20 30 40 50 60 70 80 90 100 Scale

Exercise 3D

15

15

Month

Sydney Melbourne Brisbane

January

12

13

February

12

14

March

13

15

April

12

12

11

May

12

14

10

June

12

14

July

10

15

August

10

16

September

10

15

October

12

14

November

11

12

10

December

12

11

12

Comparison of data sets

1 a English 66, Maths 63.5


b English 32, Maths 53
c The marks are more spread in Maths

than in English.

2 a

Pulse rate
Distance
1 km

2 km

among boys than among girls.


3 a Year 7: range = 0.45,
Year 12: range = 0.26
b Year 7: interquartile range = 0.15,
Year 12: interquartile range = 0.11
c The range of heights is greater in Year 7
as shown by the range and the IQR. The
heights become less spread by the time
they get to Year 12.
4 The pattern of software sales follows after
the pattern of hardware sales with a slight
time delay.

104

Mean

1.4 1.45 1.5 1.55 1.6 1.65 1.7 1.75 1.8 1.85 1.9 Scale

b Boys 1.64, girls 1.62


c Boys 0.34, girls 0.18
d Boys 0.14, girls 0.07
e The spread of heights is much greater

Quick

Standard
deviation

Boys
Girls

Data
Mean

16 a

84

5.0
126

Standard
deviation

Slow
4

Yes

No

Data
Mean

5.4

Day
13

Night
5.8

Standard
deviation

5.8

5.5

Mean

2.5

12

4.7

Standard
deviation

Machine B: x = 50.12, sn = 2.44

c Machine B has a lower standard

deviation and so is more dependable.


18 D
19

Men

Women

Total

Live alone

12

23

35

Share with
friends

16

25

21

39

60

Total
Exercise 3E

b The team seems to perform better in day

matches but performs significantly less


well when Joey is not playing.

Skewness

1 a Yes
b 3
c Yes, both equal 3
2 a No
b 59 and 2024
c No
3 a
6
5
4
3
2
1
0

0 1 2 3 4 5
Number of goals

b Yes
c 0 and 5
d Yes, both equal 2.5
4 a 12
b Positively skewed
5 a Negatively skewed
b Symmetrical
c Positively skewed
6 a Positively skewed
b Negatively skewed
7 a
25
20
15
10
5
0

103

RL Results
Joey

42 44 46 48 50 52 54 56 58 60 62 Scale

b Machine A: x = 49.96, sn = 2.90,

11
2
21 0
3
31 0
4
41 0
5
51 0
6
0

16 a English: 66; Maths: 63.5


b English: 32; Maths: 53
c Marks are more spread in Maths.
17 a 25%
b 75%
c 50%
18 a 80000
b 73500
c 19000
d 19000
e 25%
f 75%

the number of drivers over the limit


7 a 90.48%
b 55.56%
c Yes, as a much greater percentage of
games are won with Ashley playing.
8 a 9.5%
b 9.7%
c i48.7%
ii51.3%
d There is no significant difference
between the city and country results.
9 a Chemistry, 69.25
b Physics, because of the lower standard
deviation
10 a Point A: x = 61, n = 4.27,
Point B: x = 58.8, n = 12.06
b Point A because of more people driving
over the limit
c Point B because of the greater standard
deviation
11 a Aaron: x = 38.1, Sunil: x = 39.3
b Aaron: range = 76, Sunil: range = 65
c Aaron: interquartile range = 16,
Sunil: interquartile range = 57
d Aaron is more consistent because
although he has a larger range this
is caused by one outlier. Aarons
interquartile range is much less, showing
his consistency.
12 C (based on the interquartile range)
13 B
14 A

Machine B

Number of goals

b Negatively skewed
8 A
9 A
10 a
10
8
6
4
2
0

1
51 50

10 100
1
15 150
1
20 200
1
25
0

Supermarket Y, range = 90
b Both supermarkets follow a similar
pattern. There are very few customers
from midnight to 6 am. Then the
number peaks between 10 am and noon,
remaining fairly constant until 8 pm,
when the number reduces.

Machine A

Frequency

c 21.7

17 a

Frequency

Larry: interquartile range = 19

12 C
13 a July
b 13
14 a Supermarket X, range = 111

5 a Southern
b Western
c Similar peaks and troughs
6 a 43.2%
b 1.90%
c 0.93%
d 2.63%
e More evident in males with three times

Frequency

b Emad: range = 35, Larry: range = 24


c Emad: interquartile range = 23.5,

Number of people

b No
c 151200 and 201250
d Negatively skewed
11 a Chemistry is symmetrical.
b Maths is negatively skewed.

Chapter 3 Interpreting sets of data 91

Chapter Review
Multiple choice

1 B, C and D
2 A
3 A
4 B
5 C
Short answer

Marks

Frequency

50<60

13

60<70

11

70<80

80<90

10

90100

10

b 73.6
c
Marks

55<60

60<65

65<70

70<75

75<80

80<85

85<90

90<95

95100

12
minutes

1.5

23
minutes

2.5

10

15

34
minutes

3.5

13

28

45
minutes

4.5

37

56
minutes

5.5

40

b x = 3.35, n = 1.17
c
40
35
30
25
20
15
10
5
0

Q3

English

15

24

30

32

Maths

5
4
9
7662
5
0158
8766540
6
01566689
7744
7
00114
420
8
5
1
9
9
b English: median = 66.5,
Maths: median = 66
c English: range = 46, Maths: range = 50
d English: interquartile range = 18.5,
Maths: interquartile range = 11.5
8 a
Surf sales
Ski sales
D $25 000 J

frequency

$20 000
$15 000
$10 000
$5 000
$0

F
M
A

M
A

b There is a peak in surf sales through

92 Maths Quest HSC Mathematics General 2

Tel One

10 a Home: 23000 Away: 16000


b Home: 27000 Away: 19000
c Home: 19000 Away: 9000
d

Home

Away
Scale

11 a 2003: median = 7, 2004: median = 8


b 2004
c 2003
d 2004 Higher median, lower limit,

lower quartile and upper limit

12 a 9.7%
b 8.0%
c No significant difference
13 a English: x = 71, maths: x = 69.3
b English: range = 43, maths: range = 37
c English: n = 11.64, maths: n = 11.96
d English, because of the lower standard

Swell (m)

d 2
7 a Key: 5 | 6 = 56

Omtus
Company

deviation

Q1

difference in the answer whether the


class size is 5 or 10. Generally, smaller
class sizes result in more accurate results.
2 a 25.4
b 26
3 a
Number of
Cumulative

10

14

0.51.52.53.54.55.5 x
Waiting time (minutes)

d 73.5
e In this example there is not much

house calls Frequency

Price per minute

Frequency

Night

20

0
Telecomm

0.5

75%

Economy

Day
30

01
minute

25%

50<55

Waiting Class
Cumulative
time
centre Frequency frequency

Cumulative frequency

1 a

b 2.59375
c 3
d 3
4 a 80.57
b 84
c 20
d 24.4
5 x = 10.3, n = 1.64
6 a

30
00
60
00
90
12 00
0
15 00
0
18 00
0
21 00
0
24 00
0
27 00
0
30 00
0
33 00
0
36 00
00
0

Maths: mode = 7180


d Maths, because there are more scores
further away from the centre of the
distribution.
12 Check with your teacher.
13 B
14 a Median
b Mean
c Median
d Median
15 a Negatively skewed
b Symmetrical
c Positively skewed
d Negatively skewed

summer, while the ski sales are greatest


in winter, with a short peak occurring
around Christmas.

Weather
Wet

Data

North South East West

Mean

Dry

1.67

1.93

Standard 0.12
deviation

0.50

Mean

1.8

1.5

2.1

1.03

Standard 0.1
deviation

0.1

0.2

0.12

15 a 29.5
b 34.5
c Outlier reduces the mean greatly.
16 a Yes
b Both are 17.5.
c 17 and 18
17 a
Frequency

c Chemistry: mode = 4150 and 8190,

12
10
8
6
4
2
0

1 2 3 4 5 6
Class centre

b Positively skewed
18 A histogram that shows more columns to

the right of centre than to the left.

Extended response

1 a Boys: median = $105,

Girls: median = $85

b Boys: range = $75, Girls: range = $200


c Boys: interquartile range = $35,

Girls: interquartile range = $20

Boys
Girls
20
40
60
80
10
0
12
0
14
0
16
0
18
0
20
0
24
0
26
0

Scale

e Boys: x = $107, n = $22.4,

Girls: x = $97, n = $53.1

f Generally, the girls is more consistent.

The range and standard deviation for


the girls is inflated by the outlier so

the best measure of consistency is the


interquartile range which is lower for the
girls. Disregarding, the outlier the girls
standard deviation is $15.5.
2 a Sharks: median = 24, Bulldogs:
median = 24
b Bulldogs scores are more clustered
around the mean and so can be seen to
be more consistent.
c Slight positive skew

d Using a calculator, Sharks mean = 26.12,


standard deviation = 10.37
Bulldogs mean = 23.96,
standard deviation = 5.57

Chapter 3 Interpreting sets of data 93

Chapter 4

The normal distribution


CHAPTER CONTENTS
4A z-scores
4B Comparison of scores
4C Distribution of scores

4A

z-scores

Symmetrical distributions
A normal distribution is a statistical occurrence where a data set of scores
is symmetrically distributed about the mean. Most continuous variables in
a population, such as height, mass and time, are normally distributed. In a
normal distribution, the frequency histogram is symmetrical and begins to
take on a bell shape as shown by the figure on the right.
x
The normal distribution is symmetrical about the mean, which has the
same value as the median and mode in this distribution. The graph of a normal distribution will extend
symmetrically in both directions and will always remain above the x-axis.
The spread of the normal distribution will depend on the standard deviation. The lower the standard
deviation, the more clustered the scores will be around the mean. The figure below, on the left, shows a
normal distribution with a low standard deviation, while the figure below, on the right, shows a normal
distribution with a much greater standard deviation.

interactivity
int-0257
The normal
distribution

Standardised scores
To gain a comparison between a particular score and the rest of the population, we use the z-score. The
z-score (or standardised score) indicates the position of a particular score in relation to the mean. z-scores
are a very important statistical measure and later in the chapter some of their uses will be explained.
A z-score of 0 indicates that the score obtained is equal to the mean, a negative z-score indicates that
the score is below the mean and a positive z-score indicates a score above the mean.
The z-score measures the distance from the mean in terms of the standard deviation. A score that is
exactly one standard deviation above the mean has a z-score of 1. A score that is exactly one standard
deviation below the mean has a z-score of 1.
To calculate a z-score we use the formula:
xx
z=
s
where x is the score, x is the mean and s is the standard deviation.
Chapter 4 The normal distribution 95

WORKED EXAMPLE 1

In an IQ test the mean IQ is 100 and the standard deviation is 15. Dales test results give an IQ
of 130. Calculate this as a z-score.
THINK
1

Write the formula.

Substitute for x, x and s.

Calculate the z-score.

WRITE

z=

xx
s

130 100
15
=2
=

Dales z-score is 2, meaning that his IQ is exactly two standard deviations above the mean.
Not all z-scores will be whole numbers; in fact most will not be whole numbers. A whole
numberindicates only that the score is an exact number of standard deviations above or below
the mean.
WORKED EXAMPLE 2

A sample of professional basketball players gives the mean height as 192 cm with a standard
deviation of 12 cm. Dieter is 183 cm tall. Calculate Dieters height as a z-score.
Tutorial
int-2438
Worked example 2

THINK
1

Write the formula.

Substitute for x, x and s.

Calculate the z-score.

WRITE

z=

xx
s

183 192
12
= 0.75
=

The negative z-score in Worked example 2 indicates that Dieters height is below the mean but, in this
case, by less than one standard deviation.
When examining z-scores, care must be taken to use the appropriate value for the standard
deviation.If examining a population, the population standard deviation (n) should be used and if
a sample has been taken, the sample standard deviation (n 1 or sn) should be used. Remember:
Your graphics calculator displays all of this information once data is stored and calculated using the
statisticsfunction.
WORKED EXAMPLE 3

To obtain the average number of hours of study done by students in her class per week, Kate
surveys 20 students and obtains the following results.
12 18 15 14 9 10 13 12 18 25
15 10 3 21 11 12 14 16 17 20
a Calculate the mean and standard deviation (correct to 3 decimal places).
b
Robert does 16 hours of study each week. Express this as a z-score based on the above results.
(Give your answer correct to 3 decimal places.)
THINK

WRITE

Method 1: Use a calculator


a 1 Enter the data into your calculator.
2

Obtain the mean from your calculator.

96 Maths Quest HSC Mathematics General 2

x = 14.25

sn = 4.753

Obtain the standard deviation from your


calculator using the population standard
deviation.

b 1 Write the formula.


2

Substitute for x, x and s.

Calculate the z-score.

xx
s
16 14.25
=
4.753
= 0.368

b z=

Method 2: Technology-enabled
a 1 From the MENU select STAT.

Delete any existing data and enter the


scores from Worked example 3 in List 1.

Press 2 (CALC). You may need to


press 6 first for more options.

Press 6 (SET). Check that 1Var Xlist is


set to List 1 and 1Var Freq is set to 1.

Press w to return to the previous screen,


and then press 1 (1VAR) to display the
summary statistics.

b 1 Press m and then select RUN.

Press K 6 for more options and then


3 (PROB).

Again press 6 for more options and then


4 t(). This is the z-score function, so enter
16, close brackets and press w.

Chapter 4 The normal distribution 97

Exercise 4A

z-scores

1 WE1 In a Maths exam the mean score is 60 and the standard deviation is 12. Chifunes mark is 96.
Digital doc
SkillSHEET 4.1
doc-11037
Finding the mean

Digital doc
SkillSHEET 4.2
doc-11038
Finding the
standard deviation

Digital doc
SkillSHEET 4.3
doc-11039
Choosing the
appropriate
standard deviation

Calculate her mark as a z-score.


2 In an English test the mean score was 55 with a standard deviation of 5. Adrian scored 45 on the

English test. Calculate Adrians mark on the test as a z-score.


3 a In a normal distribution the mean is 32 and the standard deviation 6. Convert a score of 44 to a
z-score.
b In a normal distribution the mean is 1.2 and the standard deviation is 0.3. Convert a score of 0.6 to
a z-score.
c The mean of a distribution is 254 and the standard deviation is 39. Write a score of 214 as a
standardised score, correct to 2 decimal places.
d The mean mark on an exam is 62 and the standard deviation is 9.5. Convert a mark of 90 to a
z-score. (Give your answer correct to 2 decimal places.)
4 Tracy is a nurse and samples the mass of 50 newborn babies born in the hospital in which she works.
She finds that the mean mass is 3.5 kg, with a standard deviation of 0.4 kg. What would be the
standardised score of a baby whose birth mass was:
a 3.5 kg
b 3.9 kg
c 2.7 kg
d 4.7 kg
e 3.1 kg?
5 Ricky finds that the mean number of hours
spent watching television each week by
Year12 students is 10.5 hours, with a
standard deviation of 3.2 hours. How
many hours of television is watched by a
person who has a standardised score of:
a 0
b 1
c 2
d 1
e 3?
6 WE2 IQ tests have a mean of 100 and a standard deviation of 15. Calculate the z-score for a person
with an IQ of 96. (Give your answer correct to 2 decimal places.)
7 The mean time taken for a racehorse to run 1 km is 57.69 s, with a standard deviation of 0.36 s.
Calculate the z-score of a racehorse that runs 1 km in 58.23 s.
8 In a major exam every subject has a mean score of 60 and a standard deviation of 12.5. Clarissa
obtains the following marks on her exams. Express each as a z-score.
a English 54
b Maths 78
c Biology 61
d Geography 32
e Art 95
9 The mean time for athletes over 100 m is 10.3 s, with a standard deviation of 0.14 s. What time
would correspond to a z-score of:
a 0
b 2
c 0.5
d 3
e 0.35
f 1.6?
10 a Explain what is meant by a z-score of 1.
b Explain what is meant by a z-score of 2.
11 WE3 The length of bolts being produced by a machine needs to be measured. To do this, a sample
of 20 bolts are taken and measured. The results (in mm) are given below.
20
17

19
17

18
21

21
20

20
17

17
19

19
18

21
22

22
22

21
20

a Calculate the mean and standard deviation of the distribution.


b A bolt produced by the machine is 22.5 mm long. Express this result as a z-score. (Give your

answer correct to 2 decimal places.)


98 Maths Quest HSC Mathematics General 2

12 A garage has 50 customers who have credit accounts with them. The amount spent by each credit

account customer each week is shown in the table below.


Amount ($)

Class centre

Frequency
2
8
19
15
6

0< 20
20< 40
40< 60
60< 80
80< 100

a Copy and complete the table.


b Calculate the mean and standard deviation.
c Calculate the z-score that corresponds to a customers weekly account of:
ii $100
iii $15.40.
i $50
13 a In a distribution, the mean is 50 and the standard deviation is 10. What score corresponds to a

z-score of 0?
b In a distribution the mean score is 60. If a mark of 76 corresponds to a standardised score of 2,

what is the standard deviation?


14 MC In a normal distribution, the mean is 21.7 and the standard deviation is 1.9. A score of 20.75

corresponds to a z-score of:


A 1
B 0.5

C 0.5

D 1

15 MC In a normal distribution, the mean is 58. A score of 70 corresponds to a standardised score of

1.5. The standard deviation of the distribution is:


A 6
B 8
C 10

D 12

16 MC In a normal distribution, a score of 4.6 corresponds to a z-score of 2.4. It is known that the

standard deviation of the distribution is 0.8. The mean of the distribution is:
A 2.2
B 2.68
C 6.52
D 6.8

17 The results of 24 students sitting a Maths exam are listed below.

95
52

63
57

45
64

48
75

78
81

75
60

80
65

66
70

60
65

58
63

59
62

62
49

a Calculate the mean and standard deviation of the exam marks.


b Calculate the standardised score of the highest score and the lowest score, correct to 2 decimal places.
18 The results of Lukes exams are shown in the table below.

Subject
English
Maths
Biology
Computing studies
Visual arts
Music

Lukes mark

Mean

Standard deviation

72
72
76
60
60
50

60
55
64
70
50
58

12
13
8
5
15
10

Convert each of Lukes results to a standardised score.

Further development
19 To qualify for an elite sports camp an applicant must be able to run 3 km with a z-score less than

1.5 in comparison with the general population.


For males the mean time is 15 min with a standard deviation of 1 min 12 sec
For females the mean time is 16 min 30 sec with a standard deviation of 1 min 30 sec.
Calculate the qualification time for both males and females.

20 Explain why each of the following statements is true.


a In any normally distributed data set the z-score of the mean is equal to 0.
b In any normally distributed data set a person who performs one standard deviation better than the

mean will have a z-score of 1.


Chapter 4 The normal distribution 99

21 The following set of scores represent the ages of the members of a choir.

23

45 32 70 61 44 49 22 36 20
Find the mean and standard deviation of the data set.
Convert each of the scores to a z-score.
What is the mean and the standard deviation of the z-scores found in part b?
Explain why you can give the answer to part c without doing any calculations.
22 Ten people are surveyed for their weekly income and the results are shown below.
$485, $623, $312, $1754, $86, $740, $595, $700, $490, $642
a Find the mean and standard deviation of the data set.
b Which scores in the data set could be identified as outliers?
c Find the z-score for any scores which you have identified as outliers.
23 John obtained a mark of 76 in an exam. The standard deviation on this test was 8 and Johns score
corresponded to a z-score of 1.
Find the z-score for each of the following students.
a Anthea, who scored 60
b Bianca, who scored 88
c Carl, who scored 68
d Demetria, who scored 40
24 The mean height for an adult male is 174 cm with a standard deviation of 8.4 cm. Barney, who is
165cm tall, is generally considered by most people to be short. Fred, who is 187 cm tall, is generally
considered by most people to be tall. Compare Freds tallness with Barneys shortness.
a
b
c
d

Digital doc
WorkSHEET 4.1
doc-11040

4B

Comparison of scores

An important use of z-scores is to compare scores from different data sets. Suppose that in your Maths exam
your result was 74 and in English your result was 63. In which subject did you achieve the better result?
It may appear, at first glance, that the Maths result is better, but this does not take into account the
difficulty of the test. A mark of 63 on a difficult English test may in fact be a better result than 74 if it
was an easy Maths test.
The only way that we can fairly compare the results is by comparing each result with its mean and
standard deviation. This is done by converting each result to a z-score.
xx
z=
If for Maths x = 60 and s = 12, then
s
74 60
=
12
= 1.17
xx
And if for English x = 50 and s = 8, then z =
s
63 50
=
8
= 1.625
The English result is better because the higher z-score shows that the 63 is higher in comparison to the
mean of each subject.
WORKED EXAMPLE 4

Tutorial
int-2439
Worked example 4

Janine scored 82 in her Physics exam and 78 in her Chemistry exam. In Physics, x = 62 and
s=10, while in Chemistry, x = 66 and n = 5.
a Write both results as a standardised score. b Which is the better result? Explain your answer.
THINK

a 1 Write the formula for each subject.


2

Substitute for x, x and s.

Calculate each z-score.

b Explain that the subject with the highest

z-score is the better result.

100 Maths Quest HSC Mathematics General 2

WRITE

xx
xx
Chemistry: z =
s
s
82 62
78 66

=
=
10
5
= 2
= 2.4

a Physics: z =

b The Chemistry result is better because of the higher

z-score.

In each example the circumstances must be read carefully to see whether a higher or lower z-score is
better. For example, if we were comparing times for runners over different distances, the lower z-score
would be the better one.
WORKED EXAMPLE 5

In international swimming the mean time for the mens 100 m freestyle is 50.46 s with a
standard deviation of 0.6 s. For the 200 m freestyle, the mean time is 1 min 51.4 s with a
standard deviation of 1.4 s. Sams best time is 49.92 s for 100 m and 1 min 49.3 for 200 m. At a
competition Sam can enter only one of these events. Which event should he enter?
THINK

WRITE

Write the formula for both events.

Substitute for x, x and s. (For 200 m convert


time to seconds.)

xx
xx
200 m: z =
s
s
49.92 50.46
109.3 111.4
=
=
1.4
0.6

Calculate the z-scores.

= 0.9

The best event is the one with the lower


z-score.

Exercise 4B

100 m: z =

= 1.5

The z-score for 200 m is lower, indicating that


Sams time is further below the mean and that this
is the event that he should enter.

Comparison of scores

1 WE4 Kens English mark was 75 and his Maths mark was 72. In English the mean was 65 with a

standard deviation of 8, while in Maths the mean mark was 56 with a standard deviation of 12.
a Convert the mark in each subject to a z-score.
b In which subject did Ken perform better? Explain your answer.
2 In the first Maths test of the year the mean mark was 60 and the standard deviation was 12. In the

second test the mean was 55 and the standard deviation was 15. Barbara scored 54 in the first test
and 50 in the second test. In which test did Barbara do better? Explain your answer.

Digital doc
EXCEL Spreadsheet
doc-1412
One variable
statistics

3 MC The table below shows the mean and standard deviation in four subjects.

Subject
English
Maths
Biology
Geography

Mean
60
65
62
52

Standard deviation
12
8
16
7.5

Kellys marks were English 66, Maths 70, Biology 50 and Geography 55. In which subject did Kelly
achieve her best result?
A English
B Maths
C Biology
D Geography
4 MC The table below shows the mean and standard deviation of house prices in four Australian

cities. The table also shows the cost of building the same three-bedroom house in each of the cities.
City
Sydney
Melbourne
Adelaide
Brisbane

Mean

Standard deviation

Cost

$230000
$215000
$185000
$190000

$30000
$28000
$25000
$20000

$215000
$201000
$160000
$165000

Chapter 4 The normal distribution 101

In which city is the standardised cost of building the house least?


A Sydney
B Melbourne
C Adelaide

D Brisbane

5 WE5 Karrie is a golfer who scored 70 on course A,

which has a mean of 72 and a standard deviation of 2.5.


On course B, Karrie scores 69. The mean score on
courseB is 72 and the standard deviation is 4. On which
course did Karrie play the better round? (In golf the lower
score is better.)
6 Steve is a marathon runner. On the Olympic course in

Sydney the mean time is 2 hours and 15 minutes with a


standard deviation of 4.5 minutes. On Athens Olympic
course the mean time is 2hours and 16 minutes with
a standard deviation of 3 minutes. In Sydney Steves
time was 2 hours 17minutes and in Athens his time was
2hours 19 minutes.
a Write both times as a z-score.
b Which was the better performance? Explain your
answer.
7 MC The table below shows the mean and standard

deviation of times in the 100 m by the same group of


athletes on four different days. It also shows Matts time
on each of these days.
Day

Mean

Standard deviation

Matts time

8 Jan.
15 Jan.
22 Jan.
29 Jan.

10.21
10.48
10.14
10.22

0.15
0.28
0.09
0.12

10.12
10.30
10.05
10.11

On what day did Matt give his best performance?


A 8 Jan.
B 15 Jan.
C 22 Jan.

D 29 Jan.

8 MC In which of the following subjects did Alyssa achieve her best standardised result?

Subject

Alyssas mark

Mean

Standard deviation

English
Maths
Biology
Music

54
50
60
53

60
55
65
62

12
15
8
9

A English

B Maths

C Biology

D Music

9 Shun Mei received a mark of 64 in her Maths exam and 63 in her Chemistry exam. To determine

how well she actually did on the exams, Shun Mei sampled 10 people who sat for the same exams
and the results are shown below.
Maths:
Chemistry:

56
55

45
63

82
39

90
92

41
84

32
46

65
47

60
50

55
58

69
62

a Calculate the mean and standard deviation for Shun Meis sample in each subject.
b By converting each of Shun Meis marks to z-scores, state the subject in which she performed best.
10 Ricardo scored 85 on an entrance test for a job. The test has a mean score of 78 and a standard

deviation of 8. Kory sits a similar exam and scores 27. In this exam the mean is 18 and the standard
deviation is 6. Who is better suited for the job? Explain your answer.
11 a Cynthia scored a mark of 65 in English where the mean was 55 and the standard deviation is 8. In

Maths Cynthia scored 66 where the mean was 52 and the standard deviation 10. Convert the mark
in each subject to a z-score.
b In which subject did Cynthia achieve her best result?
102 Maths Quest HSC Mathematics General 2

Further development
12 Eamon is a swimmer who can swim 100m in 55s at the state championships. The mean time for his

age group at the championships is 58s with a standard deviation of 1.5 s. Stephanie swims 100 m in
61.5s, when the mean is 63s with a standard deviation of 1 s.
Whose performance is better? Explain your answer.
13 The mean price of a litre of milk is $1.30 with a standard deviation of 10c, and the mean price of a
loaf of bread is $3.10 with a standard deviation of 25c.
Joes convenience store charges $1.55 for a litre of milk and $3.70 for a loaf of bread. Determine
which of the milk and the bread is relatively more expensive.
14 The average number of tourists visiting Cairns each year is 240000 with a standard deviation of
15000. During 2009 the number of visitors fell to 205000.
a Find this number of visitors as a standardised score. (Give your answer correct to 2 decimal
places.)
b What does this tell you about the number of visitors for 2009?
c Suggest some possible reasons for this result.
15 The mean number of visitors on a weekday to a theme park is 2000 and on the weekend is 3500. The
number of visitors on a particular Saturday is 4550 and this corresponds to a z-score of 1.6.
a Find the standard deviation for the number of weekend visitors.
b On weekdays the standard deviation is 345. Calculate the equivalent number of
weekday visitors.
16 A liquid dispenser on a production line is responsible for pouring motor oil into 1.5-L containers.
The mean amount poured is 1.5L and the standard deviation is 8mL.
To test that the each dispenser is working properly, a sample of five containers is taken and the
contents of each measured. If any container has a z-score of greater than 2.5 or less than 1.5, the
machine is closed down until further testing takes place.
From a sample of five the following results were obtained: 1490mL, 1508mL, 1519mL, 1486mL
and 1500mL.
Determine if the machine needs to be closed down for further testing.
17 Jerry and Sammy are brothers. Jerry lives in London where the mean January temperature is 8C
and the standard deviation is 1.5C. Sammy lives in Sydney where the mean January temperature is
27C with a standard deviation of 2.8C.
During a telephone call Jerry comments on how warm it is in London when the temperature is 11C
and Sammy says that it is cool in Sydney at 22C.
Explain what each of the two brothers mean by their comments, and compare the warmth in London
with Sydneys cool weather.

4C

Digital doc
Investigation
Comparison of
subjects
doc-1413

Distribution of scores

In any normal distribution, the percentage of scores that lie within a certain number of standard
deviations of the mean is always the same, provided that the sample is large enough. This is true
irrespective of the values of the mean and standard deviation.
In any normal distribution, approximately 68% of the values will lie
within one standard deviation of the mean. This means 68% of scores
will have a z-score between 1 and 1. This can be shown on a normal
68%
curve as:
3 2 1

interactivity
int-0182
The 68-95-99.7%
rule of z-scores

Approximately 95% of the values lie within 2 standard deviations,


or have a z-score of between 2 and 2.
95%
3 2

Chapter 4 The normal distribution 103

Approximately 99.7% of scores lie within 3 standard deviations,


or have a z-score that lies between 3 and 3.
If we know that a random variable is approximately normally
distributed, and we know its mean and standard deviation, then we can
use this rule to quickly make some important statements about the way in
which the data values are distributed.
3 2 1

99.7%
0

WORKED EXAMPLE 6

Tutorial
int-2440
Worked example 6

Experience has shown that the scores obtained on a commonly used IQ test can be assumed to
be normally distributed with a mean of 100 and a standard deviation of 15. Approximately what
percentage of the distribution lies:
a between 85 and 115? b between 70 and 130? c between 55 and 145?
THINK

a 1 Calculate the z-scores for 85 and 115.


2

68% of scores have a z-score between 1 and 1.

b 1 Calculate the z-scores for 70 and 130.


2

95% of scores have a z-score between 2 and 2.

c 1 Calculate the z-scores for 55 and 145.


2

99.7% of scores have a z-score between 3 and 3.

WRITE

85 100
115 100
z =
15
15
= 1
= 1
68% of the scores will lie between
85 and 115.
70 100
130 100
z =
bz =
15
15
= 2
= 2
95% of the scores will lie between
70 and 130.
55 100
145 100
z =
c z =
15
15
= 3
= 3
99.7% will lie between 55 and 145.
az =

We can also make statements about the percentage of scores that lie in the tails of the distribution by
using the symmetry of the distribution and remembering that 50% of scores will have a z-score greater
than 0 and 50% will have a z-score less than 0.
WORKED EXAMPLE 7

In an exam x = 60 and s = 12. What percentage of candidates in the exam scored above 84?
THINK
1

Calculate 84 as a z-score.

Draw a sketch showing 95% of z-scores lie


between 2 and 2.

5% of z-scores therefore lie outside this range. Half


of these scores lie below 2 and half are above 2.

WRITE

xx
s
84 60
=
12
= 2

z =

2.5%

60
4

Give a written answer.

2.5%

95%
84

2.5% of scores are greater than 84.

Some important terminology is used in connection with this rule. We can say that if 95% of scores have
a z-score between 2 and 2, then if one member of the population is chosen, that member will very
probably have a z-score between 2 and 2.
104 Maths Quest HSC Mathematics General 2

If 99.7% of the population has a z-score between 3 and 3, then if one member of that population is
chosen, that member will almost certainly have a z-score between 3 and 3.
WORKED EXAMPLE 8

A machine produces tyres that have a mean thickness of 12 mm, with a standard deviation
of 1 mm. If one tyre that has been produced is chosen at random, within what limits will the
thickness of the tyre:
a very probably lie? b almost certainly lie?
THINK

WRITE

a 1 Tyre thickness will very probably have a

z-score between 2 and 2.

A z-score of 2 corresponds to a tyre of


10 mm thickness.

A z-score of 2 corresponds to a tyre of 14 mm


thickness.

b 1 Tyre thickness will almost certainly have a

z-score between 3 and 3.

A z-score of 3 corresponds to a tyre of 9 mm


thickness.

A z-score of 3 corresponds to a tyre of 15 mm


thickness.

a If z = 2

If z
x
x = x 2s
= 12 2 1
= 10

=
=
=
=

2
x + 2s
12 + 2 1
14

A tyre chosen will very probably have


a thickness of between 10 and 14 mm.
b If z = 3

If z
x = x 3s
x
= 12 3 1
= 9

=
=
=
=

3
x + 3s
12 + 3 1
15

A tyre chosen will almost certainly have


a thickness of between 9 and 15 mm.

Because it is almost certain that a member of the data set will lie within three standard deviations of the
mean, if a possible member of the data set is found to be outside this range one should suspect a problem.
For example, if a machine is set to deposit 200 mL of liquid into a bottle, with a standard deviation
of 5 mL, and then a bottle is found to have contents of 220 mL, one would expect there to be a problem
with the settings on the machine.
This knowledge of z-scores is then used in industry by the quality control department. In the above
example a sample of bottles would be tested and the z-scores recorded. The percentage of z-scores
between 1 and 1, 2 and 2, and 3 and 3 are checked against the above rule. If these percentages are
not correct, the machinery needs to be checked for faults.

Exercise 4C

Distribution of scores

1 WE6 The temperature on a January day in a city is normally distributed with a mean of 26 and

astandard deviation of 3. What percentage of January days lie between:


a 23 and 29
b 20 and 32
c 17 and 35?
2 The marks of students sitting for a major exam are normally distributed with mean equal to 57 and

standard deviation equal to 13. What percentage of marks on the exam were between:
a 44 and 70
b 31 and 83
c 18 and 96?
3 The mean thickness of bolts produced by a machine is 2.3 mm, with a standard deviation of

0.04mm. What percentage of bolts will have a thickness between 2.22 mm and 2.38 mm?
4 WE7 Experience has shown that the scores obtained on a commonly used IQ test can be assumed

to be normally distributed with a mean of 100 and a standard deviation of 15. What percentage of
scores lie above 115?
5 The heights of young women are normally distributed with a mean x = 160 cm and a standard

deviation equal to 8 cm. What percentage of the women would you expect to have heights:
b greater than 168 cm
c less than 136 cm?

a between 152 and 168 cm

6 The age at which women give birth to their first child is normally distributed with x = 27.5 years

and n = 3.2 years. From these data we can conclude that about 95% of women have their first child
between what ages?
Chapter 4 The normal distribution 105

7 Fill in the blanks in the following statements. For any normal distribution:
a 68% of the values have a z-score between ___ and ___
b ___% of the values have a z-score between 2 and 2
c ___% of the values have a z-score between ___ and ___.
8 MC Medical tests indicate that the amount of an antibiotic needed to destroy a bacterial infection

in a patient is normally distributed with x = 120 mg and n = 15 mg. The percentage of patients who
would require more than 150 mg to clear the infection is:
A 0.15%
B 2.5%
C 5%
D 95%

9 MC The mean mark on a test is 55, with a standard deviation of 10. The percentage of students who

achieved a mark between 65 and 75 is:


A 13.5%
B 22.5%

C 34%

D 95%

10 In a factory, soft drink is poured into cans such that the mean amount of soft drink is 500 mL with

astandard deviation of 2 mL. Cans with less than 494 mL of soft drink are rejected and not sold to
the public. What percentage of cans are rejected?
11 The distribution of IQ scores for the inmates of a certain prison is approximately normal with a mean

of 85 and a standard deviation of 15.


a What percentage of this prison population have an IQ of 100 or higher?
b If someone with an IQ of 70 or less can be classified as mentally disabled, what percentage of the
prison population could be classified as mentally disabled?
12 The distribution of blood pressures (systolic) among women of similar ages is normal with a mean of

120 (mm of mercury) and a standard deviation of 10 (mm of mercury). Determine the percentage of
women with a systolic blood pressure:
a between 100 and 140
b greater than 130
c between 120 and 130
d between 90 and 110
e between 110 and 150.
13 WE8 The mass of packets of chips is normally distributed with x = 100g and n = 2.5g. If I

purchase a packet of these chips, between what limits will the mass of the packet:
b almost certainly lie?

a very probably lie

14 The heights of army recruits are normally distributed about a mean of 172cm and a standard

deviation of 4.5cm. A volunteer is chosen from the recruits. The height of the volunteer will very
probably lie between what limits?
15 A machine is set to deposit a mean of 500g of washing powder into boxes with a standard deviation

of 10g. When a box is checked, it is found to have a mass of 550g. What conclusion can be drawn
from this?
16 The average mass of babies is normally distributed with a mean of 3.8kg and a standard deviation of

0.4kg. A newborn baby will almost certainly have a mass between what limits?

Further development
17 The force required to tear a piece of fabric is normally distributed. A sample of the fabric is tested

and it is found that the mean breaking force was 6N with a standard deviation of 0.15N.
Write down the values between which:
a 68% of the scores would lie
b 95% of the scores would lie
c 99.7% of the scores would lie.
18 MC A distribution of scores is normally distributed with a mean of 42. It is known that 68% of

scores lie between 32 and 52.


The standard deviation of the data set is:
A 5
B 10

C 15

D 20

19 The salaries of a group of business people is normally distributed. The mean salary is $65000 with

astandard deviation of $5000.


Find the percentage of business people who:
a have a salary less than $70000
b have a salary less than $50000
c have a salary more than $55000
d have a salary between $50000 and $75000.
106 Maths Quest HSC Mathematics General 2

20 In a school the number of days absent by students is normally distributed. The mean number of days

absent is 15 days with a standard deviation of 4 days.


Find the percentage of students who have:
a more than 19 days off per year
b fewer than 3 days off per year
c more than 7 days off per year
d between 11 and 23 days off per year.
21 The volume of soft drink poured into bottles is measured by the quality control department, and the
results are found to be normally distributed. The department found that the mean was 375mL and
the standard deviation was 5mL.
From a sample of 400 bottles, how many would be expected to have a volume of:
a more than 375mL
b more than 370mL
c more than 380mL
d less than 365mL?
22 The masses of each ball bearing in a batch are normally distributed. The mean mass of a ball bearing
is 25g and the standard deviation is 1g.
The manufacturer rejects all ball bearings that are less than 23g in mass and more than 26g in mass.
From a sample of 2500 ball bearings, how many ball bearings will be rejected?

Digital doc
WorkSHEET 4.2
doc-11041

Digital doc
Investigation
Examining a
normal distribution
doc-1415

Chapter 4 The normal distribution 107

Summary
z-scores

A data set is normally distributed if it is symmetrical about the mean.


A z-score measures the position of a score relative to the mean and
standard deviation.
A z-score is found using the formula
z=

xx
s

where x is the score, x is the mean, and s is the standard deviation.


Comparison of scores

Standardising both scores best compares scores from different data sets.
When comparing exam marks, the highest z-score is the best result.

Distribution of scores

A data set that is normally distributed will be symmetrical about the mean.
68% of scores will have a z-score of between 1 and 1.
95% of scores will have a z-score between 2 and 2. A score chosen from
this data set will very probably lie in this range.
99.7% of scores will have a z-score of between 3 and 3. A score chosen
from the data set will almost certainly lie within this range.

108 Maths Quest HSC Mathematics General 2

Chapter review
1 The mean time for 12-year-old boys to swim 50 m is 50.5 s with a standard deviation of 4.2 s. Kyle

swims 50 m in 44.2 s. Kyles time as a standardised score is:


A 6.3
B 1.5
C 1.5
D 6.3

M U LTIP L E
CHOICE

2 A teacher converts the marks on every test that she

gives her class to a standardised score. On a test the


mean mark was 50 and the standard deviation was
10. Adams standardised score on the test was 0.6.
Adams mark on the test was:
A 40
B 44
C 56
D 60
3 The details of Andreas half-yearly exams are shown in the table below.

Subject
English
Maths
History
Geography

Andreas
mark
65
62
75
50

Mean
50
52
58
44

Standard
deviation
12
6
15
4

In which subject did Andrea achieve her best result?


A English
B Maths
C History
D Geography
4 The details of Bretts half-yearly exams are shown in the table below.

Subject
English
Maths
History
Geography

Bretts
mark

Mean

Standard
deviation

40
48
49
42

50
52
58
44

12
6
15
4

In which subject did Brett achieve his best result?


A English
B Maths
C History
D Geography
5 A data set is normally distributed with x = 25 and s = 2.5. The percentage of scores that will lie in

the range 25 to 30 is:

A 34%
C 68%

B 47.5%
D 95%

6 A fishing boat catches a load of fish and finds the mass of

each fish. The masses of the fish are normally distributed


with a mean of 800 g and a standard deviation of 75 g.
If a fish is chosen from the catch, its mass will almost
certainly lie between:
A 725g and 875g
B 650g and 950g
C 575g and 1025g
D 800g and 1025g
Chapter 4 The normal distribution 109

Sh ort
a nswer

1 Measurements of the amount of acid in a

certain chemical are made. The results are


normally distributed such that the mean is
6.25% and the standard deviation is
0.25%. Harlan gets a reading of 5.75%.
What is Harlans reading as a z-score?
2 A set of scores is normally distributed

such that x = 15.3 and s = 5.2. Convert


each of the following members of the
distribution to z-scores.
a 15.3
b 20.5
c 4.9
d 30.9
e 10.1

3 On an exam the results are normally distributed with a mean of 58 and a standard deviation of

7.5.Jennifer scored a mark of 72 in the exam. Convert Jennifers mark to a z-score, giving your
answer correct to 2 decimal places.
4 A set of scores is normally distributed with a mean of 2.8 and a standard deviation of 0.6. Convert

each of the following members of the data set to z-scores, correct to 2 decimal places.
a 2.9
b 3.9
c 1
d 1.75
e 1.6
5 Anji conducts a survey on the water temperature at her local beach each day for a month. The results

(in C) are shown below.


20

21

19

22

21

18

17

23

17

16

20

21

18

22

17

16

20

20

22

19

22

20

20

20

21

21

22

23

24

20

a Find the mean and standard deviation of the scores.


b Find the highest and lowest temperatures in the data set and express each as a z-score.
6 The table below shows the length of time for which a sample of 100 light bulbs will burn.

Length of time
(hours)

Class centre

Frequency

0< 500

500< 1000

28

1000< 1500

59

1500< 2000

10

a Find the mean and standard deviation for the

data set.
b A further sample of five light bulbs are chosen.

The length of time for which each light bulb burned


is given below. Convert each of the following to a
standardised score.
i 1000 hours
ii 1814 hours
iii 256 hours
iv 751 hours
v 2156 hours
110 Maths Quest HSC Mathematics General 2

7 Betty sat exams in both Physics and Chemistry. In

Physics the exam results showed a mean of 48 and


a standard deviation of 12, while in Chemistry the mean
was 62 with a standard deviation of 9.
a Betty scored 66 in Physics. Convert this result to
a z-score.
b Betty scored 71 in Chemistry. Convert this result
to a z-score.
c In which subject did Betty achieve the better result?
Explain your answer.
8 In Geography Carlos scored a mark of 56, while in
Business studies he scored 58.
In Geography x = 64 and s = 10. For Business studies
x = 66 and s = 15.
a Convert each mark to a standardised score.
b In which subject did Carlos achieve the better result?
9 A psychologist records the number of errors made on
a series of tests. On a literacy test the mean number
of errors is 15.2 and the standard deviation is 4.3. On
the numeracy test the mean number of errors is 11.7
with a standard deviation of 3.1. Barry does both tests
and makes 11 errors on the literacy test and 8 errors on
the numeracy test. In which test did Barry do better?
Explain youranswer.
10 A data set is normally distributed with a mean of 40 and a standard deviation of 8. What percentage
of scores will lie in the range:
a 32 to 48
b 24 to 56
c 16 to 64?
11 The value of sales made on weekdays at a store appears to be normally distributed with a mean of
$1560 and a standard deviation of $115. On what percentage of days will the days sales lie between:
a $1445 and $1675
b $1330 and $1790
c $1215 and $1905?
12 A data set is normally distributed with a mean of 56 and a standard deviation of 8. What percentage
of scores will:
a lie between 56 and 64
b lie between 40 and 56
c be less than 40
d be greater than 80
e lie between 40 and 80?
13 A machine is set to produce bolts with a mean diameter of 5 mm with a standard deviation of
0.1mm. A bolt is chosen and it is found to have a diameter of 4.5 mm. What conclusion can be
drawn about the settings of the machine?

Chapter 4 The normal distribution 111

Ext end ed
R espons e

1 Theresa attempts to review her exam results in Physics and Chemistry. Theresa samples 10 of her

friends and finds the following results.


Physics:
65
Chemistry: 72

64
50

69
55

72
62

50
68

66
51

66
75

63
78

69
44

Find the mean and standard deviation in each subject.


Theresas marks were 65 in Physics and 67 in Chemistry. Convert each to a standardised score.
In which subject did Theresa score her best result? Explain your answer.
A student is chosen at random from the Physics class. Between what two marks will this persons
result very probably lie?
e If the marks within the class follow a normal distribution, within what two marks will
approximately 99.7% of all Chemistry scores lie?
2 A machine is set to cut lengths of metal such that the mean length of metal cut is 12.5cm with a
standard deviation of 0.05cm.
a A piece of metal is measured to have a length of 12.4cm. Express this as a standardised score.
b A second piece of metal is measured and found to have a length of 13cm. What conclusion can be
drawn from this measurement?
a
b
c
d

Digital doc
Test Yourself
doc-11042
Chapter 4

67
69

112 Maths Quest HSC Mathematics General 2

ICT activities
4A z-scores

4C Distribution of scores

interactivity
int-0257: The normal distribution. (page 95)

interactivity
int-0182: The 68-95-99.7% rule of z-scores. (page 103)

Tutorial
WE2 int-2438: Perform a calculation of a z-score. (page 96)

Tutorial
WE6 int-2440: Estimate distribution scores. (page 104)

Digital docs
SkillSHEET 4.1 (doc-11037): Finding the mean. (page 98)
SkillSHEET 4.2 (doc-11038): Finding the standard deviation. (page98)
SkillSHEET 4.3 (doc-11039): Choosing the appropriate standard
deviation. (page 98)
WorkSHEET 4.1 (doc-11040): Apply your knowledge of z-scores.
(page100)

Digital docs
WorkSHEET 4.2 (doc-11041): Apply your knowledge of distribution
scores. (page 107)
Investigation (doc-1415): Examining a normal distribution. (page 107)

4B Comparison of scores
Tutorial
WE4 int-2439: Perform a comparison of scores. (page 100)

Chapter review
Test Yourself (doc-11042): Take the end-of-chapter test to test your
progress. (page 112)

To access eBookPLUS activities, log on to www.jacplus.com.au

Digital docs
Spreadsheet (doc-1412): One variable statistics. (page 101)
Investigation (doc-1413): Comparison of subjects. (page 103)

Chapter 4 The normal distribution 113

Answers chapter 4
The normal distribution
Exercise 4A

z-scores

1 3
2 2
3 a 2
b 2
c 1.03
d 2.95
4 a 0
b 1
c 2
d 3
e 1
5 a 10.5
b 13.7
c 16.9
d 7.3
e 0.9
6 0.27
7 1.5
8 a 0.48
b 1.44
c 0.08
d 2.24
e 2.8
9 a 10.3 s
b 10.58 s
c 10.37 s
d 9.88 s
e 10.251 s
f 10.524 s
10 a One standard deviation above the mean
b Two standard deviations below the mean
11 a x = 19.55, sn = 1.76
b 1.68
12 a
Class

Amount ($)
0<20
20<40
40<60
60<80
80<100

centre
$10
$30
$50
$70
$90

Frequency
2
8
19
15
6

b x = 56, n = 20.1
c i 0.30
ii 2.2
iii 2.0
13 a 50
b 8
14 B
15 B
16 C
17 a x = 64.7, n = 11.4
b Highest score z = 2.66, Lowest score

z = 1.73

18 English 1, Maths 1.31, Biology 1.5,

Computing studies 2, Visual arts 0.67,


Music 0.8
19 Male: 13 min 12 s; female: 14 min 15 s
20 a True. Each score is measured by how
many standard deviations above or
below the mean it is. Therefore, the
z-score of the mean must be 0.
b True. A person performing one standard
deviation better than the mean could
have a z-score of 1 if a lower value is
better, e.g. the time taken to run a race.
But a person performing one standard
deviation better than the mean would
have a z-score of 1 if a higher score
is better, e.g. the marks achieved on a
Maths test.
21 a x = 40.2, n = 16.0
b 1.075, 0.3, 0.5125, 1.8625, 1.30,
0.55, 1.1375, 0.2625, 1.2625
c Mean of the scores = 0, standard
deviation = 1
d This is the mean and standard deviation
of any set of standardised scores.
22 a x = $642.70, n1 = 436.9
b $1754 and $86

c $1754 is a z-score of 2.54; $86 is a

z-score of 1.27
23 a 1
b 2.5
c 0
d 3.5
24 Barneys height corresponds to a z-score of
1.07, and Freds height corresponds to a
z-score of 1.55. It can be said that Fred is
taller than Barney is short.
Exercise 4B Comparison of scores
1 a English 1.25, Maths 1.33
b Maths mark is better as it has a higher
z-score.
2 2nd test, Barbaras z-score was 0.33
compared to0.5 in the first test.
3 B
4 D
5 Course A, z-score of 0.8 compared to
0.75 on course B
6 a Sydney 0.44, Athens 1
b In Sydney because of the lower z-score
7 C
8 B
9 a Maths x = 59.5, sn = 17.9
Chemistry x = 59.6, sn = 16.8
b Maths 0.25, Chemistry 0.20. Maths is
the better result.
10 Kory is the better candidate with a z-score
of 1.5 compared with 0.875 for Ricardo.
11 a English 1.25, Maths 1.4
b Maths
12 Eamons z-score = 2; Stephanies
z-score = 1.5. Eamons performance is
better as he is more below the mean that
Stephanie.
13 Milk z-score = 2.5, Bread z-score = 2.4.
The milk is relatively more expensive.
14 a 2.33
b The number of visitors was extremely
low.
c Possible causes include weather, the
economy, airline schedules etc.
15 a 656.25
b 2552
16 The machine must be closed for further
testing. One bottle (1486 mL, z = 1.75)
has a z-score less than 1.5.
17 In London z = 2. In Sydney z = 1.78. It
is relatively warmer in London compared
to the average there than it is cooler in
Sydney relative to the average there.
Exercise 4C

Distribution of scores

1 a 68%
b 95%
c 99.7%
2 a 68%
b 95%
c 99.7%
3 95%
4 16%
5 a 68%
b 16%
c 0.15%
6 21.1 and 33.9
7 a 68% of the values have a z-score

between 1 and 1.
b 95% of the values have a z-score
between 2 and 2.
c 99.7% of the values have a z-score
between 3 and 3.
8 B
9 A
10 0.15%

114 Maths Quest HSC Mathematics General 2

11 a 16%
b 16%
12 a 95%
b 16%
c 34%
d 15.85%
e 83.85%
13 a 95 g to 105 g
b 92.5 g to 107.5 g
14 163 cm 181 cm
15 Faulty, as the one chosen has a z-score

greater than 3

16 2.6 kg 5 kg
17 a 5.85 N and 6.15 N
b 5.7 N and 6.3 N
c 5.55 N and 6.45 N
18 B
19 a 84%
b 0.15%
c 97.5%
d 97.35%
20 a 16%
b 0.15%
c 97.5%
d 81.5%
21 a 200
b 336
c 64
d 10
22 462

Chapter Review
Multiple choice

1 B
4 D

2 B
5 B

3 B
6 C

Short answer

1 2
2 a 0
b 1
c 2
d 3
e 1
3 1.87
4 a 0.17
b 1.83
c 3
d 1.75
e 2
5 a x = 20.1, n = 2.1
b Highest = 1.86, Lowest = 1.98
6 a x = 1130, n = 334.2
b i 0.39
ii 2.05
iii 2.62
iv 1.13
v 3.07
7 a 1.5
b 1
c Physics, higher z-score
8 a Geography: 0.8, Business

studies:0.53

b Business studies: higher z-score


9 Numeracy: lower z-score
10 a 68%
b 95%
c 99.7%
11 a 68%
b 95%
c 99.7%
12 a 34%
b 47.5%
c 2.5%
d 0.15%
e 97.35%
13 Faulty, as it is more than three standard

deviations from the mean.

Extended response

1 a Physics x = 65.1, sn = 5.9

Chemistry x = 62.4, sn = 11.8

b Physics 0.02, Chemistry 0.39


c Chemistry has a higher z-score.
d 53.3 and 76.9
e 27 and 97.8
2 a 2
b Faulty, more than three standard

deviations from the mean

Chapter 5

Sampling and populations


CHAPTER CONTENTS
5A Sampling
5B Samples and means
5C Estimating populations

5A

Sampling

A sample is taken from an entire population when it is not feasible to obtain information from every
member of the target population.
For example, suppose that you want to know the average number of attempts needed for people to get
their drivers licence. It is not feasible for us to survey every driver and so we could sample a selection of
drivers and if the sample is well chosen, the information obtained should be representative of the entire
population.
The selection of an appropriate sample can be done best by one of three methods.
1. Random sample. The participants in the sample are chosen randomly. If the sample is large enough
and the process repeated the sample characteristics should be similar to the entire population.
2. Stratified sample. The sample is deliberately chosen in the same proportion as the entire population.
For example in a workplace there may be 60% male workers. If a sample is chosen from within this
workplace then 60% of the participants should be male. Within each strata or group the participants
should be chosen randomly.
3. Systematic sample. A systematic method is used to select the sample. For example, a store may
sample every tenth person that makes a purchase.

interactivity
int-0219
Classifying data

WORKED EXAMPLE 1

In each of the following state the type of sample that has been taken.
a
A school holds a public meeting. Present are 40 parents, 80 students and 20 teachers. A committee
is selected that consists of 2 parents, 4 students and 1 teacher.
b
When renewing a drivers licence every fifth person is asked the number of attempts they needed
to obtain their licence.
cA computer randomly selects two hundred telephone numbers and the people who have these
numbers are asked to complete a survey on their Internet usage.
THINK

a The committee is chosen in the same proportion as the attendees

WRITE

a This is a stratified sample.

at the meeting.
b There is a systematic method in the way the people to be asked

b This is a systematic sample.

are chosen.
c The participants in the sample are chosen randomly.

c This is a random sample.

When selecting a random sample it is essential that the sample be free from any external bias. It is
therefore best to use a random number generator to select a sample.
Chapter 5 Sampling and populations 115

Interactivity
int-0089
Random number
generator

Your calculator will have a random number generator function. You can use this function to generate a
random decimal between 0 and 1. This decimal is multiplied by the sample size and rounded up to the
next whole number to determine each participant.
WORKED EXAMPLE 2

A factory has 500 workers and one person is to represent the factory at a union conference.
If the person is to be chosen using a random number generator on a calculator, show how
this would bedone.
THINK

WRITE

Use the random number generator on your


calculator to generate a decimal.

Random number = 0.257 (your number will


most likely be different)

Multiply the decimal by 500.

0.257 500 = 128.5

Round the result up to the next whole number.

The 129th person on the list should be chosen.

In the Preliminary course we also studied stratified samples and how the selection of the participants
needs to be in the same proportion as with the entire target population.
WORKED EXAMPLE 3

The same factory with 500 workers has 350 male workers and 150 female workers. A union
committee of ten is to be selected using a stratified sample. How many of each sex should be
represented on the committee?
THINK

WRITE

Calculate the percentage of male workers and


percentage of female workers.

Percentage male =

The number of male representatives is 70%


of10.

70% of 10 = 7

The number of female representatives is 30%


of 10.

30% of 10 = 3

Answer the question.

There should be 7 men and 3 women on the


committee.

Exercise 5A

350
= 70%
500

Sampling

1 WE1 In each of the following state the type of sample that has been chosen.
a Every fiftieth person to pass through the gate at a sporting event is surveyed.
b Every person at a meeting is given a raffle ticket. Five are selected to participate in a further study.
c Participants for a poll are selected by a computer selecting phone numbers.
d From a group of 80 men and 40 women

a panel of 4 men and two women are


selected.
2 A caryard has 500 cars, 60 trucks and
180motorbikes for sale. An inspection team
is coming in to examine the vehicles for
defects. In each of the following describe
the type of sample that has been chosen.
a 25 cars, 3 trucks and 9 motorbikes are
chosen.
b A computer selects the number plates of
vehicles to be examined.
116 Maths Quest HSC Mathematics General 2

c Every third vehicle sold is examined.


d All the vehicles parked in the front row of the yard are examined.
e 50 vehicles are chosen in proportion to the number of each make in the yard.
3 WE2 Judy has to select one person from

a group of 250. She allocates each person a


number from 1to 250 and uses her
calculator to generate the random decimal
0.517. The person holding which number
should be chosen?
4 A school has 1150 students. The Principal

must select 10 students for an interview.


Use your calculators random number
generator to assign 10 numbers for the
principal to interview.
5 At the same school there are 75 teachers. A

uniform committee is to be set up consisting


of both students and teachers. If there are
12places on the committee how many
students and how many teachers should
be selected if a stratified sample is to be
chosen?
6 WE3 In an Australian Olympic team there

are 260 men and 240women. If 50 athletes are to be drug tested how many of each sex should be
chosen in a stratified sample?
7 Kingston Park is a horse stud farm where there are 10 stallions, 140mares and 100 foals.

A vet health check on 25 horses is to be conducted. If those horses to be checked are chosen in the
form of a stratified sample, how many of each category should be examined?
8 A football club consists of 10 people in management, 5 on the coaching staff, 15 support staff and

30 players. A survey of 20 people from within the club is to be conducted. How many should be
selected from each category?
9 A survey is to be conducted on 20 staff at a factory. In each of the following, state what is wrong

with the selection of samples.

a The first 20 people to arrive for work in the morning are surveyed.
b The 20 people who work the night shift are selected.
c All 20 participants are chosen from the office staff.
10 Explain the term stratified random sample.

Digital doc
WorkSHEET 5.1
doc-11043

Chapter 5 Sampling and populations 117

5B

Samples and means

When information needs to be obtained from a large population we usually complete a sample, as a
census, which involves the entire population, is impractical. We need to be as sure as possible that the
data obtained from the sample chosen reflects the data from the whole population.
WORKED EXAMPLE 4

Consider a group of four people. We are examining the average height of these four people.
The four people are:
Aiden 193 cmJake 187 cmIsaac 178 cmChaz 166 cm
a Calculate the mean height of the four boys.
b
A sample of two from the four is to be chosen. List all possible samples of two that could beselected.
c Calculate the mean of each possible sample of two.
d Calculate the mean of all samples taken.
THINK

WRITE

a Add the scores together and divide

by 4.
b There are six pairs list each by

taking them two at a time.

c Find the mean of each sample by

adding each possible pair of heights


and dividing by 2.

d Find the mean of each pair in part c.

a Mean = (193 + 187 + 178 + 166) 4

= 181 cm
b Aiden and Jake
Aiden and Isaac
Aiden and Chaz
Jake and Isaac
Jake and Chaz
Isaac and Chaz

c Aiden and Jake: Mean = (193 + 187) 2

= 190 cm
Aiden and Isaac: Mean = (193 + 178) 2
= 185.5 cm
Aiden and Chaz: Mean = (193 + 166) 2
= 179.5 cm
Jake and Isaac: Mean = (187 + 178) 2
= 182.5 cm
Jake and Chaz: Mean = (187 + 166) 2
= 176.5 cm
Isaac and Chaz: Mean = (178 + 166) 2
= 172 cm

d Mean = (190 + 185.5 + 179.5 + 182.5 + 176.5 + 172) 6

= 1086 6
= 181 cm

What can be seen here is that the mean of each possible sample of two is equal to the mean of the entire
population.
This demonstrates an important result:
The mean of all possible samples taken will be the mean of the entire population.
In a wider context this demonstrates that a repeated sample will increasingly reflect the mean of the
population that we are attempting to sample.

Exercise 5B

Samples and means

1 WE4 Consider a group of three people. The number of

televisions owned in each household is recorded.


Erin 3Tegan 5Madeline 1
a Calculate the mean number of televisions per household.
b A sample of two from the three is to be chosen. List all
possible samples of two that could be selected.
c Calculate the mean of each possible sample of two.
d Calculate the mean of all samples taken.
118 Maths Quest HSC Mathematics General 2

2 In a family of five children the number of days off from school each had

in a year is recorded.
Dylan 10, Lachlan 16, Connor 21, Cameron 5, and Aiden 1.
a Calculate the mean number of days off from school.
b How many samples of two can be chosen from the five boys?
c Calculate the mean of each possible sample of two.
d Show that the mean of each possible sample is equal to the mean
number of days off found in part a.
3 A group of five students each have their monthly mobile phone spend
recorded.
Lane $215.00 Maya $154.00 Elly $119.00 Meridee $98.00
Maddy $54.00
a Calculate the mean spend among the five girls.
b Calculate the number of samples of three girls that could be selected.
c List each sample of three and find the mean of each sample.
d Verify that the mean of each possible sample is equal to the mean
mobile phone spend found in part a.
4 The figures below show the number of people that live in 20 homes in a particular street.
House No.

Number of residents

House No.

Number of residents

1
2
3
4
5
6
7
8
9
10

2
5
6
2
1
6
4
2
3
4

11
12
13
14
15
16
17
18
19
20

4
4
5
6
5
3
2
2
3
4

a Display the results in a frequency table.


b Use the frequency table to calculate the mean number of residents per

household.
Use your calculator to randomly select three households from the street.
Find the mean of your sample of three and compare this to the mean found in part b.
Randomly select three more samples of three and find the mean of each sample.
What was the greatest and least of the four sample means found?
Find the mean of the four sample means and compare the result to the mean found in part b.
5 Craig is a big fan of horse racing. During the Melbourne Cup carnival he records the number of
horses that run in the 38 races during the week.
c
d
e
f
g

Chapter 5 Sampling and populations 119

The results are given below.


101216814119161511161614151624161213
1211161112169101112151613141516151213
a Calculate the mean number of horses per race.
b Randomly select a sample of two races and find the mean number of horses in these two races.
c Randomly select a sample of three races and find the mean number of horses in these three races.
d Randomly select a sample of four races and find the mean number of horses in these four races.
e Randomly select a sample of five races and find the mean number of horses in these five races.
f Compare each of the means obtained in parts b to e with the mean obtained in part a.
g Is there any pattern apparent as the sample size is increased?
h In part e you randomly selected a sample of five races. Find the mean of the other 33 races not
selected and compare this result with that obtained in part a.

5C

Estimating populations

Statistics are used to estimate populations


that are too difficult to count accurately.
An example of this is the number of fish
in a lake. The method used to estimate
the population is the capturerecapture
technique.
In this technique, a sample of the
population is captured, tagged and then
released back into the population. A
second sample is then captured. The
percentage of the second sample that
have been previously tagged (that is,
recaptured) becomes an estimate of the
percentage of the entire population that
has been tagged.
INVESTIGATE: Estimating a population

Make a large collection of marbles, jelly beans, matches or toothpicks.


1. Select a sample of these items and identify/tag them with a dab of liquid paper.
2. Put those selected back in with the others and mix them thoroughly.
3. Select another sample and see how many are tagged.
4. Estimate the total number.
5. Put those selected back and repeat the process to see if you get a similar answer.
6. Count the number of items to see how accurate your estimates were.
WORKED EXAMPLE 5

A group of marine biologists want to estimate the fish population in a lake. To do this, they catch
100 fish, tag them and release them back into the lake. The next day, they catch 200 fish and find
that 10 of them have been previously tagged. What would be their estimate of the population?
THINK

WRITE

Calculate the percentage of the second sample


that has been tagged.

10
100%
Percentage tagged = 200
= 5%

Use this percentage to calculate what 100% of


the population would be.

5% of population = 100
so1% of population = 20
and100% of population = 2000

Give a written answer.

Their estimate of the fish population in the lake


is2000.

120 Maths Quest HSC Mathematics General 2

Exercise 5C

Estimating populations

1 WE5 The fish population of a river is to be estimated. A sample of 400 fish are caught, tagged and

released. The next day another sample of 400 fish are caught and 40 of them have tags. Estimate the
fish population of the river.
2 A colony of bats live near a school. Wildlife officers try to estimate the bat population by catching

60bats and tagging them. These bats are then released and another 60 are caught, 9 of which had
tags. Estimate the size of the bat population living near the school.
3 A rivers fish population is to be estimated. On one day 1000 fish are caught, tagged and released.

The next day another 1000 fish are caught. Estimate the population of the river if in the second
sample of fish:
a 100 had tags
b 40 had tags
c 273 had tags.
4 A certain fish population is said to be endangered if the population falls below 15000. A sample

of 1000 fish are caught, tagged and released. The next day another sample of 1200 fish are caught,
60of which had tags. Is the fish population endangered?
5 To estimate the fish population of a lake, 300 fish were caught. These 300 fish (150 trout, 100 bream

and 50 perch) were tagged and released. A second sample of fish were then caught. Of 100 trout,
24had tags; of 100 bream, 20 had tags; and of 100 perch, 8 had tags.
a Estimate the number of trout in the lake.
b Estimate the number of bream in the lake.
c Estimate the number of perch in the lake.
6 The kangaroo population in a national park is to be estimated. On one day, 100 kangaroos were

caught and tagged before being released.


(Note: For each sample taken, the kangaroos
are released after the number with tags is
counted.)
a The next day 100 were caught, 12 of
which had tags. Estimate the population.
b The following day another estimate was
done. This time 200 were caught and 20
had tags. Estimate the population again.
c A third estimate was done by catching 150
and this time 17 had tags. What will the
third estimate for the population be?
d For a report, the average of the three
estimates is taken. Calculate this average.

Further development
7 MC The table below shows the results of a capturerecapture fish population estimate in four lakes.

Lake
Alpha
Bravo
Charlie
Delta

Number caught and


tagged in initial capture

Number caught in
second capture

Number tagged in
second capture

100
200
100
200

100
200
200
100

2
5
3
1

Which lake would be estimated to have the greatest population?


A Alpha
B Bravo
C Charlie
D Delta
8 In two lakes, a sample of 100 fish are captured and tagged. Kylie claims that when the second

population is captured the more that are tagged the greater the population. Is Kylie correct? Explain
your answer.
Chapter 5 Sampling and populations 121

9 Below are the steps for completing a capturerecapture task. Place the steps a, b, c and d in the

interactivity
int-2790
Compare statistical
reports

Digital doc
WorkSHEET 5.2
doc-11044

correct order.
a Capture a second sample.
b Capture and tag a sample of the population.
c Write the population as a percentage of the whole and use the unitary method to find the
population.
d Return the tagged samples to the general population.
10 Explain how a method similar to capturerecapture could be used to estimate a static population such
as trees in a national forest.
11 A lake is known to be populated with flathead and whiting. To estimate the population of each the
following capturerecaptures are performed.
Flathead: 100 are caught, tagged and released. In the second capture of 200 fish 4 are tagged.
Whiting: 200 are caught, tagged and released. In the second capture of 100 fish 4 are tagged.
Is the population of flathead or whiting greater?
12 Two bird populations are to be estimated.
In population A, 500 birds are tagged and released. A second capture of 200 population A birds
ismade.
In population B, 400 birds are tagged and released. A second capture of 100 population B birds
ismade.
Given that in both second captures the same numbers of tagged birds are found, which population is
greater and how many times greater is this population than the other?

122 Maths Quest HSC Mathematics General 2

Summary
Sampling

There are three types of sample:


Random sample where each participant is chosen by luck
Stratified sample where the participants are chosen such that the sample
has the same characteristics as the entire population
Systematic sample is where a particular method is used to select the
sample.
When selecting a stratified sample the participants within each strata (or
group) should be selected randomly.

Exploring samples
andmeans

When collecting information it is more practical to take a sample rather


than survey the whole population.
If we take all possible samples from a data set and then find the mean
of each sample, the mean of these samples will be the mean of the
population. This shows that if we repeat the investigation with a different
sample the results will be more accurate.
The larger the sample size the closer we can expect the mean to be to the
population mean.

Estimating populations

The capturerecapture method is used to estimate populations that cannot


be accurately counted.
A sample is caught, tagged and released back into the population.
A second sample is then captured and the percentage tagged is an estimate
of the percentage in the whole population that has been tagged.
The unitary method is then used to estimate the entire population.

Chapter 5 Sampling and populations 123

Chapter review
MULTIPL E
C HOICE

1 A television ratings survey is conducted by surveying house number 1 in every street in a particular

suburb. This is an example of what type of sample?


Random sample
Systematic sample
Stratified sample
Stratified random sample

A
B
C
D

2 When taking a sample, which of the following could be used to try and ensure the mean of the

sample is reflects the mean of the entire sample.


A Repeat the survey using several different samples.
B Taking a larger sample
C Neither A or B
D Both A and B
3 A lake has a population of fish. A sample of 50 fish is taken, tagged and released. A second

sample of 50 fish is caught and it is found that only 2 fish are tagged. The best estimate of the fish
population in the lake is
A 200
B 250
C 1250
D 2000
Sh ort
a nswer

1 In each of the following state the type of sample that has been used.
a George is collecting statistics on Rugby League games. He samples the matches that are shown

on television every Friday night.


b Veronica is researching the height of buildings in a country town. She selects 20 addresses by

opening the phone book and putting a pin in the page.


c James is researching the number of hours the employees of a company do each week. He selects

equal numbers of men and women.


2 The table below shows the makes of cars in a car yard.

Make

Number of cars

Holden
Ford
Mazda
Toyota

75
40
50
85

A sample of 20 cars is to be inspected. If a stratified sample is to be used how many cars of each
make should be selected?
3 Explain how to use a calculator to randomly select a number between 1 and 700.
4 Consider a group of four people. We are examining the average weight of these four people. The four

people are:
Luke 101 kg Lewis 92 kg Max 81 kg Jamie 68 kg
a Calculate the mean weight of the four boys.
b A sample of three from the four is to
be chosen. List all possible samples of
three that could be selected.
c Calculate the mean of each possible
sample of three.
d Calculate the mean of all samples taken.
5 Kimberley has a worm farm. To estimate

the population of her farm, she catches


150 worms and tags them before
releasing them. The next day, she catches
120worms and finds that 24 of them
havetags. Estimate the population of the
worm farm.
124 Maths Quest HSC Mathematics General 2

6 A sample of 200 fish are caught, tagged and released back into the

population. Later Barry, Viet and Mustafa each catch a sample of fish.
Barry caught 40 fish and 3 had tags.
Viet caught 75 fish and 9 had tags.
Mustafa caught 55 fish and 7 had tags.
a Find the estimate of the population that each would have calculated.
b Give an estimate for the population, based on all three samples.

1 Consider a population of six people. Rick rolls a die to choose

Ex tended
R es p ons e

two people to form a sample.


a What type of sample did Rick use?
b It is known that the mean age of the six people is 34. Ricks sample
has a mean age of 22. What could Rick do to try and ensure that his
data better reflects the population mean?
c How many samples of three people from six would it be possible to
choose?
d Explain why Rick would get more accurate results with a sample of
three people.
2 Carolyn is a marine biologist. She spends the day on a boat and 500fish
are netted. Carolyn notes the types of fish netted. There are 173 blackfish,
219 drummer and 108 mullet.
a Are Carolyns data categorical or quantitative?
b The fish are tagged and released back into the school from which they
were caught. Another 250 are then caught and it is noted that 63 have
tags. Estimate the population of the school.

Digital doc
doc-11045
Test Yourself
Chapter 5

Chapter 5 Sampling and populations 125

ICT activities
5A Sampling
interactivities
int-0219: Classifying data. (page 115)
int-0089: Random number generator. (page 116)
Digital doc
WorkSHEET 5.1 (doc-11043): Apply your knowledge of samples and
populations to problems. (page117)

5C Estimating populations
interactivity
int-2790: Compare statistical reports. (page 122)
Digital doc
WorkSHEET 5.2 (doc-11044): Apply your knowledge of samples and
populations to problems. (page 122)

126 Maths Quest HSC Mathematics General 2

Chapter review
Test Yourself (doc-11045): Take the end-of-chapter test to test your
progress. (page 125)

To access eBookPLUS activities, log on to www.jacplus.com.au

Answers chapter 5
Sampling and Populations
Exercise 5A

Sampling

1 a systematic
b random
c random
d stratified
2 a Stratified
b random
c systematic
d systematic
e stratified
3 130
4 Check with your teacher.
5 1
6 26 men and 24 women
7 1 stallion, 14 mares and 10 foals
8 3 managers, 2 coaching staff, 5 support

staff and 10 players


9 a Early arrivals are possibly the most
enthusiastic people.
b Night shift presents other problems that
other shifts may not have and hence the
survey is not balanced.
c The office staff is only one section of
the business and so the survey will not
consider the issues in other areas.
10 A random sample is where the participants
are only chosen by luck. The sample
should therefore be free from bias and be
representative of the entire population.
Exercise 5B

Samples and means

1 a 3
b Erin Tegan, Erin Madeline,

Tegan Madeline

c Erin Tegan Mean = 4, Erin

Madeline Mean = 2, Tegan Madeline


Mean = 3
d 3
2 a 10.6
b 10
c The 10 means are 13, 15.5, 7.5, 5.5,
18.5, 10.5, 8.5, 13, 11, 3
d Check with your teacher.

3 a $128
b 10
c The 10 means are $162.67, $155.67,

$141, $144, $129.33, $122.33, $123.67,


$109, $102, $90.33
d Check with your teacher.
4 a Number of
Frequency
residents (x)
(f)
fx
1

10

20

15

18

f = 20 f x = 73
b Mean = 3.65
cg Check with your teacher.
5 a 13.5
bg Check with your teacher.
Exercise 5C

as a percentage of the total area and the


population could then be found using the
unitary method.
11 The populations are approximately equal.
12 Population A is greater by a factor of 2.5.

Chapter Review
Multiple Choice

1 B
2 D
3 C
Short Answer

1 a Systematic
b random
c stratified
2 Holden 6, Ford 3, Mazda 4,

Toyota 7

3 Check with your teacher


4 a 85.5
b Luke Lewis Max, Luke Lewis

Estimating populations

1 4000
2 400
3 a 10000
b 25000
c 3663
4 No estimated population 20000
5 a 625
b 500
c 625
6 a 833
b 1000
c 882
d 905
7 D
8 Kylie is incorrect. The fewer tagged fish

means that the 100 with tags represent


a smaller percentage of the population;
hence the overall population must be
greater.
9 b d a c
10 An area of the forest could be cordoned
off, its area measured and tree population
counted. This area would then be found

Jamie, Luke Max Jamie, Lewis


Max Jamie
c 91.3, 87, 83.3, 80.3
d 85.5
5 750
6 a Barry 2667
Viet 1667
Mustafa 1571
b 1968
Extended Response

1 a random
b Repeat the experiment a number of

times.

c 20
d A larger sample will usually obtain

results closer to the population mean.

2 a Categorical
b 1984

Chapter 5 Sampling and populations 127

Chapter 6

Further applications of
area and volume
CHAPTER CONTENTS
6A Relative error
6B Area of parts of the circle
6C Area of composite shapes
6D Simpsons rule
6E Surface area of some prisms
6F Surface area of cylinders and spheres
6G Volume of pyramids, cones and spheres
6H Volume of composite solids
6I Error in measurement

6A

Relative error

How far is it from your house to school? If you live very close to school you may give your answer
in metres, or if you are a bit further away you would probably answer in kilometres. In either case the
answer you give would not be exact. In fact, no measurement is exact. We are able to measure a quantity
only to the degree of accuracy that our instruments allow.
All measurements are approximations.
In practice, we usually choose a degree of accuracy that is convenient. For example, you may
say that it is 4km from your home to school. In this case, the measurement would be given to the
nearestkilometre. The actual distance from your home to school could be anything between 3.5km
and 4.5km.
Someone who lives close to school may say it is 600m to school. This measurement is probably given
to the nearest 100m. The actual distance would be between 550m and 650m.
When rounding off measurements, the maximum error possible is half the degree of accuracy stated.
WORKED EXAMPLE 1

Hilary has her height measured at 164cm. This measurement is given to the nearest centimetre.
Between what values would her actual height be?
THINK

WRITE

The maximum error is half the degree of


accuracy used (1cm).

Maximum error = 0.5cm

Subtract 0.5cm from 164cm to find the


smallest possible measurement.

Lower limit = 164 0.5


= 163.5cm

Add 0.5cm to find the largest possible


measurement.

Upper limit = 164 + 0.5


= 164.5cm

Give a written answer.

Hilarys height is between 163.5cm and 164.5cm.

Chapter 6 Further applications of area and volume 129

WORKED EXAMPLE 2

The mass of a trailerload of soil is given as 260kg. The mass is given to the nearest 10kg.
Between what two masses would the true mass of the trailer load actually be?
THINK

WRITE

The maximum error is half the


degree of accuracy used (10kg).

Maximum error = 5kg

Subtract 5kg from 260kg to find


the smallest possible measurement.

Lower limit = 260 5


= 255kg

Add 5kg to 260kg to find the


largest possible measurement.

Upper limit = 260 + 5


= 265kg

Give a written answer.

The soils mass is between 255kg and 265kg.

The true degree of accuracy in a measurement is found by determining the maximum percentage error.
To do this we find the greatest possible error and then write that as a percentage of the measurement
given.
Percentage error =

maximum error
100%
measurement

WORKED EXAMPLE 3

A cars fuel tank has a capacity of 65 litres. If this capacity is given to the nearest litre, find the
degree of accuracy as a percentage (answer correct to 2 decimal places).
THINK

WRITE

Find the maximum error. This is half


the degreeof accuracy used (1 L).

Maximum error = 0.5 L

Write the percentage error rule.

Percentage error =

Calculate the percentage error to


2decimalplaces.

Give a written answer.

The degree of accuracy is 0.77%.

0.5
100%
65
= 0.77%

The degree of accuracy as a percentage shows how accurate a measurement is.


No measuring instrument is perfect and different people can sometimes obtain a different reading
from the same instrument. For example, one person may measure a persons height as 162cm while
another may get an answer of 163cm. To reduce the likelihood of error, we can average several readings
of the same measurement.
WORKED EXAMPLE 4

Taylor has her height measured by 8 people. They obtain the following results:
169cm, 169cm, 168cm, 170cm, 169cm, 169cm, 168cm, 168cm.
What is the average result?
THINK

WRITE

Find the total of the 8 readings.

Total = 169 + 169 + 168 + 170 + 169 + 169 + 168 + 168



= 1350

Divide the total by 8 to find the


average.

Average = 1350 8

= 168.75cm

130 Maths Quest HSC Mathematics General 2

Exercise 6A

Relative error

1 WE1 Each of the following measurements are given to the nearest centimetre. State the

limitsbetween which the true length lies.


a 5cm
b 12cm
d 59cm
e 90cm

c 34cm
f 2m

2 WE2 Each of the following measurements are given to the nearest 10m. State the limits

betweenwhich the true length lies.


a 40m
b 90m
d 300m
e 1000m

c 250m
f 2km

Digital doc
SkillSHEET 6.1
doc-11046
Writing one
quantity as a
percentage of
another

3 Each of the following measurements are given correct to 1 decimal place. State the limits

betweenwhich the true length lies.


a 5.3cm
b 9.8m
d 5.0mm
e 9.9km

c 7.2km
f 0.1m

4 WE3 The distance between two towns is given as 45km, correct to the nearest kilometre.
a Between what two limits does the true distance lie?
b What is the greatest possible error in the distance?
c Write the degree of accuracy in this measurement as a percentage, correct to

Digital doc
EXCEL Spreadsheet
doc-1461
Calculations with
percentages

2 decimal places.
5 For each of the following measurements, write the degree of accuracy as a percentage, correct

to2decimal places.
a A persons mass is given as 67kg, correct to the nearest kilogram.
b The capacity of a bucket is 7 L, correct to the nearest litre.
c The length of a park is said to be 180m, correct to the nearest 10m.
d The volume of water in a tank is 38.6 L, correct to 1 decimal place.
e The distance between Sydney and Melbourne is 1000km, correct to the nearest 100km.
6 a Measure each of the following lines to the nearest centimetre.
i ________________________
ii ____________________
iii _____________________________
b For each, find the maximum percentage error in the measurement, correct to

2 decimal places.
7 a Measure each of the lines in question 6, correct to the nearest millimetre.
b When the lines are measured correct to the nearest millimetre, what is the maximum

percentageerror for each line?


8 Janice estimates that it takes 1 hour and 20 minutes to drive to a friends house.
a If Janices estimate is to the nearest 10 minutes, between what two lengths of time does it

takeJanice to make the journey?


b What is the maximum error in Janices estimate?
c Find the degree of accuracy of Janices estimate, as a percentage, correct to 2 decimal places.
9 MC The length of a house, correct to 2 decimal places, is given as 19.95m. Which of the

followingstatements is the most accurate?


A The length of the house is between 19.9m and 20m.
B The length of the house is between 19.945m
and 19.955m.
C The length of the house is between 19.94m
and 19.96m.
D The length of the house is between 19.955m
and 20m.
10 MC The world record for 100m is 9.58 s. This is

the time correct to 2 decimal places. What is the


maximum possible error in this time?
A 0.5 s
B 0.05 s
C 0.005 s
D 0.1 s
Chapter 6 Further applications of area and volume 131

11 MC An aeroplane trip takes 17 hours, correct to the nearest hour. The degree of accuracy, correct

to2 decimal places, is:


A 0.29%
C 2.94%

B 1.76%
D 5.88%

12 MC Which of the following four measurements has the greatest degree of accuracy?
A
B
C
D

The mass of a bag of beans is 400 g, correct to the nearest 50 g.


A water container contains 10 L of water, correct to the nearest litre.
The distance from home to school is 1.6km, correct to 1 decimal place.
A roast is to cook for 1 hour and 30 min, correct to the nearest 5 minutes.

13 A paddock needs a fence which is 30m long and 20m wide, when measured to the

nearest metre.
a Between what two measurements does the length lie?
b Between what two measurements does the width lie?
c What is the smallest possible length of fencing needed?
d What is the greatest possible length of fencing needed?
e What is the maximum error in the length of fencing needed?
14 For the paddock in question 13, the area is found by multiplying the length by the width.
a By multiplying the smallest possible length and width, find the smallest possible area.
b By multiplying the largest possible length and width, find the largest possible area.
c What is the maximum error in the area of the paddock?
15 WE4 The capacity of a jug is measured by 5 people to be 750mL, 752mL, 749mL, 753mL and

748mL. Calculate the average of these 5 readings.


16 The distance between two towns is given on 4 different maps as 79km, 81km, 77km and 80km.

Calculate the average of these 4 readings.

Further development
17 The mass of a gold nugget is found to be 230 grams. Find the percentage error (correct to one

decimal place) if the measurement is correct to:


a two significant figures
b three significant figures
c four significant figures.
18 Australias richest professional foot race is the Stawell Gift run over 120m. In 2010 the track was

incorrectly measured as being 3.2 metres too long.


a Calculate the percentage error in the measurement, correct to one decimal place.
b Travis claims that an error of 3.2 metres too short is the same as an error of 3.2 metres too long.
Explain whether or not Travis is correct.
19 A measurement that is taken to the nearest centimetre is known to have a maximum percentage error

of 4.55%. What is the given measurement length?


20 Jonas claims that a measurement correct to one decimal place is ten times more accurate than a

measurement taken to the nearest whole number. By considering a measurement of 5.0 metres,
determine if Jonas is correct.
21 The length of a rectangle is given as 7 metres and breadth is given as 4 metres correct to the

nearestmetre.
a Find the area of the rectangle.
b What are the smallest possible length and breadth of the rectangle?
c What is the smallest possible area of the rectangle?
d Find the percentage error in the area of the rectangle if these measurements are used.
e What are the largest possible length and breadth of the rectangle?
f Find the largest possible area of the rectangle.
g What is the percentage error in the area of the rectangle if these measurements
are used?
h What is the greatest possible percentage error in the area of the rectangle?
22 Find the maximum possible percentage error in the area of a rectangle with a length of 4.8cm and

abreadth of 5.2cm.
132 Maths Quest HSC Mathematics General 2

INVESTIGATE: Measuring heights

In your class, get each person to measure the height of one class member. Each person writes down their
reading, without showing it to anyone else. When all readings are taken, answer the followingquestions.
1. What unit did you use to measure height?
2. What is the maximum error?
3. Calculate the percentage error using your reading.
4. What was the smallest height measured?
5. What was the greatest height measured?
6. Calculate the average reading.

6B

Area of parts of the circle

From previous work you should know that the area of a circle can be calculated using the formula:
A = r2
WORKED EXAMPLE 5

Calculate the area of a circle with a radius of 7.2cm. Give your answer correct to 2 decimal places.
THINK

WRITE

Write the formula.

A = r2

Substitute for the radius.

= (7.2)2

Calculate the area.

= 162.86cm2

A sector is the part of a circle between two radii and an arc as shown on the right.
To calculate the area of a sector we find the fraction of the circle formed
by the sector. For example, a semicircle is half of a circle and so the area
of a semicircle is half the area of a full circle. A quadrant is a quarter
of a circle and so the area is quarter that of a full circle.
For other sectors the area is calculated by using the angle between the
radii as a fraction of 360 and then multiplying by the area of the full
circle. This can be written using the formula:
A=

2
r
360

where is the angle between the two radii.


WORKED EXAMPLE 6

Calculate the area of the sector drawn on the right. Give your answer
correct to 1 decimal place.

5 cm
80
THINK

WRITE

A=

r2
360

Write the formula.

Substitute for and r.

80
52
= 360

Calculate the area.

= 17.5cm2

Chapter 6 Further applications of area and volume 133

An annulus is the area between two circles that have the same centre (i.e. concentric circles). The area
of an annulus is found by subtracting the area of the smaller circle from the area of the larger circle. This
translates to the formula A = (R2 r2), where R is the radius of the outer circle and r is the radius of the
inner circle.
WORKED EXAMPLE 7

Calculate the area of the annulus on the right. Give your answer
correct to 1 decimal place.
Tutorial
int-2411
Worked example 7

5.7 cm
3.2 cm

THINK

WRITE

Write the formula.

A = (R2 r2)

Substitute R = 5.7 and r = 3.2.

= (5.72 3.22)

Calculate.

= 69.9cm2

Exercise 6B

Area of parts of the circle

1 WE5 Calculate the area of the following circle, correct to 1 decimal place.
Digital doc
SkillSHEET 6.2
doc-11047
Area of a circle

6.4 cm

2 Calculate the area of each of the circles drawn below, correct to 2 decimal places.
a

b

c
33 mm
9 cm

7.4 m

d
26.5 cm

3.84 m

f
6.02 m

3 Calculate the area of a circle that has a diameter of 15m. Give your answer correct

to 1 decimal place.
4 WE6 Calculate the area of the sector drawn on the right. Give your answer correct

to 1 decimal place.
7.2 m

134 Maths Quest HSC Mathematics General 2

5 Calculate the area of each of the sectors drawn below. Give each answer correct to 2 decimal places.
a

b

c

5.2 cm

135

60

23 m
74 mm
20

9.2 mm

f
39 mm

240

150

19.5 m

72

6 Calculate, correct to 1 decimal place, the area of a semicircle with a diameter of 45.9cm.
7 WE7 Calculate the area of the annulus shown at right, correct to

12 cm

1 decimal place.

6 cm

8 Calculate the area of each annulus drawn below, correct to 3 significant figures.
a

b

c
9.7
m
20 cm
77 mm
18 cm

13 mm

4.2 m

9 A circular garden of diameter 5m is to have concrete laid around it. The concrete is to be 1m wide.
a What is the radius of the garden?
b What is the radius of the concrete circle?
c Calculate the area of the concrete, correct to 1 decimal place.
10 MC The area of a circle with a diameter of 4.8m is closest to:
A 15m2

B 18m2

C 36m2

D 72m2

11 MC Which of the following calculations will give the area of the sector

shown on the right?


1
A 42
8

42

1
4

1
8
1
4

82
82

45
8m

Further development
12 A circular area is pegged out and has a diameter

of 10m.
a Calculate the area of this circle, correct to
1 decimal place.
b A garden is to be dug which is 3m wide around
the area that has been pegged out. Calculate the
area of the garden to be dug. Give your answer
correct to 1 decimal place.
c In the garden a sector with an angle of 75 at the
centre is to be used to plant roses. Calculate the
area of the rose garden, correct to 1 decimal place.
Chapter 6 Further applications of area and volume 135

13 The area in front of a building is rectangular in shape, measuring 50 metres by 15 metres. At night

a security light scans the area. The security light, if positioned vertically against the wall at ground
level, illuminates an area of the wall that is a sector of a circle of radius 15m and has an angle of 60
at the centre.
a What is the area of the rectangular frontage?
b What is the area that is illuminated at any one time by the security light? Give your answer correct
to 1 decimal place.
c What percentage of the frontage is illuminated at any one time? Give your answer correct to
1decimal place.
14 A circular island is in the centre of a circular lake such that the surface of the water in the lake forms

an annulus. The radius of the lake is 10m greater than the radius of the island.
a Given that the island has a radius of 20m, find the area of the surface of the water. Give your
answer correct to the nearest square metre.
b Tori claims that the surface area of the lake will remain the same regardless of the two radii as
long as the difference of 10metres remains unchanged. Is Tori correct? Use calculations to justify
your response.
15 An arc of length 1cm stands on a circle of radius 1cm. Find the size of the angle subtended at the

centre correct to the nearest degree.

6C
Interactivity
int-0005
Area
int-2350
Area of a triangle

Area of composite shapes

A composite shape is a shape that is made up of two or more regular shapes. The area of a composite
shape is found by splitting the area into two or more regular shapes and calculating the area of each
separately before adding them together. In many cases it will be necessary to calculate the length of a
missing side before calculating the area. There will sometimes be more than one way to split the
composite shape.

WORKED EXAMPLE 8

Find the area of the figure on the right.

6 cm
18 cm
10 cm
12 cm

THINK
1

WRITE

Copy the diagram and divide the shape into two rectangles.

6 cm
A1 8 cm
18 cm
A2

10 cm

12 cm
2

Calculate the length of the missing side in rectangle 1.


(Writethis on the diagram.)

18 10 = 8cm

Calculate the area of rectangle 1.

A1 = 6 8
= 48cm2

Calculate the area of rectangle 2.

A2 = 10 12
= 120cm2

Add together the two areas.

Area = 48 + 120

= 168cm2

136 Maths Quest HSC Mathematics General 2

Composite areas that involve triangles may require you to also make a calculation using Pythagoras
theorem.

WORKED EXAMPLE 9

Find the area of the figure on the right.

13 m

10 m

Tutorial
int-2412
Worked example 9

24 m
THINK
1

WRITE

Draw the triangle at the top and cut the isosceles


triangle in half.

13 m
a
12 m

a2 = c2 b2
= 132 122
= 169 144
= 25
a = !25
= 5m

Calculate the perpendicular height using


Pythagoras theorem.

Calculate the area of the triangle.

A = 12 24 5
= 60m2

Calculate the area of the rectangle.

A = 24 10
= 240m2

Add the two areas together.

Area = 60 + 240

= 300m2

Composite areas can also be calculated by using subtraction rather than addition. In these cases we
calculate the larger area and subtract the smaller area in the same way as we did with annuluses in the
previous section.

WORKED EXAMPLE 10

Find the shaded area in the figure on the right.


6 cm

20 cm

30 cm
THINK

WRITE

Calculate the area of the rectangle.

A = 30 20
= 600cm2

Calculate the area of the circle.

A = 62
= 113.1cm2

Subtract the areas.

Area = 600 113.1



= 486.9cm2

Chapter 6 Further applications of area and volume 137

Area of composite shapes

Exercise 6C

4m

1 WE8 Copy the figure on the right into your workbook and calculate

its area by dividing it into two rectangles.


Digital doc
SkillSHEET 6.3
doc-11048
Areas of squares,
rectangles and
triangles

18 m
11 m
20 m
2 Find the area of each of the figures below. Where necessary, give your answer correct to 1 decimal place.
a
b
c
5 cm
7 cm
18 cm

19 cm

12 cm

16 cm
25 cm

6 cm
40 cm

5 cm
22 cm

12 cm

4 cm

8 cm

8 cm

4 cm
4 cm

16 cm

3 Look at the triangle on the right.


a Use Pythagoras theorem to find the perpendicular height of
Digital doc
SkillSHEET 6.4
doc-11049
Using Pythagoras
theorem

10 cm

17 cm

the triangle.
b Calculate the area of the triangle.

6 cm

4 Below is an isosceles triangle.

15 cm

8m

12 m
a Use Pythagoras theorem to find the perpendicular height of the triangle, correct to 1 decimal place.
b Calculate the area of the triangle.
5 Calculate the area of each of the triangles below. Where necessary, give your answer correct to

1decimal place.
Digital doc
EXCEL Spreadsheet
doc-1306
Pythagoras


26 m

25 cm

48 cm
24 m
124 mm

Digital doc
GC program
Casio
doc-1307
Mensuration

6 WE9 Find the area of each of the composite figures drawn below.
a

b
c
25 mm
13 cm
17 m
15 mm
13 m

12 cm
30 m

138 Maths Quest HSC Mathematics General 2

52 mm
48 mm
54 mm

7 MC The area of the composite figure on the right is closest to:


A
B
C
D

139m2
257m2
314m2
414m2

10 m

8 MC The area of the figure drawn on the right is:


A
B
C
D

36m2
54m2
72m2
144m2

12 m

6m
9 A block of land is in the shape of a square with an equilateral triangle on top. Each side of the block

of land is 50m.
a Draw a diagram of the block of land.
b Find the perimeter of the block of land.
c Find the area of the block of land.
10 WE10 In each of the following, find the area of the shaded region. Where necessary, give your
answer correct to 1 decimal place.
a

b
c
12 m
4 cm

10 cm

9 cm
3 cm

16 cm

8m

9 cm

112 mm

cm
1
7.

3.1 m

1.9 m
7.4 m

36 mm
40 mm

10 cm
95 mm

11 An athletics track consists of a rectangle with two semicircular ends. The dimensions are shown in

the diagram below.


70 m
90 m

82 m

The track is to have a synthetic running surface laid. Calculate the area which is to be laid with the
running surface, correct to the nearest square metre.
12 A garden is to have a concrete path laid around it. The garden is rectangular in shape and measures
40m by 25m. The path around it is to be 1m wide.
a Draw a diagram of the garden and the path.
b Calculate the area of the garden.
c Calculate the area of the concrete that needs to be laid.
d If the cost of laying concrete is $17.50 per m2, calculate the cost of laying the path.

Further development
13 Find the area of each of the following figures.
a
b
10 cm

c
20 m

30 cm

25 cm

20 cm
40 m
4 cm
24 cm

25 cm

Chapter 6 Further applications of area and volume 139

12 cm

mm

cm

32

12

c
15

76

f
6 cm

40 cm
14 Convert the following areas to the units given in brackets.
a 20000mm2 (cm2)
b 3500000cm2 (m2)
2
2
c 0.005m (cm )
d 0.035m2 (mm2)
2
2
e 13400m (km )
f 375000m2 (hectares)
2
2
g 2750000000mm (m )
h 0.043km2 (m2)
15 Find the area of the regular hexagon shown in the diagram below.

0m

2.4

4.16 m
16 A cutting blade for a craft knife has the dimensions shown in the diagram. What is the area of steel

in the blade (to the nearest mm2)?

20 mm

32 mm
5 mm
38 mm

17 Emma left her horse tied to a railing in a paddock while she chatted to a friend.
a The horse is tied to one end of the railing as shown below.

Railing

2m
2m

Find the area that the horse has access to correct to 1 decimal place.
b The horse is now tied to the centre of the railing as shown below.
r=1m
r=2m
r=1m

Find the area that the horse now has access to.
140 Maths Quest HSC Mathematics General 2

18 Find the areas of the following figures (to 1 decimal place).


a

b

16 m
14 m
12 m

19 m

4.5 m

2m

95 mm

32 cm
m
c
16 cm
20

24

cm

11 m

17 m

6.5

80 mm

20 cm

24 mm

14 m

5.5 m

30 m

42 cm

22 m

25 m

d 36 mm

3.5 m
10 m

6D

Simpsons rule

Simpsons rule is a method used to approximate the area of an irregular


figure. Simpsons rule approximates an area by taking a straight boundary
and dividing the area into two strips. The height of each strip (h) is
measured. Three measurements are then taken perpendicular to the
straight boundary, as shown in the figure on the right. The formula for
Simpsons rule is:
h
A (df + 4dm + dl)
3
where h = distance between successive measurements
df = first measurement
dm = middle measurement
dl = last measurement.

dm

dl

df
h

WORKED EXAMPLE 11

Use Simpsons rule to approximate the area shown on the right.

30 m
10 m
THINK

90 m

WRITE

Calculate h.

h = 90 2 = 45

Write down the values of df , dm and dl.

df = 10, dm = 30, dl = 0

Write the formula.

h
A (df + 4dm + dl)
3

Substitute.

Calculate.

= 15 130
1950m2

45
(10 + 4 30 + 0)
3

Chapter 6 Further applications of area and volume 141

Could Simpsons rule be used to estimate the areas of these irregular shapes from nature?
Simpsons rule can be used to approximate the area of an irregular shape without a straight edge. This
is done by constructing a line as in the diagram in the worked example below and approximating the
area of each section separately.

WORKED EXAMPLE 12

Use Simpsons rule to find an approximation for the area shown on


the right.
30 m

30 m

30 m

10 m

17 m
THINK

WRITE

Write down the value of h.

h = 30

For the top area, write down the values of df , dm and dl.

df = 0, dm = 30, dl = 10

Write the formula.

h
A (df + 4dm + dl)
3

Substitute.

Calculate the top area.

10 130
1300m2

For the bottom area, write down the values of df,


dm and dl.

df = 0, dm = 17, dl = 0

Write down the formula.

h
A (df + 4dm + dl)
3

Substitute.

Calculate the bottom area.

10 68
680m2

10

Add the two areas together.

Area 1300 + 680



1980m2

30
(0 + 4 30 + 10)
3

30
(0 + 4 17 + 0)
3

Simpsons rule approximates an area, it does not give an exact measurement. To obtain a better
approximation, Simpsons rule can be applied several times to the area. This is done by splitting the
areain half and applying Simpsons rule separately to each half.
142 Maths Quest HSC Mathematics General 2

105 m
THINK

WRITE

Calculate h by dividing 105 by 4. (We are using


4 sub-intervals.)

h = 105 4
= 26.25

Apply Simpsons rule to the left half. Write the


values of df, dm and dl.

df = 32, dm = 31, dl = 24

Write the formula.

Substitute.

Calculate the approximate area of the left half.

8.75 180
1575m2

Apply Simpsons rule to the right half. Write the


values of df, dm and dl.

df = 24, dm = 29, dl = 30

Write the formula.

Substitute.

Calculate the approximate area of the right half.

8.75 170
1487.5m2

Add the areas together.

Area 1575 + 1487.5



3062.5m2

10

Exercise 6D

30 m

29 m

24 m

31 m

Use two applications of Simpsons rule to approximate the area on


the right.

32 m

WORKED EXAMPLE 13

Tutorial
int-2413
Worked
example 13

h
A (df + 4dm + dl)
3
26.25
A
(32 + 4 31 + 24)
3

h
A (df + 4dm + dl)
3
26.25
A
(24 + 4 29 + 30)
3

Simpsons rule

1 WE11 The diagram on the right

18 m

9m

approximate the area of this


section of the creek.

40 m

is of a part of a creek.
a State the value of h.
b State the value of df, dm and dl.
c Use Simpsons rule to

Digital doc
SkillSHEET 6.5
doc-11050
Substitution into
formulas

60 m

Chapter 6 Further applications of area and volume 143

16 m

0m

72 m

40 m

12 m

28 m

6m

12 m

10 m

35 m

2 Use Simpsons rule to approximate each of the areas below.


a

b

54 m

48 m
3 WE12 The irregular area on the right has been divided into two areas

80 m

5 m 18 m

7m

30 m

19 m 11 m

A1

labelled A1 (upper area) and A2 (lower area).


a Use Simpsons rule to find an approximation for Al.
b Use Simpsons rule to find an approximation for A2.
c What is the approximate total area of the figure?

A2

21 m

31 m
27 m

27 m

40 m

21 m

16 m

12 m 10 m

16 m

27 m

23 m

45 m

14 m 6 m

17 m

45 m

12 m

22 m 11 m

4 Use Simpsons rule to find an approximation for each of the areas below.
a

b

c

18 m 18 m 18 m 18 m

87 m

102 m

22 m

33 m

21 m 21 m 21 m 21 m

45 m

63 m

54 m

60 m 60 m 60 m 60 m

11 m

50 m

44 m

20 m

71 m

42 m

8 Use Simpsons rule twice to approximate each of the areas drawn below.
a

b

c

10 m 10 m 10 m 10 m

144 Maths Quest HSC Mathematics General 2

27 m

cross-section.

36 m

of land.
b Use Simpsons rule twice to obtain a better approximation for the

32 m
15 m

9 The figure on the right is of a cross-section of a waterway.


a Use Simpsons rule once to find an approximate area of this section

25 m

36 m

10 m

45 m

50 m

22 m

51 m

Simpsons rule gives an approximate area of:


A 1200m2
B 2400m2
C 3495m2
90 m
D 6990m2
6 MC If we apply Simpsons rule twice, how many measurements from the traverse line need to be
taken?
A 4
B 5
C 7
D 9
7 WE13 Use Simpsons rule twice to approximate the area
on the right.

7m

5 MC Consider the figure drawn on the right.

20 m
30 m
35 m
36 m
38 m
41 m
45 m
30 m
24 m

10 Apply Simpsons rule four times to approximate the area on the right.

9m 9m 9m 9m 9m 9m 9m 9m

Further development
11 Explain why Simpsons rule cannot be used to find the area of the figure below.

18 m

12 m
10 m

21 m

10 m

15 m

10 m

12 The figure below shows the entrance to a cave.

7m
1.8 m

1.9 m
6m

6m

a Find the area of the entrance.


b The cave is 25metres long and approximately has the same cross-section for its entire depth.

Approximate the volume of the cave.


13 The figure below shows the cross-section of a river.
15 m

15 m
8m

Find the area of the cross-section.


14 The figure below shows a semicircle of radius 12cm.

12 cm

a Find the area of the semicircle correct to the nearest cm2.


b Joe finds the area of the semicircle using Simpsons rule, taking h = 12cm, the middle value as

12cm, while the first and last values are both zero. What answer does Joe get?

c Find the percentage error in using Simpsons rule to find the area of this semicircle.
15 A botanist needs to estimate the number of trees in a certain

headland area. It is known that there are approximately 32 trees


in every 100 square metres. A diagram of the area is drawn below.

35 m
20 m

17 m
60 m

Complete the estimate of the number of trees to the nearest 10 trees.


Chapter 6 Further applications of area and volume 145

16 The figure below is a top view of an in ground swimming pool.

The pool is symmetrical and has a length of 16 metres. At its two widest points the width of the pool
is 10 metres and at the narrow point in the middle it is 5 metres wide. The pool is 2.5 metres deep at
all points.
Use Simpsons rule to estimate the volume of the pool.

6E

Surface area of some prisms

Area usually refers to the space inside a 2-dimensional shape. Surface area refers to the total area
occupied by the faces of a 3-dimensional shape. Surface area is measured in square units as are
2-dimensional area problems. In general, the surface area of a solid needs to be calculated by adding the
area of each face separately. However, for some solids there is a unique formula.

Cube
A cube has six identical faces, each of which is a square. Consider a cube of side lengths.
Each face can have its area calculated using the formula A = s2. Therefore, we have
the formula for the surface area (SA) of a cube:
SA = 6s2

WORKED EXAMPLE 14

Find the surface area of the cube at right.

4.3 cm
THINK

WRITE

Write the formula.

SA = 6s2

Substitute the side length.

= 6 4.32

Calculate the surface area.

= 110.94 cm2

Rectangular prism
Consider a rectangular prism with a length of l, a breadth of b and a
height of h.
Each pair of opposite faces are equal. Using the formula for a rectangle:
Front and back A = l h
Top and bottom A = l b
Left and right
A=bh
Adding these gives the formula for the surface area of a rectangular prism:
SA = 2(lh + lb + bh)
146 Maths Quest HSC Mathematics General 2

h
l

WORKED EXAMPLE 15

Find the surface area of the rectangular prism at right.


9.7 m
3.2 m

4.3 m
THINK

WRITE

Write the formula.

SA = 2(lh + lb + bh)

Substitute the length, breadth and height.

= 2(4.3 9.7 + 4.3 3.2 + 3.2 9.7)

Calculate the surface area.

= 173.02m2

For other prisms the surface area is found by adding the area of each face separately.
WORKED EXAMPLE 16

Find the surface area of the following


triangular prism.

13 cm
5 cm
10 cm
12 cm

THINK
1

WRITE

A = 12bh

Find the area of the triangular base.

= 12 12 5
= 30cm2
2

Find the area of each rectangular side.

A = lb
= 5 10
= 50cm2

Add the two bases (front and back) and the


three rectangles.

SA = 2 30 + 50 + 120 + 130
= 360cm2

Exercise 6E

A = lb
A = lb
= 12 10
= 13 10
= 120cm2
= 130cm2

Surface area of some prisms

1 WE14 Find the surface area of each of the cubes below.


a

b

5 cm
9 cm

32 cm

Digital doc
SkillSHEET 6.6
doc-11051
Surface area
of cubes and
rectangular prisms

2.7 cm
62 mm
2.8 m

Chapter 6 Further applications of area and volume 147

Digital doc
SkillSHEET 6.7
doc-11052
Surface area of
triangular prisms

2 WE15 Find the surface area of each of the following rectangular prisms.
a

b

c
12 cm
3m
4m
7.5 m
7m

42 cm

21 cm

3.5 m
2.5 m

4 cm
3.9 cm

4.1 cm

42 mm

20 cm
13 cm
14 cm

7 mm
7 mm

3 Oliver is making a box in the shape of a rectangular prism. The box is to be 2.5m long, 1.2m

wideand 0.8m high. Calculate the surface area of the box.


4 Calculate the surface area of an open box in the shape of a cube, with a side length of 75cm.

(Hint:Since the box is open there are only five faces.)


5 A room is in the shape of a rectangular prism. The floor is 5m long and 3.5m wide. The room

has aceiling 2.5m high. The floor is to be covered with slate tiles, the walls are to be painted blue
andthe roof is to be painted white.
a Calculate the area to be tiled.
b Each tile is 0.25m2. Calculate the number of tiles needed.
c Calculate the area to be painted blue.
d Calculate the area to be painted white.
e One litre of paint covers an area of 2m2. How many litres of paint are needed to paint
the room?
6 MC Two cubes are drawn such that the side length on the second cube is double the side length

onthe first cube. The surface area of the larger cube will be:
A twice the surface area of the smaller cube
B four times the surface area of the small cube
C six times the surface area of the small cube
D eight times the surface area of the small cube
7 WE16 Calculate the surface area of the triangular

prism below.

5 cm
4 cm
3 cm

2 cm

8 Calculate the surface area of the prism below.

3.2 m
1m
2m
6m
148 Maths Quest HSC Mathematics General 2

4m

9 Find the surface area of the cube shown at right.


10 Find the surface area of a rectangular prism with a length

of 8cm, a breadth of 5cm and a height of 6cm.

9c

11 Find the surface area of the triangular prism below.

8 cm

10 cm

20 cm
6 cm

Further development
12 Find the width of the prism in the figure below:
8m
w=?
12 m
TSA = 592 m2
13 A cardboard box is designed to contain a clothes dryer

which is 1m 1m 1m. The box is to allow an extra


5% on each side to allow the dryer to be placed in and
taken out of the box. Find the surface area of the box.

14 A bread bin has the following design.

90
cm
60 cm

Find the surface area of the bread bin.


15 The figure below shows the packaging for a chocolate bar.
Digital doc
WorkSHEET 6.1
doc-11053

ARIS-CHOC
19.5 cm

3.5 cm

Find the area of cardboard used if the area is increased by 10% for overlap.

6F

Surface area of cylinders and spheres

From earlier work you should remember that surface area is the area of all surfaces
of a 3-dimensional shape.
Consider a closed cylinder with a radius (r) and a perpendicular height (h).
The surface of the cylinder consists of two circles and a rectangle.
Area of top = r2
Area of bottom = r2
The rectangular side of the cylinder will have a length equal to the circumference
of the circle (2r) and a width equal to the height (h) of the cylinder.
Area of side = 2rh
The surface area of the closed cylinder can be calculated using the formula:
SA = 2r 2 + 2rh

interactivity
int-2782
Surface area of
asphere

Chapter 6 Further applications of area and volume 149

WORKED EXAMPLE 17

Calculate the surface area of the closed cylinder drawn on the right.
Give your answer correct to 1 decimal place.
10 cm
9 cm
THINK

WRITE

Write the formula.

SA = 2r2 + 2rh

Substitute the values of r and h.

= 2 92 + 2 9 10

Calculate the surface area.

= 1074.4cm2

For cylinders, before calculating the surface area you need to consider whether the cylinder is open or
closed. In the case of an open cylinder there is no top and so the formula needs to be written as:
SA = r2 + 2rh
Note: On the formula sheet in the exam, only the formula for the closed cylinder is provided. You will
need to check the question and adapt the formula yourself if necessary.
WORKED EXAMPLE 18

Calculate the surface area of an open cylinder with a radius of 6.5cm and a height of 10.8cm.
Give your answer correct to 1 decimal place.
THINK

WRITE

Write the formula.

SA = r2 + 2rh

Substitute the values of r and h.

= (6.5)2 + 2 6.5 10.8

Calculate the surface area.

= 573.8cm2

A sphere is a round 3-dimensional shape, and the only measurement given


is the radius (r). The surface area of a sphere can be calculated using the
formula:
SA =

4r2

WORKED EXAMPLE 19

Calculate the surface area of the sphere drawn on the right. Give the answer
correct to 1decimal place.
2.7 cm

THINK

WRITE

Write the formula.

SA = 4r2

Substitute the value of r.

= 4 (2.7)2

Calculate the surface area.

= 91.6cm2

150 Maths Quest HSC Mathematics General 2

Exercise 6F

Surface area of cylinders and spheres

1 WE17 Calculate the surface area of a closed cylinder with a radius of 5cm and a height of 11cm.

12 cm

20 cm

5 cm

1.6 m

Give your answer correct to 1 decimal place.


2 Calculate the surface area of each of the closed cylinders drawn below. Give each answer correct to
1decimal place.
a

b

c

Digital doc
SkillSHEET 6.8
doc-11054
Circumference of
acircle

1.1 m

3 cm

5.9 cm

20 cm

1.5 m

5.9 cm
r

2.3 m

r = 5 cm

3 Calculate the surface area of a closed cylinder with a diameter of 3.4m and a height of 1.8m. Give

your answer correct to 1 decimal place.


4 WE18 Calculate the surface area of an open cylinder with a radius of 4cm and a height of 16cm.
Give your answer correct to the nearest whole number.
5 Calculate the surface area of each of the following open cylinders. Give each answer correct to
1decimal place.
a

b

c

9.6 cm

r = 4.1 cm

20 cm
f

50 cm

23.2 cm
2.4 cm

3.2 m

30 cm

22 cm

13.3 cm

4m

4 cm

6 Find the outside surface area of a cylinder open at both ends with a radius of 5cm and a height

of 10cm.
7 A can of fruit is made of stainless steel. The can has a radius of 3.5cm and a height of 7cm. A label

is to be wrapped around the can.


a Calculate the amount of steel needed to make the can (correct to the nearest whole number).
b Calculate the area of the label (correct to the nearest whole number).
8 WE19 Calculate the surface area of a sphere with a radius of 3cm. Give your answer correct to the
nearest whole number.
9 Calculate the surface area of each of the spheres drawn below. Give each answer correct to 1 decimal
place.
a

b

c
8 cm

2.1 cm

14 cm

Chapter 6 Further applications of area and volume 151

1m

3.4 cm

1.8 m

10 Calculate the surface area of a sphere with a diameter of 42cm. Give your answer correct to the

nearest whole number.


11 MC An open cylinder has a diameter of 12cm and a height of 15cm. Which of the following

calculations gives the correct surface area of the cylinder?


B 2 62 + 2 6 15
D 2 122 + 2 12 15

A 62 + 2 6 15
C 122 + 2 12 15

12 MC Which of the following figures has the greatest surface area?


A
B
C
D

A closed cylinder with a radius of 5cm and a height of 10cm


An open cylinder with a radius of 6cm and a height of 10cm
A cylinder open at both ends with a radius of 7cm and a height of 10cm
A sphere with a radius of 6cm

13 An open cylinder has a diameter and height of 12cm.


a Calculate the surface area of the cylinder (correct to the nearest whole number).
b A sphere sits exactly inside this cylinder. Calculate the surface area of this sphere (correct to the

nearest whole number).

Further development
14 A cylindrical can is to contain three tennis balls each having a diameter of 6cm.
a Calculate the surface area of each ball.
b The three balls fit exactly inside the can. State the radius and height of the can.
c The can is open and made of stainless steel, except the top which will be plastic. Calculate the

area of the plastic lid (correct to the nearest whole number).


d Calculate the amount of stainless steel in the can (correct to the nearest whole number).
e Calculate the area of a paper label that is to be wrapped around the can (correct to the nearest
whole number).
15 Calculate the surface area of the hemisphere drawn below given that it is open at the base.

22 cm

Give your answer correct to the nearest hundred cm2.


16 Calculate the outside surface area of the hemisphere in question 15 if it is

closed at the base.


17 Find the surface area of the largest sphere that can be placed inside the cylinder

on the right.
18 A tennis ball has a diameter of 7cm. Calculate

the surface area of a cylindrical tennis ball canister that is


to hold four tennis balls exactly.
19 Soccer balls have a diameter of 30cm.
a The soccer ball is to be placed in the

smallest possible cubic box. Calculate the


surface area of this box.
b Calculate the percentage of material saved by
placing the ball in a spherical box of diameter
30cm. Give your answer to the nearest whole
number.
152 Maths Quest HSC Mathematics General 2

8 cm
4 cm

Computer Application 1: Minimising surface area


Access the spreadsheet Volume from the Maths Quest HSC Mathematics General 2 eBookPLUS.
A cylindrical drink container is to have a capacity of 1 litre (volume = 1000cm3). We are going to
calculate the most cost-efficient dimensions to make the container. To do this, we want to make the
container with as little material as possible. In other words, we want to minimise the surface area of the
cylinder. The spreadsheet should look as shown below.
1. In cell B3 enter the volume of the cylinder, 1000.
2. In cell A6 enter a radius of 1. In cell A7 enter a radius of 2 and so on up to a radius of 20.
3. The formula that has been entered in cell B6 will give the height of the cylinder corresponding to the
radius for the given volume.
4. The surface area of each possible cylinder is in column D. Use the charting function on the
spreadsheet to graph the surface area against the radius.
5. What are the most cost-efficient dimensions of the drink container?

Challenge exercise
Use one of the other worksheets to find the most efficient dimensions to make a rectangular prism of
volume 1000cm3 and a cone of volume 200cm3.

Volume of pyramids, cones


andspheres

Digital doc
EXCEL Spreadsheet
doc-1311
Volume

Digital doc
Investigation
doc-2747
Packaging

6G

Pyramids

interactivity
int-1150
Maximising the
volume of a cube

The volume of any pyramid is one-third of the volume of the corresponding prism.
This leads us to the general formula for the volume of any pyramid:
V = 13Ah
where A is the area of the base and h is the height of the pyramid.
Chapter 6 Further applications of area and volume 153

WORKED EXAMPLE 20

Find the volume of the pyramid on the right.


7 cm
THINK

Area = 45 cm2

WRITE

We are given A and h, so use the general formula.

V = 13Ah

Substitute the value of A and h.

= 13 45 7

Calculate V.

= 105cm3

In other cases we may need to calculate the area of the base before we are able to use the general
formula for the volume of a pyramid.

Cones
A cone is a circular pyramid. By substituting the formula for the area of a circle into the general formula
for the volume of a pyramid, we find the formula for the volume of any cone.
A = r2 when substituted into V = 13 Ah becomes
V = 13r2h


WORKED EXAMPLE 21

Find the volume of the cone on the right, correct to 2 decimal places.

8.5 cm
3.2 cm

Tutorial
int-2482
Worked
example21

THINK

WRITE

Write the formula.

V = 13r2h

Substitute the radius and height.

= 13 3.22 8.5

Calculate the volume.

= 91.15cm3

Spheres
A sphere is a solid that looks like a ball. To find the volume of a sphere we need only the radius. The
volume is calculated using the formula:
V = 43r3
WORKED EXAMPLE 22

Find the volume of a sphere with a radius of 9.5cm, correct to the nearest cm3.
THINK

WRITE

Write the formula.

V = 43r3

Substitute the radius.

= 43 9.53

Calculate the volume.

= 3591cm3

154 Maths Quest HSC Mathematics General 2

Volume of pyramids, cones and spheres

Exercise 6G

1 WE20 Find the volume of each of the pyramids below.


a

b
8m

6m
A = 47 cm2

A = 25 cm2

d
2.5 m

9 cm
A = 62 cm2
A = 13.5 m2

2 For each of the following pyramids, calculate the volume by first calculating the area of the

baseshape.

a
8 cm

6 cm

15 cm

8 cm

14 cm

12 m

8 cm
6 cm
5 cm

6m
12 cm

10 m

1
3

3 Use the formula V = Ah to find the volume of the following cones.

Digital doc
EXCEL Spreadsheet
doc-1482
Volume of a cone

6 cm
A = 46 cm2

10 cm
A = 30 cm2
c

14 cm

52 mm

A = 12 mm2

A = 150 cm2
4 WE21 Find the volume of each of the following cones, correct to the nearest whole number.
a

b
10 cm
5 cm

12 cm
12 cm

42 cm

33 mm
8 mm

42 cm

Chapter 6 Further applications of area and volume 155

5 A cone has a base with a diameter of 9cm and a height of 12cm. Calculate the volume of that cone,

correct to 1 decimal place.


6 WE22 Calculate the volume of each of the following spheres, correct to 1 decimal place.
a

b

c

d
3.2 m
6 cm
8 cm
12.5 m

7 Calculate the volume of a sphere with a diameter of 2.3cm. Answer correct to 2decimal places.
8 MC Which of the following solids could not be described as a pyramid?
A

B

9 MC A triangular pyramid and a square pyramid both have a base area of 20cm2 and a height of

15cm. Which has the greater volume?


A The triangular pyramid
C Both have equal volume

B The square pyramid


D This cant be calculated.

10 MC A spherical balloon has a volume of 500cm3. It is then inflated so that the diameter of the

balloon is doubled. The volume of the balloon will now be:


B 2000cm3
C 3000cm3

A 1000cm3

D 4000cm3

11 Find the volume of the solid on the right. Answer correct to 1 decimal place.
12 A hollow rubber ball is to be made with a radius of 8cm, and the rubber to

4 cm

be used is 1cm thick.


a What would be the radius of the hollow inside?
b Calculate the volume of the ball.
c Calculate the volume of space inside the ball.
d Calculate the amount of rubber (in cm3) needed to make the ball.

12 cm

13 The figure on the right is a truncated cone, that is, a cone with the top cut off.
a Calculate the volume of the cone before it was truncated.
b The portion cut off was itself a cone. Calculate its volume.
c Calculate the volume of the truncated cone.
14 Use the same method as in question 13 to find the volume

15 cm
3 cm
6 cm
6 cm
5 cm

of the truncated pyramid shown at right.

3 cm

1 cm

15 The figure at right is of an ice-cream cone, containing a spherical scoop

3 cm

2.5 cm

of ice-cream.
a Calculate the volume of the cone.
b Calculate the volume of the scoop of ice-cream.
c Calculate the total volume of the shape. (Hint: Only half the sphere

sits above the cone.)


156 Maths Quest HSC Mathematics General 2

8 cm

Further development
16 Find the volume of each of the following correct to one decimal place.
a
b
c

12.6
cm
22.4 cm
8.4
cm
18.5
cm

60 cm

64
cm

12 cm

80 cm
7 cm

17 The following diagram shows a tennis canister that contains four tennis balls each of

diameter 7cm.
a What is the radius and height of the canister?
b Find the volume of the canister correct to the nearest cm3.
c Find the amount of empty space in the canister.
18 A cosmetic eye mask is 12mm thick and filled will a special liquid. Find the volume of

the liquid given that the mask has a cross sectional area of 140cm2.

19 Fifty small chocolates are to be placed inside a spherical ball. Given that each chocolate

has a volume of 0.8cm3, find the diameter of the spherical ball.

20 One hundred spherical marbles of diameter 1cm are put into a larger sphere

of diameter 10cm. Find the percentage of space inside the larger sphere that
is occupied.
21 The vase drawn on the right is a frustum, which is a cone with part cut off.

12 cm
24
cm

The height of the frustum is 24cm, which is two-thirds the height of the full cone.
Find the volume of the frustum.

6H

Volume of composite solids

Many solid shapes are composed of two or more regular solids. To calculate the volume of such a figure,
we need to determine the best method for each particular part. Many irregular shapes may still be prisms.
A prism is a shape in which every cross-section taken parallel to the base shape is equal to that
base shape. The base shape must be a polygon.

interactivity
int-2754
Volume of prisms

The formula for the volume of a prism is:

V = Ah
where A is the area of the base shape and h is the height.
Remember that the base of the prism is not necessarily the bottom. The base is the shape that is
constant throughout the prism and will usually be drawn as the front of the prism. This means that the
height will be drawn perpendicular to the base. To calculate the volume of any prism, we first calculate
the area of the base and then multiply by the height.
Chapter 6 Further applications of area and volume 157

WORKED EXAMPLE 23

4 cm

6 cm

12 cm

Find the volume of the figure drawn on the right.

10 cm

Divide the front face into two rectangles.

4 cm
12 cm

WRITE

A1
A2

6 cm

THINK

3 cm

10 cm
2

Calculate the area of each.

A1 = 4 12
= 48cm2

Add the areas together to find the value of A.

A = 48 + 36
= 84cm2

Write the formula.

Substitute A = 84 and h = 3.

= 84 3

Calculate.

= 252cm3

A2 = 6 6
= 36cm2

V=Ah

If the shape is not a prism, you may need to divide it into two or more regular 3-dimensional shapes. You
could then calculate the volume by finding the volume of each shape separately. You will need to use
important volume formulas that appear on the formula sheet:
Cone: V = 13r2hCylinder: V = r2h Pyramid: V = 13Ah Sphere: V = 43r3

WORKED EXAMPLE 24

Calculate the volume of the figure drawn on the right, correct to


2 decimal places.
2.4 cm
1.2 cm
THINK

WRITE

The shape is a cylinder with a hemisphere on top.

Write down the formula for the volume of a


cylinder.

Substitute r = 1.2 and h = 2.4.

= (1.2)2 2.4

Calculate the volume of the cylinder.

= 10.857cm3

Write down the formula for the volume of


a hemisphere. (This is the formula for the
volume of a sphere divided by 2.)

158 Maths Quest HSC Mathematics General 2

V = r2h

V = 43r3 2

Substitute r = 1.2.

= 43 (1.2)3 2

Calculate the volume of the hemisphere.

= 3.619cm3

Add the two volumes together.

Volume = 10.857 + 3.619


= 14.48cm3

In many cases a volume question may be presented in the form of a practical problem.
WORKED EXAMPLE 25

A water storage tank is in the shape of a cube of side length 1.8m, surmounted by a cylinder of
diameter 1m with a height of 0.5m. Calculate the capacity of the tank, correct to the nearest
100litres.
THINK
1

WRITE

Draw a diagram of the water tank.

1m

0.5 m

1.8 m
2

Calculate the volume of the cube using the


formula V = s3.

V = s3
= 1.83
= 5.832m3

Calculate the volume of the cylinder using the


formula V = r2h.

V = r2h
= 0.52 0.5
= 0.393m3

Add the volumes together.

Volume = 5.832 + 0.393


= 6.225m3

Calculate the capacity of the tank using


1m3 = 1000 L.

Capacity = 6.225 1000


= 6225 L

Give an answer in words.

The capacity of the tank is approximately


6200litres.

Volume of composite solids


18 cm

1 WE23 Look at the figure drawn on the right.


a Find the area of the front face.
b Use the formula V = A h to calculate the volume of the prism.
2 Find the volume of the following prisms (to 2 decimal place).

18 mm

[Base area: 35 mm2]

6 cm

5 cm

Exercise 6H

20 cm

4 cm

Digital doc
SkillSHEET 6.9
doc-11055
Volume of cubes
and rectangular
prisms

15 cm

[Base area: 28 cm2]

Chapter 6 Further applications of area and volume 159

6 mm

15 mm

14 mm
26.5 mm

8 mm

6 mm

(Note: This is called as


annular cylinder)

15 cm

10 cm

3 Calculate the volume of each of the figures drawn below.


a

b
5 cm

4 cm

12 cm

20 cm
c

5 cm

0.7 m

25 cm

12 cm

2.3 m
40 cm

3 cm
2.1 m

4 m 0.5 m

12 cm

20 cm

0.4 m

0.6 m
5m

1m
2m

1.5 m

1.5 m

4 WE 24 Consider the figure on the right.


Digital doc
SkillSHEET 6.10
doc-11056
Volume of
triangular prisms

The shape consists of a cube with a square pyramid on top.


a What is the volume of the cube?
b What is the volume of the square pyramid?
c What is the total volume of this figure?

2m

5 The figure on the right is a cylinder with a cone mounted on top.


a Calculate the volume of the cylinder, correct to the nearest cm3.
b Calculate the volume of the cone, correct to the nearest cm3.
c What is the total volume of the figure?

40 cm

50 cm
12 cm

160 Maths Quest HSC Mathematics General 2

6 Calculate the volume of each of the figures drawn below, correct to 1 decimal place.

c
Digital doc
SkillSHEET 6.11
doc-11057
Volume of
cylinders

3 cm
34 cm
r

5 cm

r =12 cm

50 cm

7 MC Which of the figures drawn below is not a prism?

B
Digital doc
SkillSHEET 6.12
doc-11058
Volume of a
sphere

8 MC The volume of the figure on the right is closest to:


A
B
C
D

718cm3
1437cm3
2155cm3
2873cm3

14 cm
7 cm

9 A fish tank is in the shape of a rectangular prism.

The base measures 45cm by 25cm. The tank is


filled to a depth of 15cm.
a Calculate the volume of water in the tank in cm3.
b Given that 1cm3 = 1mL calculate, in litres,
the amount of water in the tank.
10 WE25 A hemispherical wine glass of radius 2.5cm

is joined to a cylinder of radius 1cm and height


5cm. The glass then rests on a solid base.
a Draw a diagram of the wine glass.
b Calculate the capacity of the glass, to the
nearest 10mL.
c How many glasses of wine can be poured
from a 1 litre bottle?
11 The figure on the right is the cross-section of an annular cylinder for a concrete pipe used as a

sewage outlet.
a Calculate the area of a cross-section of the pipe, shown in blue, correct to
2 decimal places.
b Calculate the amount of concrete needed to make a 10-m length
of this pipe.
12 A commemorative cricket ball has a diameter of 7cm. It is to be preserved in

3m
2.5 m

a cubic case that will allow 5mm on each side of the ball.
a What will the side length of the cubic case be?
b Calculate the amount of empty space inside the case, to the nearest
whole number.
c Calculate the percentage of space inside the case occupied by the ball, to the nearest whole
number.

Chapter 6 Further applications of area and volume 161

Further development

Digital doc
SkillSHEET 6.13
doc-11059
Volume of a
pyramid

6 mm

13 A diamond is cut into the shape of two square-based pyramids as shown on

the right. Each mm3 of the diamond has a mass of 0.04 g. Calculate the mass
of the diamond.

6 mm

14 Find the volume of these objects (to the nearest whole unit).
8 cm
a

b
m

5 cm
12 cm

9c

5 cm

r=

6 cm
c

5 cm

2.5 cm

3 cm

2 cm

3 cm

f
2m

4.2 m

20 cm

12 m

35 cm

5m

12 cm
21 m
19 m

3m

60 m
21 m
1.6 m

and a height of 5cm (not including the cups base). Find the volume of the cone
to the nearest millilitre, where 1cm3 = 1mL.
16 Tennis balls have a diameter of 6.5cm and are packaged in a cylinder that can

hold four tennis balls. Assuming the balls just fit inside a cylinder, find:
a the height of the cylindrical can
b the volume of the can (to 1 decimal place)
c the volume of the four tennis balls (to 1 decimal place)
d the volume of the can occupied by air
e the fraction of the cans volume occupied by the balls.
162 Maths Quest HSC Mathematics General 2

14 cm
15 cm

15 The medicine cup on the right has the shape of a cone with a diameter of 4cm

17 MC The ratio of the volume of a sphere to that of a cylinder of similar

dimensions, as shown in the diagram, is best expressed as:


4
2
A
B
3
3
3
4

3
2

18 A model aeroplane is controlled by a tethered string of 10 metres length.

The operator stands in the middle of an oval. (Give all answers to the
nearest whole unit.)
a What is the maximum area of the oval occupied by the plane in
flight?
b If the plane can be manoeuvred in a hemispherical zone, find:
i the surface area of the airspace that the plane can occupy
ii the volume of airspace that is needed by the operator for controlling
the plane.
c Repeat part b with a new control string with a length of 15 metres.

6I

Digital docs
Investigation
doc-2748
Maximising
volume

Error in measurement

Earlier in this chapter we saw that all measurements are approximations. The degree of accuracy
inany measurement is restricted by the accuracy of the measuring device and the degree of
practicality.
We have previously seen that the maximum error in any measurement is half of the smallest unit of
measurement. This error is compounded when further calculations such as surface area or volume
are made.

WORKED EXAMPLE 26

THINK

8 cm

In the rectangular prism on the right, the length, breadth


and height have been measured, correct to the nearest
centimetre.
a Calculate the volume of the rectangular prism.
b Calculate the greatest possible error in the volume.

15 cm
20 cm

WRITE

a Calculate the volume of the rectangular prism.

a V=lwh

b 1 Write the smallest possible dimensions of the

b Smallest possible dimensions:

= 20 15 8
= 2400cm3

prism.

l = 19.5, w = 14.5, h = 7.5

Calculate the smallest possible volume.

V=lwh
= 19.5 14.5 7.5
= 2120.625cm3

Write the largest possible dimensions of the


prism.

Largest possible dimensions:


l = 20.5, w = 15.5, h = 8.5

Calculate the largest possible volume.

V=lwh
= 20.5 15.5 8.5
= 2700.875cm3

Calculate the maximum error.

Maximum error = 2700.875 2400


= 300.875cm3

Chapter 6 Further applications of area and volume 163

As can be seen in the above example, a possible error of


0.5cm in the linear measurement compounds to an error
of 300.875cm3 in the volume measurement.
Errors in measurement will compound errors in all
further calculations.

WORKED EXAMPLE 27

THINK

WRITE

a 1 Calculate the area of the pool floor.

Calculate the area of the ends.

Area of ends = 7.5 1.5


= 11.25m2

Calculate the area of the sides.

Area of sides = 10.2 1.5


= 15.3m2

Calculate the total area to be cemented.

Total area = 76.5 + 2 11.25 + 2 15.3


= 129.6m2

b 1 Use the incorrect measurement to repeat

all the above calculations.

answer.

Exercise 6I

Area of floor = 9.4 7.5


= 70.5m2
Area of ends = 7.5 1.5
= 11.25m2
Area of sides = 9.4 1.5
= 14.1m2
Total area = 70.5 + 2 11.25 + 2 14.1
= 121.2m2
Error = 129.6 121.2
= 8.4m2

Find the difference between the two


answers.

c Write the error as a percentage of the correct

Digital doc
SkillSHEET 6.14
doc-11060
Error in linear
measurement

Area of floor = 10.2 7.5


= 76.5m2

8.4
100%
129.6
= 6.5%

Percentage error =

Error in measurement

1 WE26 In the figure on the right each measurement has been taken to

the nearest centimetre.


a Calculate the volume of the figure.
b Calculate the maximum error in the volume calculation.

12 cm

Tutorial
int-2414
Worked
example27

A swimming pool is built in the shape of a rectangular prism with a length of 10.2m, a width of
7.5m and a depth of 1.5m. The floor and the sides of the pool need to be cemented.
a Calculate the area that is to be cemented.
bThe concreter incorrectly measured the length of the pool as 9.4m. Calculate the error in the
area calculation.
c Calculate the percentage error (correct to 1 decimal place) in the area calculation.

16 cm
2 The radius of a circle is measured as 7.6cm, correct to 1 decimal place.
a What is the maximum possible error in the measurement of the radius?
b Calculate the area of the circle. Give your answer correct to 1 decimal place.
c Calculate the maximum possible error in the area of the circle.
d Calculate the maximum possible error in the area of the circle as a percentage of the area.

164 Maths Quest HSC Mathematics General 2

6 cm

3 A cube has a side length of 16mm, correct to the nearest millimetre.


a Calculate the volume of the cube.
b Calculate the smallest possible volume of the cube.
c Calculate the largest possible volume of the cube.
d Calculate the maximum possible percentage error in the volume of the cube.
e Calculate the surface area of the cube.
f Calculate the smallest possible surface area of the cube.
g Calculate the largest possible surface area of the cube.
h Calculate the maximum possible percentage error in the surface area of the cube.
4 A cylinder has a radius of 4cm and a height of 6cm with each measurement being taken correct to

the nearest centimetre.


Calculate the volume of the cylinder (correct to the nearest whole number).
Calculate the smallest possible volume of the cylinder (correct to the nearest whole number).
Calculate the largest possible volume of the cylinder (correct to the nearest whole number).
Calculate the greatest possible percentage error in the volume of the cylinder.
5 For the cylinder in question 4, calculate the greatest possible percentage error in the surface area of
the cylinder.
6 The radius of a sphere is 1.4m with the measurement taken correct to 1 decimal place.
a Calculate the volume of the sphere, correct to 1 decimal place.
b Calculate the maximum possible error in the volume of the sphere.
c Calculate the maximum percentage error in the volume.
d Calculate the surface area of the sphere, correct to 1 decimal place.
e Calculate the maximum possible error in the surface area of the sphere.
f Calculate the maximum percentage error in the surface area.
7 WE27 An open cylindrical water tank has a radius of 45cm and a height of 60cm.
a Calculate the capacity of the tank, in litres (correct to the nearest whole number).
b If the tanks radius is given as 50cm, correct to the nearest 10cm, calculate the error in the
capacity of the tank.
c Calculate the percentage error in the capacity of the tank.
8 A rectangular prism has dimensions 56cm 41cm 17cm.
a Calculate the volume of the prism.
b Calculate the surface area of the prism.
c If the dimensions are given to the nearest 10cm, what will the dimensions of the prism be givenas?
d Calculate the percentage error in the volume when the dimensions are given to the nearest 10cm.
e Calculate the percentage error in the surface area when the dimensions are given to the nearest 10cm.
9 The four walls of a room are to be painted. The length of the room is 4.1m and the width is 3.6m.
Each wall is 1.8m high.
a Calculate the area to be painted.
b One litre of paint will paint an area of 2m2. Each wall will need two coats of paint. Calculate the
number of litres of paint required to complete this job.
c Karla incorrectly measures the length of the room to be
3.9m. If Karla does all her calculations using this
incorrect measurement, how many litres will she
be short of paint at the end of the job?
10 The dimensions of a rectangular house are 16.6m by 9.8m.
a Simon takes the dimensions of the house to the nearest
metre for all his calculations. What dimensions does
Simon use?
b Simon plans to floor the house in slate tiles. What is the
area that needs to be tiled?
c The tiles cost $27.50/m2 and Simon buys an extra 10%
toallow for cutting and breakage. Calculate the cost of
thetiles.
d How much extra has Simon spent than would have been
necessary had he used the original measurements of
thehouse?
a
b
c
d

Chapter 6 Further applications of area and volume 165

Further development
11 The dimensions of a rectangular courtyard are 20 metres by 12 metres, correct to the nearest metre.

The area is to be paved with pavers that are squares of side length 50cm.
a Calculate the number of pavers that will be needed to ensure that the entire courtyard is paved,
allowing for possible measurement error in the courtyard measurements.
b If this number of pavers are ordered what would be the maximum number of pavers that could be
left over at the end of the job?
12 The area of a square is measured as being 4900m2, correct to the nearest 100m2. Find:
a the side length of the square
b the maximum possible side length (correct to 1 decimal place)
c the minimum possible side length (correct to 1 decimal place)
d the maximum percentage error in the side length (correct to 2 decimal places).
13 The volume of a sphere is found to be 23000cm3, correct to the nearest 1000cm3. Find the
maximum percentage error in:
a the volume of the sphere
b the radius of the sphere.
Give your answers correct to 2 decimal places.
14 A cylinder has given radius of 10cm and a height of 30cm, correct to the nearest centimetre.
a Find the volume of the cylinder, correct to the nearest cm3.
b A liquid is to be poured into the cylinder. The liquid can expand by as much as 10% in hot
weather. Allowing for possible error in measurement, what is the maximum amount of the liquid
that can be poured into the cylinder such that none will spill in the event of expansion? Give your
answer correct to the nearest 100mL.
15 a Find the area of a sector of a circle of radius 15cm and subtending a 74 angle at the centre.
b Find the length of the arc formed.
c Find the maximum percentage error in
i the area of the sector
ii the arc length
given that the radius was measured to the nearest centimetre and the angle was measured to
the nearest degree.
16 The figure below is of a field. The area is to be approximated using Simpsons rule.
5m

Digital doc
WorkSHEET 6.2
doc-11061

30 m

30 m
18 m
67 m

a Estimate the area.


b Given that each measurement is taken correct to the nearest metre, find the smallest possible area

of the field.
c Find the largest possible area of the field.
d Find the maximum percentage error.

166 Maths Quest HSC Mathematics General 2

Summary
Relative error

All measurements are approximations.


Every measuring instrument is limited in the degree of accuracy that it allows.
The maximum error is half the degree of accuracy used.
A true gauge of the accuracy of a measurement is to calculate the maximum error as a
percentage of the measurement taken.

Area of parts of the circle

The area of a circle can be calculated using the formula A = r2.


The area of a sector is found by multiplying the area of the full circle by the fraction of
the circle occupied by the sector. This is calculated by looking at the angle that the sector
makes with the centre.
An annulus is the area between two circles. The area is calculated by subtracting the area of
the smaller circle from the area of the larger circle or by using the formula A = (R2 r2),
where R is the radius of the large circle and r is the radius of the small circle.
The area of an ellipse is calculated using the formula A = ab, where a is the length of
the semi-major axis and b is the length of the semi-minor axis.

Area of composite shapes

The area of a composite figure is calculated by dividing the figure into two or more
regular figures.
When calculating the area of a composite figure, some side lengths will need to be
calculated using Pythagoras theorem.

Simpsons rule

Simpsons rule is used to find an approximation for an irregular area.


h
The formula for Simpsons rule is A (df + 4dm + dl).
3
To obtain a better approximation for an area, Simpsons rule can be applied twice. This is
done by dividing the area in half and applying Simpsons rule separately to each half.

Surface area of prisms

The surface area is the total area of all faces on the solid shape.
Surface area formulas
Cube:
SA = 6s2
Rectangular Prism SA = 2(lh + lb + bh)
Many solid shapes have their surface area calculated by separately calculating the area of
each face.

Surface area of cylinders and


spheres

The surface area of a closed cylinder is found by using the formula SA = 2r2 + 2rh.
If the cylinder is an open cylinder, the surface area is found using SA = r2 + 2rh.
The surface area of a sphere is calculated using the formula SA = 4r2.

Volume of pyramids, cones


and spheres.

All pyramids can have their volume calculated using the formula V = 13Ah.
The volume of a cone is found using V = 13r2h.
The volume of a sphere is found using the formula V = 43r3.

Volume of composite solids

The volume of solid prisms is calculated using the formula V = A h.


The volume of a cone is found using the formula V = 13r2h.
The volume of a cylinder is found using the formula V = r2h.
The volume of a sphere is found using the formula V = 43r3.
The volume of a pyramid is found using the formula V = 13Ah.
Other solids have their volume calculated by dividing the solid into regular solid shapes.

Error in measurement

All measurements are approximations. The maximum error in any measurement is half
the smallest unit used.
Any error in a measurement will compound when further calculations using the
measurement need to be made.

Chapter 6 Further applications of area and volume 167

Chapter review
1 MC A cube has a side length of 4cm.

20 m

25 m

15 m

30 m

I: The surface area of the cube is 64cm2.


II: The volume of the cube is 96cm3.
Which of the above statements is correct?
A I only
B II only
C Both I and II
D Neither I nor II
2 The field drawn below is to have its area approximated by two applications of Simpsons rule.
The value of h is:
A 16
B 20
C 40
D 80
10 m

mult ip le
ch oice

80 m
3 The following figure is an open cylinder. Which of the calculations below will

correctly give the surface area of the cylinder?


A 52 + 2 5 20
C 102 + 2 10 20

B 2 52 + 2 5 20
D 2 102 + 2 10 20

4 A closed cylinder is measured as having a radius of 1.2m and a height of 1.4m.

The maximum error in the calculation of the surface area is:


A 1.2m2
B 1.5m2
C 1.6m2
Sh ort
a nswer

20 cm
10 cm

D 19.6m2

1 An elevator has a capacity of 1.3 tonnes. If 18 people who each weigh an average of 66kg are on the

elevator, how much under the capacity is the total weight?


2 In each of the following, a measurement and its degree of accuracy are given. State the limits
between which the measurement lies.
a 34cm, correct to the nearest centimetre
b 8.9kg, correct to 1 decimal place
c 500km, correct to the nearest 100km
d 2.25 L, correct to 2 decimal places
e 800km, correct to the nearest 10km
3 For each of the measurements in question 2, find the degree of accuracy as a percentage, correct to
1decimal place.
4 Calculate the area of each of the circles below. Give each answer correct to 1 decimal place.
a

b

c
3.7 cm

52 mm

1.7 m

5 Calculate the area of each of the figures below. Give each answer correct to 1 decimal place.
a

b

c
92 mm

237
12.5 cm

30
4.8 m

6 Calculate the area of each of the annuluses below. Give each answer correct to 1 decimal place.
a

b

c
34 cm
3.7 m
81 mm
94 mm

168 Maths Quest HSC Mathematics General 2

1.3 m

17 cm

7 Calculate the area of the figure on the right.

15 cm

35 cm

10 cm
12 cm

10 cm
8 Calculate the area of each of the figures below. Where appropriate,

give your answer correct to 2 decimal places.


a 0.7 m

b
1.5 cm

3.9 m

0.9 m

4.1 m

3 cm
1.5 cm

36 cm
6 cm

50 m

9 Use Simpsons rule to approximate the area below.

13 m
42 m
42 m

57 m

96 m

62 m

57 m

31 m

2m

29 m

30 m

36 m

14 m

10 Use Simpsons rule to find an approximation for each of the areas below.
a

b
c

21 m

42 m

24 m
11 By dividing the area shown below into two sections, use Simpsons

30 m

30 m
25 m

19 m 11 m

50 m

62 m

27 m

rule to find an approximation for the area.

23 m

2.1 m
0.8 m

3.9 m
4.2 cm

38 m

9m
15 m 15 m 15 m 15 m

13 Find the surface area of each of the following solids.


a

b

44 m

33 m

12 Use Simpsons rule twice to find an approximation for the area below.

4.6 m
0.9 m

1.8 m

Chapter 6 Further applications of area and volume 169

14 Calculate the surface area of each of the figures below, by calculating the area of each face separately

and adding them.


6 cm

5m
4m

10 cm
5 cm

4 cm

15 m

2m

12 cm

3.5 m

3m

5m

12 m

15 Below is a diagram of an Olympic swimming pool.


a Calculate the area of one side wall.
b Use the formula V = A h, to calculate the volume of the pool.
c How many litres of water will it take to fill the pool?

(1m3 = 1000 L)

d The walls and floor of the pool need to be painted. Calculate

50 m
22 m

3m

1m

2m

the area to be painted.


16 Calculate the surface area of each of the closed cylinders drawn below, correct to
1 decimal place.

c
60 cm

25 cm

10 cm
7 cm

1.1 m

4 cm

17 Calculate the surface area of an open cylinder with a diameter of 9cm and a height of 15cm.

Give your answer correct to the nearest whole number.


18 Calculate the surface area of a sphere with:
a a radius of 5cm
c a diameter of 156mm.

b a radius of 2.4m

Give each answer correct to the nearest whole number.


1

19 Use the formula V = 3Ah to calculate the volume of each of the pyramids below.

a 9 cm

A = 16 cm2

19 mm

A = 126 mm2

2.3 m

A = 6.9 m2

20 Calculate the volume of each of the pyramids, cones and spheres below.

a
25 m


2.6 m

3.2 m
7.9 m

36 m

52 mm
19 mm
d

19.5 mm
23.5 mm

170 Maths Quest HSC Mathematics General 2

e
23 mm

f
70 cm

21 Calculate the volume of the solid drawn below.

1.9 m

3.1 m

0.5 m

2.7 m

0.6 m

22 Calculate the volume of each of the solids drawn below. Where necessary, give your answer correct

to the nearest whole number.

17 cm

22 cm

40 cm

12 cm

3 cm
9 cm

20 cm

12 cm

19 cm

15 cm

10 cm

3 cm
3 cm
10 cm

23 Calculate the volume of the following figure, correct to 2 decimal places.

15 cm
9 cm
24 A sphere has a diameter of 16cm when measured to the nearest centimetre.
a State the maximum error made in the measurement of the

radius.
b Calculate the volume of the sphere. Answer correct to the

nearest whole number.


c Calculate the maximum percentage error in the volume of

the sphere.
25 An aluminium soft drink can has a diameter of 8cm and a
height of 10cm.
a Calculate the capacity of the can, in millilitres, correct to
the nearest 10 millilitres.
b The machine that cuts the aluminium for the can is
mistakenly set to 12cm. Calculate the percentage error
in the capacity of the can (correct to the nearest whole
number).
1 The figure on the right shows a section of a concrete drainage pipe.
a Calculate the area of the annulus, correct to 1 decimal place.
b Calculate the volume of concrete needed to make a 5m length of this pipe

(correct to 1 decimal place).


c Calculate the volume of water that will flow through the 5m length of the pipe

Ex tended
R es p ons e

2.5 m
1.5 m

(in litres, to the nearest 100 L).


d Calculate the surface area of a 5m section of pipe (correct to the nearest m2).

(Hint: Include the area of the inside of the pipe.)


Chapter 6 Further applications of area and volume 171

2 The diagram on the right shows the cross-section of a river.


a Use two applications of Simpsons rule to find the approximate area

172 Maths Quest HSC Mathematics General 2

4.9 m

9.2 m

60 m
5.1 m

Digital doc
doc-11062
Test yourself
chapter 6

of the rivers cross-section.


b If the river flows with this cross-section for approximately 800m,
calculate the volume of the river.
c The length of the river has been approximated to the nearest 100m.
Calculate the maximum percentage error in calculating this volume.

ICT activities
6ARelative error

6G Volume of pyramids, cones and spheres

DIGITAL DOCS
SkillSHEET 6.1 (doc-11046): Writing one quantity as a percentage of
another. (p. 131)
EXCEL Spreadsheet (doc-1461): Calculations with percentages.
(p.131)

INTERACTIVITY
int-1150: Maximising the volume of a cube. (page 153)
TUTORIAL
WE21 int-2482: Learn how to calculate the volume of a cone.
(page154)

6BArea of parts of the circle

DIGITAL DOC
EXCEL Spreadsheet (doc-1482): Volume of a cone. (page 155)

TUTORIAL
WE7 int-2411: Learn to calculate the area of an annulus.
(page134)

6H Volume of composite solids

DIGITAL DOC
SkillSHEET 6.2 (doc-11047): Area of a circle. (page 134)

6CArea of composite shapes


INTERACTIVITies
int-0005: Area. (page 136)
int-2350: Area of a triangle. (page 136)
TUTORIAL
WE9 int-2412: Learn how to find the area of a composite figure.
(page 137)
DIGITAL DOCS
SkillSHEET 6.3 (doc-11048): Areas of squares, rectangles and
triangles. (page 138)
SkillSHEET 6.4 (doc-11049): Using Pythagoras theorem.
(page 138)
EXCEL Spreadsheet (doc-1306): Pythagoras. (page 138)
GC program Casio (doc-1307): Mensuration. (page 138)

6D Simpsons rule
DIGITAL DOCS
WE13 int-2413: Learn how to apply Simpsons rule.
(page 143)
SkillSHEET 6.5 (doc-1308): Substitution into formulas.
(page 143)

6E Surface area of some prisms


DIGITAL DOCS
SkillSHEET 6.6 (doc-11051): Surface area of cubes and rectangular
prisms. (page 147)
SkillSHEET 6.7 (doc-11052): Surface area of triangular prisms.
(page148)
WorkSHEET 6.1 (doc-11053): Apply your knowledge of measurement
to problems. (page 149)

INTERACTIVITY
int-2754: Volume of prisms. (page 157)
DIGITAL DOC
SkillSHEET 6.9 (doc-11055): Volume of cubes and rectangular prisms.
(page 159)
DIGITAL DOCS
SkillSHEET 6.10 (doc-11056): Volume of triangular prisms. (page 160)
SkillSHEET 6.11 (doc-11057): Volume of cylinders. (page 161)
SkillSHEET 6.12 (doc-11058): Volume of a sphere. (page 161)
SkillSHEET 6.13 (doc-11059): Volume of a pyramid. (page 162)
Investigation (doc-2748): Maximising volume. (page 163)

6IError in measurement
TUTORIAL
WE27 int-2414: Learn how to apply area concepts to a real
situation. (page 164)
DIGITAL DOCS
SkillSHEET 6.14 (doc-11060): Error in linear measurement. (page 164)
WorkSHEET 6.2 (doc-11061):Apply your knowledge of measurement
to problems. (page 166)

Chapter review
DIGITAL DOC
Test Yourself (doc-11062): Take the end-of-chapter test to test your
progress. (page 172)

To access eBookPLUS activities, log on to www.jacplus.com.au

6F Surface area of cylinders and spheres


INTERACTIVITY
int-2782: Surface area of a sphere. (page 149)
DIGITAL DOCS
SkillSHEET 6.8 (doc-11054): Circumference of a circle. (page 151)
EXCEL Spreadsheet (doc-1311): Volume. (page 153)
Investigation (doc-2747): Packaging. (page 153)

Chapter 6 Further applications of area and volume 173

Answers chapter 6
Further applications of
area and volume
Exercise 6A

Relative error

1 a 4.5cm to 5.5cm
b 11.5cm to 12.5cm
c 33.5cm to 34.5cm
d 58.5cm to 59.5cm
e 89.5cm to 90.5cm
f 199.5cm to 200.5cm
2 a 35m to 45m
b 85m to 95m
c 245m to 255m
d 295m to 305m
e 995m to 1005m
f 1995m to 2005m
3 a 5.25cm to 5.35cm
b 9.75m to 9.85m
c 7.15km to 7.25km
d 4.95mm to 5.05mm
e 9.85km to 9.95km
f 0.05m to 0.15m
4 a 44.5km to 45.5km
b 0.5km c
1.11%
5 a 0.75% b
7.14% c
2.78%
d 0.13% e
5%
6 a i 4cm
ii 3cm
iii 5cm
b i 12.5%
ii 16.67%
iii 10%
ii 27mm
iii 52mm
7 a i 40mm
b i 1.25%
ii 1.85%
iii 0.96%

8 a 1 h 15 min to 1 h 25 min
b 5 minutes c
6.25%
9 B
10 C
11 C
12 D
13 a 29.5m to 30.5m
b 19.5m to 20.5m
c 98m d
102m e
2m
14 a 575.25m2 b
625.25m2
c 25.25m2
15 750.4mL
16 79.25km
17 a 2.2% b
0.2% c
0.02%
18 a 2.67%
b Travis is correct as the percentage error

is the same.
19 11cm
20 Jonas is correct as a whole number gives a
percentage error of 10% while one decimal
place gives 1%.
21 a 28m2 b
6.5m and 3.5m
c 22.75m2 d
18.75%
e 7.5m and 4.5m
f 33.75m2
g 20.5%
h 20.5%
22 2%
Exercise 6B

Area of parts of the circle

1 128.7cm2
2 a 254.47cm2
b 3421.19mm2
c 172.03m2
d 2206.18cm2
e 46.32m2
f 113.85m2
3 176.7m2

Exercise 6D Simpsons rule


4 40.7m2
5 a 14.16cm2
1 a 30m
b 6451.26mm2
b df = 40m, dm = 9m, dl = 18m
c 92.33m2
c 940m2
d 110.79mm2
2 a 1296m2 b
1560m2 c
936m2
e 796.39m2
3 a 620m2 b
880m2 c
1500m2
f 955.67mm2
4 a 2535m2 b
1184m2 c
2934m2
6 827.3cm2
5 C
7 339.3cm2
6 B
8 a 239cm2 b
240m2
7 2514m2
c 18100mm2
8 a 2970m2 b
11840m2
9 a 2.5m b
3.5m c
18.8m2
c 1386m2
10 B
9 a 768m2 b
640m2
11 B
10 2484m2
b 122.5m2
c 25.5m2
12 a 78.5m2
11 The figure cannot be divided into an even
13 a 750m2 b
117.8m2 c
15.7%
number of equal strips.
14 a 1571m2
12 a 63.4m2 b
1585m3
b Tori is incorrect. Your explanation
13 160m2
should include another example of
14 a 226cm2 b
192cm2
two radii differing by 10m, giving a
c 15.04%
different answer to that in part (a).
15 350 trees
15 57
16 300m3
Exercise 6C

Area of composite shapes

Exercise 6E

Surface area of some prisms

1 248m2
1 a 150cm2 b
486cm2
2
2 a 222cm2 b
375cm2
c 6144cm d
43.74cm2
c 335cm2 d
228.5cm2
e 23064mm2 f
47.04m2
2
e 44.6cm2 f
130.3cm2
2 a 122m
b 107.5m2
3 a 8cm b
84cm2
c 3276cm2 d
95.98cm2
2
4 a 5.3m
b 31.8m2
e 1444cm f
1274mm2
5 a 120m2 b
168cm2
3 11.92m2
c 6658.2mm2
4 28125cm2
6 a 174cm2 b
510m2
5 a 17.5m2 b
70
c 4032mm2
c 42.5m2
d 17.5m2
7 A
e 30 L
8 B
6 B
9 a
b
250m c
3582.5m2 7 36cm2
8 95.6m2
9 486cm2
10 236cm2
11 568cm2
50 m
2
2
12 10m
10 a 80m b
109.7cm
13 6.615m2
c 12.2cm2 d
58.4cm2
2
2
14 2.65m2
e 4600mm f
20.1m
2
15 236.89cm2
11 2513m

42 m

Exercise 6F Surface area of cylinders


and spheres
1 502.7cm2
25 m
2 a 282.7cm2 b
18.7m2
27 m
2
40 m
c 3141.6cm d
785.4cm2
e 437.4cm2 f
54.9m2
2
2
2
3 37.4m
b 1000m c
134m d
$2345
4 452cm2
13 a 306cm2 b
625cm2
2
2
5 a 395.4cm2 b
1616.5cm2
c 1428.3cm
d 1147.6cm
c 2199.1cm2 d
367.9cm2
e 17121.8mm2
f 670.9cm2
2
2
2
e 640.9cm f
52.8m2
14 a 200cm
b 350m
6 314.16cm2
c 50cm2 d
35000mm2
7 a 231cm2 b
154cm2
e 0.0134km2
f 37.5 ha
8 113cm2
g 2750m2 h
43000m2
9 a 804.2cm2 b
55.4cm2
15 14.98m2
c 2463.0cm2
d 12.6m2
16 661mm2
e 145.3cm2
f 40.7m2
17 a 12.6m2 b
9.4m2
10 5542cm2
18 a 1026m2 b
10.6m2
11 A
c 266m2 d
4209.1mm2
12 B
e 1252cm2 f
172.9m2

12 a

174 Maths Quest HSC Mathematics General 2

1m

13 a 565cm2 b
452cm2
10 a
Chapter Review
2.5 cm
14 a 113cm2
Multiple choice
b r = 3cm, h = 18cm c
28cm2
2
2
1
D
d 368cm e
339cm
5 cm
2 B
15 3041cm2
3 A
16 4562cm2
4 C
17 201cm2
b 50mL c
20
18 693cm2
Short answer
11 a 8.64m2 b
86.4m3
19 a 5400cm2 b
47.64%
1 112kg
12 a 8cm b
332cm3 c
35%
Exercise 6G Volume of pyramids, cones
2 a 33.5cm to 34.5cm
13 5.76 g
and spheres
b 8.85kg to 8.95kg
14 a 3054cm3 b
840cm3
1 a 6666.7cm3 b
9400cm3
c 450km to 550km
c 48cm3 d
42cm3
c 186cm3 d
11.25m3
d 2.245 L to 2.255 L
e 10379cm3 f
312m3
2 a 96cm3 b
560cm3
e 795km to 805km
g 9448m3 h
33m3
c 120m3 d
100cm3
3 a 1.5% b
0.6%
15 770mL
3 a 100cm3 b
92cm3
c 10% d
0.2%
16 a 26cm b
862.8cm3
c 700cm3 d
208mm3
e 0.6%
c 575.2cm3 d
287.6cm3
2
4 a 262cm3 b
1810cm3
4 a 43.0cm2 b
8494.9mm2
e
3
2
c 2212mm3 d
77585cm3
c 2.3m
17 B
5 254.5cm3
5 a 2215.9mm2 b
18.1m2
18 a 314m2
2
2
3
6 a 904.8cm3 b
2144.7cm3
c 323.2cm
b i 628m
ii 2094m
c 8181.2m3 d
137.3m3
6 a 7147.1mm2 b
37.7m2
c i 1414m2
ii 7069m3
2
7 6.37cm3
c 2723.8cm
Exercise 6I Error in measurement
8 A
7 705cm2
1 a 1152cm3
9 C
8 a 5.75m2 b
27cm2
b 188.625cm3
2
10 D
c
1804.94
c
m
2 a 0.05cm b
181.5cm2
11 335.1cm3
9 1722m2
c 2.4cm2 d
1.3%
3
12 a 7cm b
2144.7cm
10 a 840m2 b
2672m2
3 a 4096mm3
2
c 1436.76cm3 d
707.9cm3
c 5548m
b 3723.875mm3
13 a 565.487cm3 b
84.83cm3
11 4190m2
3
c
4492.125
m
m
c 480.7cm3
12 2010m2
d 9.7%
14 14.33cm3
13 a 105.84cm2 b
25.98m2
e 1536mm2
2
15 a 52.4cm3 b
65.45cm3
c 19.44m d
18.18m2
f 1441.5mm2
2 b
c 85.1cm3

1
4
a
54
m
352cm2
g 1633.5mm2
2
16 a 1241.4cm3 b
1075.2cm3
c 340m
h 6.3%
c 768.9cm3 d
43306.7cm3
15 a 75m2 b
1650m3
4 a 302cm3 b
212cm3
17 a r = 3.5cm, h = 28cm
c 1650000 L d
1316.2
c 414cm3 d
37%
b 1078cm3 c
360cm3
16 a 747.7cm2 b
728.8cm2
5
24%
2
18 168cm3
c 11.7m
6 a 11.5m3 b
1.3m3
19 4.2cm
17 488cm2
c 11.3% d
24.6m2
20 10%
18 a 314cm2 b
72m2
e 1.8m2 f
7.3%
21 5227.6cm3
c 76454mm2
7 a 382 L b
89 L
19 a 48cm3 b
798mm3
Exercise 6H Volume of composite
c 23%
3
c
5.29
m
solids
8 a 39032cm3
20 a 10800m3 b
10.95m3
1 a 178cm2 b
712cm3
b 7890cm2
3 d
c
19
6
58
m
m
2339mm3
3
3
2 a 630mm b
420cm
c 60cm 40cm 20cm
3
e 50965mm f
179594cm3
c 3152.68mm3
d 23%
3

2
1
3.438
m
3
d 1319.47mm
e 11.5%
22 a 5797cm3 b
14283cm3
3 a 700cm3 b
3000cm3
9 a 27.72m2 b
28 L c
1L
3
c
1260
c
m
3
3
c 3720cm d
2.128m
10 a 17m 10m b
170m2
23 5343.85cm3
e 12.75m3 f
18m3
c $5142.50 d
$221.43
24 a 0.25cm b
2145cm3
3
3
4 a 8m b
2m
11 a 1025 b
128
c 9.7%
c 10m3
12 a 70m b
70.4m
25 a 500mL b
20%
5 a 22619cm3 b
6032cm3
c 69.6m d
0.57%
c 28651cm3
Extended response
13 a 2.17% b
0.74%
6 a 19000.4cm3 b
103.7cm3
14 a 9425cm3 b
7500mL

1 a 12.6m2 b
62.8m3
c 157724.9cm3
15 a 145.3cm2 b
19.4cm
c 62800mL d
125.7m2
7 B
c i 7.5%
ii 4%
2
2 a 292m b
233600m3
8 D
16 a 2910m2 b
2832m2
3
c
6.25%
9 a 16875cm b
16.875 L
c 2989m2 d
2.7%

Chapter 6 Further applications of area and volume 175

Chapter 7

Applications of trigonometry
CHAPTER CONTENTS
7A Review of right-angled triangles
7B Using the sine rule to find side lengths
7C Using the sine rule to find angles
7D Using the cosine rule to find side lengths
7E Using the cosine rule to find angles
7F Area of a triangle
7G Bearings
7H Radial surveys

7A Review

of right-angled triangles

Previously we have studied right-angled triangles and discovered that we can calculate a side length of a
triangle when given the length of one other side and one of the acute angles.
To do this we need to use one of the formulas for the three trigonometric ratios.
opposite side
sin =
hypotenuse

adjacent

hypotenuse

opposite

cos =
tan =

Interactivity
int-2405
SOHCAHTOA

adjacent side
hypotenuse
opposite side
adjacent side

WORKED EXAMPLE 1

Find the length of the side marked a in the following triangle.


13.2 cm

a
THINK
1

43

WRITE

Label the relevant sides of the diagram.


Hypotenuse
13.2 cm

43

Adjacent

Chapter 7 Applications of trigonometry 177

Choose the cosine ratio and write the formula.

Substitute all known information into the formula.

adjacent side
hypotenuse
a
cos 43 =
13.2

Make a the subject of the formula.

a = 13.2 cos 43

Calculate the value of a.

cos =

9.65m

WORKED EXAMPLE 2

Find the length of the side marked x in the figure on the right
(correct to 1 decimal place).

42

29.2 cm
THINK

WRITE

Method 1: Technology-free
1

Label the relevant sides of the diagram.


42

hyp
x

29.2 cm
opp

sin =

opposite side
hypotenuse

Choose the sine ratio and write the formula.

Substitute the values for the opposite side, the hypotenuse


andfortheangle.

sin 42 =

Make x the subject of the formula.

Calculate the value of x.

x sin 42 = 29.2
29.2
x=
sin 42
x = 43.6m

Method 2: Technology-enabled
1

From the MENU select EQUA.

Press 3 (SOLV).

Delete any equation, enter the equation sin 42 = 29.2 X


and press w.
Note: Your calculator may display a different value of X at this stage.
This is just the last value of X stored in the calculators memory.

178 Maths Quest HSC Mathematics General 2

29.2
x

Press 6 (SOLV) to solve the equation.

The same formulas can be used to calculate the size of an angle if we are given two side lengths in the
triangle.
WORKED EXAMPLE 3

Calculate the size of the angle marked in the figure on the right
(correct to the nearest degree).

47 mm
35 mm

THINK

WRITE

Method 1: Technology-free
1

Label the sides of the triangle.

Choose the tangent ratio and write the formula.

Substitute values for the opposite side and the adjacent side.

Make the subject of the formula.

Calculate .

Opposite = 47mm
Adjacent = 35mm
opposite side
tan =
adjacent side
tan =

47
35

= tan 1 a
= 53

47
b
35

Method 2: Technology-enabled
1

From the MENU select EQUA.

Press 3 (SOLV).

Delete any existing equation, then enter the equation tan X = 47


$ and press w.
Note: Your calculator may display a different value of X at this
stage. This is just the last value of X stored in the calculators
memory.

Press 6 (SOLV) to solve the equation.

Chapter 7 Applications of trigonometry 179

Using these results, we are able to solve problems that involve more than one right-angled
triangle.

WORKED EXAMPLE 4

Tutorial
int-2415
Worked example 4

Greg stands 70m from the base of a building and measures the
angle of elevation to the top of the building as being 35. Julie is
standing 40m from the base of the building on the other side
of the building as shown in the following figure.
h
35

40 m

70 m

a
Calculate the height of the building, correct to 2 decimal places.
b
Calculate the angle of elevation of the top of the building that

Julie would measure, correct to the nearest degree.

THINK

a 1 Draw the triangle showing the angle of elevation from

WRITE

where Greg is standing and label the sides.


h
35
70 m

opposite side
adjacent side
h
tan 35 =
70
tan =

Choose the tangent ratio and write the formula.

Substitute for and the adjacent side.

Make h the subject of the formula.

h = 70 tan 35

Calculate the value of h.

h = 49.01m

b 1 Draw the triangle from where Julie is standing and label

b
49.01 m

the sides.

Choose the tangent ratio and write the formula.

Substitute for the opposite side and the adjacent side.

Make the subject of the formula.

Calculate , correct to the nearest degree.

180 Maths Quest HSC Mathematics General 2

40 m

opposite side
adjacent side
49.01
tan =
40
tan =

= tan 1 a
= 51

49.01
b
40

One of the main


B
applications of rightAngle of depression
angled trigonometry
will be solving
problems that involve
angles of elevation
and/or angles of
depression.
The angle of
elevation is the angle
through which you must
look up to see an object A
that is higher than
Angle of elevation
yourself. Similarly the
angle of depression is the angle through which you must look down to see an object lower than yourself.
In any example the angle of elevation is equal to the angle of depression.That is, the angle
ofelevationfrom point A to point B, is the same as the angle of depression from point B to point A.
WORKED EXAMPLE 5

Jeff is on a yacht and sights the top


of a lighthouse at an angle of elevation
of 56 as shown. If Jeff is 50m away fromthe
lighthouse, calculate the heightofthe lighthouse
correct to the nearest metre.

56
50 m
THINK
1

WRITE

Draw a diagram to represent the situation.


Label the sides and select the correct
trigonometric ratio.
Opposite
x

56
50 m
Adjacent

tan =
2

Substitute the known values into the formula.

Make x the subject of the formula.

Calculate the value of x.

Write the answer to the question.

tan 56 =

Opposite
Adjacent
x
50

50 tan 56 = x
x 74
The lighthouse is approximately 74metres tall.

Chapter 7 Applications of trigonometry 181

Exercise 7A

Review of right-angled triangles

1 WE1, 2 Calculate the length of the side marked with the pronumerals in each of the following,

correctto 1 decimal place.


a

38

314 mm

23

13.2 cm

142 mm

Digital doc
SkillSHEET 7.1
doc-11063
Right-angled
trigonometry
finding a side
length

61
11.4 m

17
5
d

50

9.1 m

19.2 cm

2 WE3 Calculate the size of each of the angles marked with the pronumerals, correct to the nearest

degree.

9.5 m

113 cm

71 mm

11.4 m

61

cm

36 mm

Digital doc
SkillSHEET 7.2
doc-11064
Using the inverse
trigonometric
ratios

3 Andrew walks 5km from point P to point Q. At the same time Bianca walks from P to R such that

PQ is perpendicular to PR. Given that PQR = 28:

Digital doc
SkillSHEET 7.3
doc-11065
Rounding angles
to the nearest
degree

Digital doc
SkillSHEET 7.4
doc-11066
Right-angled
trigonometry
finding an angle

a draw a diagram of PQR


b calculate the distance walked by Bianca, correct to the nearest metre
c calculate the distance that Andrew would need to walk in a straight line to meet Bianca, correct to

the nearest metre.


4 WE4 Sally and Tim are both sighting the top of a building, as shown in

the figure on the right. Sally is 40m from the base of the building and
sights the angle of elevation to the top of the building as 35. Tim is
60m from the base of the building.
a Calculate the height of the building, correct to 2 decimal
places.
b Calculate the angle of elevation at which Tim will sight the
building.

35
40 m
S

50 m

6 The angle of depression from the top of a cliff to a boat sailing 100m

offshore is 32. Calculate the height of the cliff, correct to the nearest
metre.
100 m

32
h

182 Maths Quest HSC Mathematics General 2

5 George and Diego are both flying a kite from the same point.

Georges kite is flying on 50m of string and the string makes a 70


angle with the ground. Diegos kite is flying on a 60m piece of string
and is at the same height as Georges kite, as shown in the figure on the
right. Calculate the angle that the string from Diegos kite makes with
the ground. Give your answer correct to the nearest degree.

60 m

70

60 m

7 A lighthouse is 40m tall and the beacon atop the lighthouse is sighted

by a ship 150m from shore, as shown in the figure on the right.


40 m
Calculate the angle of elevation at which the lighthouse is sighted
from the ship, correct to the nearest degree.
8 From a point 65m above the ground, a second point is sighted on the
ground at a distance of 239m.
a Draw a diagram of this situation.
b Calculate the angle of depression from the first point to the second point.

150 m

Further development
9 MC The shadow cast by a statue 2metres tall is 0.6metres. The angle of the sun to the ground is

closest to:
A 17
C 72

B 18
D 73

10 In the diagram find x (to 1 decimal place), and (to the nearest degree).
11 The sun is overhead, casting a shadow of length 90cm from a 1.75m

60

8m

6.5 m

scarecrow, which is no longer standing upright. Determine the angle


(to the nearest degree) that the scarecrow makes with the ground.
12 A kite is hovering in strong winds, 10m vertically above the ground. It is being held in place by a
taut 12m length of rope from the kite to the ground. Find the angle (to the nearest degree) that the
rope makes with the ground.
13 A ramp joins two points, A and B. The horizontal distance between A and B is 1.4m, and B is 30cm
vertically above the level of A.
a Find the length of the ramp (in metres to 2 decimal places).
b Find (to the nearest degree) the angle that the ramp makes with the horizontal.
14 A chairlift follows a direct line from a mountain peak (altitude 1400m) to a ridge (altitude 960m).
If the horizontal distance between the peak and the ridge is 510m, find the angle of descent (to the
nearest degree) from one to the other.

Using the sine rule to find


sidelengths

7B

In this section you will be learning about trigonometry as it applies to triangles which are not right
angled.In right-angled triangles you have been able to find the length of sides and the size of angles
usingthe sine,cosine and tangent ratios and in doing so you have applied these ratios to acute
anglesonly.
A non-right-angled triangle can be drawn with three acute angles, or with one obtuse angle and two
acute angles.
For this reason we need to explore trigonometric ratios as they apply to obtuse angles.

Trigonometric ratios for obtuse angles


Many non-right-angled triangles have one obtuse angle. In the following sections we will be solving
non-right-angled triangles and will need to investigate the trigonometric ratios for obtuse angles.
1. Use your calculator to give each of the following, correct to 3 decimal places.
(a) sin 100
(b) cos 100
(c) tan 100
(d) sin 135
(e) cos 135
(f) tan 135
(g) sin 179
(h) cos 179
(j) tan 179
2. Which of the answers to question 1 are positive and which are negative?
3. Calculate the sine, cosine and tangent of several other obtuse angles and see if the established pattern
continues.
4. Can you develop a rule for the sign of trigonometric ratios of obtuse angles?
Every obtuse angle has a corresponding acute angle for which the three trigonometric ratios are
related.
Chapter 7 Applications of trigonometry 183

5. Find each of the following using your calculators, giving your answers correct to three decimal places.
(a) sin 60 and sin 120
(b) cos 40 and cos 140
(c) tan 20 and tan 120
6. What do you notice about each of the angle pairs in question 5?
7. Use what you have discovered to complete each of the following using an acute angle.
(a) sin 150 = sin ___
(b) sin 115 = sin ___
(c) sin 166 = sin ___

Finding side lengths


The trigonometry we have studied so far has been applicable to only
right-angled triangles. The sine rule allows us to calculate the lengths
of sides and the size of angles in non-right-angled triangles. Consider
the triangle drawn on the right.
The sine rule states that in any triangle, ABC, the ratio of each
side to the sine of its opposite angle will be equal.

C
C
b
A

a
B

a
b
c
=
=
sin A sin B sin C

Derivation of the sine rule


C

A, B and C represent the three angles in the triangle ABC and a, b and c
represent the three sides, remembering that each side is named with the
lower-case letter of the opposite vertex.
Construct a line from C to a point, D, perpendicular to AB. CD is the
perpendicular height of the triangle, h.
Now consider ACD and BCD separately.
C

a
h

D
c

D D

Use the formula for the sine ratio:


opp
opp
sin =
sin = hyp
hyp
h
h
sin A =
sin B =
a
b
h = b sin A
h = a sin B
We are now able to equate these two expressions for h.
a sin B = b sin A
Dividing both sides by sin A sin B we get:
a sin B
b sin A
=
sin A sin B sin A sin B
a
b
=
sin A sin B
Similarly, we are able to show that each of these is also equal to

c
. Try it!
sin C

This formula allows us to calculate the length of a side in any triangle if we are given the length of
one other side and two angles. When using the formula we need to use only two parts of it.
184 Maths Quest HSC Mathematics General 2

WORKED EXAMPLE 6

Calculate the length of the side marked x in the triangle on the right,
correct to 1 decimal place.

A
80

B
THINK

40

16 cm

WRITE

Method 1: Technology-free
1

Write the formula.

Substitute a = x, b = 16, A = 80 and B = 40.

Make x the subject of the equation by multiplying by sin 80.

Write the value of x.

a
b
=
sin A sin B
x
16
=
sin 80 sin 40
16 sin 80
x=
sin 40
x = 24.5cm

Method 2: Technology-enabled
1

From the MENU select EQUA.

Press 3 (SOLV).

Delete any existing equation, enter the equation


X sin 80 = 16 sin 40, and then press w.
Note: Your calculator may display a different value of X
at this stage. This is just the last value of X stored in the
calculators memory.

Press 6 (SOLV) to solve the equation.

WORKED EXAMPLE 7

Find the unknown length, x cm, in the


triangle at right (to 1 decimal place).
THINK
1
2

130
30

7 cm
WRITE

Draw the triangle. Assume it is non-right-angled.

Label the triangle appropriately for the sine rule.

130
C

30

Tutorial
int-0465
Worked example 7

c = 7 cm
A

b=x

Chapter 7 Applications of trigonometry 185

Confirm that it is the sine rule that can be used as you have the angle
opposite to the unknown side and a known side ratio.
angle

a
b
c
=
=
sin A sin B sin C
b=x
B = 130
c = 7cm C = 30

Substitute known values into the two ratios.

x
7
=
sin 130 sin 30

Make x the subject and evaluate.

Write the answer.

7 sin 130
sin 30
x = 10.7246
x = 10.7
x=

The unknown length is


10.7cm, correct to
1 decimal place.

Note: Some questions may ask for you to give the answer in a form other than a number and as such
thegraphics calculator method cannot be used. For example, the question above could be worded to,
16 sin 80
say, show x =
, in which case you must manipulate the equation to arrive at the desired
sin 40
expression.
To use the sine rule we need to know the angle opposite the side we are finding and the angle opposite
the side we are given. In some cases these are not the angles we are given. In such cases we need to use
the fact that the angles in a triangle add to 180 to calculate the required angle.

WORKED EXAMPLE 8

Calculate the length of the side labelled m in the figure on the right,
correct to 4 significant figures.

A
65
m

B
THINK
1

Calculate the size of angle C.

Write the formula.

Substitute a = 16, c = m, A = 65 and C = 40.

Make m the subject of the equation.

Calculate.

75
16 m

WRITE

C = 180 65 75
= 40
a
c
=
sin A sin C
16
m
=
sin 65 sin 40
m=

16 sin 40
sin 65

= 11.35m

As mentioned in the previous investigation, we need to apply the sine rule to obtuse-angled triangles.
Insuch examples the method used is exactly the same with the substitution of an obtuse angle.
Using the sine rule allows us to solve a number of more complex problems. As with our earlier
trigonometry problems, we begin each with a diagram and give a written answer to each.
186 Maths Quest HSC Mathematics General 2

WORKED EXAMPLE 9

Georg looks south and observes an aeroplane at an angle of


elevation of 60. Henrietta is 20km south of where Georg is and
she faces north to see the aeroplane at an angle of elevation of 75.
Calculate the distance of the aeroplane from Henriettas
observation point, to the nearest metre.

60

G
THINK

75

20 km

WRITE

Calculate the size of GAH.

A = 180 60 75
= 45

Write the formula.

Substitute g = x, a = 20, G = 60 and H = 75.

Make x the subject.

x=

Calculate the value of x.

x = 24.495km

Give a written answer.

g
a
=
sin G sin A
x
20
=
sin 60 sin 45
20 sin 60
sin 45

The distance of the aeroplane from Henriettas


observation point is 24.495km.

Using the sine rule to find side lengths

Exercise 7B

1 Write down the sine rule formula as it applies to each of the triangles below.
a
b X

c P
A
c

Digital doc
SkillSHEET 7.5
doc-11067
Angle sum of a
triangle

2 WE6 Use the sine rule to calculate the length of the side marked with the pronumeral in each of the

following, correct to 3 significant figures.


A
a
b
x

16 cm

50

45

1.9 km
C

R
52

63

59

84

Digital doc
SkillSHEET 7.6
doc-11068
Solving fractional
equations

89 mm

3 WE7 In each of the following, use the sine rule to calculate the length of the side marked with the

pronumeral, correct to 1 decimal place, by first finding the size of the third angle.
x
H
74 74

N
80

18.2 mm

m
62 P

35.3 cm
I

B
19.4 km

a G

85

y
27

Chapter 7 Applications of trigonometry 187

4 MC Look at the figure drawn on the right. Which of the following

expressions gives the value of m?


42 sin 117
A m =

sin 28
C m =

42 sin 28

sin 117

B m =

42 sin 117
sin 35

D m =

42 sin 35
sin 117

42 cm
35

5 MC Look at the figure drawn on the right. Which of the following

expressions gives the value of n?


42 sin 117
A n =

sin 28
C n =

42 sin 117
B n =
sin 35

42 sin 28

sin 117

D n =

28

35

28
42 m

42 sin 35
sin 117

6 ABC is a triangle in which BC = 9cm, BAC = 54 and ACB = 62. Calculate the length of side

AB, correct to 1 decimal place.

7 WE8 XYZ is a triangle in which y = 19.2m, XYZ = 42 and XZY = 28. Calculate x, correct

to3 significant figures.

8 WE7 X and Y are two trees, 30m apart on one side of a river. Z is a tree on the opposite side of the

river, as shown in the diagram below.


Z

59
X

72
30 m

It is found that XYZ = 72 and YXZ = 59. Calculate the distance XZ, correct to 1 decimal
place.

Further development
9 WE9 From a point, M, the angle of elevation to the top of a building, B, is 34. From a point, N,

20m closer to the building, the angle of elevation is 49.


a Draw a diagram of this situation.
b Calculate the distance NB, correct to 1 decimal place.
c Calculate the height of the building, correct to the nearest metre.

10 Look at the figure on the right.

80 sin 30
.
sin 40
80 sin 30 sin 70
b Show that h can be found using the expression
.
sin
40
c Calculate h, correct to 1 decimal place.
a Show that XY can be given by the expression

11 Steel trusses are used to support a heavy gate at the entrance to a

h
W

30
80 m

70
X
0.8 m

shipping yard.
The struts in the truss shown are each made from 0.8m steel lengths
130 130 130
and are welded at the contact points with the upper and lower sections
of the truss.
a On the lower section of the truss, what is the distance (to the nearest centimetre) between each
pair of consecutive welds?
b What is the height (to the nearest centimetre) of the truss?
12 A scenic flight leaves Town A and flies west of north for the 80km direct journey to Town B. At

Town B the plane turns 92 to the right to fly east of north to Town C. From here the plane turns
129 to the right and flies the 103km straight back to Town A. Find the distance (to the nearest km)
of the direct flight from Town B to Town C.
188 Maths Quest HSC Mathematics General 2

7C Using the
Finding angles

sine rule to find angles

Using the sine rule result, we are able to calculate angle sizes as well. To do this, we need to be given the
length of two sides and the angle opposite one of them. For simplicity, in solving the triangle we invert
the sine rule formula when we are using it to find an angle. The formula is written:
sin A sin B sin C
=
=
a
b
c
Your formula sheet has the sine rule to find a side length. You need to invert this formula when finding
an angle. As with finding side lengths, we use only two parts of the formula.
WORKED EXAMPLE 10

Find the size of the angle, , in the figure on the right, correct to the
nearest degree.

A
6 cm

110

20 cm

B
THINK

WRITE

Method 1: Technology-free
1

Write the formula.

Substitute A = 110, C = , a = 20 and c = 6.

Make sin the subject of the equation.

Calculate a value for sin .

Calculate sin1(0.2819) to find .

sin A sin C
=
a
c
sin 110 sin
=
20
6
6 sin 110
sin =
20
sin = 0.2819
= 16

Method 2: Technology-enabled
1

From the MENU select EQUA.

Press 3 (SOLV).

Delete any existing equation, enter the equation


sin 110 20 = sin X 6 and press w.
Note: Your calculator may display a different value of X
at this stage. This is just the last value of X stored in the
calculators memory.

Press 6 (SOLV) to solve the equation.

Chapter 7 Applications of trigonometry 189

The ambiguous case of the sine rule


Elesson
eles-0051
Ambiguous case of
the sine rule

As we saw earlier that a triangle can be drawn with three acute angles or one obtuse angle with two
acute angles
Consider finding the angle in the triangle below
10

15 m
C

17 m

sin A sin B
=
a
b
sin sin 10
=
17
15
17 sin 10
sin =
15
0.1968

Using your calculator gives the result 11.


However sin 169 = sin 11. (Check this on your calculator)
So there are two possible answers to this question, one acute (11, giving angles in the triangle 10,
11 and 159) and one obtuse (169, giving angles 10, 169 and 1).
11

10
B

159
17 m
10

15 m
C

A
169
17 m

15 m
1

In this course, when asked to find an angle, if two answers are possible you will be told whether to find
the acute or obtuse angle.
WORKED EXAMPLE 11

Find the size of the obtuse angle in the following triangle.

10 m

20

A
THINK
1

Write the formula for the sine rule.

Substitute the known values for b, c and sin C.

Make sin B the subject.

Calculate the value of sin B.

Use your calculator to find the acute angle B as the possible


acute angle.

Subtract the acute angle from 180 to find the obtuse angle .

12 m

WRITE

sin B sin C
=
c
b
sin B sin 20
=
12
10
sin B =

12 sin 20
10

0.4104
B = sin 1 1 0.4104 2
24
180 24
= 156

Note: When using the graphics calculator, you do not need to remember to invert the sine rule.
If you enter 20 sin 110 = 6 sin x, the graphics calculator will still solve the equation.
As with finding side lengths, some questions will be problems that require you to draw a diagram to
extract the required information and then write the answer.
190 Maths Quest HSC Mathematics General 2

WORKED EXAMPLE 12

P is a point that is due north of a second point Q. A ship (S) is located 12.4km from
Pand31.2km from Q such that QPS = 43.
Find PQS correct to the nearest degree.
THINK
1

Tutorial
int-2416
Worked
example12

WRITE

Draw a diagram.

12.4 km
43 S
31.2 km

Q
2

Write the formula.

sin Q sin P
=
q
p

Substitute for p, q and P.

sin Q sin 43
=
12.4
31.2

Make sin Q the subject.

sin Q =

Calculate a value for sin Q.

sin Q = 0.271

Calculate sin1(0.271) to find Q.

Exercise 7C

12.4 sin 43
31.2

Q = 16

Using the sine rule to find angles

1 WE10 Find the size of the angle marked with a pronumeral in each of the following, correct to the

nearest degree.
a

29.5 m
46 cm

R
Q 60
18.9 m

V
23.6 km

23.6 km

75

27.6 cm
86

C sin =

36 sin 13

7

7 sin 36
B sin =
13
D sin =

117

170

27 mm

156 mm

2 MC Which of the statements below give the correct value for sin ?

13 sin 36
A sin =

7

153 mm

79 mm

16.5 cm

32 cm

100
B

7
36

13

7 sin 13
36
Chapter 7 Applications of trigonometry 191

3 MC In which of the triangles below is the information insufficient to use the sine rule?
A

14.8 m

12.7 m

57

45
12.6 m

115

16.2 m
6.2 m

12.7 m

8.7 m
9

12.9 m

4 In Questions ac find the size of the angle marked , correct to the nearest degree.
a

b
c
44 cm

65 cm
23
41 m

4.9 m
31
60

32 m
3.6 m
5 WE11 Find the size of the angle in the triangle drawn below.
4.1 m

4.5 m

22

Give your answer correct to the nearest degree.


6 Find the size of the angle in each of the triangles below given that is obtuse.
a

b

10.1 cm
3.6 m
3.3 cm

19
20
9.2 m
c
12.1 cm
8.7 cm
33

7 In PQR, q = 12cm, r = 16cm and PRQ = 56. Find the size of PQR, correct to the nearest degree.
8 In KLM, LM = 4.2m, KL = 5.6m and KML = 27. Find the size of LKM, correct to the

nearest degree, given that the angle is obtuse.

9 WE12 A, B and C are three towns marked on

a map. Judy calculates that the distance


between A and B is 45km and the distance
between B and C is 32km. CAB is 45.
Calculate ACB, correct to the nearest
degree.
10 A surveyor marks three points X, Y and Z in

the ground. The surveyor measures XY to be


13.7m and XZ to be 14.2m. XYZ is 60.
a Calculate XZY to the nearest degree.
b Calculate YXZ to the nearest degree.
11 Two wires support a flagpole. The first wire

is 8m long and makes a 65 angle with the


ground. The second wire is 9m long.
Find the angle that the second wire makes
with the ground.
192 Maths Quest HSC Mathematics General 2

12 Construct a suitable triangle from the following instructions and find all unknown sides and angles.

The smallest side is 17cm and one of the other sides is 25cm. The smallest angle is 32.
13 MC A yacht sails the three-leg course shown.
15 km
The smallest angle between any two legs within the course, to the nearest
degree, is:
78
A 34
B 55
C 45
D 78
13 km
14 MC The correct expression for angle S in the given triangle is:
A sin 1 a

40

sin 41
30

B sin 1 a

30

sin 41
40

18 km

41
30

40

41 sin 41
30
D sin 1 a
b
b
30
40 sin 41
15 Ben is planning to hike in the mountains out of the snow season. From a position in front of the ski
lodge, Ben can see the chairlift station and the start of the ski run in the distance. He notes that in
moving his eye from the ski lodge station to the start of the run, the angle is 34. Ben then walks in
a straight line to the chairlift station, a distance of 365m, turns and walks the 230m straight line
distance to the start of the ski run. From here, what angle (to the nearest degree) would he note
between the ski lodge and the chairlift station?
C sin 1 a

Using the cosine rule to find


sidelengths

7D

Finding side lengths

When given the length of one side and two angles in a triangle, we can use the sine rule to find another
side length. However, in many cases we do not have this information and need another method of
calculating the side lengths. The cosine rule allows us to calculate the length of the third side of a
triangle when we are given the length of the other two sides and the included angle.
a2 = b2 + c2 2bc cos A
b2 = a2 + c2 2ac cos B
c2 = a2 + b2 2ab cos C
The formula sheet gives the third version of this formula only. The others are an adaptation of the samerule.
It is important to notice that the formula is given in terms of a2, b2 or c2. This means that to find the
value of a, b or c we need to take the square root of our calculation.

Derivation of the cosine rule


Consider ABC on the right. In this triangle, h is the perpendicular height
of the triangle and meets AB at D. We will let AD = x, and therefore
BD = c x.
Using Pythagoras theorem on BCD: a2 = (c x)2 + h2

a2 = c2 2cx + x2 + h2[1]
2
2
2
From ACD: b = x + h
Therefore:
h2 = b2 x2
Substituting for h2 in [1]: a2 = c2 2cx + x2 + b2 x2

a2 = c2 2cx + b2[2]
Now in ACD: cos A =

C
a
B

c x

h
D
c

x
b

Therefore:
x = b cos A
Substituting for x in [2]: a2 = c2 2c(b cos A) + b2

a2 = c2 + b2 2bc cos A
This becomes the formula for the cosine rule. A similar formula can be used for finding sides b and c.
You may like to try it for yourself.
Chapter 7 Applications of trigonometry 193

1 Start with ABC and draw a perpendicular line from A to BC.


2 Use this diagram and follow the method shown to obtain the following version of the cosine rule:

b2 = a2 + c2 2ac cos B.

3 Can you obtain c2 = a2 + b2 2ab cos C?


WORKED EXAMPLE 13

Find the length of the side marked b in the triangle below, correct to 1 decimal place.
B
70
10 m

A
THINK

12 m

b
WRITE

Method 1: Technology-free
b2 = a2 + c2 2ac cos B

Write the formula with b2 as the subject.

Substitute a = 12, c = 10 and B = 70.

= 122 + 102 2 12 10 cos 70

Calculate the value of b2.

= 161.915

Find b by taking the square root of b2.

Method 2: Technology-enabled
1

From the MENU select EQUA.

Press 3 (SOLV).

Delete any existing equation, enter B2 = 122 +


102 2 12 10 cos 70 and then press
w.

Press 6 (SOLV) to solve the equation.

b = !161.915
= 12.7 m

As with sine rule questions, we can apply the cosine rule to obtuse-angled triangles. You should recall
from the earlier investigation that the cosine ratio of an obtuse angle is negative. The method of solution
remains unchanged.
194 Maths Quest HSC Mathematics General 2

WORKED EXAMPLE 14

Find the unknown length (to 2 decimal places), x, in the following triangle.
7 cm

Tutorial
int-0468
Worked
example14

80
6 cm
THINK

WRITE

Identify the triangle as non-right-angled.

Label the triangle appropriately for the sine


rule or cosine rule.

B
a=x

c=7
A

80
b=6

Identify that it is the cosine rule that is


required as you have the two sides and the
angle in between.

b = 6 A = 80
c = 7
a=x

Substitute the known values into the cosine


rule formula.

a2 = b2 + c2 2bc cos (A)


x2 = 62 + 72 2 6 7 cos (80)
x2 = 36 + 49 84 cos (80)
x2 = 70.4136

Remember to get the square root value, x.

Evaluate the length and include units with the


answer.

x = !70.4136
= 8.391

x = 8.39
The unknown length is 8.39cm, correct to
2decimal places.

The cosine rule also allows us to solve a wider range of practical problems. The important part of
solving such problems is marking the correct information on your diagram. If you can identify two side
lengths and the included angle, you can use the cosine rule.
WORKED EXAMPLE 15

A surveyor standing at a point, X, sights a point, M, 50m away and a point, N, 80m away. If the
angle between the lines XM and XN is 45, calculate the distance between the points M and N,
correct to 1 decimal place.
THINK
1

WRITE

Draw a diagram and mark all given


information on it.

X
50 m

45

80 m
N

M
2

Write the formula with x2 as the subject.

Substitute m = 80, n = 50 and X = 45.

Calculate the value of x2.

Calculate x by taking the square root of

Give a written answer.

x2 = m2 + n2 2mn cos X
= 802 + 502 2 80 50 cos 45
= 3243.15

x2.

x = !3243.15
= 56.9m

Chapter 7 Applications of trigonometry 195

Using the cosine rule to find side lengths

Exercise 7D

1 Write down the cosine rule formula as it applies to each of the triangles below. In each case, make

the bold pronumeral the subject.


a
b
A
c

m
l

2 WE13 Find the length of the side marked with a pronumeral in each of the following, correct to

3significant figures.
a

12 m
35

42

13 cm

r
C

14 m

21 cm

12 m

60

12 m

R
Y

3 WE14 In each of the following obtuse-angled triangles, find the length of the side marked with the

pronumeral, correct to 1 decimal place.


b A
X

112 cm

114 cm

110

Z
x

R
q

6.1 m
B

130

9.7 m

63 mm

160
43 mm Q

4 MC In which of the following triangles are we unable to use the cosine rule to find x?
A

B
14.8 cm
132
16.4 m
16.2 cm
x
x
32
18.2 m

63
8.3 km

9.6 km

10.5 m
63

9.7 m

5 MC Look at the triangle drawn on the right. The value of x, correct to

1 decimal place, is:


A 7.2m
B 7.3m
C 52.4m
D 52.5m

50
8m

9m

6 MC Lieng is asked to find the value of a, correct to 1 decimal place,

in the figure drawn on the right. Below is Liengs solution.


Line 1: a2 = 122 + 82 2 12 8 cos 60
Line 2:
= 144 + 64 192 cos 60
Line 3:
= 208 192 cos 60
Line 4:
= 16 cos 60
Line 5:
=8
Line 6:a = 2.8m
Liengs solution is incorrect. In which line did she make her error?
A Line 2
B Line 3
C Line 4

60
8 cm

12 cm

a
D Line 5

7 In ABC, a = 14cm, c = 25cm and ABC = 29. Calculate b, correct to 1 decimal place.
196 Maths Quest HSC Mathematics General 2

8 In PQR, PQ = 234mm, QR = 981mm and PQR = 128. Find the length of side PR, correct to

3significant figures.

9 Len and Morag walk separate paths that diverge from one another at an angle of 48. After three

hours Len has walked 7.9km and Morag 8.6km. Find the distance between the two walkers at this
time, correct to the nearest metre.
10 A cricketer is fielding 20m from the batsman and at an angle of 35 to the pitch. The batsman hits

a ball 55m and straight behind the bowler. How far must the fieldsman run to field the ball? (Give
your answer to the nearest metre.)
11 The sides of a parallelogram are 5.3cm and 11.3cm. The sides meet at angles of 134 and 46.
a Draw a diagram of the parallelogram showing this information and mark both diagonals on it.
b Calculate the length of the shorter diagonal, correct to 1 decimal place.
c Calculate the length of the long diagonal, correct to 1 decimal place.
12 The cord supporting a picture frame is 58cm long. It is hung over

58 cm

a single hook in the centre of the cord and the cord then makes an
angle of 145 as shown in the figure on the right. Calculate the length
of the backing of the picture frame, to the nearest centimetre.

145

Further development

13 During a sailing race, the boats followed a course as shown.


14 Two circles, with radii 4cm and 7cm, overlap slightly as shown.

7 km

105

Findthelength, x, of its third leg (to 1decimal place).


10 km

If the angle between the two radii that meet at the point of
intersection of the circumferences is 110, find the distance
between the centres of the circles (to 1 decimal place).

15 Two hikers set out from the same point. The hikers walk 1200m

and 1500m respectively and diverge at an angle of 100. How far apart
to the nearest metre are the two hikers?
16 An advertising balloon is attached to two ropes 120m and 100m long.

4 cm

7 cm
110

The shorter rope makes a 70 angle with the ground and is attached
tothe bottom of the balloon. The longer rope makes an 80 angle
withthe horizontal and is attached to the top of the balloon. How tall is
the balloon? Give your answer correct to 2 decimal places.

7E Using the
Finding angles

Digital doc
doc-11069
WorkSHEET 7.1

cosine rule to find angles

We can use the cosine rule to find the size of the angles within a triangle. Consider the cosine rule formula.
a2 = b2 + c2 2bc cos A
We now make cos A the subject of this formula.
a2 = b2 + c2 2bc cos A
2
a + 2bc cos A = b2 + c2
2bc cos A = b2 + c2 a2
b2 + c2 a2
cos A =
2bc
In this form, we can use the cosine rule to find the size of an angle if we are given all three side lengths.
We should be able to write the cosine rule in three forms depending upon which angle we wish to find.
b2 + c2 a2
cos A =
2bc
a2 + c2 b2
cos B =
2ac
a2 + b2 c2
cos C =
2ab
Again, the formula sheet gives the third version of this formula only. The others are an adaptation of
the same rule.
Chapter 7 Applications of trigonometry 197

WORKED EXAMPLE 16

Find the size of angle B in the triangle, correct to the nearest degree.
A
7 cm

5 cm

9 cm

THINK

C
WRITE

Method 1: Technology-free
1

Write the formula with cos B as the subject.

Substitute a = 9, b = 5 and c = 7.

Calculate the value of cos B.

Make B the subject of the equation.

Calculate B.

a2 + c2 b2
2ac
92 + 72 52
cos B =
297
105
cos B =
126
= 0.8333
B = cos1(0.8333)
cos B =

B = 34

Method 2: Technology-enabled
1

From the MENU select EQUA.

Press 3 (SOLV).

Delete any existing equation, enter the equation


cos B = (92 + 72 52) (2 9 7), and then
pressw.

Press 6 (SOLV) to solve the equation.

Your formula sheet will give you two versions of the cosine rule, one for finding a side length and one
for finding an angle. When using the equation solver it does not matter which version you use to find a
side or an angle.
Try using the solver on the equation 52 = 92 + 72 2 9 7 cos B.
As we found earlier, the cosine ratio for an obtuse angle will be negative. So, when we get a negative
result to the calculation for the cosine ratio, this means that the angle we are finding is obtuse. Your
calculator will give the obtuse angle when we take the inverse.
198 Maths Quest HSC Mathematics General 2

WORKED EXAMPLE 17

Find the size of angle Q in the triangle, correct to the nearest degree.
Q
4 cm

3 cm

6 cm

THINK

WRITE

cos Q =

p2 + r2 q2
2pr

cos Q =

32 + 42 62
243

Write the formula with cos Q as the subject.

Substitute p = 3, q = 6 and r = 4.

Calculate the value of cos Q.

Make Q the subject of the equation.

11
24
= 0.4583
Q = cos1(0.4583)

Calculate Q.

Q = 117

cos Q =

In some cosine rule questions, you need to work out which angle you need to find. For example, you
could be asked to calculate the size of the largest angle in a triangle. To do this you do not need to
calculate all three angles. The largest angle in any triangle will be the one opposite the longest side.
Similarly, the smallest angle will lie opposite the shortest side.

WORKED EXAMPLE 18

Find the size of the largest angle in the triangle.


R
3.4 m

4.9 m

S
5.7 m
THINK
1

ST is the longest side, therefore angle R is the largest angle.

Write the formula with cos R the subject.

Substitute r = 5.7, s = 4.9 and t = 3.4.

Calculate the value of cos R.

Make R the subject of the equation.

Calculate R.

Give a written answer.

T
WRITE

cos R =

s2 + t2 r2
2st

4.92 + 3.42 5.72


2 4.9 3.4
3.08
cos R =
33.32
= 0.0924
R = cos1(0.0924)
cos R =

R = 85
The largest angle in the triangle is 85.

Chapter 7 Applications of trigonometry 199

WORKED EXAMPLE 19

Two paths diverge from a point, A. The first path goes for 1.25km to a point, B. The second path
goes for 1.4km to a point, C. B and C are exactly 2km apart. Find the angle at which the two
paths diverge.
THINK
1

WRITE

Draw a diagram.

2 km

1.25 km

A
2

Write the formula with cos A as the subject.

Substitute a = 2, b = 1.4 and c = 1.25.

Calculate the value of cos A.

Make A the subject of the equation.

Calculate the value of A.

Give a written answer.

Exercise 7E

1.4 km

cos A =

b2 + c2 a2
2bc

cos A =

1.42 + 1.252 22
2 1.4 1.25

0.4775
3.5
= 0.1364
A = cos1(0.1364)

cos A =

= 98
The roads diverge at an angle of 98.

Using the cosine rule to find angles

1 For each of the following, write the cosine rule formula as it applies to the triangle drawn with the

bold angle as the subject.


a

b P

P
B

2 WE16 Find the size of the angle marked with the pronumeral in each of the following triangles,

correct to the nearest degree.


a

A
8 cm

11 cm

3.2 m

2.8 m

M
4.5 m

5.4 m

C
B

13 cm

4.0 m

6.2 m

3 WE17 In each of the obtuse-angled triangles below find the size of the angle marked with the

pronumeral, to the nearest degree.


b

6m

9.6 m

8m

4.2 m
11 m

c
12.9 m

6.1 m
4.2 m

200 Maths Quest HSC Mathematics General 2

9.2 m

4 MC Look at the figure drawn below.


3 cm

5 cm
7 cm

Which of the following correctly represents the value of cos ?


32 + 72 52
32 + 72 52
A cos =

B cos =
237
257
2
2
2
2
3 +5 7
5 + 72 32
C cos =

D cos =
235
257
5 MC In which of the following is the angle obtuse?
A

B
3 cm

4 cm

3 cm

5 cm

4 cm

3 cm

4 cm

4 cm

4 cm

3 cm

4 cm

6 cm

6 In PQR, p = 7m, q = 9m and r = 6m. Find QRP, correct to the nearest degree.

7 In KLM, k = 85mm, l = 145mm and m = 197mm. Find the size of the smallest angle, correct to

the nearest degree.


8 WE18 Calculate the size of all three angles (correct to the nearest degree) in a triangle with side
lengths 12cm, 14cm and 17cm.
9 WXYZ is a parallelogram. WX = 9.2cm and XY = 13.6cm. The diagonal WY = 14cm.
a Draw a diagram of the parallelogram.
b Calculate the size of WXY, correct to the nearest degree.
10 WE19 Two roads diverge from a point, P. The first road is 5km long and leads to a point, Q. The
second road is 8km long and leads to a point, R. The distance between Q and R is 4.6km. Calculate
the angle at which the two roads diverge.
11 A soccer goal is 8m wide.
a A player is directly in front of the goal such that he is 12m from each post. Within what angle
must he kick the ball to score a goal?
b A second player takes an angled shot. This player is 12m from the nearest post and 17m from the
far post. Within what angle must this player kick to score a goal?

Chapter 7 Applications of trigonometry 201

12 The backing of a picture frame is 50cm long and is hung over a picture

52 cm

hook by a cord 52cm long as shown in the figure on the right.


Calculate the angle made by the cord at the picture hook.

Further development
50 cm

13 A garden bed is in the shape of a triangle, with sides

of length 3m, 4.5m and 5.2m.


a Calculate the smallest angle.
b Hence, find the area of the garden.
14 A hockey goal is 3m wide. When Sophie is 7m from one post and 5.2m from the other, she shoots

for goal. Within what angle must the shot be made if it is to score a goal?
15 A plane flies 80km and then turns 120 right and flies a further 150km.
a How far is the plane from its starting point?
b What angle must the plane then turn to return to its starting point?
16 Three circles of radii 5cm, 6cm and 8cm are positioned so that they just touch one another. Their

centres form the vertices of a triangle. Find the largest angle in the triangle.
17 From the top of a vertical cliff 68m high, an observer notices a yacht at sea. The angle of depression

to the yacht is 47. The yacht sails directly away from the cliff, and after 10min the angle of
depression is 15. How fast is the yacht sailing?

7F

Area of a triangle

You should be familiar with finding the area of a triangle using the
formula Area = 12 bh. In this formula, b is the base of the triangle and h is
the perpendicular height. This formula cannot be used in triangles where
we do not know the perpendicular height. Trigonometry allows us to find
the area of such triangles when we are given the length of two sides and
the included angle.
Consider the triangle drawn on the right. In this triangle:

A
c

h
D

C
a

1
Area = bh[1]
2

(a = base of triangle, h = height)


Now consider ACD. Since this triangle is right angled:
opp
hyp
h
sin C =
b
h = b sin C

sin C =

Substituting for h in [1]:


1
Area = absin C
2
This becomes the formula for the area of a triangle. There are three equivalent formulas for the area of
a triangle.
Area = 12 ab sin C
Area = 12 ac sin B
Area = 12 bc sin A
The formula sheet gives the first version of this formula only. The others are an adaptation of the same
rule. These formulas allow us to find the area of any triangle where we are given the length of two sides
and the included angle. The included angle is the angle between the two given sides. The formula chosen
should be the one that uses the angle you have been given.
202 Maths Quest HSC Mathematics General 2

WORKED EXAMPLE 20

Find the area of the triangle (to 2 decimal places).


Tutorial
int-0469
Worked
example20

9m
37 6 m

THINK
1

Identify the shape as a triangle with two known


sides and the angle in between.

WRITE

A
b=9

B
37 a = 6
C

Identify and write down the values of the two sides,


a and b, and the angle in between them,C.

a=6
b=9
C = 37

Identify the appropriate formula and substitute the


known values into it.

Areatriangle =

1
ab sin (C)
2
1
= 6 9 sin (37)
2
= 16.249

Write the answer in correct units.

The area of the triangle is 16.25m2, correct to


2decimal places.

WORKED EXAMPLE 21

Two paths diverge at an angle of 72. The paths lengths are 45m and 76m respectively.
Calculate the area between the two paths, correct to the nearest square metre.
THINK
1

WRITE

Draw a diagram.
45 m
72

Area =

76 m

1
ab sin C
2

Write the formula.

Substitute a = 45, b = 76 and C = 72.

Calculate.

= 1626m2

Give a written answer.

1
45 76 sin 72
2

The area between the paths is 1626m2.

Chapter 7 Applications of trigonometry 203

Exercise 7F

Area of a triangle

1 Write down the formula for the area of a triangle in terms of each of the triangles drawn below. Write

the formula using the bold angle.


a

b X
B

2 For each of the triangles drawn below, state whether the area would be best found using the formula

Area =
a

1
1
ab sin C or Area = bh.
2
2

6 cm
1.9 m

60
12 cm

2.6 m

6.2 m
9.1 m

8.3 m

60
12.4 m
3 WE20 Find the area of each of the following triangles, correct to 1 decimal place.
a
b

c
11 cm

196 mm

40

207 mm

117 mm
120
92 mm

10

12 cm

1
1
ab sin C or Area = bh to find the area of each of the following triangles.
2
2
Where necessary, give your answer correct to 1 decimal place.
a

b

c

4 Use either Area =

38 cm

32 cm
19 cm
66
38 cm

14 cm

32 cm

1
5 MC In which of the following triangles can the formula Area = ab sin C not be used to find the
2
area of the triangle?
A

4 cm

4 cm

60
9 cm

9 cm

4 cm

75

9 cm
120
4 cm

204 Maths Quest HSC Mathematics General 2

9 cm

6 MC The area of the triangle on the right (correct to 1 decimal place) is:
A
B
C
D

4.4cm2
14.7cm2
17.1cm2
20.5cm2

5 cm

7 cm

78
6 cm

7 In PQR, p = 4.3cm, q = 1.8cm and PRQ = 87. Calculate the area of

PQR, correct to 4 significant figures.


A
8 The figure on the right is of a parallelogram, ABCD.
a Copy the diagram into your workbook and draw the diagonal AC on 2.5 m
your diagram.
70
b By considering the parallelogram as two equal triangles, calculate
D
5.2 m
its area, correct to 1 decimal place.
9 On the right is a diagram of a pentagon inscribed in a circle of radius 5cm.
a Calculate the size of each of the angles made at the centre.
b Calculate the area of the pentagon, correct to the nearest square
centimetre.

10 WE21 A surveyor sights the four corners of a block of land and

makes the following notebook entry. Calculate the area of the block of
land, correct to the nearest square metre.

18 m
20 m
90
80 70
15 m
120
25 m

Further development
11 Find the area of an equilateral triangle with a side length of 10cm.
12 MC

4m

30 6 m
50

The correct expression for the area of the shape above is:
1
6 4 sin 80
2
1
B
6 4 cos 80
2
1
C 6 4 sin 30
2
1
D 6 4 sin 100
2
A

Chapter 7 Applications of trigonometry 205

13 MC The correct expression for the area of the octagon shown is:

11 sin 45
88 sin 67.5
88 sin 45
11 sin 67.5
14 Consider the triangle ABC drawn below.
A
B
C
D

5.5

B
15 m

105

18 m

a Find the area of the triangle.


1
b Use your answer to part a together with the formula A = bh to find the shortest distance of the

2
point A from the line BC.
15 The triangle PQR has side lengths PQ = 15cm, QR = 22cm and PQR = 75.
a Find the area of the triangle.
b Betty draws the triangle by mistake with PQR = 105. Show that Betty will still get the correct
answer.
c Explain why the same answer for the area of the triangle is obtained.
16 Penny is making a triangular display case as shown in the diagram.
30 cm
30 cm
The two sides are to be 30cm in length. Find the angle between the two
sides that will maximise the area of the triangular cross-section.

7G

Bearings

A bearing is an angle used to describe direction. Bearings are used in navigation and are a common
application of trigonometry to practical situations. We can therefore apply our trigonometrical formulas
to make calculations based upon these bearings. There are two types of bearing that we need to be able
to work with: compass bearings and true bearings.

Compass bearings
Interactivity
int-0190
Navigation and
specifications of
locations

N
NW

Compass bearings use the four points of the compass. With compass
bearings there are four main directions: north, south, east and west. In
between each of these main directions there are four others: north-east,
south-east, south-west and north-west. Each of these directions is at 45 to
two of the four main directions.
Trigonometry can then be used to solve problems about distances and
angles using these eight basic directions.

NE

SW

SE
S

WORKED EXAMPLE 22

A ship (A) is 10 nautical miles due east of a lighthouse. A second ship (B) bears SE of the
lighthouse and is due south of the first ship. Calculate the distance of the second ship from the
lighthouse, correct to 1 decimal place.
THINK
1

Draw a diagram labelling the sides of the triangle.

WRITE

adj
10 M

45

hyp
x

opp

206 Maths Quest HSC Mathematics General 2

Choose the cosine ratio and write the formula.

Substitute for and the adjacent side.

Make x the subject of the equation.

adj
hyp
10
cos 45 =
x
cos =

x cos 45 = 10
x=

Calculate the value of x, correct to 1 decimal place.

Give a written answer.

10
cos 42

= 14.1 M
The second ship is 14.1 nautical miles from
the lighthouse.

These eight compass points do not allow us to make calculations about more precise directions. For this
reason an alternative method of describing bearings is needed for any direction other than these basic
eight points.

True bearings

A true bearing is an angle measured from north in a clockwise direction.


As there are 360 in a revolution, all true bearings are represented as a
three-digit number between 000 and 360. For example, east is at a
bearing of 090, south has a bearing of 180 and west 270.
When given information about a bearing, we can solve problems using
trigonometry by constructing a right-angled triangle.

270

090

180
WORKED EXAMPLE 23

A ship sails on a bearing of 130 for a distance of 10 nautical miles. Calculate how far south of its
starting point the ship is, correct to 2 decimal places.
THINK
1

Draw a diagram completing a right-angled triangle


and label the sides.

WRITE

N
130
50
adj
x

hyp
10 M

opp

cos =

adj
hyp

Choose the cosine ratio and write the formula.

Substitute for and the hypotenuse.

Make x the subject of the equation.

x = 10 cos 50

Calculate the value of x.

x = 6.43 M

Give a written answer.

cos 50 =

x
10

The ship is 6.43 nautical miles south of its


starting point.

We can also use our methods of calculating angles to make calculations about bearings. After solving the
right-angled triangle, however, we need to provide the answer as a bearing.
Chapter 7 Applications of trigonometry 207

WORKED EXAMPLE 24

On a hike Lisa walked south for 3.5km and then turned west for 1.2km. Calculate Lisas bearing
from her starting point.

WRITE

Draw a diagram and label the sides of the triangle.

hyp

3.5 km
adj

THINK

1.2 km
opp

opp
adj
1.2
tan =
3.5
= 0.3429
tan =

Choose the tangent ratio and write the formula.

Substitute for the opposite and adjacent sides and


simplify.

Make the subject of the equation.

Calculate .

From the diagram we can see the angle lies


between south and west. South has a bearing of
180, and so we must add 19 to 180 to calculate
the true bearing.

Bearing = 180 + 19
= 199

Give a written answer.

Lisa is at a bearing of 199 from her starting


point.

= tan1(0.3429)
= 19

With many bearing questions it will be necessary to use the work completed or sine rule and/or
cosinerule.
WORKED EXAMPLE 25

Tutorial
int-0473
Worked
example25

Soldiers on reconnaissance set off on a return journey from their base camp. The journey
consists of three legs. The first leg is on a bearing of 150T for 3km; the second is on a bearing of
220T for 5km. Find the direction (to the nearest minute) and distance (correct to 2 decimal
places) of the third leg by which the group returns to its base camp.
THINK
1

Draw a diagram of the journey and indicate or


superimpose a suitable triangle.

WRITE

N
Base
camp

150
3 km
N

220T
5 km

208 Maths Quest HSC Mathematics General 2

Identify the side of the triangle to be found.


Redraw a simple triangle with the most important
information provided.

B
3 km

110

150

30

30
30

5 km

110
220

40

40

A
3

Identify that the problem requires the use of the


cosine rule, as you are given two sides and the
angle in between.

a = 3km b = 5km C =110 c = x km

Substitute the known values into the cosine rule and


evaluate.

c2 = a2 + b2 2ab cos (C)


x2 = 32 + 52 2 3 5 cos (110)
x2 = 44.260604
x = !44.260 604
= 6.65

For direction, we need to find the angle between the


direction of the second and third legs using the sine
or cosine rules.

B
N

40

A
40

Substitute the known values into the rearranged


cosine rule.

a = 3 b = 5 c = 6.65 or !44.260 604


cos (A) =

b2 + c2 a2
2bc

52 + 44.260 604 32
2 5 !44.260 604
cos (A) = 0.9058
A = 25.07
= 254

Note: Use the most accurate form of the length of


side c.

cos (A) =

Calculate the angle of the turn from the north


bearing.

= 40 254
= 1456
Bearing is N1456E.

Write the answer in correct units and to the required


level of accuracy.

The distance covered in the final leg is


6.65km, correct to 2 decimal places, on a
bearing of N1456E, correct to the nearest
minute.

Chapter 7 Applications of trigonometry 209

Exercise 7G

Bearings

1 Specify the following directions as compass bearings.


a

b
N

25
W

E
10
S

S
c

E
310

E
12

S
e

12
W

12
W

S
2 Specify the following directions as true bearings.
a

b
N

22 12

45
S

S
c

67 12

135
W

210 Maths Quest HSC Mathematics General 2

112 12
W
SW

22 12
S

3 WE22 A road runs due north. A hiker leaves the road and walks for 4.2km in a NW direction.
a Draw a diagram of this situation.
b How far due east must the hiker walk to get back to the road? (Give your answer correct to

Digital doc
SkillSHEET 7.7
doc-11070
Converting
nautical miles to
kilometres

3decimal places.)
4 A driver heads due south for 34km, then turns left and drives until he is SE of his starting point.
a Draw a diagram to show the drivers journey.
b Calculate the distance the driver travelled in an easterly direction from his starting point.
5 Two boats, A and B, sail from a port. A heads due west, while B heads NW for a distance of

43nautical miles, where it drops anchor. Boat A drops anchor due south of boat B.
a Draw a diagram showing the positions of boats A and B.
b Calculate the distance between boats A and B in nautical miles, correct to 1 decimal place.
c Calculate the distance in kilometres between A and B.
6 MC A true bearing of 315 is equivalent to a compass

bearing of:
A NE
B NW
C SE
D SW
7 MC A compass bearing of SE is equivalent to a true bearing

of:
A
B
C
D

045
135
225
315

walks 5km due north to a point, A, and Bertrand walks on a bearingof


052 to a point, B. Bertrand lets off a flare and Adrian notices Bertrand
is now due east of him, as shown in the diagram on the right. Calculate
the distance between the two hikers, correct to 1 decimal place.

5 km

8 WE23 Two hikers, Adrian and Bertrand, set out on a walk. Adrian

52

9 A yacht sights a lighthouse on a bearing of 060. After sailing another

eight nautical miles due north, the yacht is due west of the lighthouse.
a Draw a diagram of this situation.
b Calculate the distance from the yacht to the lighthouse when it is due west of it (correct to
1decimal place).
10 An aeroplane takes off from an airport and flies on a bearing of 220 for a distance of 570km.

Calculate how far south of the airport the aeroplane is (correct to the nearest kilometre).
11 A camping ground is due east of a car park. Eden and Jeff walk 3.8km due south from the camping

ground until the car park is on a bearing of 290.


a Draw a diagram showing the car park, the camping ground, and Eden and Jeffs position.
b Calculate the distance Eden and Jeff need to walk directly back to the car park, correct to
1decimal place.
12 MC A ship is on a bearing of 070 from a lighthouse. The bearing of the lighthouse from the ship

will be:
A 070

B 160

C 200

D 250

Chapter 7 Applications of trigonometry 211

13 MC A camping ground is SW of a car park. The bearing of the car park from the camping ground

will be:
A NE

B NW

C SE

D SW

14 WE24 A search party leaves its base and head 4km due west before turning south for 3.5km.
a Draw a diagram of this situation.
b Calculate the true bearing of the search party from its base, correct to the nearest degree.
15 WE25 A ship is two nautical miles due west of a harbour. A yacht that sails 6.5 nautical miles from

that harbour is due north of the ship. Calculate the true bearing (correct to the nearest degree) of the
course on which the yacht sails from the harbour.
16 A plane takes off at 10.00 am from an airfield and flies at 120km/h on a bearing of 325. A second
plane takes off from the same airfield and flies on a bearing of 100 at a speed of 90km/h. How far
apart are the planes at 10.25 am?
17 a Two lighthouses are 17kilometres apart on an eastwest line. From lighthouseA, a ship is seen
on a bearing of 130. From lighthouse B, the same ship is spotted on a bearing of 200. Which
lighthouse is the ship closer to? How far is that lighthouse from the ship?
b Two lighthouses are 25 kilometres apart on a southnorth line. From lighthouse A, a ship is
reported on a bearing of 082T. The same ship is detected from lighthouse B on a bearing of
165T. Which lighthouse is closer to the ship and how far is that lighthouse from the ship?
c Two fire-spotting towers are 33km apart on an eastwest line. From Tower A a fire is spotted on a
bearing of 63, while from Tower B the same fire is spotted on a bearing of 290T. How far away
from the nearer tower is the fire? Which tower is this?
18 Two lighthouses are 25km apart on a northsouth line. The northern lighthouse spots a ship on a
bearing of 1200. The southern lighthouse spots the same ship on a bearing of 050T.
a Find the distance from the northern lighthouse to the ship.
b Find the distance from the southern lighthouse to the ship.
19 A light aircraft has strayed into a major
air corridor. It has been detected by two air
traffic control towers.
Tower 1 has the light aircraft on a bearing of
315T.
N
Tower 2 has the light aircraft on a bearing of
Tower 2
north.
E
The two towers are 300 kilometres apart on a
N
300 km
NE line as shown. How far is the light plane
from each tower?
Tower 1

Further development
20 Two army camps A and B are on the same eastwest line. Radio tower T is located 20km from camp

A, SE of camp A. The tower is a distance of 15km from camp B. Find the bearing of the radio tower,
T, from camp B.
21 Maria cycles 12km in a direction 292 and then 7km in a direction of 034E.
a How far is she from her starting point?
b What is the bearing of the starting point from her finishing point?
22 For each of the following, find how far north/south and east/west position A is from position O.
N75E
a
b
N 40E
100 m

N 50E

100 m
N

200 m

200 m
O

212 Maths Quest HSC Mathematics General 2

c
240T

d O

30 km

A
750 m

N10W
150 m

25 cm

S60E

160T

A
23 The distances covered in a yachting regatta are shown in the diagram.

Find (to the nearest metre):


a how far south the yacht is from the starting point
b how far west the yacht is from the starting point
c the distance from the starting point
d the direction of the final leg to return to the starting point.

N
Starting
point

0.5 km

N50W

N40W

3 km

1.5 km
S15W

7H

Radial surveys

In the preliminary course we examined the offset survey. In this survey method an area is measured
by drawing a traverse line and measuring offsets at right angles to the traverse line. Because the offset
survey created right-angled triangles, the length of each boundary could be calculated using Pythagoras
1
theorem and the area could be calculated using the formula Area = bh.
2
An alternative survey method to this is a radial survey. One type of radial survey is the plane table
radial survey. The following steps are taken in a plane table survey.
1. A table is placed in the centre of the field to be surveyed, each corner of the field is sighted and a line
is ruled on the paper along the line of sight.

2. The distance from the plane table to each corner is then measured.
20

25

26 m

28 m

3. The angle between each radial line is then measured and the radial lines joined to complete the
diagram.
m

115
60

115

70

28

26 m

20

25

Chapter 7 Applications of trigonometry 213

The field will then be divided into triangles. The length of each side of the field can then be calculated
by using the cosine rule. The perimeter of the field is then found by adding the lengths of each side.

WORKED EXAMPLE 26

The figure on below is a plane table survey of a block of land. Calculate the perimeter of the
block of land, correct to the nearest metre.
A
23
m

30 m

110
125 4017 m
85

28 m

D
THINK

WRITE

Apply the cosine rule in AXB to calculate


the length of AB.

For AXB:
x2 = a2 + b2 2ab cos X
= 302 + 232 2 30 23 cos 110
= 1900.99
x = 43.6m
The length of AB is 43.6m.

Apply the cosine rule in BXC to calculate the


length of BC.

For BXC:
x2 = b2 + c2 2bc cos X
= 172 + 302 2 17 30 cos 40
= 407.63
x = 20.2m
The length of BC is 20.2m.

214 Maths Quest HSC Mathematics General 2

Apply the cosine rule in CXD to calculate


the length of CD.

For CXD:
x2 = c2 + d2 2cd cos X
= 282 + 172 2 28 17 cos 85
= 990.03
x = 31.5m
The length of CD is 31.5m.

Apply the cosine rule in DXA to calculate


the length of DA.

For DXA:
x2 = d2 + a2 2da cos X
= 232 + 282 2 23 28 cos 125
= 2051.77
x = 45.3m
The length of DA is 45.3m.

Calculate the perimeter by adding the length of Perimeter = 43.6 + 20.2 + 31.5 + 45.3
each side and rounding the answer to the nearest
= 140.6m
metre.
= 141m (correct to the nearest metre)

A similar approach is used to calculate the area of such a field. The area of each triangle is found using
the formula Area = 12 ab sin C. The total area is then found by adding the area of each triangle.
WORKED EXAMPLE 27

52

Calculate the area of the field on the right. Give A


your answer correct to the nearest square metre.

48

96
144 120

67 m

C
THINK

WRITE

Calculate the area of AXB.

1
For AXB: Area = ab sin X
2
1
= 48 52 sin 96
2
= 1241.2m2

Calculate the area of BXC.

For BXC: Area =

Calculate the area of CXA.

For CXA: Area =

Calculate the total area by adding the area of


each triangle.

Total area = 1241.2 + 1392.6 + 1023.9


= 3657.7m2
= 3658m2 (correct to the nearest m2)

1
bc sin X
2
1
= 67 48 sin 120
2
= 1392.6m2

1
ca sin X
2
1
= 52 67 sin 144
2
= 1023.9m2

An alternative to the plane table radial survey is the compass radial survey. In this survey the bearing of
each radial line is calculated and this bearing is used to calculate the angle between each radial, as in the
Chapter 7 Applications of trigonometry 215

worked example below. The method of calculating the perimeter and area of the field is then the same
asfor the plane table radial survey.
WORKED EXAMPLE 28

The figure on the right shows a compass radial


survey of a block of land.
a Calculate the size of AXB.
b
Hence, calculate the distance AB, correct to the
nearest metre.

338

B
49

58

067

55

57 m

114

239

THINK

WRITE

a A is 22 west of North, B is 67 east of North.

a 22 + 67 = 89

b 1 Write the cosine rule formula.

b For AXB:

x2 = a2 + b2 2ab cos X

Substitute for a, b and X.

= 492 + 582 2 49 58 cos 89

Calculate the value of x2.

= 5665.8

Calculate x.

Write your answer.

x = 75m (correct to the nearest metre)


The distance AB is 75m.

Radial surveys

Exercise 7H

1 WE26 The figure below is a plane table radial survey of a block of land. Use the cosine rule to

15

m
20

10
m

calculate the perimeter of the block of land, correct to the nearest metre.
100
70 80
110

25

80

95 75 m
115
150

92 m

1
12

45
m

100 m

114

m
60

55 m

70
85 80
125
45

2 Calculate the perimeter of each of the following areas, correct to the nearest metre.
a
b

c

90 40
60
89 m
140 30

78

3 WE27 The figure below is a plane table survey of a block of land. Calculate the area of the block,

correct to the nearest square metre.

11
216 Maths Quest HSC Mathematics General 2

160
0m
60 100 8
40

100 m

0m

90 m

4 For each of the plane table surveys shown in question 2 calculate the area, correct to the nearest

square metre.
A

315

50

5 WE28 The figure on the right is a compass radial survey of a field.


a Calculate the size of AXB.
b Hence, use the cosine rule to calculate the distance AB, correct to

60

40 m

the nearest metre.

m
X
70 m

170

040

110 C

D
6 Calculate the perimeter of the field given by the compass radial survey

below. Give your answer correct to the nearest metre.


350

100 m
90 m

30 m

110

250

7 Calculate the perimeter of each of the compass radial surveys shown below.
a 327
b 339

c 319
030

020

m
8

63

085

226

49 m

38

097

215

m
29

99 m

38 m

10

42

72

114

53

24 m

052

170

196

8 For each of the compass radial surveys in question 7 calculate the area, correct to the nearest square

metre.

Further development
9 The figure below is that of a triangular field.
A000
20 m
B240

30 m

35 m
C120

a Find the area of the field correct to 2 decimal places.


b Find the perimeter of the field correct to 2 decimal places.
c An alternative formula for the area of a triangle is A = !s(s a) (s b) (s c), where a, b and

c are the side lengths of the triangle and s =

with that obtained in question 9a.

a+b+c
. Use this formula and compare the result
2

10 ABCD is a square field. O is a point in the centre of the square such that it is 10metres from each corner.
a Use the cosine rule to find each side length.
b Find the area of triangle OAB and find the area of the square by multiplying this result by the

number of triangles that make up the square.

c Compare the answer to part b to the area of the square found using A = l 2.
Chapter 7 Applications of trigonometry 217

11 A surveyor, at point S sights two trees, A and B, on the opposite side of a river as being a distance of

52m and 64m away on bearings of 320 and 030 respectively.


a Find the distance AB correct to 2 decimal places.
b Find the area of the triangle ABS correct to 2 decimal places.
c Use your answers to a and b to find the width of the river.
12 Jason is at point A. Peta is 100m from Jason on a bearing 090 at point X. Jason observes a tower at
point B on a bearing 050 and a tree at a bearing of 150 at Y. Peta notes the bearing of the tower as
310 and tree as 240.
a Use this information to draw a 1:1000 sketch.
b Use this diagram to find the length of:
i AX
ii BX
iii AY
iv YB
v XY
c An observer is at X. Give the bearing of:
i A
ii B
iii Y
d Calculate the area of ABXY to the nearest 100m2.
13 Cameron sets up a plane table at O and from this point makes the following notes about the
surrounding features A to E.
Feature
A
B
C
D
E

Distance to feature
25m
35m
20m
40m
65m

Bearing of feature from O


045
120
180
250
325

What surveying method is Cameron using?


Draw a neat sketch representing this information. Use a 1:1000 scale.
An observer is at E. From E, what would you expect to be the bearings of A, B, C and D?
Estimate the area enclosed by the perimeter linking features of A to E. Express to the nearest
10m2.
14 Margaret places a plane table directly over point A and sights B, 60m from her on a bearing of 020,
and D, 50m away on a bearing of 100. She moves the plane table to B and notes the bearing of C,
70m from her, to be 090.
a Draw a neat 1:1000 sketch showing this information.
b Margaret then moves the plane table to C. What bearing is expected of:
i D
ii B?
c What is the distance from C to:
i A
ii D?
d Determine the area of ABCD to the nearest 100m2.
a
b
c
d

Investigate: Conducting a radial survey

Digital doc
doc-11071
WorkSHEET 7.2

Choose an appropriate area in or near your school to conduct a radial survey.


1. Set up a table in the centre of the area and tape a large piece of paper to the table.
2. Mark a point in the middle of the piece of paper and sight each corner of the field from this point,
ruling a line from the point in that direction.
3. Use a tape or trundle wheel to measure the distance from the table to each corner of the field.
4. Use your protractor to measure the angle between each radial line.
5. Calculate the area and the perimeter of the field.

218 Maths Quest HSC Mathematics General 2

Summary
Review of right-angled
triangles

The formulas to be used when solving right-angled triangles are:


opposite side
sin =
hypotenuse
cos =
tan =

adjacent side
hypotenuse
opposite side

adjacent side
To calculate a side length, you need to be given the length of one other side and one angle.
To calculate the size of an angle, you need to be given two side lengths.
If a question is given as a problem, begin by drawing a diagram and give a written answer.
Sine rule to find side
lengths

The sine rule allows us to calculate sides and angles in non-right-angled


triangles.
When finding a side length you need to be given the length of one
b
other side and two angles.
A
a
b
c
A
The sine rule formula is
=
=
sin A sin B sin C

C
C
a
c

Sine rule to find angles

When finding an angle you need to be given two side lengths and one angle.
sin A sin B sin C
The sine rule formula when finding an angle is
=
=
a
c
b

Cosine rule

The cosine rule allows you to calculate the length of sides and size of angles of non-right-angled
triangles where you are unable to use the sine rule.
To find a side length using the cosine rule, you need to be given the length of two sides and the
included angle and use the formula c2 = a2 + b2 2ab cos C.
To find an angle using the cosine rule, you need to be given the length of all three sides and use
a2 + b2 c2
the formula cos C =
.
2ab

Area of a triangle

When you do not know the perpendicular height of a triangle, you can calculate the area using the
1
formula Area = ab sin C.
2
To calculate the area using this formula, you need to be given the length of two sides and the
included angle.

Bearings

Bearings are a measure of direction.


A compass bearing uses the four main points of the compass, north, south, east and west, as well
as the four intermediate directions, north-east, north-west, south-east, south-west.
More specific directions are given using true bearings. A true bearing describes a direction as a
three-digit angle taken in a clockwise direction from north.
Most bearing questions will require you to draw a diagram to begin the question and require a
written answer.

Chapter 7 Applications of trigonometry 219

A plane table radial survey sights each corner of a field and draws a
radial line in that direction. This divides the field into triangles.
20
The length of each radial line and the angle between radial lines are
m
then measured.
The cosine rule can then be used to calculate the length of each
boundary.
1
The formula Area = ab sin C can be then used to calculate the area
2
of the field.
A compass radial survey takes the bearing of each radial line and this
is then used to calculate the angles between them.
A

338

B
49

58

067

55

239

220 Maths Quest HSC Mathematics General 2

57 m

114

25

115
60

115

26 m

Surveying

70

28

Chapter review
1 In the following figure, which of the following will give the value of x?

13 sin 36
A x =
sin 64
13 sin 64
13 m
B x =
64
sin 36
13 sin 64
C x =
x
sin 80
36
13 sin 80
D x =
sin 64
2 In the following figure, which of the following will give the value of cos ?
62 + 72 82
A cos =
267
7m
6m
2
6 + 82 72
B cos =
268
2

7 + 82 62
C cos =
8m
278
62 + 72 82
D cos =
278
3 Maurice walks 3 km on a true bearing of 225. To return to his starting point he must walk on a
compass bearing of:
A north-east
B north-west
C south-east
D south-west
4 The following figure is a compass radial survey. AXB is:
A
B
C
D

B 80

305 A

35
55
85
135

M U LT IP L E
C H O IC E

C 174
1 Find the length of the side marked with the pronumeral in each of the right-angled triangles below,

correct to 1 decimal place.


a

17.2 cm

t
72

S ho rt
a nsw er

7.9 cm

42 km
45

60
x

2 In each of the following right-angled triangles, find the size of the angle marked with the

pronumeral, correct to the nearest degree.


8.3 km
a

b

c
35 cm

16 m

24.8 cm

20.1 km
9m

3 An aeroplane at an altitude of 2500m sights a ship at an angle of depression of 39. Calculate, to the

nearest metre, the horizontal distance from the aeroplane to the ship.
Chapter 7 Applications of trigonometry 221

4 When a yacht is 500m from shore, the top of a cliff

is sighted at an angle of elevation of 12.


a Calculate the height of the cliff, correct to the nearest
metre.
b Calculate what the angle of elevation of the top of the
cliff will be when the yacht is 200m from shore.
5 Two aircraft are approaching an airport. The Qantas
plane (Q) is 40km due north of the runway (R), while
a Jetstar plane (J) is due east of the Qantas plane and
north-east of the runway. Calculate the distance of
the Jetstar plane from the runway. (Give your answer
correct to the nearest metre.)
6 A car rally requires cars to travel for 25km on a bearing
of 240. The cars are then required to travel due north
until they are due west of the starting point. Calculate
the distance from the cars to the starting point. (Give
your answer correct to 1 decimal place.)
7 A yacht sails due west for 45 nautical miles before
turning north for 23 nautical miles.
a Calculate the bearing of the yacht from its starting
point.
b On what bearing must the yacht sail to return to its starting point?
8 Use the sine rule to calculate each of the sides marked with a pronumeral, correct to 3 significant
figures.
e
a a

b
c
d
70

31

4.6 cm

4.6 km

28

117

19

136 mm

20

9 In XYZ: x = 9.2cm, XYZ = 56 and YXZ = 38. Find y, correct to 1 decimal place.
10 Use the sine rule to calculate the size of the angle marked with a pronumeral, correct to the nearest

degree.

a
8 cm

9 cm

9.7 cm

123
4.1 cm

63

c
7.1 m 9

1.2 m

11 In ABC: b = 46cm, c = 37cm and BAC = 72. Find the area of the triangle, correct to the nearest

square centimetre.
12 Find the area of a triangular field with two sides of 80m and 98m, which meet at an angle of 130
(correct to the nearest hundred square metres).
13 Use the cosine rule to find each of the following unknown sides, correct to 3 significant figures.
a
b

c
6.9 cm
6.2 cm

9m
a
50

b
5.7 m
117

11 m

222 Maths Quest HSC Mathematics General 2

4.6 m

128
c

14 In LMN: LM = 63cm, MN = 84cm and LMN = 68.

Find the length of LN, correct to 1 decimal place.


15 During a stunt show two aeroplanes fly side by side until they
suddenly diverge at an angle of 160. After both planes have
flown 500m what is the distance between the planes, correct
to the nearest metre?
16 Use the cosine rule to find the size of the angle in each of the
following, correct to the nearest degree.
a

6 cm

6 cm

6 cm
b

4.2 m

5.3 m

7.9 m
c

9 cm

7 cm

15 cm
17 In XYZ: x = 8.3m, y = 12.45m and z = 7.2m. Find YZX,

60 m

to the nearest degree.


18 Two wooden fences are 50m and 80m long respectively.
Their ends are connected by a barbed wire fence 44m long.
Find the angle at which the two wooden fences meet.
19 The following figure is a plane table radial survey of a field.
a Use the cosine rule to calculate the perimeter of the field.
b Calculate the area of the field.
20 The figure below is a compass radial survey.
a Calculate the perimeter of the field.
340
b Calculate the area of the field.

70 m

30

40
80
120 50
110

0
15

160 m

080

140 m

260

90 m

190

1 The distance between football goal posts is 7m. If Soon Ho is 20m from one goal post and 25m

from the other:


a draw a diagram showing the goal posts and Soon Hos position.
b calculate the angle within which Soon Ho must kick to score a goal. (Give your answer correct to
the nearest degree.)
2 An observer sights the top of a building at an angle of elevation of 20. From a point 30m closer to
the building, the angle of elevation is 35 as shown in the following figure.
a Calculate the size of ATB.
30 sin 20
b Show that the distance BT can be given by the expression BT =
.
sin 15

Ex tended
R es p ons e

Chapter 7 Applications of trigonometry 223

c Show that the height of the building can be given by the expression h =
d Calculate the height of the building correct to 1 decimal place.
T
h
A

Digital doc
Test Yourself
doc-11072
Chapter 7

20
30 m

35

3 The figure below shows a compass radial survey of a field.


a Calculate the length of the boundary CD, correct to 1 decimal place.
b Calculate the area of AXB, correct to the nearest square metre.
A 345

110 m
80 m

D 250

224 Maths Quest HSC Mathematics General 2

X 30 m
30
m

B
085

125 C

30 sin 20 sin 35
.
sin 15

ICT activities
7AReview of right-angled triangles

7FArea of a triangle

Interactivity
int-2405: SOHCAHTUA. (page177)

Tutorial
WE 20 int-0469: Calculate the bearing of a ship from a specified
position. (page203)

Tutorial
WE 4 int-2415: Use trigonometric ratios to solve problems.
(page180)

7GBearings

Digital docs
SkillSHEET 7.1 (doc-11063): Right-angled trigonometry finding a
side length. (page 182)
SkillSHEET 7.2 (doc-11064): Using the inverse trigonometric ratios.
(page 182)
SkillSHEET 7.3 (doc-11065): Rounding angles to the nearest degree.
(page 182)
SkillSHEET 7.4 (doc-11066): Right-angled trigonometry finding
anangle. (page 182)

Digital doc
SkillSHEET 7.7 (doc-11070): Converting nautical miles to kilometres.
(page211)

7BUsing the sine rule to find side lengths

7HRadial surveys

Tutorial
WE 7 int-0465: Learn how to apply Simpsons rule. (page185)
Digital docs
SkillSHEET 7.5 (doc-11067): Angle sum of a triangle. (page 187)
SkillSHEET 7.6 (doc-11068): Solving fractional equations. (page 187)

7CUsing the sine rule to find angles


Elesson
eles-0051: Ambiguous case of the sine rule. (page 190)
Tutorial
WE 12 int-2416: Calculate the bearing of a ship from a specified
position. (page191)

Interactivity
int-0190: Navigation and specifications of locations. (page206)
Tutorial
WE 25 int-0473: Apply knowledge of bearings to solve problems.
(page208)

Digital doc
WorkSHEET 7.2 (doc-11071): Apply your knowledge of the sine and
cosine rules to problems. (page218)

Chapter review
Digital doc
Test Yourself (doc-11072): Take the end-of-chapter test to test your
progress. (page 224)

To access eBookPLUS activities, log on to www.jacplus.com.au

7DUsing the cosine rule to find side lengths


Tutorial
WE 14 int-0468: Learn how to apply the cosine rule. (page195)
Digital doc
WorkSHEET 7.1 (doc-11069): Apply your knowledge of trigonometry
to problems. (page 197)

Chapter 7 Applications of trigonometry 225

Answers chapter 7

b 2.659km
4 a 28.01m
5 52
6 62m
7 15
8 a

c 5.663km
b 25

b 16

239 m

65 m

9 D
10 = 20, x = 4.0m
11 59
12 56
13 a 1.43m
14 41

b 12

Exercise 7B Using the sine rule to find


side lengths
a
b
c
1 a
=
=
sin A sin B sin C
y
x
z
b
=
=
sin X sin Y sin Z
q
p
r
c
=
=
sin P sin Q sin R

2 a 14.8cm
3 a 10.0mm
4 B
5 C
6 9.8cm
7 27.0m
8 37.8m
9 a B

b 1.98km
b 22.1cm

c 112mm
c 39.6km

b2 + c2 a2
2bc
p2 + r2 q2
b cos Q =
2pr
a2 + m2 p2
c cos P =
2am

b 43.2m
c 33m
10 a WYX = 40

XY =

80 sin 30
sin 40

h
XY

80 sin 30
sin 70
sin 40

c 58.5m
11 a 145cm
12 68km
Exercise 7C

angles

2 a 85
b 83
3 a 103
b 137
4 A
5 D
6 42
7 23
8 82, 54, 44
9 a
W 9.2 cm
X

80
XY
=
sin 40 sin 30

b In YXZ sin 70 =
h=

1 a 43
d 75

Using the cosine rule to find

13.6 cm

b 34cm

Using the sine rule to find


b 35
e 37

c 27
f 2

9.2 cm

b 73
10 32
11 a 38.94

226 Maths Quest HSC Mathematics General 2

14 cm
13.6 cm

b 25.26

c 4660.9mm2
4 a 133cm2
b 555.4cm2
c 608cm2
5 D
6 B
7 3.865cm2
A
B
8 a
70

5.2 m

c 45
c 10

b 12.2m2
9 a 72
b 59cm2
10 710m2
11 43.3cm2
12 C
13 B
14 a 130.4cm2
b 14.5m
15 a 159.4cm2
b Check with your teacher.
c Because sin 75 = sin 105
16 90
Exercise 7G

1 a cos A =

N 20 m

In WYX

c 15.5cm
12 55cm
13 13.6km
14 9.2cm
15 2218m
16 27.65m

angles

1
yz sin X
2
1
c Area = am sin G
2
1
2 a Area = ab sin C
2
1
b Area = bh
2
1
c Area = bh
2
1
d Area = bh
2
3 a 42.4cm2
b 3522.6mm2

b Area =

Using the cosine rule to find


side lengths
1 a a2 = b2 + c2 2bc cos A
b r2 = p2 + q2 2pq cos R
c n2 = l2 + m2 2lm cos N
2 a 8.05m
b 14.3cm
c 12.0m
3 a 185.1cm
b 14.4m
c 104.4mm
4 D
5 A
6 C
7 14.4cm
8 1140mm
9 6.742km
10 40m
11 a

b 8.5cm
134
46
11.3 cm

b S22.2E

Area of a triangle
1
ab sin C
2

1 a Area =

Exercise 7D

Exercise 7E

34

49

Exercise 7F

b 6.73m2

5 km

148
a 356'
22.94
a 130km
7049'
1.14km/h

2.5

b 56

28

b 110.9mm
d 409.9mm
f 19.3cm
c 57

c 131

12
13
14
15
16
17

Bearings

1 a N25E
c N50W
e N78E
2 a 135T
c 135T
e 202.5T
3 a
x
4.2 km

4 a

34 km

Review of right-angled

c 32

cm

Exercise 7A

triangles
1 a 12.2cm
c 10.0m
e 29.8m
2 a 27
3 a R

2 B
3 B
4 a 43
b 50
5 19.96
6 a 119
b 95
7 38
8 20
9 84
10 a 57
b 63
11 54
12 51.2, 96.8, 31.85
13 C
14 B
15 63

5.3

Applications of
trigonometry

45

45
x

b 34km
5 a B
x

43 M

45

b 30.4 M
c 56.3008km
6 B
7 B

b S10W
d S78W
f S78E
b 247.5T
d 292.5T
f 247.5T
b 2.970km

8M

8 6.4km
9 a

60

b 13.9 M
10 437km
11 a CP

70

b 11.1km

290

3.5 km

12 D
13 A
14 a

3.8 km

CG

4 km

b 229

15 342
16 80.98 km
17 a B, 11.6km
b A, 6.5km
c A, 15.4km
18 a 20.38km
b 23.04km
19 199.47N
20 199.47
21 a 12.57km
b 5341E
22 a 276.6m north, 64.28m east
b 154.44m north, 249.80 east
c 38.49km south, 17.43km west
d 227.28m south, 623.47m east
23 a 1428m
b 1358m
c 1970m
d N4334E
Exercise 7H

1 102m
2 a 286m

c 540m

3 3087m
4 a 106m
5 56.569km
6 21.7km
7 a 297
8 a 1.67cm
c 9.81km
9 12.4cm
10 a 52
11 809cm2
12 3000m2
13 a 8.64m
c 11.8cm
14 84.0cm
15 985m
16 a 60
17 34
18 29
19 a 284m
20 a 783m

b 117
b 81.7mm

b 21

c 68

b 8.80m

b 112

c 139

b 4020m2
b 34910m2

25

20

b Approximately at an angle of 13
2 a ABT = 15

Multiple choice

1 D
2 B
3 A
4 D
1 a 8.6cm
2 a 61

b 28

Extended response

1 a A

Chapter review

30 sin 20
sin 15
30 sin 20
c h =
sin 35
sin 15
d Approximately 22.7m
3 a Approximately 100.3m
b Approximately 1625m2
b BT =

Short answer

Radial surveys
b 432m

3 12992m2
4 a 4923m2
b 8861m2
2
c 18382m
5 a 85
b 75m
6 389m
7 a 214m
b 531m
c 301m
8 a 2719m2
b 12425m2
c 5809m2
9 a 1017.58m2
b 148.16m
c You should obtain the same result.
10 a 14.14m
b 50m2, 200m2
c You should obtain the same result.
11 a 67.26m
b 1563.65m2
c 46.5m
12 a Check with your teacher.
b i 100m
ii 66m
iii 50m
iv
90m
v 86m
c i 270
ii 310
iii 240

d 4300m2
13 a Radial
b Check with your teacher.
c A, 123; B, 136; C, 152; D, 180
d 3160m2
14 a Check with your teacher.
b i 212
ii 270

c i 107m
ii 77m

d 3800m2

b 13.6km
b 66

c 11.2cm
c 45

Chapter 7 Applications of trigonometry 227

Chapter 8

Spherical geometry
CHAPTER CONTENTS
8A Arc lengths
8B Great circles and small circles
8C Latitude and longitude
8D Distances on the Earths surface
8E Time zones

8A

Arc lengths

An arc is a section of the circumference of a circle. To calculate an arc length we must first revise finding
the circumference of a circle. The circumference of a circle can be found using either of the formulas:
C = d, where d is the diameter

C = 2r, where r is the radius.
WORKED EXAMPLE 1

Calculate the circumference of a circle that has a radius of 6m. Give your answer correct to
2decimal places.
THINK

WRITE

Write the formula.

C = 2r

Substitute the value of r.

C=26

Calculate the circumference.

C = 37.70m

The length of an arc can be calculated as the fraction of the circle determined by
the angle subtended by the arc at the centre, as shown in the figure on the right.
The arc length, l, can be calculated using the formula:
l=

2r
360

where = number of degrees in the central angle.

WORKED EXAMPLE 2

Calculate the length of the arc shown on the right, correct to


1 decimal place.

7.1 cm
60

Tutorial
int-2443
Worked example 2

Chapter 8 Spherical geometry 229

THINK

WRITE

Write the formula.

Substitute the value of and r.

2r
360
60
=
2 7.1
360

Calculate the arc length.

= 7.4cm

l=

The arc length formula is used to make calculations about the distance between points on the Earths
surface.

WORKED EXAMPLE 3

The radius of the Earth at the equator is approximately 6400km.


aCalculate the circumference of the Earth at the equator, correct to the nearest kilometre.
bTwo points on the equator subtend at a 5 angle at the centre of the Earth. Calculate the
distance between them, correct to the nearest kilometre.
THINK

WRITE

a C = 2r

a 1 Write the formula.


2

Substitute the value of r.

= 2 6400

Calculate the circumference.

= 40212km
b d=

b 1 Write the formula.

2r
360
5
2 6400
360

Substitute for and r.

Calculate the distance.

= 559km

Exercise 8A

Arc lengths

1 WE1 Calculate the circumference of each of the following circles, correct to

1 decimal place.
Digital doc
SkillSHEET 8.1
doc-11073
Circumference of
acircle

4 cm

62 mm

9m

e
13.9 km

8.3 m

4.1 km

2 Calculate the circumference of a circle with a radius of 100km. Give your answer correct to the

nearest 10km.
230 Maths Quest HSC Mathematics General 2

3 Calculate the circumference of each of the following circles. Give your answer correct to

3significant figures.

14 cm

60 km

39 m

117 mm

3.7 km

219 km

4 Calculate the circumference of a circle that has a diameter of 3000km. Give your answer correct to

the nearest 100km.


5 Use the formula C = d to find the diameter of a circle with a circumference of 100cm. Give your

answer correct to 2 decimal places.


6 Find (correct to 3 significant figures):
a the diameter of a circle with a circumference of 80m
b the radius of a circle with a circumference of 42.3cm
c the diameter of a sphere with a circumference of 2500km.
7 WE2 Calculate the length of the arc shown on the right, correct to
2 decimal places.

50
10 cm
8 Calculate the lengths of each of the arcs drawn below, correct to 1 decimal place.
a

b

c

43

mm

Digital doc
SkillSHEET 8.2
doc-11074
Calculating arc
length

45
120
72 cm

13 m

220

72 km
150

.2 cm

7.9 km
4

9 WE3 The radius of the Earth at the equator is 6400km.


a Calculate the circumference of the Earth at the equator, correct to the nearest 100km.
b Two points on the equator subtend an angle of 40 at the centre. Calculate the distance between

them, correct to the nearest 10km.


10 A circle has a radius of 30km.
a Two points on the circle subtend an angle of 100 at the centre. Calculate the length of the arc

joining them, correct to the nearest kilometre.


b The outer arc between the two points subtends an angle of 260. Calculate the length of the outer

arc, correct to the nearest kilometre.


c Show that the sum of the lengths of the two arcs is equal to the circumference of the circle.
Chapter 8 Spherical geometry 231

11 A sphere has a radius of 40cm.


a Calculate the circumference of the sphere, correct to the nearest centimetre.
b Calculate the distance between two points on the sphere that subtend an angle of 90 at the centre,

correct to 1 decimal place.


12 Calculate the distance, correct to 1 decimal place, between two points on a sphere:
a of radius 10cm, which subtend an angle of 30 at the centre
b of radius 2m, which subtend an angle at 122 at the centre
c of radius 6400km, which subtend an angle of 51 at the centre.
13 Calculate the distance between two points on the Earths surface that subtend an angle of 1 at the

centre. Give your answer correct to the nearest kilometre.

Further development
14 Find the radius of each of the following circles given the circumference. Give each answer correct to

3 significant figures.
a 100cm

b 26.7m

c 12000km

15 An arc has a length of 24cm and subtends an angle of 45 at the centre of the circle. Find, correct to

1 decimal place, the radius of the circle on which the arc stands.
16 A 42-metre long arc stands on a circle of radius 30 metres. Find the angle that is subtended at the

centre of the circle by this arc. Give your answer correct to the nearest degree.
17 A sphere has a diameter of 50cm. Two points A and B on the surface of the sphere subtend an angle

of 125.
a Calculate the distance AB correct to the nearest centimetre.
b Calculate the length of the larger arc AB (i.e. the arc formed by travelling from A to B in the
opposite direction).
232 Maths Quest HSC Mathematics General 2

18 The Earth has a radius of approximately 6400km. Two points on the Earths surface are 5000km

apart.
a Calculate the angle subtended by these two points at the centre of the Earth. Give your answer to
the nearest minute.
b A circle between the two points can be drawn on the Earths surface that has a radius of 4000km.
Calculate the angle that is subtended at the centre of this circle.
19 Two points on the surface of the Earth are 1675km apart. A circle can be drawn on the surface of the

Earth that passes through the two points and the two points subtend an angle of 1926' at the centre
of this circle. Find the radius of this circle, correct to the nearest 10km.

8B

Great circles and small circles

Consider the sphere drawn on the right. The axis of the sphere is a diameter of
that sphere. The ends of the axis are called the poles.
If we draw any lines around the sphere passing through both poles,
a great circle is formed. A great circle is the largest possible circle that can
be drawn around the sphere.

Pole Axis

Great circle
Pole

The length of a great circle is found using the formulas for the cir-cumference of a circle:
C = d, where d is the diameter of the sphere
or C = 2r, where r is the radius of the sphere.

WORKED EXAMPLE 4

Calculate the length of a great circle on a sphere with a radius of 40cm. Give your answer
correct to the nearest centimetre.
THINK

WRITE

C = 2r

Write the formula.

Substitute the radius of the sphere.

= 2 40

Calculate the length of the great circle.

= 251cm

Now consider a circle drawn perpendicular to the axis of the sphere. Only one circle, called the
equator, will be a great circle. The centre of the equator will be the centre of the sphere as
shown below.

Equator

Chapter 8 Spherical geometry 233

Other circles that are perpendicular to the axis of the sphere will be smaller than a great circle and are
called small circles.
Small circle
Equator

To calculate the length around a small circle, we need to know the small circles radius. The small
circle will have a radius smaller than that of the great circle, as shown in the figure below.
Radius of small circle

Radius of great circle

WORKED EXAMPLE 5

Calculate the length of the small circle on the right, correct to


1 decimal place.
Tutorial
int-2444
Worked example 5

2m
1.5 m
THINK

WRITE

C = 2r

Write the formula for the circumference of a circle.

Substitute for r.

= 2 1.5

Calculate the circumference.

= 9.4cm

Exercise 8B

Great circles and small circles

1 WE4 Calculate the length of a great circle around a sphere of radius 7cm. Give your answer correct

to 2 decimal places.
2 Calculate the length of a great circle on each of the spheres drawn below. Give each answer correct

to 3 significant figures.
a

9m
30 m
74 mm
3 The Earth is a sphere with a radius of approximately 6400km. Calculate the length of a great circle

around the Earths surface, correct to the nearest 10km.


4 Calculate the length of a great circle that lies on a sphere with a diameter of 1m. Give your answer

correct to the nearest centimetre.


234 Maths Quest HSC Mathematics General 2

5 Below are the diameters of the other planets in our solar system. Calculate the length of a great circle

on the surface of each planet, correct to the nearest 10km.


a Mercury 4878km b Venus 12100km
c Mars 6796km
e Saturn 120540km f Uranus 51118km
g Neptune 49100km

d Jupiter 142984km

6 WE5 Calculate the length of a small circle on a sphere of radius 4m if the radius of the small circle

is 2m. Give your answer correct to 2 decimal places.


7 Calculate the length of each of the small circles below, correct to 1 decimal place.
a

b

c
1100 km
6.4 m
80
25
7.1 m

6400 km

9m
10
8.8 m

8 MC The diameter of the moon is 3476km. The length of a great circle on the moons surface is

closest to:
A 5460km

B 10920km

C 21840km

D 43680km

9 The diameter of the sphere drawn on the right is 50cm. Calculate the

50 cm

distance along the surface from one pole to the other. Give your answer
correct to the nearest centimetre.

10 Calculate the distance between the north and south poles along the Earths

surface, correct to the nearest 100km. (Take the radius of the Earth to be
6400km.)

11 A and B are two points on a small circle of radius 60cm, as shown in the

figure on the right.


a Calculate the circumference of the small circle, correct to the nearest
centimetre.
b Calculate the length of the arc, AB, correct to the nearest centimetre.
12 Calculate the length of each arc below, correct to 3 significant figures.
a

b
X

60 cm

1000 km
100 Y
50 m
60
Q
P

Further development
13 The sphere below has four circles on it, labelled (a), (b), (c) and (d).
(b)
(c)

(a)
(d)

Label each as being either a great circle or a small circle.


Chapter 8 Spherical geometry 235

14 The figure on the right shows a great circle and a small circle drawn on a

Digital doc
WorkSHEET 8.1
doc-11075

sphere of radius 50cm.


y
a The small circle is at an angle of elevation of 40 to the great circle.

50 cm
Find the size of the angle marked .
40
50 cm
b Use trigonometry to find the radius of the small circle correct to the
nearest centimetre.
c Find the circumference of the small circle correct to the nearest
centimetre.
15 The radius of a small circle can be found using the formula r = Rcos, where r is the radius
ofthesmall circle, R is the radius of the great circle and is the angle subtended by the small
circlewith the great circle at the centre of the sphere.
a Verify this formula using the information given in question 14.
b Use the formula to find the radius and circumference of a small circle that subtends a 25
angle at the centre of the sphere radius 5.2m with a great circle. Give each answer correct
to1decimalplace.
16 The Earth has a radius of approximately 6400km.
a Use the formula given in question 15 to find the radius of a small circle that makes
a 20 angle with a great circle. Give your answer correct to the nearest
kilometre.
b X and Y are two points on the small circle in 16a that subtend an angle of 40 at the centre
ofthesmall circle. Find the length of arc XY correct to the nearest 10km.
17 a Find the radius of a small circle that makes a 45 angle with a great circle of radius 6400km.
Give your answer correct to the nearest kilometre.
b Find the distance to the nearest kilometre between two points on the small circle in 17a
thatsubtend a 75 angle at the centre of the small circle.
18 X and Y are two points on a great circle upon the Earths surface that subtend a 65 angle at the
centre. X and Z lie on the same small circle that has a radius of 3200km and subtendan angleof
65 at the centre of the same small circle. Jude claims thatthedistanceXYwill be
twicethedistance XZ. Is Jude correct? Explain your answer.

236 Maths Quest HSC Mathematics General 2

8C

Latitude and longitude

As the Earth is a sphere, great circles and small circles on the surface of the
Earth are used to locate points on the surface.
Consider the axis of the Earth to be the diameter joining the North Pole and the
South Pole. The only great circle that is perpendicular to this axis is theequator.
The angular distance either north or south of the equator is the latitude.
Small circles parallel to the equator are called parallels of latitude. These
small circles are used to describe how far north or south of theequatora
placeis.For example Sydney lies close to the small circle 30S.
This means Sydney subtends a 30 angle at the centre of the Earth and is
southof the equator.
The maximum latitude for any point on the Earth is 90N or 90S. The
north and south poles lie at these points.
For latitude, the equator is the line of reference for all measurements.
To locate a place on the globe in an eastwest direction, the line of
30S
reference is the Greenwich Meridian. The Greenwich Meridian is half a
great circle running from the North to the South Pole.

North Pole

Equator

eLESSON
eles-0138
Understanding
map scales and
distances

South Pole

30

Sydney

Equator

G re

enwich Meridia

North Pole

South Pole

The Greenwich Meridian is named after Greenwich, a suburb of London through which the circle runs.
All other places on the globe are located by the half great circle on which theylie. These half great
circles are called meridians of longitude.
Each meridian of longitude is identified by the angle between it and the Greenwich Meridian and by
whether it is east or west of Greenwich.

40

The meridian of longitude opposite the Greenwich Meridian is the International Date Line. The
International Date Line has longitude 180 eithereast or west. On either side of the International Date
Line the day changes.(This will be explained in more detail later in the chapter.)
For the convenience of some small island nations and Russia, the International Date Line is bent so as
not to pass through them.
World maps or globes are drawn with both parallels of latitude and meridians of longitude shown. Any
location on a map or globe can be given a pair of coordinates: the first is the parallel of latitude that it lies
on, the second is the meridian of longitude. For example, the coordinates of Sydney are 30S, 150E.
Greenwich
Meridian

International
Date Line

Chapter 8 Spherical geometry 237

238 Maths Quest HSC Mathematics General 2

30S

30N

60N

30W

FRANCE

30W

1000

ATLANTIC
OCEAN

Equator

30E

Johannesburg

NAMIBIA

2000

3000

SOMALIA

IRAQ
Baghdad

30E

4000 km

60E

MADAGASCAR

KampalaUGANDA
KENYA

ANGOLA

NIGER

Cairo
LIBYA
EGYPT

SOUTH AFRICA
Cape Town

MALI

ALGERIA

ITALY
Rome
PORTUGAL Madrid
SPAIN

FINLAND
ICELAND
SWEDEN
NORWAY
Oslo
UNITED
KINGDOM
Moscow
IRELAND
NETHERLANDS
Amsterdam
London

60W

INDIAN
OCEAN

90E

Perth

120E

Sydney

Brisbane

150E

Hobart

Adelaide
Melbourne

AUSTRALIA

EAST
TIMOR
Darwin

INDONESIA

PAPUA NEW
GUINEA
Port Moresby

PHILIPPINES
Manila

TAIWAN

180

180

120W

150W

PACIFIC
OCEAN

120W

Hawaii (U.S.A.)

90W

Montreal

60W

BRAZIL

30W

URUGUAY
Buenos Aires
ARGENTINA

60W

Santiago

CHILE

BOLIVIA

PERU
Lima

ATLANTIC
OCEAN

PARAGUAY

JAMAICA

New York

30W

COLOMBIA
Quito
ECUADOR

90W

MEXICO
Mexico City

UNITED STATES
OF AMERICA

CANADA

Los Angeles

Vancouver

Alaska
(U.S.A.)
Anchorage

150W

NEW
ZEALAND
Wellington

FIJI
Suva

ARCTIC OCEAN

150E

NORTH KOREA
SOUTH KOREA
Tokyo
JAPAN

120E

SINGAPORE

Shanghai

Beijing

90E

CHINA

SRI LANKA
Colombo

INDIA

RUSSIA

60E

International Date Line

30S

30N

60N

WORKED EXAMPLE 6

Identify the major cities closest to each of the following locations using the map on page 238.
a 30S, 30E
b 30N, 120E
c 45N, 75W
THINK

WRITE

a Look for the city closest to the intersection of the 30S parallel of latitude

a Johannesburg

andthe 30E meridian of longitude.


b Look for the city closest to the intersection of the 30N parallel of latitude

b Shanghai

andthe 120E meridian of longitude.


c Look for the city closest to the intersection of the 45N parallel of latitude

c Montreal

andthe 75W meridian of longitude.

WORKED EXAMPLE 7

Write down the approximate coordinates of each of the following cities using the map on page 238.
a Singapore
b Perth
c Los Angeles
THINK

WRITE

a 1 Use the parallels of latitude drawn to estimate the latitude.


2

Use the meridians of longitude drawn to estimate the longitude.

b 1 Use the parallels of latitude drawn to estimate the latitude.

Use the meridians of longitude drawn to estimate the longitude.

Use the parallels of latitude drawn to estimate the latitude.

Use the meridians of longitude drawn to estimate the longitude.

Exercise 8C

1N, 104E
b

32S, 115E
c

35N, 118W

Latitude and longitude

For the following questions use the map on page 238.


1 WE6 Write down the name of the city closest to each of the following pairs of coordinates.
a 30N, 30E
b 30N, 120E
c 15S, 135E
d 45N, 75W
e 50N, 0
f 37S, 175E
g 35N, 140E
h 40N, 115E
1
i 222S, 43W
j 60N, 11E
2 WE7 State the approximate latitude and longitude of each of the following major cities or islands.
a Melbourne
b New York
c Jamaica
d Johannesburg
e Rome
f Buenos Aires
g Baghdad
h Moscow
i Singapore
j Suva

Further development
3 For each of the following points on the Earths surface, state whether the two points lie on (or close

to) the same great circle or the same small circle.


a Shanghai and Los Angeles b Montreal and Lima
d Manila and Shanghai
e Baghdad and Shanghai

c Rome and Oslo


f Singapore and Quito
Chapter 8 Spherical geometry 239

4 The following cities are close to lying on the same great circle. Use the map on page 238 to estimate

the angle that is subtended at the centre of the Earth.


a Perth and Beijing
b Kampala and Singapore
c Quito and Kampala
d Cairo and Kampala
5 The following cities are close to lying on the same small circle. Use the map on page 238 to estimate

the angle subtended at the centre of the small circle.


a Baghdad and Los Angeles
b Cape Town and Sydney
c Oslo and Anchorage
d Santiago and Sydney

Important parallels of latitude


Four significant parallels of latitude on the surface of the Earth are the:
1. Arctic Circle
2. Antarctic Circle
3. Tropic of Cancer
4. Tropic of Capricorn.
Find out the latitude of these small circles and state the significance of each.

8D

Distances on the Earths surface

From the previous section on latitude and longitude it can be seen that
angular geometry is of great importance when making measurements on the
Earths surface.
Now consider a meridian of longitude on the Earths surface with two
points on it. The angular distance between them will be the difference between
their latitudes.
The angular distance is calculated by subtracting the latitudes of points if
both are on the same side of the equator and adding the latitudes if on opposite
sides of the equator.

WORKED EXAMPLE 8

The coordinates of A are (20S, 130E) and the coordinates of B are (15N, 130E). Find the
angular distance between them.
THINK

A and B are on opposite sides of the equator so


add the latitudes.

240 Maths Quest HSC Mathematics General 2

WRITE

Angular distance = 20 + 15
= 35

Now consider two points on the same great circle that have an angular distance of 1 minute. (Remember
60 = 1.) The distance between these two points is defined to be 1 nautical mile (M). Therefore, an
angular distance of 1 on a great circle will equal 60 nautical miles.
1 nautical mile 1.852km
Using this information, we are able to calculate the distance between two points on a great circle on
the Earths surface in both nautical miles and kilometres.

WORKED EXAMPLE 9

P and Q are two points on the Earths surface with coordinates (27N, 30W) and (39N, 30W)
respectively.
a Calculate the distance between P and Q in nautical miles.
b Use 1 M 1.852km to give the distance, PQ, to the nearest kilometre.
THINK

WRITE

a 1 P and Q are on the same great circle.

Calculate the angular distance, PQ.

Angular distance = 39 27
= 12

Convert the angular distance to nautical miles


using 1 = 60 M.

Distance = 12 60
= 720 M

b Multiply 720 by 1.852 to convert to kilometres.

Tutorial
int-2445
Worked example 9

b Distance = 720 1.852

= 1333km

We can also calculate the distance between two points on the same great circle, in kilometres, using the
fact that the radius of the Earth is 6400km.

WORKED EXAMPLE 10

X and Y are two points on the Earths surface with coordinates (32N, 120E) and (45S, 120E).
Calculate the distance, XY, correct to the nearest 100km.
THINK

WRITE

Calculate the angular distance, XY.

Angular distance = 32 + 45
= 77

Use the arc length formula to calculate the distance between


Xand Y, correct to the nearest 100km.

l=

2r
360

77
2r
360

77
2 6400
360

= 8600km

In marine and aerial navigation, speed on the Earths surface is measured in knots.
1 knot = 1 nautical mile/hour

Chapter 8 Spherical geometry 241

WORKED EXAMPLE 11

The coordinates of two points on the Earths surface are given by the coordinates A (50N, 120W)
and B (30S, 120W). Calculate the time taken for a ship to sail the shortest distance between
thesetwo points at an average speed of 40 knots.
THINK

WRITE

Calculate the angular distance between A and B.

Angular distance = 50 + 30
= 80

Calculate the distance, AB, in nautical miles.

Distance = 80 60
= 4800 M

Use the formula time =


takenfor the journey.

Exercise 8D

distance
to calculate the time
speed

Time =

distance
speed

4800
40
= 120 hours (5 days)
=

Distances on the Earths surface

1 WE8 Two points, A and B, on the Earths surface are at (30N, 25W) and (20S, 25W). Calculate

the angular distance between A and B.


2 In each of the following calculate the angular distance between the pairs of points given.
a (70N, 150E) and (30N, 150E)
b (25N, 40W) and (15S, 40W)
c (64N, 0) and (7S, 0)
d (42S, 97W) and (21S, 97W)
e (0, 60E) and (0, 20W)
3 The city of Durban is at approximately (30S, 30E) while Cairo is at approximately (30N, 30E).

What is the angular distance between Durban and Cairo?


4 WE9 P and Q are two points on the Earths surface with coordinates (45N, 10W) and (15N,

10W) respectively.
a Calculate the distance between P and Q in nautical miles (M).
b Use 1 M = 1.852km to calculate the distance, PQ, correct to the nearest km.
5 Calculate the distance between each of the points below in nautical miles.
a A (10N, 45E) and B (25S, 45E)
b C (75N, 86W) and D (60S, 86W)
c E (46S, 52W) and F (7S, 52W)
d G (34N, 172E) and H (62S, 172E)
6 The city of Osaka is at (37N, 135E) while Alice Springs is at (23S, 135E).
a Calculate the distance between Osaka and Alice Springs in nautical miles.
b Use 1 M = 1.852km to write this distance, correct to the nearest kilometre.
1

7 The Tropic of Cancer is at latitude 222N while the Tropic of Capricorn is at latitude 222 S.

Calculate the distance between these two tropics along the same great circle in:
a nautical miles
b kilometres (correct to the nearest km).

8 WE10 M and N are two points on the Earths surface with coordinates (56N, 122W) and

(3S,122W). Calculate the distance, MN, correct to the nearest 100km, using the arc length
formula. (Take the radius of the Earth to be 6400km.)
9 Calculate the distance between each of the points below, correct to the nearest kilometre, using the

arc length formula and taking the radius of the Earth to be 6400km.
a P (85S, 89E) and Q (46S, 89E)
b R (24N, 0) and S (12S, 0)
c T (34S, 17W) and U (0, 17W)
242 Maths Quest HSC Mathematics General 2

10 MC Perth is at approximately (31S, 115E) while Hong Kong is at approximately (22N, 115E).

The distance between Perth and Hong Kong is approximately:


A 9 M
B 53 M
C 540 M

D 3180 M

11 MC Rachel is a flight navigator. She is responsible for calculating the distance between

Stockholm(60N, 18E) and Budapest (47N, 18E). Rachel calculates the distance using the
arclength formula, assuming the radius of the Earth is 6400km. Rachels answer would be
closestto:
A 1445km
B 1452km
C 11952km
D 11890km
12 WE11 The coordinates of two points on the Earths surface are X (40S, 30E) and Y (10S, 30E).

Calculate the time taken for a plane to fly from X to Y at a speed of 240 knots.
13 Quito (0, 78W) and Kampala (0, 32E) are two cities on the Equator.
a Calculate the angular distance between Quito and Kampala.
b Calculate the distance between them in nautical miles.
c Use 1 M = 1.852km to find the distance, correct to the nearest 100km.
d Calculate the time taken to fly from Quito to Kampala at a speed of 480 knots.
14 Calculate the distance between the North Pole and the South Pole in nautical miles.
15 The city of Kingston is at approximately (18N, 76W). Ottawa is at approximately (46N, 76W).
a Calculate the angular distance between Kingston and Ottawa.
b Calculate the distance between Kingston and Ottawa in nautical miles.
c Use 1 M = 1.852km to calculate the distance, correct to the nearest kilometre.
d Use the arc length formula to calculate the distance between the two cities, correct to the nearest

kilometre.
e Explain the discrepancy between the two answers.

Further development
16 The point X has coordinates (25N, 20E).
a Give the coordinates of two possible points on the globe that have a difference in latitude of

35to point X.
b Find the distance (in km) between X and these two points
17 Two places on the equator have an angular distance of 200. What is the shortest distance between

these two places (to the nearest kilometre)?


18 Find the distance in kilometres to the nearest 100km from each of the following places to the closer

pole. Indicate which pole (north or south) is the closer.


a Warwick (28S, 152E)
b Vancouver (49N, 123W)
c St Moritz (46N, 10E)
d Thursday Island (10S, 142E)
19 Find the distance (in km) between the following places.
a (40S, 130E) and (40S, 159E)
b (70N, 15E) and (70N, 100E)
c (50S, 66W) and (50S, 106W)
d (80S, 67W) and (80S, 89W)
e (20S, 150E) and (20S, 54W)
f (30N, 28E) and (30N, 39W)
20 X and Y are two points on the same small circle with coordinates (35N, 45E) and (35N, 10E).
a Use the formula r = Rcos to find the radius of the small circle upon which these two points lie,

given that = 35.

b Find the angle subtended between these two points at the centre of the small circle.
c Find the distance along the small circle between these two points.
21 Point A on the Earths surface has coordinates (45S, 75E). Point B is 5000km due north of A. Find

the coordinates of this point to the nearest degree.

8E

Time zones

As the Earth rotates, different parts of the globe are experiencing day and night. This means that each
meridian of longitude on the Earths surface should have a different time of day. To simplify this, the
Earth is divided into time zones.
Time zones are all calculated in relation to Greenwich. The time on the Greenwich Meridian is
known as Greenwich Mean Time (GMT). Time zones are then stated in terms of the number of

interactivity
int-0006
Day, night and
time zones

Chapter 8 Spherical geometry 243

hours they are ahead or behind GMT. All places with longitudes east of Greenwich are ahead of
GMT,while all place with longitudes west of Greenwich are behind GMT. For example, Eastern
Standard Time is GMT +10, meaning that Sydney is 10 hours ahead of GMT. When GMT is noon,
EST is10.00 pm.
The International Date Line is 12 hours ahead of Greenwich when travelling east and 12 hours behind
when travelling west, so this totals 24 hours, or one day. Therefore, the day changes on either side of the
International Date Line.
The time difference between two places is calculated by subtracting the comparative time with GMT.
WORKED EXAMPLE 12

Sydney is GMT +10 while New York is GMT 5. Calculate the time difference between Sydney
and New York.
THINK

WRITE

Subtract the comparative times.

Time difference = 10 (5)


= 15

State the difference and which city is ahead in time.

Sydney is 15 hours ahead of New York.

Once we have calculated the time difference, we are able to calculate the time in one place given the
time in another. To calculate the time in a city further ahead of GMT we add time, or to calculate the
time in a city further behind GMT we subtract time.
WORKED EXAMPLE 13

Perth is GMT +8 while Cape Town is GMT +1. When it is 11.00 am in Cape Town, what is the
time in Perth?
THINK

WRITE

Calculate the time difference and state which city is


ahead.

Time difference = 8 1

= 7 hours
Perth is 7 hours ahead of Cape Town.

Add the time difference to the time in Cape Town to


calculate the time in Perth.

Time in Perth = 11.00 am + 7 hours



= 6.00 pm

The time as calculated by the longitude is called the standard time. Time zones are calculated to
approximate all the standard times within a region.

Australian time zones


At times when daylight saving time does not apply, Australia is divided into three time zones, Eastern
Standard Time (EST), Central Standard Time (CST) and Western Standard Time (WST).
1. Which states are in each of the three time zones?
2. What is the time difference between each of these zones?
3. In which states does daylight saving time apply in summer?
4. When daylight saving is in force in each of the states that have daylight saving, how many time zones
is Australia divided into?
As there are 24 hours in a day and 360 of longitude (180E and 180W), we can calculate that:
1 hour = 15 of longitude
1 = 4 minutes
We are now able to compare the time in various cities given the longitude of each.
244 Maths Quest HSC Mathematics General 2

WORKED EXAMPLE 14

Calculate the time in Los Angeles (34N, 120W) when it is 8.00 am on Wednesday in Sydney
(33S, 150E).
THINK

WRITE

Tutorial
int-2446
Worked
example14

Calculate the difference in longitudes.

Longitude difference = 150 + 120



= 270

Convert this angular distance into hours


using 1 = 4 minutes.

Time difference = 270 4



= 1080 minutes

= 18 hours

Subtract the time difference from the


time inSydney.

Time in Los Angeles = 8.00 am Wednesday 18 hours



= 2.00 pm Tuesday

It is important to note that, for convenience, places that have almost the same longitude have the same
time. An example of this is Australias time zones where all of Queensland, New South Wales, Victoria
and Tasmania are in the same standard time zone although there is a difference of 12 in longitude from
the easternmost and westernmost points in this zone.
These calculations can then be used to calculate the arrival and departure times for international travel.
WORKED EXAMPLE 15

A plane leaves London (50N, 0) at 9.00 am Sunday, London time, and flies to Sydney (33S, 150E).
The flight takes 20 hours. Calculate the time in Sydney when the plane arrives.
THINK

WRITE

Calculate the longitude difference


between Sydney and London.

Longitude difference = 150 0



= 150

Use 1 = 4 minutes to calculate the


time difference.

Time difference = 150 4



= 600 minutes

= 10 hours

Calculate the time in Sydney when the


plane is departing London by adding
the time difference.

When the plane leaves London at 9.00 am


(London time)
Time in Sydney = 9.00 am Sunday + 10 hours

= 7.00 pm Sunday

Add the flying time to calculate the


time when the plane lands.

Plane arrives at 7.00 pm Sunday + 20 hours



= 3.00 pm Monday.

More challenging examples will require you to allow for daylight saving time. When daylight saving
time applies, we add one hour to the standard time at that location.

Exercise 8E

Time zones

1 WE12 The time zone in New Zealand is GMT +12 while in Turkey it is GMT +2. Calculate the

time difference between New Zealand and Turkey.

2 Calculate the time difference between each of the following locations.


a Tokyo GMT +9 and New York GMT 5
b Los Angeles GMT 8 and Dakar GMT 1
c Rio De Janeiro GMT 3 and Perth GMT +8
d Hawaii GMT 11 and Fiji GMT +11

Digital doc
SkillSHEET 8.3
doc-11076
Converting units
of time

3 WE13 Sydney is GMT +10, while San Francisco is GMT 8. When it is 5.00 pm on Tuesday in

Sydney, what is the time in San Francisco?

Chapter 8 Spherical geometry 245

4 For each of the following calculate:


a the time in Perth (GMT +8) when it is 10.00 pm in Sydney (GMT +10)
b the time in Washington (GMT 5) when it is 4.00 am Saturday in Sydney (GMT +10)
c the time in Auckland (GMT +12) when it is 7.00 am Wednesday in Johannesburg (GMT +2)
d the time in Sydney (GMT +10) when it is 6.00 am Tuesday in Salt Lake City (GMT 7)
e the time in Adelaide (GMT +9.5) when it is 8.15 pm Sunday in the Cook Islands (GMT 10).

5 Jane is in Sydney (GMT +10) and wants to telephone her friend in Paris (GMT) at 7.00 pm Friday,

Paris time. At what time must she call from Sydney?

6 Carl is holidaying in Hawaii (GMT 11). If he wants to call his parents in Sydney (GMT +10) at

8.00 pm on Wednesday, what time must he call from Hawaii?

7 Neville is in Sydney (GMT +10). He wants to set his video recorder to tape the Superbowl which

is being played in Atlanta (GMT 5) and televised live in Sydney. The Superbowl is due to begin
at7.00pm on Sunday in Atlanta. At what day and time will Neville need to set his video to begin taping?

8 Sydney is GMT +10.


a What is the time in Sydney when it is noon GMT?
b When daylight saving time is applied, describe the time zone in Sydney as compared to GMT.
c During daylight saving time, what will the time be in Sydney when it is:
i 4.00 am Monday GMT
ii 9.00 pm Thursday GMT?
9 Sydney is GMT +10 and Los Angeles is GMT 8. Calculate the time difference between Sydney

and Los Angeles when:


a both cities are on standard time
b Sydney has daylight saving time and Los Angeles is on standard time
c Los Angeles has daylight saving time and Sydney is on standard time.

10 WE14 Beijing is at approximately (40N, 120E). Rome is at approximately (40N, 15E).

Calculate the difference in standard time between Beijing and Rome.


11 Calculate the time difference in standard time between each of the following cities.
a Mumbai (19N, 73E) and Casablanca (23N, 82W)
b Tokyo (36N, 140E) and Adelaide (23S, 134E)
c Miami (26N, 80W) and Seattle (47N, 122W)
12 MC At a point on the Earths surface, the coordinates are (45N, 135W). The standard time at this

point would be:


A GMT 3

B GMT +3

C GMT 9

D GMT +9

13 MC It is 11.00 am Tuesday at a point X with coordinates (32S, 90W). At a point, Y, with

coordinates (51N, 120E), what is the time if daylight saving time applies at Y?
A 9.00 pm Monday
B 10.00 pm Monday
C 1.00 am Wednesday
D 2.00 am Wednesday
246 Maths Quest HSC Mathematics General 2

14 WE15 A plane leaves Sydney (32S, 150E) at 2.00 pm on Tuesday. If it is an 18-hour flight to Los

Angeles (33N, 120W), at what time will the plane touch down in Los Angeles?
15 A plane leaves Perth (32S, 120E) on an 8-hour flight to Cape Town (33S, 15E) at 3.00 pm
Wednesday.
a At what time will the plane arrive in Cape Town?
b The return flight leaves Cape Town at 5.00 pm Saturday. At what time will it arrive in Perth?
16 A flight leaves Melbourne (40S, 150E) at 5.00 pm Tuesday on an 18-hour flight to Frankfurt
(50N, 15E). Calculate the time of arrival in Frankfurt if it is:
a daylight saving time in Melbourne
b daylight saving time in Frankfurt.

Further development
17 Robert lives in eastern Australia (longitude 150E) and

likes to watch cricket. In June he watches cricket from


England (longitude 0). The game starts at 11.00 am
daylight saving time in England and Robert watches the
game from 8:00 pm eastern standard time.
In January when England is on standard time and
eastern Australia is on daylight saving time, Kyle, who
is in England, wants to watch a game of cricket from
Australia that also starts at 11.00 am. What is the time in
England when Kyle is watching the match?
18 A plane that flies from Sydney (30S, 150E) and flies to Buenos Aires (30S, 60W) leaves Sydney
at 1.00 pm Wednesday. The flying time is 13 hours.
a What day and time does the plane arrive in Buenos Aires?
b What is the difference in time between arrival in Buenos Aires and departure in Sydney?
c What will be the difference between departure and arrival for the return journey?
19 Zdenka lives in Sydney (30S, 150E) and needs to telephone Kim, who lives in Los Angeles
(30N, 120W).
a Kim tells Zdenka to work out when to call her by adding five hours and going back one day. When
Zdenka calculates the time difference she thinks Kim is out by 1 hour. Show that Zdenka is correct.
b Kim says to Zdenka that the one hour difference is because of daylight saving time. Assuming
Kims advice to Zdenka is correct, determine if Sydney or Los Angeles is on daylight saving time.
c Devise a rule similar to Kims for when the opposite city has daylight saving time.
20 Kin Yong likes to play the stock exchanges from around the world from his home computer in
Sydney (30S, 150E). In each city the stock exchange operates from 9.00 am until 5.00 pm local
time.
a Calculate the hours in Sydney that each of the following stock exchanges are open.
i Hong Kong (120E) ii London (0)
iii New York (75W)
iv Auckland (180)
b Explain what Kin Yong means when he says, The stock market never closes.
21 Explain why:
a Melbourne (33S, 148E) and Brisbane (29S, 151E) have the same time even though they have
different coordinates
b the time difference is greater between Sydney and Honolulu than Sydney and London even
though Honolulu is closer to Sydney than London
c it is possible to fly west and arrive at your destination at an earlier time than it was when you left.

Digital doc
WorkSHEET 8.2
doc-11077

The keepers of time


Information about the world time zones can be found by clicking on the Royal Observatory Greenwich
weblink in your eBookPLUS. The Royal Observatory Greenwich is recognised worldwide as the
keeper of time. Information can be found here about all time zones throughout the world and other
factsabouttime.

Chapter 8 Spherical geometry 247

Summary
Arc lengths

The circumference of a circle is found using either C = d or C = 2r.


The length of an arc can be found using the formula:
l=

2r
360

where is the number of degrees in the central angle.


Great circles and
small circles

An axis of the sphere is any diameter. A diameter must pass through the centre of the sphere.
Theendpoints of the axis are called the poles.
A great circle is any circle of maximum diameter that can be drawn on the surface of a sphere.
Thecircumference of a great circle can be found using the formula C = 2r.
A small circle is any circle on the surface of the sphere that is smaller than a great circle.

Latitude and
longitude

A point on the Earths surface is located using a pair of coordinates.


A point is located as being either north or south of the equator using latitude. Latitude is the angle
subtended at the centre of the sphere by the small circle on which the point lies.
A point is located as being either east or west of the Greenwich Meridian by the meridian of
longitude. Each meridian of longitude is measured by the angle between the meridian of longitude
and the Greenwich Meridian.

Distances on the
Earths surface

If two points lie on the same great circle, the angular distance between them can be found by finding
the difference between their latitudes.
The distance between two points can then be found in nautical miles using 1 = 60 M.
1 M 1.852km.
The distance between two points can also be found in kilometres using the arc length formula.
Speed can be measured in knots, where 1 knot = 1 M/h.

Time zones

The longitude of a city determines its time zone.


The standard time at any location can be calculated using 15 = 1 hour or 1 = 4 minutes.
Points to the east of the Greenwich Meridian have standard time ahead of GMT. Points to the west
ofthe Greenwich Meridian have standard time behind GMT.

248 Maths Quest HSC Mathematics General 2

Chapter review
1 A circle has a diameter of 12cm. An arc is drawn on the circumference of the circle such

thatthearcsubtends an angle of 45 at the centre of the circle. The length of the arc is
closest to:
A 4.7cm
B 9.4cm
C 14.1cm
D 37.7cm

M U LT IP L E
C H O IC E

2 A great circle on the surface of a planet has a circumference of approximately 10700km. The

diameter of the planet would be closest to:


A 850km
C 3400km

B 1700km
D 6800km

3 Point X on the Earths surface has coordinates (69S, 12E), while point Y is at (8S, 12E).

Thedistance between X and Y is:


A 61 M
C 3660 M

B 77 M
D 4620 M

4 The coordinates of two points, M and N, on the surface of the Earth are (45N, 45W) and

(30S, 60E). Which of the following statements is correct about the time difference between
M and N?
A M is 5 hours behind N.
B M is 5 hours ahead of N.
C M is 7 hours behind N.
D M is 7 hours ahead of N.
1 Calculate the circumference of each of the following circles, correct to 1 decimal place.
a

b

c
19.2 cm

8.6 cm

S ho rt
a nsw er

92 mm

2 Calculate the length of each of the following arcs, correct to 1 decimal place.
a

b

c
9m
40

260
6.2 cm
135
9.3 m

3 A sphere has a radius of 7.5cm.


a Calculate the circumference of the sphere, correct to 1 decimal place.
b Calculate the distance between two points on the surface of the sphere that subtend an angle of

60 at the centre. Give your answer correct to 2 decimal places.


4 Calculate the circumference of each of the following spheres, correct to 1 decimal place.
a

b

c
33 cm

7 cm

9.2 m

5 Calculate the circumference of a great circle that lies on the surface of a sphere with a radius of

9km. (Give your answer correct to 1 decimal place.)


6 A sphere has a diameter of 45cm. Calculate the distance between the poles on this sphere. Give your

answer correct to the nearest centimetre.


Chapter 8 Spherical geometry 249

7 Calculate the circumference of each of the small circles drawn below, correct to 1 decimal place.
a

b

c
2.8 m
48
7 cm

8.1 m

1500 km
3000 km

20
6.6 cm

8 Use the world map on page 238 to identify the cities at each of the following locations.
a (14N, 121E)
b (12S, 76W)
c (33S, 71W)
9 Use the world map on page 238 to give the approximate coordinates of each of the following cities.
a Madrid
b Singapore
c Hobart
10 The points X and Y on the Earths surface have coordinates (32N, 120E) and (26S, 120E).

Calculate the angular distance between X and Y.


11 A ship gives its coordinates as (56N, 14W) and is sailing to a port at (40N, 14W).

a Calculate the angular distance through which the ship must sail to reach port.
b Calculate the distance the ship must sail, in nautical miles.
c Use 1 M = 1.852km to calculate the distance, correct to the nearest kilometre.
12 The angular distance between two points on the same great circle is 120. Calculate the time that it

would take to sail between these points at a speed of 48 knots.


13 Santiago has approximate coordinates

(33S, 70W), while Santo Domingo has


approximate coordinates (18N, 70W).
a Calculate the distance between Santiago
and Santo Domingo:
i in nautical miles
ii correct to the nearest kilometre.
b Calculate the time taken to fly from
Santiago to Santo Domingo at a speed of
480 knots.
14 The coordinates of Perth are approximately (32S, 115E), while Hong Kong is approximately at
(22N, 115E). Taking the radius of the Earth to be 6400km, use the arc length formula to calculate
the distance between Perth and Hong Kong, correct to the nearest 100km.
15 Calculate the time difference between each of the following cities.
a Sydney (GMT +10) and Istanbul (GMT +2)
b Perth (GMT +8) and New York (GMT 3)
c Ottawa (GMT 5) and Fiji (GMT +12)
250 Maths Quest HSC Mathematics General 2

16 In Dhahran (GMT +4) the time is 10.00 pm on Wednesday. Calculate the time in Tokyo (GMT +9).

17 Ann is on a skiing holiday in Winnipeg, Canada (GMT 6). She needs to call her parents at 7.30 pm

on Tuesday night, Sydney time. At what time should she make the call from Winnipeg?

Chapter 8 Spherical geometry 251

18 Kingston, Jamaica is at approximately (18N, 75W) while Oslo, Norway is at approximately

(60N, 15E). Calculate the time:


a in Oslo when it is 5.00 am in Kingston

b in Kingston when it is 5.00 pm in Oslo.

19 A plane is flying from Munich (48N, 15E) to New York (41N, 75W). The flight departs Munich

at 6.00 pm and takes 7 hours. Calculate the time of arrival in New York.

Ext end ed
R esp ons e

1 The city of St Petersburg is at approximately (60N, 30E), while the city of Johannesburg has

approximate coordinates of (25S, 30E).

a Calculate the angular distance between St Petersburg and Johannesburg.


b Taking the radius of the Earth to be equal to 6400km, calculate the distance between

StPetersburg and Johannesburg, correct to the nearest 100km.


c In Sydney (30S, 150E) daylight saving time applies. Calculate the time difference between

StPetersburg and Sydney.


2 The approximate coordinates of Tokyo are

Digital doc
Test Yourself
doc-11078
Chapter 8

(36N, 140E), while San Francisco is at


approximately (36N, 120W).
a Do San Francisco and Tokyo lie on the same
great circle or the same small circle? Explain
your answer.
b An aeroplane takes 8 hours to fly between
Tokyo and San Francisco. If a plane leaves
Tokyo at 10.00 pm on Saturday, Tokyo time,
what day and time will it arrive in San
Francisco?
c If the return flight leaves San Francisco at 8.00am Tuesday, what day and time will it arrive
in Tokyo?

252 Maths Quest HSC Mathematics General 2

Activities
8AArc lengths

8ETime zones

Tutorial
WE2 int-2443: Perform a calculation of an arc length.
(page 229)

Interactivity
int-0006: Day, night and time zones. (page 243)

Digital docs
SkillSHEET 8.1 (doc-11073): Circumference of a circle. (page 230)
SkillSHEET 8.2 (doc-11074): Calculating arc length. (page 231)

8BGreat circles and small circles


Tutorial
WE5 int-2444: Perform a calculation of a circle. (page 234)
Digital docs
WorkSHEET 8.1 (doc-11075): Apply your knowledge of spherical
geometry to problems. (page 236)

8D Distances on the Earths surface

Tutorial
WE14 int-2446: Perform a calculation using time zones.
(page245)
Digital docs
SkillSHEET 8.3 (doc-11076): Converting units of time. (page 245)
WorkSHEET 8.2 (doc-11077): Apply your knowledge of time zones.
(page 247)

Chapter review
Test Yourself (doc-11078): Take the end-of-chapter test to test your
progress. (page 252)

To access eBookPLUS activities, log on to www.jacplus.com.au

Elesson
eles-0138: Understanding map scales and distances. (page 237)
Tutorial
WE9 int-2445: Perform a calculation of a distance on the Earths
surface. (page 241)

Chapter 8 Spherical geometry 253

Answers chapter 8
Spherical geometry

g Tokyo h
Beijing
3 11:00 pm Monday
i Rio de Janeiro j
Oslo
4 a 8:00 pm
Exercise 8A Arc lengths
2 These answers are approximate.
b 1:00 pm Friday
1 a 25.1cm b
56.5m
a (38S, 145E)
c 5:00 pm Wednesday
c 389.6mm d
25.8km
b (40N, 75W)
d 11:00 pm Tuesday
e 87.3km f
52.2m
c (18N, 76W)
e 3:45 pm Monday
2 630km
d (26S, 28E)
5 5:00 am Saturday
3 a 44.0cm b
123m
e (42N, 12E)
6 11:00 pm Tuesday
c 188km d
368mm
f (35S, 57W)
7 10:00 am Monday
e 11.6km f
688km
g (33N, 44E)
8 a 10:00 pm b
GMT + 11
4 9400km
h (55N, 40E)
c i 3:00 pm Monday
5 31.83cm
i (2N, 104E)
ii 8:00 am Friday
6 a 25.5m b
6.73cm c
796km j (18S, 178E)
9 a 18 h b
19 h c
17 h
7 8.73cm
3 a Small circle b
Great circle
10 7 h
8 a 33.8mm b
20.4m
c Great circle d
Great circle
11 a 10 h 20 min b
24 min
c 150.8cm d
27.6cm
e Small circle f
Great circle
c 2 h 48 min
e 5.0km f
20.7km
4 a 71 b
71
12 C
9 a 40200km b
4470km
c 110 d
30
13 D
10 a 52km b
136km
5 a 162 b
132
14 2:00 pm Tuesday
c Circumference of circle = 2r =
c 159 d
140
15 a 4:00 pm Wednesday
188km = sum of two arcs
b 8:00 am Sunday
Exercise 8D Distances on the Earths
11 a 251cm b
62.8cm
16 a 1:00 am Wednesday
surface
12 a 5.2cm b
4.3m
b 3:00 am Wednesday
1
50
c 5696.8km
17 Midnight

2 a
40
b
40
c
71
13 112km
18 a Noon Wednesday
d
21
e
80
14 a 15.9cm b
4.25m
b 1 hour earlier
3
60
c 1910km
c 27 hours

4 a
1800
M
b
3334
k
m
15 30.6cm
19 a Check with your teacher.
5 a 2100 M b
8100
M
16 80
b Sydney
c 2340 M d
5760
M
17 a 54.5cm b
102.5cm
c Add 7 hours and go back 1 day
6 a 3600 M b
6667km
18 a 4446 b
3549
20 a i 11.00 am to 7.00 pm
7 a 2700 M b
5000
k
m
19 4940km
ii 7.00 pm to 3.00 am
8 6600km
Exercise 8B Great circles and small
iii 12.00 am to 8.00 am
9 a 4356km b
4021km
circles
iv 7.00 am to 3.00 pm
c 3798km
1 43.98cm
b The stock market is always open in
10 D
2 a 56.5m b
465mm c
188m
some part of the world.
11 B
3 40210km
21 a The difference in their latitudes is so
12 7 hours 30 minutes
4 314cm
similar they are placed in the same
13 a 110
5 a 15320km b
38010km
timezone.
b 6600 M
c 21350km d
449200km
b Sydney is 150E and Honolulu is
c 12200km
e 378690km f
160590km
155W, meaning that there is a 20-hour
d 13 hours 45 minutes
g 154250km
time difference. Honolulu is closer than
14 10800 M
6 12.6m
London to Sydney but is on the opposite
15 a 28 b
1680 M
7 a 55.3m b
40.2m
side of the International Date Line.
c 3111km d
3128km
c 6911.5km
c When flying west across the
e 1 M 1.852km and radius Earth
8 B
International Date Line, you go back
6400km. We are therefore working with
9 79cm
one day.
approximations.
10 20100km
16 a (60N, 20E) (10S, 20E)
11 a 377cm b
94.25cm
b 3889.20km
Chapter Review
12 a 1750km b
52.4m
17 17779.20km
Multiple choice
13 a Great circle b
Small circle
18 a 6889.44km to South Pole
1
A
c Great circle d
Great circle
b 4555.92km to North Pole
2 C
14 a 50 b
38cm c
239cm c 4889.28km to North Pole
3 C
15 a Check with your teacher.
d 8889.6km to South Pole
4 C
b r = 4.7cm, C = 29.6
19 a 2482km b
3247km

16 a 6014km b
4200km
c 2872km d
427km
Short answer
17 a 4525km b
5924km
e 16374km f
6481km
1 a 120.6cm b
54.0cm
18 Jude is correct. The distance XY is
20 a 5243km b
35
c 289.0mm
7260km, and the distance YZ = 3630km,
c 3203km
2 a 6.3m b
28.1cm
so XY is twice YZ.
21 (0, 75E)
c 21.9m
Exercise 8C

Exercise 8E Time zones


3 a 47.1cm b
7.85cm
Latitude and longitude
4 a 207.3cm b
44.0cm
1 a Cairo b
Shanghai
1 10 h
c 57.8m
c Darwin d
Montreal
2 a 14 h b
7h
e London f
Auckland
c 11 h d
22 h
5 56.5km

254 Maths Quest HSC Mathematics General 2

Extended response
12 6 days 6 hours
6 71cm
13 a i 3060 M
7 a 41.5cm b
17.6m
1 a 85 b
9500km c
8h
ii 5667km
c 9424.8km
2 a Small circle. They lie on the same small
b 6.375 h or 6 hrs 22.5 mins
8 a Manila b
Lima
circle because they lie on the same
14 6000km
c Santiago
parallel of latitude.
15 a 8 h b
11 h c
17 h
9 a (41N, 3W)
b 12:20 pm Saturday
16 3:00 am Thursday
b (1N, 104E)
c 9:40 am Wednesday.
17 3:30 am Tuesday
c (43S, 147E)
18 a 11:00 am the same day
10 58
b 11:00 am the same day
11 a 16 b
960 M
19 7:00 pm the same day
c 1778km

Chapter 8 Spherical geometry 255

Chapter 9

Multi-stage events and


applications of probability
CHAPTER CONTENTS
9A Counting techniques
9B Tree diagrams
9C Probability and counting techniques
9D Expected outcomes

9A Counting techniques
Fundamental counting principle
Tom is a football fan. Tom likes all codes of football and enters a competition to select the winner of
the NRL (16 teams), AFL (18 teams) and Super Rugby (15 teams) competitions. In how many different
ways can Tom select his entry in the competition?
During the Preliminary course we studied at the Fundamental Counting Principle, which states that
The total number of ways that a succession of choices can be made is found by multiplying the
number of ways each choice can be made.
In the case of Tom selecting the winners of the three football codes, the number of possible
selectionsis:

Interactivity
int-0089
Random number
generator

Number of possible selections = 16 18 15


= 4320
WORKED EXAMPLE 1

Maya has 5 albums on her MP3 player. The albums have 18, 12, 15, 10 and 7 songs on them
respectively. If Maya is to select one song to play from each album calculate the number of ways
in which Maya can select the five songs.
Think

Calculate the number of combinations by


multiplying the number of possibilities at
eachstage.

Write

Number of combinations = 18 12 15 10 7
= 226 800

The fundamental counting principle depends upon each event being independent. In the case where
Tom is selecting football teams, the result of the NRL has no effect on the result of either the AFL or Super
Rugby competitions. The sets from which each selection is made have no common elements.
If several selections are being from the same group the events are no longer independent and a
different method of counting needs to be used.

Ordered arrangements
1. Select three people and stand them in a line.
2. Now get the three people to stand in a different order.
3. In how many different orders can the three people be placed?
Chapter 9 Multi-stage events and applications of probability 257

4. Repeat the above process with four people in the line.


5. Is there a pattern? Can you calculate the number of different ways in which five people can be arranged?
There are 10 people standing in a line. In how many ways can they be arranged? To calculate this we
need to consider the number of ways that each place in the line can be filled. To do this we need to
calculate the number of people remaining after we fill each place in the line.
There are 10 people who could fill the first position.
Once the first position has been filled, there are nine people remaining to fill the second position.
Once the second position has been filled, there are eight people remaining to fill the third position.
This pattern continues until there is only one person left who can fill the last position.
Calculating this: 10 9 8 7 6 5 4 3 2 1 = 3628800.
A shorter way of writing 10 9 8 7 6 5 4 3 2 1 is to write 10!, that is, 10 factorial.
Your calculator will have a factorial function, usually labelled x!. Make sure that you know where this
function is on your calculator.
WORKED EXAMPLE 2

Calculate the value of 8!.


THINK

WRITE

Enter 8 and press " on the calculator.

8! = 40 320

WORKED EXAMPLE 3

Six people are standing in a line. In how many ways can the six people be arranged?
THINK
1
2

WRITE

The answer is 6!

6! = 720

Give a written answer.

The people can be arranged in 720 ways.

Tree diagrams and ordered arrangements


Four people, Anji, Belinda, Kristen and Summer, are to be
placed in order.
1. Calculate the number of different ways these four girls can
be placed in a line.
2. Draw a tree diagram and use it to list the ways that the four
girls can be placed in order.
3. Check that the number of elements in the sample space
found from your tree diagram corresponds to the answer
obtained in part 1.
In Worked examples 2 and 3, we have been ordering an
entire group. In some cases we may wish to order only part of
the group. Consider the case of an Olympic swimming final.
There are eight swimmers and we wish to know the number of
ways that the gold, silver and bronze medals can be awarded.
There are eight possible winners of the gold medal.
With the first place filled, there are seven possible winners of the silver medal.
With both first and second places filled, there are six possible winners of the bronze medal.
Calculating this: number of arrangements = 8 7 6
= 336
This type of arrangement is known as an ordered selection. It occurs when the order in which the
choices are made is important.
In the worked example below, a captain and a vice-captain are to be chosen. If Benito is captain and
Imran is vice-captain, this is a different selection to Imran as captain and Benito as vice-captain.
258 Maths Quest HSC Mathematics General 2

To calculate the number of ordered selections that can be made, we multiply, starting from the number
of possible first selections, then reducing by one with each multiplication until each position is filled.
WORKED EXAMPLE 4

In a cricket team of eleven players, a captain and vice-captain are to be chosen. In how many
ways can this be done?
THINK
1

There are 11 possible choices of captain.

Once the captain is chosen, there are 10 choices


remaining for vice-captain.

WRITE

Number of arrangements = 11 10
= 110

Committee selections
On a committee of five people, a president and a vice-president are to be chosen. The five committee
members are Andreas, Brett, Cathy, Dharma and Emiko.
1. Use the method shown in Worked example 4 to calculate the number of ways in which the president
and the vice-president can be chosen.
2. Now use a tree diagram to list the sample space of all possible selections of president and vice-president.
3. Check that the number of elements in the sample space corresponds to the answer obtained in part 1
of this investigation.
Consider a case where two representatives to a committee are chosen from a class of 20 students. This
is an example of an unordered selection. If Sue is chosen, followed by Graham, this is the same choice
as if Graham is chosen and then Sue.
To calculate the number of unordered selections that can be made, we calculate the number of ordered
selections that can be made and then divide by the number of arrangements of these selections. This is
calculated using factorial notation as in Worked example 3. In the case of choosing the committee:
Number of ordered selections is 20 19 = 380.
Two people can be arranged in two (2!) ways.
Number of unordered selections = 380 2
= 190
WORKED EXAMPLE 5

From a group of eight athletes, three are to be chosen to represent the club at a carnival. In how
many ways can the three representatives be chosen?
THINK

WRITE

Calculate the number of ordered selections that


can be made.

Ordered selections = 8 7 6
= 336

Calculate the number of arrangements of the


representatives.

Arrangements = 3 2 1
=6

Divide the ordered selections by the


arrangementsof the representatives.

Unordered selections = 336 6


= 56

Tutorial
int-2424
Worked example 5

Unordered selection
A rowing team has six members: Mark, Norman, Olaf, Pieter, Quentin and Raymond. Two are to be
chosen to be the crew in a pairs race.
1. Use the method described in Worked example 5 to calculate the number of pairs that could be chosen.
2. Use a tree diagram to list the ordered selections and then write the sample space of unordered
selections by ignoring any repeated pair.
3. Check that the number of elements of the sample space corresponds to the answer obtained in part 1
of this investigation.
Chapter 9 Multi-stage events and applications of probability 259

Exercise 9A

Counting techniques

1 WE1 April is going out to dinner. The banquet menu has a selection of 5 entrees, 6 main courses,

4desserts and a selection of 3 drinks. How many meal combinations is it possible to choose?
Digital doc
SkillSHEET 9.1
doc-11079
Fundamental
counting principle

2 At a high school there are 5 classes in each year from 7 to 12. If one class from each year is to be

chosen to participate in a survey, how many combinations of classes is it possible to choose.


3 Merridee and Bryce are heading out on their first date. Each is nervous and fussy about what to wear.
a Merridee has to coordinate one skirt from a selection of 6, one blouse from a selection of 8 and

a pair of shoes from a choice of 30. How many possible combinations of outfit could Merridee
choose?
b Bryces task is to select a shirt from a choice of 8, a pair of pants from a choice of 5, one of three
ties. Bryce only has one pair of shoes. How many combinations could Bryce possibly select?
c When they go out together how many possible combinations of outfits are possible?
4 WE2 Use your calculator to calculate the value of the following.
a 3!
b 5!
c 9!
5 WE3 Four people are involved in a race. In how many different orders can they complete

the race?
6 The letters A, B, C, D and E are written on cards. In how many different orders can the cards be

placed?
7 A three-digit number is formed using the digits 3, 6 and 8. If no number can be repeated, how many

numbers is it possible to form?


8 In a race of 10 people, in how many different ways can the first three places be filled?
9 WE4 In a school, a captain and vice-captain are to be elected. The four nominations are Geri, Reika,

Melanie and Victoria. In how many different ways can the captain and vice-captain be chosen?
10 In the Melbourne Cup there are

24 horses. In how many different


ways can the three placings be filled?
11 WE5 Seven people try out for three

places on a debating team. In how


many ways can the team of three be
chosen from the group of seven?
12 How many different groups of four

can be selected from ten people?


13 In his pocket Trevor has six coins:

a $2 coin, $1 coin, 50c coin,


20ccoin,10c coin and 5c coin. If
Trevor randomly chooses two coins,
how many different sums of money
are possible?
14 On a restaurant menu there is a choice of three entrees, six main courses and four desserts. In how

many ways can a person choose an entree, main course and dessert from the menu?
15 MC Which of the following is an example of an unordered selection?
A Five students are placed in order of their exam results.
B From a group of five students, a contestant and a reserve are chosen for a Mathematics

competition.
C From a group of five students, two are chosen to represent the class on the SRC.
D From a group of five students, two are awarded 1st and 2nd prizes in Mathematics.

16 MC The numbers 1, 2, 3 and 4 are used to form a three-digit number such that no digit can be used

more than once. The number of three-digit numbers that can be formed is:
A 4
B 6
C 12
D 24
17 MC Gavin, Dion, Michael, Owen and Shane try out for two places on a tennis doubles team. The

number of teams that can be chosen is:


A 5
B 10
260 Maths Quest HSC Mathematics General 2

C 20

D 25

18 A small play has three characters. Six people, Wendy, Rebecca, Thai, Yasmin, Andrea and Ophelia,

audition for the three parts.


a How many different groups of three can be chosen for the play?
b In how many different ways can the three parts be allocated to the three girls?
19 At the Olympic qualifying trials,

nine cyclists compete for a place on the


team.
a In how many different orders can the
competition finish?
b How many different ways can 1st, 2nd
and 3rd place be filled?
c Two cyclists are chosen to represent
Australia on the team. How many
different teams of two can be chosen?

Further development
20 A rowing crew consists of four rowers who sit in a definite order. How many different crews are

possible if 5 people try out for selection?


21 The school musical needs a producer, director, musical director and script coach. Nine people have

volunteered for any of these positions. In how many different ways can the positions be filled?
Note:One person cannot take on more than 1 position.
22 There are four people in a race. Explain why the number of ways that the first three places can be

filled is the same as the number of ways that all four places can be filled.
23 A rugby union squad has 12 forwards and 10 backs in training. A team consists of 8 forwards and

7backs. How many different teams can be chosen from the squad?
24 Lotto is a gambling game played by choosing 6 numbers from 45. Gamblers try to match their

choice with those numbers chosen at the official draw. No number can be drawn more than once and
the order in which the numbers are selected does not matter.
a How many different selections of 6 numbers can be made from 45?
b Suppose the first numbers drawn at the official draw are 42, 3 and 18. How many selections of
6numbers will contain these 3 numbers?
c Suppose the first numbers drawn at the official draw are 42, 3, 18 and 41. How many selections of
6 numbers will contain these 4 numbers?
Note: This question ignores supplementary numbers.
25 Explain why the number of ways that three people can be selected from 10 is the same as the number

of ways that seven people can be selected from 10.

9B

Tree diagrams

If an event has more than one stage to it, then a tree diagram can be drawn to list the sample space
accurately. In a tree diagram, the tree branches out once for each stage of the experiment. At each stage
the number of branches is the same as the number of possible outcomes.
To list the sample space we then follow the tree to the end of each branch and record the outcome at
each stage.
WORKED EXAMPLE 6

A coin is tossed three times. Draw a tree diagram and use it to list the sample space for this
experiment.
THINK
1

There are three stages to the experiment.

At each stage the outcome can be heads or tails.

WRITE

Chapter 9 Multi-stage events and applications of probability 261

Draw the tree diagram branching out three


times with two branches at each stage.

1st
coin
toss

2nd
coin
toss

3rd
coin
toss
Heads
Tails
Heads
Tails
Heads
Tails
Heads
Tails

Heads
Heads
Tails
Heads
Tails
Tails
4

List the sample space by following the path to


each end branch.

S = { HHH, HHT, HTH, HTT, THH, THT,


TTH,TTT}

In the above example, each stage of the


experiment (each toss of the coin) is
independent of the other stages. That is to say,
the outcome of one toss does not affect the
outcome of another toss. In many examples, the
outcome of one stage will affect the outcome of
another. Consider Worked example7. Here we
are forming a two-digit number such that no
digit may be repeated. Once a number has been
chosen as the first digit, it cannot be chosen as
the second digit. Therefore, the first stage of the
experiment does affect the second stage.

WORKED EXAMPLE 7

A two-digit number is formed using the digits 4, 5, 7 and 9 without repetition. Draw a tree
diagram and use it to list all possible numbers that can be formed.
THINK

WRITE

There are two stages to the experiment.

For the first stage there will be four branches


and since one number is chosen there will be
three branches for the second stage.

Draw the tree diagram.

List the sample space by following the branches to


each end point on the tree diagram.

1st digit
4
5
7
9

2nd digit
5
7
9
4
7
9
4
5
9
4
5
7

Sample space
45
47
49
54
57
59
74
75
79
94
95
97

Once the tree diagram is completed, the probability of an event can be calculated using the formula:
P(event) =

262 Maths Quest HSC Mathematics General 2

number of favourable outcomes


total number of outcomes

WORKED EXAMPLE 8

A coin is tossed and a die is rolled. Calculate the probability of tossing a tail and rolling a
number greater than 4.
THINK

WRITE

There are two stages to the event.

At the first stage there are two outcomes and at


the second stage there are six outcomes.

Coin toss
Heads

Draw the tree diagram.

Tails

Die roll
1
2
3
4
5
6
1
2
3
4
5
6

List the sample space by following the branches


to each end point on the tree diagram.

Calculate the probability using the probability


formula. There are two favourable outcomes
T5 and T6.

2
P(tail and no. > 4) = 12

Simplify.

Exercise 9B

Sample space
Heads 1
Heads 2
Heads 3
Heads 4
Heads 5
Heads 6
Tails 1
Tails 2
Tails 3
Tails 4
Tails 5
Tails 6

= 16

Tree diagrams

1 WE6 A family consists of four children. Draw a tree diagram to show all possible combinations of

boys and girls.


2 Two dice are cast. Draw a tree diagram that will allow you to list

the sample space of all possible outcomes.


Digital doc
SkillSHEET 9.2
doc-11080
Listing the sample
space

3 There are two bags each containing a red, blue, yellow and green

marble. One marble is to be chosen from each bag. Draw a tree


diagram that will allow you to calculate the sample space.
4 A school is to send one male and one female representative to a

conference. The boys nominate George, Frank, Stanisa and Ian; the
girls have nominated Thuy, Petria, Joan, Wendy and Amelia. Draw a tree diagram and list the sample
space for all possible choices of representatives.
5 WE7 A two-digit number is to be formed using the digits 1, 2, 4, 5 and 7 such that no digit may be

repeated. Draw a tree diagram to list all possible numbers that can be formed.
6 A committee needs to elect a president, secretary and treasurer. The four nominations for these

positions are Belinda, Dean, Kate and Adrian. Given that no person is allowed to hold more than one
position, use a tree diagram to list all ways in which these three positions can be filled.

Digital doc
SkillSHEET 9.3
doc-11081
Informal
description of
chance

7 The digits 3, 5, 7 and 8 are used to form a three-digit number. If no digit can be used more than once

list the sample space.


8 MC From a group of five nominations a school captain and vice-captain are to be elected. The

number of ways that the captain and vice-captain can be chosen is:
A 5
B 10
C 20

D 25

Digital doc
SkillSHEET 9.4
doc-11082
Equally likely
events

9 WE8 The four aces from a deck of cards are placed face down on a table. One card is chosen

followed by a second card without the first card being replaced. Calculate the probability that the ace
of hearts is one of the two cards chosen.
10 A two-digit number is formed using the digits 2, 3, 4 and 7 without repetition.
a Use a tree diagram to list the sample space.
b Calculate the probability that the number formed is greater than 35.

Digital doc
EXCEL Spreadsheet
doc-1364
Tree diagrams

Chapter 9 Multi-stage events and applications of probability 263

11 A tennis team consists of three men, Andre, Yevgeny and Jonas and two women, Martina and

Lindsay. From the team the captain and the vice-captain are to be chosen. Calculate the probability
that the captain and vice-captain are:
a Andre and Lindsay
b both men
c the same sex
d different sex.
12 Find the probability that all three children in a family will be the same sex.

13 MC A three-digit number is formed using the digits 5, 6, 8 and 0. No digit can be repeated and the

0cant be first. The probability of the number formed being greater than 800 is:
1
1
3
1
A
B
C
D
4
3
16
2
14 An airline offers holidays to three destinations: Brisbane, Gold Coast or Cairns. The holiday can

be taken during two seasons: Peak season or Off-peak season. The customer has the choice of three
classes: Economy, Business or First class. There is no First class to Cairns, however.
a Use a tree diagram to list all combinations of holiday that could be taken by choosing a
destination, season and class.
b Terry takes a mystery flight, which means he is allocated a ticket at random from the above
combinations. Calculate the probability that Terrys ticket:
i goes to Brisbane
ii is First class
iii is in Peak season, flying First class.

Further development
15 A three-digit number is to be formed using the digits 5, 7 and 8.
a If the same digit cannot be used twice, how many three-digit numbers can be formed?
b If repetition is allowed, how many numbers can be formed?
c Kevin claims that the probability of an even number will be the same regardless of whether

repetition of digits is allowed or not. Is Kevin correct? Explain why or why not.
16 In a family of four children there can be:

more boys
more girls
an equal number of boys and girls.
a Is each of these outcomes equally likely to occur?
b Dan claims that if there is an even number of children the probability of there being an equal
number of boys and girls is the same. Is Dan correct? Explain your answer.
17 Give a brief explanation of why, when two dice are rolled, that each total is not equally likely

tooccur.
18 Tanya is allergic to peanuts. On a menu there are 4 entrees, 6 main courses and 4 desserts. Of these,
Digital doc
WorkSHEET 9.1
doc-11083

2 entrees, 3 main courses and 2 desserts contain traces of peanuts. Tanya says that that the number of
combinations of meals that she may choose is halved. Is Tanya correct? Explain your answer.
19 Ingrid tosses a coin 10 times and claims that there is exactly a 90% chance that she will throw at

least one Head. Is Ingrid correct? Explain your answer.

264 Maths Quest HSC Mathematics General 2

9C

Probability and counting techniques

Once the counting techniques done in the previous section have been completed, we can calculate the
probability of certain events occurring. To do this we go back to using the probability formula:
P(event) =

number of favourable outcomes


total number of outcomes

Interactivity
int-2787
A pack of cards
int-0085
Random numbers

WORKED EXAMPLE 9

The letters A, H, M, S and T are written on cards. The cards are shuffled and then laid out face
up. Calculate the probability that the cards form the word MATHS.
THINK

WRITE

The five cards can be arranged in 5! ways.

No. of arrangements = 5!
=54321
= 120

MATHS is one way of arranging the letters


and so we use the probability formula.

1
P(MATHS) = 120

We also need to be able to calculate the probability of a particular ordered or unordered arrangement
occurring.
WORKED EXAMPLE 10

From Francis, Gary, Harley, Ike and Jacinta, a school captain and vice-captain need to be
elected. Calculate the probability that Ike and Jacinta occupy the two positions.
THINK
1

Calculate the number of ordered selections that are


possible.

Ike and Jacinta in the two positions can be arranged


in two ways.

Divide the ordered selections by the number of


arrangements.

Substitute into the probability formula.

WRITE

No. of ordered selections = 5 4


= 20
No. of arrangements = 2 1
No. of unordered selections = 20 2
= 10
1
P(Ike and Jacinta) = 10

WORKED EXAMPLE 11

A bag contains a red, green, yellow, blue, orange and purple marble. Three marbles are selected
from the bag. Calculate the probability that the red, yellow and orange marbles are chosen.
THINK
1

Calculate the number of ordered selections.

Calculate the number of arrangements.

Calculate the number of unordered selections.

The red, yellow and orange marble is one possible


selection.

Substitute into the probability formula.

WRITE

No. of ordered selections = 6 5 4


= 120
No. of arrangements = 3 2 1
=6
No. of unordered selections = 120 6
= 20

P(red, yellow and orange) =

Tutorial
int-2425
Worked
example 11

1
20

Chapter 9 Multi-stage events and applications of probability 265

Popular gaming
There are many different forms of lottery that depend upon ordered or unordered arrangements.
1. Lotto This requires the player to select six numbers out of 45. In how many ways can the
sixnumbers be chosen? Remember order does not matter.
2. Similar games to Lotto are:
a. Oz Lotto seven numbers are chosen from 45.
b. The Pools six numbers are chosen from 38.
In how many ways can the six numbers for each of these games be chosen?
3. Powerball This requires the player to choose six numbers from 40 in an unordered selection.
Aseventh ball (the powerball) is chosen from a second barrel containing 20 balls. In how many ways
can this be selected?
4. Lotto Strike The player must select the first four balls drawn from 45 in the correct order. In how
many ways can this ordered selection be made?

Exercise 9C

Probability and counting techniques

1 WE9 Four people, Craig, Barry, Anne and Dimitri, are arranged in a line. Calculate the probability

that the four people are arranged in alphabetical order.


Digital doc
SkillSHEET 9.5
doc-11084
Single event
probability

2 The numbers 4, 5, 6, 7 and 8 are arranged to form a five-digit number such that no digit can be

repeated. Calculate:
a how many five-digit numbers can be formed
b the probability that the number formed is 54867
c the probability that the number formed is 86574.
3 A three-digit number is formed using the digits 6, 8 and 9 and no digit may be repeated. Calculate

the probability that the number formed is:


a 896
b even

c greater than 800.

4 WE10 There are five candidates in an election for SRC president. The second placed candidate

will be made vice-president of the SRC. If Lauren and Meta are two of the candidates, calculate the
probability that they will occupy the two positions.
5 Seven surfers enter a competition. If two of the surfers are Kurt and Paul, calculate the

probabilitythat:
a Kurt comes first and Paul comes second
b Paul comes first and Kurt comes second
c Kurt and Paul fill the first two places.

266 Maths Quest HSC Mathematics General 2

6 From the digits 1 to 9 a two-digit number is formed such that no digit can be repeated. Calculate the

probability that the number formed is:


a 67
b greater than 80

c less than 30.

7 WE11 From a deck of cards, the four aces are laid face down on a table. Two of the aces are then

turned face up. Calculate the probability that the two aces turned face up are the ace of clubs and the
ace of spades.
8 An ice-cream parlour offers a choice of 25 flavours. A triple

scoop ice-cream places three different flavours on top of each other.


Ifthe flavours are chosen randomly, find the probability that the
ice-cream is:
a vanilla, chocolate and strawberry in that order
b vanilla, chocolate and strawberry in any order.
9 Six boys try out for three places on a debating team.

The boys are Gavin, David, Andrew, Rhyse, Julius


and Elliot.
a How many teams of three is it possible to choose?
b Calculate the probability that Gavin, Andrew and
Elliot are on the team.
10 The letters M, A, I, D and G are written on cards and two of these are to be chosen. Calculate the

probability that the two cards chosen are:


a both vowels
b both consonants

c one vowel and one consonant.

Further development
11 To win Lotto you must correctly select the six correct winning numbers from a possible 45 numbers.
a Find the probability of winning Lotto when selecting a single set of six numbers.
b To increase their chances of winning some players take a systems entry. This means selecting more

than six numbers. For example, a system 7 is where 7 numbers are chosen and is equivalent to all
combinations of six numbers that can be chosen within 7. Find the probability of winning Lotto if:
i a system 7 entry is played
ii a system 8 entry is played
iii a system 15 entry is played.
12 A second game that is played within Lotto is called Lotto Strike. This involves selecting the first four

Lotto balls drawn in the correct order. Determine whether Lotto or Lotto Strike is harder to win.
13 Oz Lotto is another similar game where the player has to select seven numbers from 45.

John thinks that, because you need to select one extra ball, Oz Lotto will be 17 or 14.3% harder
towin.
Peter thinks that, because you need to select one extra ball, Oz Lotto will be 16 or 16.7% harder
towin.
Bruce thinks that both Peter and John are incorrect and that Oz Lotto will be about 450% harder
to win.
Who is correct? Explain your answer.

14 Powerball is a game where the player must select six numbers out of 40 from one barrel and then a

single ball out of 20 from a second barrel.


a Find the number of ways that the first set of five numbers can be selected.
b Find the number of ways that the single number can be selected.
c Find the probability of winning Powerball with a single entry.
15 Keno is a popular game where 20 numbers are selected from 80. The major jackpot is won by a

player selecting 10 numbers and those 10 numbers being among the 20 drawn.
a In approximately how many ways can the player select 10 numbers from 80? Give your answer in
scientific notation correct to 3 significant figures.
b How many winning combinations of 10 numbers from 20 are there?
c Find the probability of selecting a winning combination in scientific notation correct to
3significant figures.
16 In the game of Keno find the approximate probability, as a decimal, of winning the:
a 2 number game
b 3 number game
c 5 number game.
Chapter 9 Multi-stage events and applications of probability 267

9D

Expected outcomes

Suppose that we toss a coin 100 times. How many times would you expect the coin to land Heads? As
each outcome is equally likely, we would expect there to be 50 Heads and 50 Tails. How can this be
shown to be true?
The number of times that we expect a certain outcome to occur is found by multiplying the
probabilityof each outcome by the number of trials. In the above case, the probability of the coin
landing heads is 12, and this is multiplied by the number of trials (100). The result is an expectation of
50Heads in 100 tosses of the coin.
The expected outcome is the number of times that we expect a particular outcome to occur in a
certain number of trials.
WORKED EXAMPLE 12

A die is rolled 120 times. How many 6s would you expect to occur in 120 rolls of the die?
THINK

WRITE

Calculate the probability of rolling a 6.

P(six) = 16

Multiply the probability of a 6 by the number


oftrials.

Expected number of 6s = 16 120



= 20

If the expected number of 6s is 20 in 120 rolls of a die, this does not mean that this is what will occur.
It may be that on one occasion we may get 25 sixes in 120 rolls, another occasion we may get only
10sixes. However, we expect that if we repeat the experiment often enough, we would get an average of
20 sixes in 120 rolls.

Rolling a die
1. Each person is to take a die and roll it 120 times and record the
number of 6s rolled.
2. What is the most number of 6s rolled by anyone in 120 rolls of
thedie?
3. What is the least number of 6s rolled by anyone in 120 rolls of
thedie?
4. What is the average number of 6s rolled by the class in 120 rolls of the
die? How does this compare with the expected outcome of 20?
The expected outcome does not need to be a whole number. In many
cases this will not be so. Consider the example below.
WORKED EXAMPLE 13

Roger draws a card from a standard deck, notes the suit and replaces the card in the deck. If
Roger repeats this process 50 times, how many spades can Roger expect to have drawn?
THINK

WRITE

Calculate the probability of drawing a spade.

P(spade) = 14

Calculate the expected number of spades by


multiplying the probability by the number of trials.

Expected number of spades = 14 50



= 12.5

Obviously, after drawing 50 cards, Roger could not have drawn 12.5 spades. The number of spades
drawn must of course be a whole number. However, if this experiment were repeated a number of times,
we would expect to have drawn an average of 12.5 spades in every 50 cards.
The expected outcome method can be applied to any probability experiment. This includes multistage
events in which it may be necessary to draw a tree diagram or probability tree to calculate the probability
of a particular outcome.
268 Maths Quest HSC Mathematics General 2

WORKED EXAMPLE 14

A psychologist is conducting a study on the upbringing of boys. For the study, the psychologist
selects 100 couples with exactly three children. How many of these couples would the
psychologist expect to have three boys?
THINK
1

WRITE

Draw a tree diagram showing the sample space


for three children.

Boy
Boy
Girl
Boy
Girl
Girl

Boy
Girl
Boy
Girl
Boy
Girl
Boy
Girl

Calculate the probability of three boys.

P(three boys) = 18

Calculate the expected number by multiplying


theprobability of three boys by the number of
couples in the study.

Expected number of couples with three boys


=18100
= 12.5

Exercise 9D

Tutorial
int-2426
Worked
example 14

Expected outcomes

1 WE12 Calculate the number of times that a coin can be expected to land Tails in 40 tosses.
2 A die is rolled 300 times. Calculate the expected number of 6s to be rolled.
3 A card is drawn from a standard deck, its suit is noted and the card is replaced in the deck. Calculate

the expected number of hearts in 100 selections.


4 A barrel contains five red marbles, four blue marbles and a green marble. A marble is drawn from
the barrel. Its colour is noted, and it is then replaced in the barrel. In 70 selections from the barrel,
how many times would we expect to select:
a a red marble
b a blue marble
c a green marble?
5 WE13 Lorna spends a night at the greyhounds. There are 10 races, and in each race there are eight
greyhounds. Lorna bets on number 5 in every race. Calculate the number of winning greyhounds that
Lorna can expect to back.
6 A card is drawn from a standard deck; the card is then
noted and replaced in the deck. This is repeated
100times. Calculate the number of times (where
necessary, correct to 2 decimal places) that we could
expect to select:
a a club
b a red card
c an ace
d a court card (ace, king, queen or jack)
e the king of diamonds.

Digital doc
SkillSHEET 9.6
doc-11085
Single event
probability

Digital doc
SkillSHEET 9.7
doc-11086
Tree diagrams

Digital doc
SkillSHEET 9.8
doc-11087
Probability trees

7 Kevin buys a ticket in a meat raffle every week. There are 100 tickets and four prizes.
a Calculate the probability of Kevin winning a prize in the raffle.
b How many prizes can Kevin expect to win in one year?
8 Janice buys a ticket in every lottery. In each lottery there are 180000 tickets, a first prize and

3384cash prizes. One lottery is drawn every weekday for 52 weeks a year. Calculate the number of
times in 10 years that Janice can expect to win:
a first prize (as a decimal, correct to 3 significant figures)
b a cash prize (as a decimal, correct to 3 significant figures).
Chapter 9 Multi-stage events and applications of probability 269

9 MC A meeting is attended by 350 men and 150 women. At the meeting 100 people will be chosen

to make a speech. What is the expected number of women to make speeches?


A 15
B 30
C 50
D 150
10 MC A tennis club runs a raffle each week with 100 tickets. Fumiko buys one ticket each week. The

expected number of raffle wins over a period of 50 weeks is:


A 0.01
B 0.5
C 1

D 20

11 WE14 Four coins are tossed simultaneously in the air. If this were repeated 80 times, on how many

occasions would you expect the coins to land with four Heads?
12 The digits 2, 5, 6, 7 and 9 are written on cards and placed face down. Three are then chosen and

arranged to form a three-digit number. If this is repeated 150 times, what is the expected number of:
a odd numbers
b numbers greater than 600
c multiples of five?
13 Two dice are rolled 100 times. Copy and complete the table below to calculate the expected number

of occurrences of each total in 100 rolls of the dice. Give each answer correct to 1 decimal place.
Digital doc
EXCEL Spreadsheet
doc-2749
Die rolling

Outcome
Probability
Expected no.

10

11

12

14 A barrel contains 15 blue marbles and 5 red marbles. Two marbles are selected from the barrel, the

first not being replaced in the barrel before the second is chosen. This experiment is repeated 100
times. On how many occasions (correct to 2 decimal places) would you expect the two marbles
chosen to be:
a both blue
b both the same colour
c different colours
d selected with at least one being blue?

Further development
15 A die is biased as shown in the table below. What average outcome would you expect for a large

number of rolls of this die?


Number

Probability

1
2
3
4
5
6

0.2
0.2
0.2
0.2
0.1
0.1

16 Anthony is a basketball player and has a probability of 0.7 of shooting a basket from the free-throw

line. During a match Anthony goes to the free-throw line 10 times. On each occasion he receives two
free throws.
a What is the expected number of successful free throws that Anthony will make?
b What is the expected number of times that Anthony will be successful with both free throws?
17 Eldrick is a golfer and is playing the par three 17th hole. Eldrick has a 80% chance of hitting his first

shot onto the green and a 45% of sinking a putt from any point on the green.
a Find the probability that Eldrick is able to get the ball in the hole in two shots. (This score of one
under par is called a birdie.)
b If Eldrick plays the 17th hole four times during a tournament, what is the expected number of
birdies?
18 Every Friday night a local club runs a meat raffle. In the raffle there are 1000 tickets, and

Rhondabuys 5 tickets. There are 70 prizes in the raffle. Find the number of prizes that Rhonda can
expect to win:
a in one night
b over a year if she buys the same number of tickets each week.
270 Maths Quest HSC Mathematics General 2

19 A criminal gang has the technology to copy peoples ATM cards. They do not have the technology to

identify peoples PINs. If they try to use the card they can have three attempts at the PIN before the
bank will deactivate the card.
a What is the probability that the gang is able to guess the 4-digit PIN?
b How many cards does the gang need to copy before they can expect to be able to guess one PIN?
20 A telephone insurance salesperson has a 0.16 probability of being able to sell an insurance policy.
How many telephone calls does the salesperson need to make so they can expect to sell 60 insurance
policies?

Digital doc
Worksheet 9.2
doc-11088

Computer Application 1: Simulations


A simulation is where a computer gives results to an experiment that are similar to those that would
occur if the experiment were actually performed. For example, if a coin is tossed 100 times, a computer
can randomly choose Heads or Tails in a fraction of a second. In each case, the probability of each
outcome is 12 and we are saved the process of actually tossing the coin.
1. Access the spreadsheet Simulations from your Maths Quest General Mathematics HSC Course
eBookPLUS.

Digital doc
EXCEL Spreadsheet
doc-2750
Simulations

2. The first worksheet has a coin toss simulation. In cell B3 enter the number of times you wish to toss
the coin, in cell F4 enter the expected number of heads and in cell F5 enter the expected number
oftails.
3. How do the simulation results compare with the expected outcome? Complete 10 simulations
and average the results. Is this answer closer to the expected number of outcomes that you have
calculated?
4. Repeat this process for each one of the other simulations on rolling a die and rolling two dice.

Chapter 9 Multi-stage events and applications of probability 271

Summary
Counting techniques

The fundamental counting principle states that the number of ways a multiple selection can be
made is found by multiplying the numbers of ways each individual selection can be made.
The number of ways that n objects can be arranged in order is:
n! or n (n 1) (n 2) ... 2 1.
In an ordered selection, a number of objects are chosen and are arranged in order. The number of
ordered selections can be calculated by multiplying the number of first choices that can be made by
the number of second choices possible and so on until all choices have been included.
In an unordered selection, the order in which the objects have been chosen is not important. The
number of unordered selections that are possible is calculated by dividing the number of ordered
selections by the number of ways the ordered selection can be arranged.
Once the number of selections has been determined, the probability of particular selections can be
determined.

Tree diagrams

A tree diagram is used in any probability experiment where there is more than one stage to the
experiment.
The sample space can be determined from a tree diagram by following the paths to the end of each
branch.
The probability of an event can then be calculated by the probability formula:
P 1 event 2 =

number of favourable outcomes


total number of outcomes

Probability and
counting techniques

When we have calculated the number of arrangements and the number of ordered or unordered
selections that are possible, we can then calculate the probability of a certain selection using the
probability formula.

Expected outcomes

The expected number of times that an event will occur in a number of trials is calculated by
multiplying the number of trials by the probability of that event occurring.
The expected number of outcomes is the average number of times that the event is expected to
occur. It does not mean this is the number of times the event will occur.

272 Maths Quest HSC Mathematics General 2

Chapter review
1 Which of the following is an example of an ordered selection?
A
B
C
D

multip l e
c ho ic e

A team of four people is chosen from a group of 12.


Two representatives from a class of 30 students are elected to the SRC.
A class of 30 students elect a class captain and vice-captain.
From a barrel of 44 balls, six are chosen.

2 Six people are arranged in a line. The number of ways in which this can be done is:
A 6

B 12

C 120

D 720

3 In a race there are six runners. In how many ways can the first three places be filled?
A 6

B 12

C 120

D 620

4 A group of six people consists of Darren, Shintaro, Jim, Damien, John and Allan. From these six

people a group of three is chosen. The probability of choosing Darren, Jim and John is:
3
1
1
1
A
B
C
D
6
20
12
120
5 A bag contains 3 red marbles, 13 blue marbles and 4 yellow marbles. A marble is chosen from the

bag and then replaced in the bag. In 90 selections, the expected number of blue marbles selected is:
A 13
B 20
C 58.5
D 59
S ho rt
a nsw er

1 Two coins are tossed in the air.


a Draw a tree diagram.
b Use the tree to list the sample space for this experiment.
2 The digits 5, 7, 8 and 9 are used to form a two-digit number. Use a tree diagram to list the sample

space if:
a no digit can be used more than once

b digits can be repeated.

3 There are three births in the maternity ward of a hospital. Calculate the probability that the

babiesare:
a all boys

b two boys and a girl

c more girls than boys.

4 A two-digit number is formed using the digits 4, 6, 7, 8 and 9. No digit is allowed to be repeated.
a Use a tree diagram to list the sample space.
b Find the probability that the number formed is:
i 86
ii odd
iii greater than 65.
5 In a barrel there are three black marbles and three white marbles. A marble is drawn and its colour

noted, and it is then replaced in the barrel. A second marble is then drawn. Find the probability of
selecting:
a two marbles of the same colour
b at least one black marble.
Chapter 9 Multi-stage events and applications of probability 273

6 A rowing crew has eight rowers. In how many different ways can the crew be seated in the boat?
7 From the rowing crew of eight, a captain and vice-captain are to be selected. Calculate the number of

different ways the captain and vice-captain can be selected.


8 From the rowing crew of eight, four are to be chosen to crew a four-person boat. How many crews of

four can be chosen from the group of eight?

9 From the digits 1, 2, 3, 4 and 5:


a how many five-digit numbers can be formed if repetition is not allowed
b how many three-digit numbers can be formed if repetition is not allowed?
10 The letters D, S, T, U and Y are shuffled and placed in a line on a table. Calculate the probability that

the word STUDY is formed.


11 Thirty-six coins are tossed in the air. Calculate the expected number of coins landing Heads.
12 A die is rolled 60 times. Calculate the expected number of:
a 6s
b even numbers
c numbers less than 3.
13 A card is chosen from a standard deck, noted and replaced in the deck. In 100 trials, calculate (where

necessary, correct to 2 decimal places) the expected number of:


a red cards
b spades
c aces
d court cards
e black jacks.

274 Maths Quest HSC Mathematics General 2

14 Two dice are rolled. The score in each roll is the total of the two dice. In 90 rolls of the dice,

calculate the expected number of:


a two
b sevens
c tens
d doubles
e totals greater than 8.
15 In game, three coins are tossed in the air. In 100 tosses of the coins, on how many occasions would
you expect the coins to land:
a three Heads
b two Tails and one Head
c more Heads than Tails?
16 Two students from Richard, Sandra, Talia and Ingo have to make a speech. They draw straws to see
who will go first and second.
a How many different ways can the first and second speaker be arranged?
b What is the probability that Ingo speaks first and Talia speaks second?
17 Six teams A, B, C, D, E and F contest a basketball competition. The top four sides play in the semifinals, and later two will contest the grand final.
a In how many different ways can the top four sides be arranged?
b What is the probability that the top four teams finish D, C, F and A?
c How many pairs of teams is it possible to meet in the grand final?
d What is the probability of A playing B in the grand final?
e What is the probability that C plays in the grand final?
18 Zita is doing an exam when she realises that she has almost run out of time. She has not answered
the last 10 questions.
a If each question requires True or False as an answer and Zita guesses each answer, what is the
probability that she guesses all 10 correctly?
b If each question is multiple choice and requires the choice of (A), (B), (C) or (D), what is the
probability that Zita will guess all 10 correctly?
19 Mr And Mrs Morrisey have a son and are planning his education. In the local area there are
6pre-schools, 3 primary schools and 2 high schools. How many different combinations of schools
do the Morriseys have to choose from?
1 Three coins are tossed in the air.
a Draw a tree diagram to list the sample space.
b Use your tree diagram to calculate the probability of tossing two heads and one tail.
c Calculate the probability of tossing at least one head.

Ex tended
R es p ons e

2 A golf team is to be made by selecting two males from a group of 5 and two females from a

groupof4.
a How many teams is it possible to select?
b What is the probability that Issac is one of the male players and Tegan is one of the female

players? What is the probability that Issac and Tegan are both selected?
c If a new team is selected to play each weekend over a year on how many occasions can:
i Issac expect to play
ii Tegan expect to play?

Digital doc
Test Yourself
doc-11089
Chapter 9

Chapter 9 Multi-stage events and applications of probability 275

ICT activities
9A Counting techniques
INTERACTIVITY
int-0089: Random number generator. (page 257)
Tutorial
WE5 int-2424: Learn about ordered selections. (page 259)
Digital doc
SkillSHEET 9.1 (doc-11079): Fundamental counting principle.
(page 260)

9BTree diagrams
Digital docs
SkillSHEET 9.2 (doc-11080): Listing the sample space. (page 263)
SkillSHEET 9.3 (doc-11081): Informal description of chance.
(page263)
SkillSHEET 9.4 (doc-11082): Equally likely events. (page 263)
EXCEL Spreadsheet (doc-1364): Tree diagrams. (page 263)
WorkSHEET 9.1 (doc-11083): Apply your knowledge of multi-stage
events to problems. (page 264)

9CProbability and counting techniques


INTERACTIVITIES
int-2787: A pack of cards. (page 265)
int-0085: Random numbers. (page 265)

276 Maths Quest HSC Mathematics General 2

Tutorial
WE11 int-2425: Learn to calculate probabilities. (page 265)
Digital doc
SkillSHEET 9.5 (doc-11084): Single event probability. (page 266)

9DExpected outcomes
Tutorial
WE14 int-2426: Learn to calculate expected outcomes. (page 269)
Digital docs
SkillSHEET 9.6 (doc-11085): Single event probability. (page 269)
SkillSHEET 9.7 (doc-11086): Tree diagrams. (page 269)
SkillSHEET 9.8 (doc-11087): Probability trees. (page 269)
EXCEL Spreadsheet (doc-2749): Die rolling. (page 270)
WorkSHEET 9.2 (doc-11088): Apply your knowledge of multi-stage
events to problems. (page 271)
EXCEL Spreadsheet (doc-2750): Simulations. (page 271)

Chapter review
Digital doc
Test Yourself (doc-11089): Take the end-of-chapter test to test your
progress. (page 275)

To access eBookPLUS activities, log on to www.jacplus.com.au

Answers chapter 9
Multi-stage events
and applications of
probability
Exercise 9A

there is only one place left and only one


way of filling it; hence, the number of
arrangements is the same.
23 59400
24 a 8145060
b 11480
c 820
25 When selecting three people from
ten people, seven people remain. For
every group of three selected there is a
corresponding group of seven not selected.
Hence the number of groups of three is the
same as the number of groups of seven.
1

1st
child

3rd
child

Boy

Boy
Girl

Girl

Boy
Girl

Boy

Boy
Girl

Girl

Boy
Girl

Boy

Girl

1st die
1

R
B
Y
G
R
B
Y
G
R
B
Y
G
R
B
Y
G

Yellow

Green

Male

Female
Thuy
Petria
Joan
Wendy
Amelia
Thuy
Petria
Joan
Wendy
Amelia
Thuy
Petria
Joan
Wendy
Amelia
Thuy
Petria
Joan
Wendy
Amelia

George

Frank

Stanisa

Ian

S = {George Thuy, George Petria,


George Joan, George Wendy,
George Amelia, Frank Thuy,
Frank Petria, Frank Joan, Frank
Wendy, Frank Amelia, Stanisa
Thuy, Stanisa Petria, Stanisa Joan,
Stanisa Wendy, Stanisa Amelia,
Ian Thuy, Ian Petria, Ian Joan,
Ian Wendy, Ian Amelia}
5

1st digit

2nd digit
2
4
5
7
1
4
5
7
1
2
5
7
1
2
4
7
1
2
4
5

4th
child

Boy
Girl
Boy
Girl
Boy
Girl
Boy
Girl
Boy
Girl
Boy
Girl
Boy
Girl
Boy
Girl

2nd die
1
2
3
4
5
6
1
2
3
4
5
6
1
2
3
4
5
6
1
2
3
4
5
6
1
2
3
4
5
6
1
2
3
4
5
6

2nd bag

Blue

Tree diagrams

2nd
child

1st bag
Red

Counting techniques

1 360
2 15625
3 a 1440
b 120
c 172800
4 a 6
b 120
c 362880
5 24
6 120
7 6
8 720
9 12
10 12144
11 35
12 210
13 15
14 72
15 C
16 D
17 B
18 a 20
b 6
19 a 362880
b 504
c 36
20 120
21 3024
22 After the first three places have been filled

Exercise 9B

4
5
7

President Secretary Treasurer

Belinda

Dean
Kate
Adrian

Dean

Belinda
Kate
Adrian

Kate

Belinda
Dean
Adrian

Adrian

Belinda
Dean
Kate

Kate
Adrian
Dean
Adrian
Dean
Kate
Kate
Adrian
Belinda
Adrian
Belinda
Kate
Dean
Adrian
Belinda
Adrian
Belinda
Dean
Dean
Kate
Belinda
Kate
Belinda
Dean

7 S = {357, 358, 375, 378, 385, 387, 537,

538, 573, 578, 583, 587, 735, 738,


753, 758, 783, 785, 835, 837, 853,
857, 873, 875}

8 C
1
2

1st digit

10 a

2
3
4
7

2nd digit
3
4
7
2
4
7
2
3
7
2
3
4

1

20
2
c
5
1
12
4

7
12

3
10
3
d
5

11 a

13 B
14 a S = {Bris Peak EC, Bris Peak

BC, Bris Peak FC, Bris Offpeak EC, Bris Off-peak BC,
Bris O
ff-peak FC, GC
Peak EC, GC Peak BC, GC
Peak FC, GC Off-peak EC,
GC Off-peak BC, GC
Off-peak FC, Cairns Peak EC,
Cairns Peak BC, Cairns Offpeak EC, Cairns Off-peak BC}
3
1
1
b i
ii
iii
4
8
8
15 a 6
b 27
c Kevin is correct. In each case the
probability is 13 as there is one chance
inthree that the 8 is in the final
position.
5
16 a P(more boys) = , P(equal numbers)
16
6
5
=16
, P(more girls) = 16
. Each outcome

isnot equally likely.


b Dan is incorrect. If there are two

children, the probability of an equal


number of boys and girls is 12.
17 Each number is not equally likely
to occur.
18 Tanya is incorrect as the number of
choicesthat she can make is reduced
from96 to 12.
1023
19 P(at least one head) =
99.9%
1024
No.Ingrid is wrong.

Exercise 9C

techniques
1
24

Probability and counting

2 a 120
1
6

3 a
1
10

1

120
2
b
3

1
120
2
c
3

1
42
2
b
9

1

42
1
6 a
72
1
7
6
1
8 a

13 800

5 a

1
21
2
c
9

1
2300
1
9 a 20 b
20
1
3
3
10 a
b
c
10
10
5
1
11 a
8 145 060
7
28
5005
b i
ii 8 145

iii
8 145 060
060
8 145 060

Chapter 9 Multi-stage events and applications of probability 277

12 Lotto is harder to win as Lotto Strike

player has a probability of 3 5751 880 of


wining.
13 Bruce is correct as 45C7 = 45379620,
which is approximately 5.57 times greater
than lotto. That is, Oz Lotto is 457%
harder to win.
14 a 3838380
b 20
c

1
76 767 600

1 20
2 50
3 25
4 a 35
5 1.25
6 a 25
d 30.77
1
7 a

b 10000

6 40320
7 56
8 70
9 a 120
1
10

20 375

b 28

1st coin

b 50
e 1.92

c 7.69

2nd coin
Heads
Tails

b 30
b 25
e 3.85
b 15
e 25
b 37.5

16 a 12

Tails

Tails

1 a

1st coin

2nd coin

Heads

95, 97, 98}

87, 88, 89, 95, 97, 98, 99}


3
1
b
c
8
2
4 a S = {46, 47, 48, 49, 64, 67, 68, 69, 74, 76,
78, 79, 84, 86, 87, 89, 94, 96, 97, 98}

c 30

10

11

12

Probability

1
36

1
18

1
12

1
9

5
36

1
6

5
36

1
9

1
12

1
18

1
36

2.8

5.6

8.3

8.3

5.6

2.8

11.1 13.9 16.7 13.9 11.1

278 Maths Quest HSC Mathematics General 2

Heads
Tails

Tails

3 a
b 90

c 7.5
c 50

c 15

Extended response

b S = {
 55, 57, 58, 59, 75, 77, 78, 79, 85,
1
8

c 20
c 7.69

1
12
1
b
360
1
e
3
1
b
1048576

19 36

Heads

Outcome

Expected no.

11 18
12 a 10
13 a 50
d 30.8
14 a 2.5
d 15
15 a 12.5

1
d
15
1
18 a

1024

b S = {HH, HT, TH, TT}


2 a S = {57, 58, 59, 75, 78, 79, 85, 87, 89,
b 48.9

3
4

b 60

17 a 360

Heads

c 7

iii

120

Short answer

1 a

2
5
3
b
4

ii

1
2

b 4.9
b 1.44
b 18.2

1

10 000

1

20

5 a

Multiple choice

25

13

19 a

b i

1 C
2 D
3 C
4 B
5 C

Expected outcomes

b 2.08
8 a 0.0144
9 B
10 B
11 5
12 a 90

b 60.53
d 94.74

Chapter Review

15 a 1.65 1012
b 184756
c 1.12 107
16 a 0.06
b 0.014
c 6.45 104
Exercise 9D

14 a 55.26
c 39.47
15 3.1
16 a 14
17 a 0.36
18 a 0.35

Heads
Tails

2 a 16
1 1 4
b , ,
10

6 15

c i 5.2 times
ii 0.6666 times

3rd coin
Heads
Tails
Heads
Tails
Heads
Tails
Heads
Tails

3
8

7
8

Chapter 10

Further algebraic skills


and techniques
CHAPTER CONTENTS
10A Addition and subtraction of algebraic expressions
10B The index laws
10C Expanding and simplifying
10D Solving equations
10E Equations and formulas

Addition and subtraction


ofalgebraic expressions

10A

An algebraic expression is a mathematical statement made up of terms, operation symbols and


sometimes brackets. For example, 2c + 4p is an algebraic expression made up of two terms.
Like terms are terms that contain the same pronumerals, and can be collected (added or subtracted) in
order to simplify the expression.
For example 6a and a are like terms, but 6a and 6 are not like terms because one is an algebraic term
and the other is a constant (or number).
Similarly, 4x and 8x are like terms but 4x and 4x2 are not like terms.
Only like terms can be added or subtracted.
WORKED EXAMPLE 1

Simplify the following.


a 4j 5c + c + 3jb d2 + 9d 12d 15
THINK

a 1 Write the expression.

WRITE

a 4j 5c + c + 3j

Identify the like terms and group them together.

= 4j + 3j 5c + c

Simplify by collecting like terms.

= 7j 4c

b 1 Write the expression.


2

Simplify by collecting like terms.

b d2 + 9d 12d 15

= d2 3d 15

Adding and subtracting algebraic fractions


The methods for working with algebraic fractions are the same as those used for numerical fractions.
To add or subtract algebraic fractions we perform the following steps.
1. Find the lowest common denominator (LCD) by finding the lowest common multiple (LCM) of the
denominators.
2. Rewrite each fraction as an equivalent fraction with this common denominator.
3. Express as a single fraction.
4. Simplify the numerator.
Chapter 10 Further algebraic skills and techniques 279

WORKED EXAMPLE 2

Simplify the following expressions.


2x x
x+1 x+4
a
b
+
5
3
2
3
THINK

WRITE

a 1 Write the expression.

Rewrite each fraction as an equivalent fraction using


the LCD. The LCM of 3 and 2 is 6.

Express as a single fraction.

Simplify the numerator.

2x x

3
2

2 1 2x 2 3 1 x 2

6
6

4x 3x
6
x
=
6
x+1 x+4
+
b
3
5

b 1 Write the expression.

Rewrite each fraction as an equivalent fraction using


the LCD. The LCM of 3 and 5 is 15.

Express as a single fraction.

Simplify the numerator by expanding grouping


symbols and collecting like terms.

31x + 12 51x + 42
+
15
15

31x + 12 + 51x + 42
15

3x + 3 + 5x + 20
15
8x + 23
=
15

Adding and subtracting algebraic


expressions

Exercise 10A

1 Simplify the following.


a 2k + 3k + 2c + 9c
c 4d + 3c + 5d + 2c
e 7g + 2j + 5g + 11j
g 9n + 4 + 3n + 7
i 4nv + 8u + 7nv + 2u

b
d
f
h

2 WE1 Simplify the following.


a 10m 7m + 5c 3c
d 7t 11t + 7 5
g 6p 9 2p 5
j 2j + 4c 3j c
m 5y2 + 3y + 2y2 7y

b
e
h
k
n

6m + 9m + 10f + 6f
2f + 3h + 2f + 5h
12d + 8 + 3d + 5
5y + 6h2 + h2 + 9y

14a 6a + 11f 8f
10r + 15 11r 10
10w 6w + 14 10
2k + 14m 5k 10m
4x3 2x3 + 3x4 9x4

3 MC
a 2q + 6p 5q p simplifies to:

c
f
i
l
o

5k + 11 3k 9
6v 8v + 12 17
4c 8 + 3c 9
d + 5c 2c 3d
c2 2c2 + 5 + 7

A 7q + 5p

B 7q 5p

C 3q + 5p

D 3q 5p

A 9r 9y

B r 9y

C r + 9y

D 9r + 9y

A 2j2 2j

B 2j4 2j

C 3j2 2j

D 2j6

b 4r 6y 3y + 5r simplifies to:
c j2 6j 3j2 + 4j simplifies to:
4 Simplify the following.
a x2 + 5x + 2x + 6
c v2 2v 8v 6
e u2 + 4u 3u 12
280 Maths Quest HSC Mathematics General 2

b d2 3d + 7d 10
d a2 4ab + 2ab + b2
f 5n4 12n2 + 6n2 25

5 WE2 Simplify the following expressions.

2y y

3
4
2w
w
e

14 28
x+1 x+3
i
+

2
5
a

y y

8 5
y
y
f

20 4
x+2 x+6
j
+

3
4
b

4x x

3
4
12y y
g
+
7
5
2x 1 2x + 1
k


6
5
c

8x 2x
+
9
3
10x 2x
h
+
5
15
3x + 1 5x + 3
l
+
3
2
d

Further development
6 Given that x = 8a 7b and y = 7b 8a what is the value of
a x + y
b x y
c y x

d 3x 4y?

7 Find the sum and difference of (5x y + 2z) and (y 4z x).


8 Find the:
a sum of x and its reciprocal
b the difference between x and its reciprocal.
9 Simplify

10B

3x 5y
+ .
5y 3x

The index laws

When a number or pronumeral is repeatedly multiplied by itself, it can be written in a shorter form
called the index form with a base and power or index. For example, 3 3 3 3 = 34.
Patterns in powers when multiplying and dividing pronumerals can be summarised using index laws.
When multiplying algebraic terms with the same bases the indices should be added.
First Index law: am an = am + n

Interactivity
int-2360
Indices

WORKED EXAMPLE 3

Simplify 5e10 2e3.


THINK

WRITE

Write the expression.

5e10 2e3

The order is not important when multiplying, so place the numbers first.

= 5 2 e10 e3

Multiply the coefficients and add the indices.

= 10e13

When there is more than one pronumeral involved in the question, we apply the First index law to each
pronumeral separately.
WORKED EXAMPLE 4

Simplify 7m3 3n5 2m8 n4.


THINK

WRITE

Write the problem.

7m3 3n5 2m8 n4

The order is not important when multiplying, so place


numbers first and group the same pronumerals together.

= 7 3 2 m3 m8 n5 n4

Simplify by multiplying the coefficients and using the First


IndexLaw for bases that are the same (add indices).

= 42m11n9

Since division is the inverse of multiplication, we subtract indices when dividing bases that are thesame.
Second Index Law: am an = am n

Chapter 10 Further algebraic skills and techniques 281

WORKED EXAMPLE 5

Simplify 712 74.


THINK

WRITE

Write the problem and express it as a fraction.

Simplify by applying the Second Index Law.

712
74
= 78

As with multiplication of algebraic expressions, when dividing we divide the coefficients normally
before applying the Second Index Law to each pronumeral separately.
WORKED EXAMPLE 6

Simplify 36d7 12d4 giving your answer index form.


THINK

WRITE

Write the problem.

36d7 12d4

Divide the numbers and apply the Second Index Law


to the pronumerals.

= 3d3

Raising a power to another power


If we are given (72)3, this can be written in factor form as 72 72 72.
We can then simplify using the First Index Law as 72 + 2 + 2 = 76.
Similarly, if we are given (45)3, this means 45 45 45 = 415.
You will notice that (72)3 = 72 3
= 76
5
And(4 )3 = 45 3
= 415.
When raising a power to another power, multiply the indices.
Third Index Law: (am)n = am n
WORKED EXAMPLE 7

Simplify the following.


a(74)8b(3a2b5)3
THINK

a 1 Write the problem.


2

Simplify by applying the Third Index Law.

b 1 Write the problem.


2

Simplify by applying the Third Index Law. For


each term inside the grouping symbols Simplify
thecoefficient.

WRITE

a (74)8

= 732
b (3a2b5)3

= 33a6b15
= 27a6b15

Zero index
Any base that has a power of zero has a value of one. It does not matter whether the base is a number
or a pronumeral. If the power is zero; its value is one. We can show this by looking at the following
example which can be simplified using two different methods.
282 Maths Quest HSC Mathematics General 2

Method 1
23 23 =
=

Method 2

222
222

23 23 =

8
8

23
23

= 20 (using the Second Index Law)

=1

=1

Since the two results should be the same, 20 must equal 1.


Any base that has an index (power) of zero is equal to 1.
Fourth Index Law: a0 = 1
WORKED EXAMPLE 8

Find the value of 260.


THINK

WRITE

Write the problem.

260

Any base with an index of zero is equal to one.

=1

WORKED EXAMPLE 9

Find the value of (12a9b7c2)0.


THINK

WRITE

Write the problem.

(12a9b7c2)0

Any base with an index of zero is equal to one.

=1

WORKED EXAMPLE 10

Find the value of 12b2c0.


THINK

WRITE

Write the problem.

12b2c0

Only c has a power of zero, so replace it with a 1 and simplify.

= 12b2

Exercise 10B

The index laws

1 WE3 Simplify each of the following.


a 5p7 5p4
b 5x2 3x6
7
d 3p 6p
e 10t3 t2 7t
2 WE4 Simplify each of the following.
a 2a2 3a4 e3 e6
b 4p3 2h7 h5 p4
2
2
5
d 2gh 3g h
e 5p4q2 6p2q6

c 9y6 7y4
f 4q2 q5 5q8
c 2m3 5m2 8m3

3 WE5 Simplify each of the following using the Second Index Law.
a 53 52
b 79 72
c s8 s4
6
45
42
d z z
e i i
f 675 674
g 1023 1019
j

h78

h

c13

c9
b77
k

b7
h

i
l

t456
t423
f 1000
f 100

Chapter 10 Further algebraic skills and techniques 283

4 WE6 Simplify each of the following, giving your answer in index form.
a 4x5 x3
b 6y7 y5
c 9w12 w5
e 20f 12 2f 3

48g8

f 100h100 10h10

12b7

8b2
5 Simplify each of the following.
i

6g5

15p12

g 40j15 20j5
k

3p8

45p14

9p4
100n95
l
70n5

81m6

18m2

18r6

3r2
60b7
100r10
d

e

20b
10r6
6 WE7 Simplify each of the following.
a (x2)3
b (f 8)10
12
12
d (r )
e (a2b3)5
g (g3h2)5
h (3w9q2)4
7 Simplify each of the following.
a (p5)2 (q3)2
b (r5)3 (w3)3
d (j6)3 (k4)3
e (q2)2 (r4)5
4
4
7
3
g (f ) (g )
h (t5)2 (u4)2
8 WE8 Find the value for each of the following.
a 30
b 50
c p0
9 WE9 Find the value of each of the following.
a (3p)0
b 50t0
c (2w2)0
10 Find the value of each of the following.
a 3p0
b 5x0 + 7
c (a0)6 + 3
11 WE10 Find the value of each of the following.
a 4r3g0
b 2p0 + 3g0
a

d 12q34 4q30

45a5
9a2
9q2
f
q
c

c (s25)4
f (pq3)5
i (5e5r2q4)2
c (x5)2 (n3)6
f (h3)8 (j2)8
i (i3)5 (j2)6
d d0
d (5x8)0
d (2y0)3 + 9
c 6t0 + 7f0

Further development
12 Simplify each of the following.
a 2a2 3a4 e3 e4
d 2gh 3g2h5
g 9y8d y5d3 3y4d7
j 10h10v2 2h8v6 3h20v12

b 4p3 2h7 h5 p3
e 5p4q2 6p2q7
h 7b3c2 2b6c4 3b5c3

c 2m3 5m2 8m4


f 8u3w 3uw2 2u5w4
i 4r2s2 3r6s12 2r8s4

13 Simplify each of the following.


a
d

8p6 3p4
16p5
27x9y3

12xy2
8p3 7r2 2s

g
6p 14r

12b5 4b2

18b2

16h7k4

12h6k

27a9 18b5 4c2



18a4 12b2 2c

25m2 4n7
15m2 8n
12j8 6f 5
8j3 3f 2
81f 15 25g12 16h24
27f 9 15g10 12h30

14 Simplify each of the following.

3b4 2
5h10 2
2k5 3
b a
c a
b
b
b
d3
2j2
3t8
7p9 2
5y7 3
4a3 4
d a
e a
f a
b
b
b
22
13
8q
3z
7c5
15 Simplify each of the following giving your answer in fractional form.
a x5 x8
b b6 b10
c z z7
d q2 q9
20pq
5m2
e m0 m4
f 12m3 4m5
g

h
2
4p
30m3
a a

284 Maths Quest HSC Mathematics General 2

16 Use the index laws to simplify each of the following. Give each of your answers in fractional form.
a a3 a4
b 12p2 3p3
c 7g5h2 3gh1
d 4p 5p2
e s2 s3

10C

f 42p2q3 6p2q

g 6r2 2r4

h 45a2b3 c 3abc

Expanding and simplifying

Brackets are used to group terms and are sometimes referred to as grouping symbols. When there are
multiples of a group of terms, for example two groups of (x + 4y), this is written as 2(x + 4y).
Expanding the algebraic expression is achieved by multiplying the term outside the brackets by each
of the terms inside.
For example, 2(x + 4y) = 2 x + 2 4y
= 2x + 8y

Interactivity
int-0075
Expanding
brackets

Alternatively, we could have written:


2(x + 4y) = 2(x) + 2(4y)
= 2x + 8y
When more than one set of brackets appears in an expression, we can often simplify by collecting any
like terms that result from expanding the brackets.
For example: 2(x + 5y) + 3(2x y)
= 2(x) + 2(5y) + 3(2x) + 3(y)
= 2x + 10y + 6x 3y
= 8x + 7y
WORKED EXAMPLE 11

Expand:
a 7(m 4)b 6(a 3).
THINK

WRITE

a 7(m 4)

a 1 Write the expression.


2

Multiply each term inside the brackets by the term outside.

b 6(a 3)

b 1 Write the expression.


2

= 7(m) + 7(4)
= 7m 28

Multiply each term inside the brackets by the term outside.

= 6(a) 6(3)
= 6a + 18

WORKED EXAMPLE 12

Expand and simplify 6(m 4r) 2(2m + 7r).


THINK

WRITE

Write the expression.

6(m 4r) 2(2m + 7r)

Multiply each term inside the brackets by the term outside.

= 6m 24r 4m 14r

Simplify by collecting like terms.

= 2m 38r

Exercise 10C

Expanding and simplifying

1 WE11 Expand the following.


a 5(k + 1)
d 8(d 9)
g 4(5m 2)
j 5(m + n)
m b(c d)

b
e
h
k
n

7(m + 4)
12(h 5)
5(6t + 5)
8(4y 3f )
k(i + ef )

c
f
i
l
o

4(y + 7)
2(k 6)
8(2k 11)
6(3v + 7w)
6p(2j 3m)

Chapter 10 Further algebraic skills and techniques 285

2 Expand the following.


a 3(c + 1)
d 8(c + d)
g 10(5 y)

b 5(d + 2)
e 4(3k 2m)
h k(k + 2)

3 WE12 Expand and simplify.


a 2(c + 4) + 3(c + 5)
d 2(j 5) 3(j + 2)
g 10(c + 4) + 6(c 9)
j 4(2h + 7) 10(h 5)
m 10(h 1) (3h 5)

b
e
h
k
n

4 Expand and simplify.


a y(y 6) + 2(y 6)
d 3h(h + 1) + 5(h + 1)

b w(w + 2) 6(w + 2)
e f (3f + 2) 8(3f + 2)

c 6(m + 11)
f 7(2 3x)
i x(x 3)

12(k + 1) + 5(k + 6)
7(t + 2) 5(t + 5)
6(d 3) 5(d 2)
2(3y 8) + (4y 3)
c + 3 + 5(c + 7)

5 MC
a y(3 2y) simplifies to:

c
f
i
l
o

8(m + 2) + 5(m + 5)
9(m + 7) + 4(m 6)
12(w 4) 8(w + 8)
5(2x + 3) (x 12)
2(m 9) (3m 4)

c x(2x 5) 4(2x 5)
f 2a(4a 3) + 3(4a 3)

A 3 2y2

B 6 y2

C 3y 6y2

D 3y 2y2

A 5k 15

B 5k + 15

C 5k + 3

D 5k 8

A 2b 16

B 2b + 8

C 2b 1

D 2b 8

b 5(k 3) simplifies to:

c 4(b 3) 2(b 2) simplifies to:

Further development
6 Expand and simplify the following expressions.
a 2(x + 2y) + 3(2x y)
c 7(2a + 3b) + 4(a + 2b)
e 6(a 2b) 5(2a 3b)
g 4(2p 4q) 3(p 2q)
i 7(2x 3y) (x 2y)

b
d
f
h
j

7 Expand and simplify the following expressions.


a 4(m + 2n) + 3(2m n)
c 2(3x + 2y) + 3(5x + 3y)
e 5(p 2q) (2p q)

b 3(2x + y) + 4(3x 2y)


d 5(4p + 2q) + 2(3p + q)
f 3(a 2b) (2a + 3b)

8 Expand and simplify the following expressions.


a a(b + 2) + b(a 3)
c c(d 2) + c(d + 5)
e 3c(d 2) + c(2d 5)
g 2m(n + 3) m(2n + 1)
i 3m(2m + 4) 2(3m + 5)
k 6m(2m 3) (2m + 4)
m 7x(5 x) + 6(x 1)
Digital doc
Worksheet 10.1
doc-11090

x(y + 4) + y(x 2)
p(q 5) + p(q + 3)
7a(b 3) b(2a + 3)
4c(d 5) + 2c(d 8)
5c(2d 1) (3c + cd)
2p(p 4) + 3(5p 2)

9 a3a(5a + b) + 2b(b 3a)


b 4c(2c 6d) + d(3d 2c)
c 2y(5y 1) 4(2y + 3)

10D
Interactivities
int-2778
Solving equations
int-2373
Solving equations
int-2764
Solving equations

b
d
f
h
j
l

4(2p + 3q) + 2(p 2q)


5(3c + 4d) + 2(2c + d)
5(2x y) 2(3x 2y)
2(c 3d) 5(2c 3d)
4(3c + d) (4c + 3d)

Solving equations

Equations are mathematical statements that show two equal expressions. This means that the left-hand
side and the right-hand side of the equation are equal.
Linear equations contain pronumerals whose highest power is 1. For example, 2x + 1 = 5.
Linear equations can be solved using inverse operations.
+ and are inverse operations
and are inverse operations
When solving equations, the last operation performed on the pronumeral when building the equation
is the first operation undone by applying the inverse operation.

286 Maths Quest HSC Mathematics General 2

WORKED EXAMPLE 13

Solve the following.


a 5y 6 = 79b

4x
=5
9

THINK

WRITE

a 5y 6 = 79

a 1 Write the equation.

5y = 85

Apply the inverse operation by adding 6 to both sides.

Apply the inverse operation by dividing both sides by 5 to obtain y.

b 1 Write the equation.

y = 17
b

4x
=5
9
4x = 45

Multiply both sides by 9.

Divide both sides by 4 to obtain x.

x = 1114

Many equations have the pronumeral on both sides. In such cases they are solved by transposing all
pronumerals to the left-hand side of the equation, as shown in the examples below.
Interactivity
int-0077
Keeping equations
balanced

WORKED EXAMPLE 14

Solve the following equations.


a 5h + 13 = 2h 2
b 14 4d = 27 d
THINK

WRITE

a 5h + 13 = 2h 2

a 1 Write the equation.


2

Subtract 2h from both sides.

Subtract 13 from both sides.

Divide both sides by 3.

3h + 13 = 2
3h = 15
h = 5
b 14 4d = 27 d

b 1 Write the equation.


2

Add d to both sides.

Subtract 14 from both sides.

Divide both sides by 3.

Exercise 10D

14 3d = 27
3d = 13
d = 413

Solving equations

1 WE13 Solve the following equations.


a a + 61 = 85
b k 75 = 46
d r 2.3 = 0.7

e h + 0.84 = 1.1

g t 12 = 7

1 1
h q + =
3 2

c g + 9.3 = 12.2
f i + 5 = 3

i x 2 = 2

2 Solve the following equations.


a

f
= 3
4

i
= 6
10

d 9v = 63

e 6w = 32

g 4a = 1.7

m 7
=
19 8

c 6z = 42

k
5
=
12 6
y
3
i
=5
8
4
f

Chapter 10 Further algebraic skills and techniques 287

3 Solve the following equations.


a !t = 10
d f 2 = 1.44

15

22
4 Solve the following.
a 5a + 6 = 26
d 7f 18 = 45
g 6s + 46 = 75
g !g =

b y2 = 289

c !q = 2.5

4
7
196
h j2 =

961
e !h =

f p2 =

9
64
7

i a2 = 2 9

b 6b + 8 = 44
e 8q + 17 = 26
h 5t 28 = 21

c 8i 9 = 15
f 10r 21 = 33
i 8a + 88 = 28

5 Solve the following.

2 + 6 = 16
4
m
d
12 = 10
9
6 Solve the following.
a 6(x + 8) = 56
d 3(2k + 5) = 24
a

g
+ 4 = 9
6
n
e + 5 = 8.5
8

r
+6=5
10
p
f
1.8 = 3.4
12

b 7(y 4) = 35
e 5(3n 1) = 80

c 5(m 3) = 7
f 6(2c + 7) = 58

7 Solve the following.

3k
= 15
5
8u
d
= 3
11
a

7p
= 8
10
4v
f
= 0.8
15

9m
= 18
8
11x
e
= 2
4
b

8 MC

p
+ 2 = 7 is:
5
A p = 5
B p = 25
b If 5h + 8 = 53, then h is equal to:
1
A
B 12.2
5
c The exact solution to the equation 14x = 75 is:
A x = 5.357142857
5
C x = 5
14
9 Solve the following equations.
a x = 5
b 2 d = 3
d 7 x = 4
e 5h = 10
v
r
1
=
g = 4
h
12 4
5
10 Solve the following equations.
a 6 2x = 8
b 10 3v = 7
a The solution to the equation

d 3 2g = 1

C p = 45

D p = 1

C 225

D 9

B x = 5.357 (to 3 decimal places)


D x = 5.4

e 5 4t = 17

8j
k
= 9
h 3 = 6
3
4
11 WE14 Solve the following equations.
a 6x + 5 = 5x + 7
b 7b + 9 = 6b + 14
d 8f 2 = 7f + 5
e 10t 11 = 5t + 4
g 12g 19 = 3g 31
h 7h + 5 = 2h 6
g

12 Solve the following equations.


a 5 2x = 6 x
d k 5 = 2k 6
g 14 5w = w + 8
288 Maths Quest HSC Mathematics General 2

b 10 3c = 8 2c
e 5y + 8 = 13y + 17
h 4m + 7 = 8 m

c 5 p = 2
f 6t = 30
i 4g = 3.2
c 9 6l = 3

3e
= 14
5
4f
i + 1 = 8
7
f

c 11w + 17 = 6w + 27
f 12r 16 = 3r + 5
i 5a 2 = 3a 2
c 3r + 13 = 9r 3
f 17 3g = 3 g
i 14 5p = 9 2p

13 Solve the following equations.


a 3(x + 5) = 2x
d 10(u + 1) = 3(u 3)
g 5(2d + 9) = 3(3d + 13)

b 8(y + 3) = 3y
e 12(f 10) = 4(f 5)
h 5(h 3) = 3(2h 1)

14 MC
a The solution to 8 4k = 2 is:

1
2

A k = 2

b The solution to

1
2

B k = 2

6n
+ 3 = 7 is:
5

1
1
B n = 3
3
3
c The solution to p 6 = 8 4p is:
2
4
A p =
B p = 2
5
5
A n = 3

c 6(t 5) = 4(t + 3)
f 2(4r + 3) = 3(2r + 7)
i 2(4x + 1) = 5(3 x)

1
2

C k = 1

D k = 1

1
3

C n = 8

D n = 8

2
3

C p = 4

D p =

1
2

1
3

2
3

Further development
15 Solve each of the following linear equations.
a 6(4x 3) + 7(x + 1) = 9
c 8(5 3x) 4(2 + 3x) = 3
e 6(4 + 3x) = 7(x 1) + 1
g 8(x + 4) + 2(x 3) = 6(x + 1)
i 7.2(3x 1) + 2.3(5 x) = 34.3
k 9(2x 5) + 5(6x + 1) = 100

b
d
f
h
j
l

9(3 2x) + 2(5x + 1) = 0


9(1 + x) 8(x + 2) = 2x
10(4x + 2) = 3(8 x) + 6
6(2x 3) 2(6 3x) = 7(2x 1)
5(2x 1) 3(6x + 1) = 8
7(2x + 7) 5(2x + 1) = 2(4 x)

16 Solve each of the following linear equations.

x 4x 1
+
=
3
3
5
3x x 1
d
+ =
8 4
5
2 x 3x
g =
7 8
8
1 4 5
j
+ =
3 x x
a

x x 3
=
4 5 4
2x x 3
e
=

3
4
6
4 1 2
h =
x 6 x
b

2x 4
x
+6=
2
5

17 Solve each of the following linear equations.

3(x + 1) 5(x + 1)
2(x + 1) 3(2x 5)
+
= 4
b
+
= 0
7
3
8
2
8(x + 3) 3(x + 2)
5(7 x) 2(2x 1)
d
=

e
=
+ 1
7
4
2
5
5(x 2) 6(2x 1) 1
9(2x 1) 4(x 5)
g

= h
=

7
3
3
3
5
3
5
5
3
1
1
j
+
=

k
=

x
x+1 x4 x+1
x1
x1
a

18 Solve each of the following linear equations.

x+2 x5
=

3
6
2x 4 11 x
d
=

2
5
a

x + 11 2(x + 14)
=

3
9
4x + 66 13 3x
e
=

3
4
b

19 Solve each of the following linear equations.

6x + 7 5x + 1
=
+ 1
4
5
9x + 28 7x 6
d
=
5
2
5
a

2x + 29 x + 44
=
+ 2
3
8
7x 9 21 x
e
=
18
9
3
b

x 4x

=2
7
4
5x
2x
f
8=
8
3
15
2
i
4=
x
x
c

4x 1 2x + 5

=0
3
2

2(4x + 3) 6(x 2) 1

=
2
2
5
2(6 x) 9(x + 5) 1
f
=
+
3
3
6
3
8
1
i
+
=
x1 x+1 x+1
5 1
4
l
=
x
2x 1 x
c

x 1 3x 17
=
8
12
2(7
3x)
x + 10
f
=
9
5
c

7x 9 5x + 3
=
+6
2
4
17 x 49 + 2x
f
=
+5
2
5
c

Chapter 10 Further algebraic skills and techniques 289

10E

Equations and formulas

The purpose of solving an equation is to find the value of an unknown value that makes that statement
true. In solving an equation every process that has been performed on the pronumeral is reversed until
itbecomes the subject of the equation. In many cases, an equation arises as the result of substitution
intoa formula.
WORKED EXAMPLE 15

In the formula C = 2r, find the value of r when C = 100, correct to 2 significant figures.
THINK

WRITE

Method 1: Technology-free
1

Write the formula.

Substitute the value of C.

Divide each side by 2 and round the answer off to


2 significant figures.

C = 2r
100 = 2r
100
r=
2
= 16

Method 2: Technology-enabled
1

From the MENU select EQUA.

Press 3 3 (Solver).

Delete any existing equation, and enter the equation that


arises after the substitution is made. To enter 100 = 2r
press 1 a a ! [=] 2 ! [] a [R].
Note: At this stage you may have a different value of R,
but this is to be ignored.

Press 6 to solve the equation.

Some equations involve powers and roots. In the solution to an equation, remember that the opposite
function to taking a square is to take the square root and vice versa. When solving such an equation, both
the positive and negative square roots are possible solutions.
For example, the equation x2 = 9 has the solution x = 3. This differs from !9, which equals 3.

WORKED EXAMPLE 16

In the equation d = 5t2, find the value of t when d = 320.


THINK
1

Write the formula.

290 Maths Quest HSC Mathematics General 2

WRITE

d = 5t2

Substitute the value of d.

Divide each side by 5.

Take the square roots of each side, considering both the positive
and negative answers.

320 = 5t2
t2 = 64
t = 8

Note: If the solver function on your graphics calculator is used, only the positive solution is given. It is
important to remain aware that equations of this type have a positive and negative solution.
With such examples, consider both the positive and negative cases only where appropriate. In practical
cases where measurements are being considered, only the positive answer is given.
Using the same process as this, the subject of a formula can be changed. The subject of the formula is
the single pronumeral usually written on the left-hand side of the formula. For example, in the formula
A = r2, A is the subject. It is possible to make another pronumeral the subject of the equation by moving
all other numbers and pronumerals to the other side of the formula, as if we were solving an equation.
WORKED EXAMPLE 17

Make x the subject of the formula y = 5x 2.


THINK

WRITE

Write the equation.

Add 2 to each side.

Divide each side by 5 (and write the new subject of the formula
on the left-hand side).

y = 5x 2
y + 2 = 5x
x=

y+2
5

This method is also used for quadratic formulas but, as with equation solving, it is important to
remember to use both the positive and negative square root where appropriate.
WORKED EXAMPLE 18

The formula A = 4r2 is used to find the surface area of a sphere. Make r the subject of the formula.
THINK

WRITE

Write the formula.

Divide both sides by 4.

Take the square root of each side. As r is the radius, a length,


we consider only the positive square root.

Exercise 10E

A = 4r2
A
= r2
4
r=

A
4

Equations and formulas

1 WE15 The formula C = d is used to calculate the circumference of a circle. Find the diameter of a

circle that has a circumference of 40 cm. Give your answer correct to 3 significant figures.

2 The formula P = 2l + 2w is used to calculate the perimeter of a rectangle. Calculate the length of a

rectangle that has a perimeter of 152 m and a width of 38 m.

3 In each of the following, find the value of the unknown after substitution into the formula. Where

appropriate, give your answer correct to 1 decimal place.


h
a A = (a + b); find h when A = 145, a = 15 and b = 25.
2
b A = l w; find w when A = 186 and l = 15.
c V = r2h; find h when V = 165.2 and r = 3.6.
d T = a + (n 1)d; find n when T = 260, a = 15 and d = 11.
e v2 = u2 + as; find s when v = 5.5, u = 2.4 and a = 1.2.

Digital doc
SkillSHEET 10.1
doc-11091
Raising a power to
a power

Digital doc
SkillSHEET 10.2
doc-11092
Solving linear
equations

Chapter 10 Further algebraic skills and techniques 291

4 In the formula A = 6s2, find the value(s) of s when A = 150.

5 WE16 The formula A = r2 is used to calculate the area of a circle. Find the radius of a circle,

correct to 2 decimal places, given that the area of the circle is 328 cm2.
6 Substitute into each of the formulas and solve the equation to find the value of the unknown. Where
necessary, give your answer correct to 2 decimal places.
a V = r2h; find r when V = 1.406 25 and h = 2.5.
b A = (R2 r2); find R when A = 12 and r = 1.
4
c V = r3, find r when V = 136.
3
1
d E = mv2; find v when E = 254 and m = 23.
2
e P = I 2R; find I when P = 0.54 and R = 1.5.
7 WE17 Make x the subject of the formula y = 2x + 1.
8 Make l the subject of the formula A = l b.
h
9 In the formula A = (a + b):
2
a make a the subject of the formula
b make h the subject of the formula.
10 WE18 Make r the subject of the formula A = r2.
11 In the formula E = mc2:
a make m the subject of the formula
b make c the subject of the formula.
Questions 12 to 14 refer to the following information.
The volume of a square-based pyramid with the side of the base, s, and the height, h, is given by
1
the formula V = s2h.
3
12 MC The side length of the base of a square-based pyramid with the height, h, and volume, V, is
given by:
V
3V

D s =
h
3h
13 MC The height of a square-based pyramid with the side of the base 5 cm and the volume 75 cm3 is:
A 8 cm
B 9 cm
C 10 cm
D 12 cm
14 MC If both the side of the base and the height are doubled the volume is:
A doubled
B increased by 4 times C increased by 6 times D increased by 8 times
15 In each of the following, make the subject of the formula the pronumeral indicated in brackets.
4
a V = r2 [r]
b v2 = u2 + as[u]
c V = r3[r]
3
L
d T = 2
[L]
e c2 = a2 + b2[a]
g
V

h

A s = 3

B s =

h

3V

C s =

Further development

16 In each of the following make the pronumeral in brackets the subject and then substitute the given

value to find the value of the given pronumeral.


a A = l2 [l]
A = 60
4
b V = r3 [r]
V = 1000
3
c F = mg kv2 [v]
F = 250, m = 60, g = 10, k = 0.1
d v = u + at [a]
v = 25, u = 0, t = 6
e S = r(r + h) [h]
S = 120, r = 2, = 3.14
L
[l ]
g
l2 d2
g f =
[d ]
4l
f T = 2

292 Maths Quest HSC Mathematics General 2

T = 4, g = 9.8, = 3.14
f = 2, l = 15

h H = U + PV [V ]

H = 26, U = 4.5, P = 2

c2
[c]
K = 6.9, = 0.05
1
Hi v
j
= [u]
Hi = 34, Ho = 4, v = 40
Ho u
17 The perimeter P of a rectangle of length l and width w may be found using the equation P = 2(l + w).
a Find the perimeter of a rectangle of length 16 cm and width 5 cm.
b Rearrange the equation to make w the subject.
c Find the width of a rectangle that has perimeter 560 mm and length 240 mm.
18 The net force F measured in newtons (N) acting on a mass m measured in kilograms (kg) is found
using the equation F = ma, where a is the acceleration of the mass measured in metres per second.
a Find the net force required to accelerate a 2.5 kg rock at the rate of 4 m/s2.
b Make a the subject of the equation.
c Find the acceleration produced by a 700 N force acting on a 65 kg person.
i K =

1 1 1
+ + ,
u v f
where u is the distance of the object from the lens and v is the distance of the image from the lens.
a Make f the subject of the equation.
b Make u the subject of the equation.
c How far from the lens is the image when an object is 30 cm in front of a lens of focal length
25cm?

19 The object and image positions for a lens of focal length f are related by the formula

Digital doc
Worksheet 10.2
doc-11093

Chapter 10 Further algebraic skills and techniques 293

Summary
Addition and subtraction
of algebraic expressions

Only like terms can be added or subtracted.


In all addition and subtraction questions the sign belongs with the term that follows.
When adding or subtracting algebraic fractions each fraction must first be written over a
common denominator.

The index laws.

First Index law am an = am + n


Second Index Law: am an = am n
Third Index Law: (am)n = am n
Fourth Index Law: a0 = 1

Expanding and
simplifying

When expanding brackets multiply each term inside the brackets by the term immediately
outside the brackets.
When there is a negative term outside the brackets change the sign of each term inside the
brackets.
After expanding collect all like terms.

Solving equations

An equation is an incomplete mathematical sentence. To solve an equation the object is to find


the value of the pronumeral.
When solving an equation we perform the same operation to both sides of the equation to
isolate the pronumeral.
Equations that have pronumerals on both sides are solved by moving all pronumerals to the left
hand side.

Equations and formulas

After substituting into a formula an equation may be created when you are finding the value of
a pronumeral that is not the subject.
Using the same method as solving equations a formula can be rearranged to make another
pronumeral the subject.

294 Maths Quest HSC Mathematics General 2

Chapter review
1 The number of like terms in the expression 8xy2 + 2x + 8y2 5 is:
A 0

B 1

2 The expression 6d + 3r 4d r simplifies to:


A 2d + 2r

B 10d + 2r

C 2

D 4

C 10d 4r

D 2d + 4r

m u lti p l e
c hoic e

3 The solution to the equation 3x 6 = 7 x is:

1
1
1
1

B
C 3
D 6
4
2
4
2
4 The total surface area of a cone is given by the formula A = r (r + s), where r is the radius and s is
the slant height of the cone. The formula with s as the subject is:
A r
A
Ar
A
A s =
r
B s =

C s =

D s =
+r
r
r
r
r
A

5 The total surface area of the square-based pyramid with side of the base b and the height of the

triangular face h is given by A = b2 + 2bh. If the total surface area of the pyramid is 64 cm and the
length of the side of the base is 4 cm, the height of the triangular face is:
A 6 cm
B 10 cm
C 20 cm
D 24 cm

1 Simplify each of the following expressions.


a 8p + 9p
c 5y2 + 2y 4y
e 9s2t 12s2t
g 11c2d 2cd + 5dc2
i n2 p2q 3p2q + 6
2 Simplify each of the following.
a

5y y

3
2

b
d
f
h
j

+
+
8ab + 3b2 + 2ab
5x + 6xy x + 3xy
7x2y 8 2x2y + 2
8ab + 2a2b2 5a2b2 + 7ab
7m2

2m2

x+4 x+2
+

2
5

m2

5
1

3x 5x

3 Simplify each of the following


a b7 b3
b m2 m9
4
5
e h h h
f 2q5 3q2 q10

c k3 k5
g 5w3 7w2 w14

d f2 f8 f4
h 2e2p3 6e3p5

4 Simplify each of the following.


a a5 a2
b t5 t

c r19 r12

d 27q7 3q3

10a

2

5m4

m

24xy

6y

20x2y
15xy2

5 Simplify.
a (b4)2
e (a5b2)3

b (a8)3
f (m7n12)2

c (k7)10
g (2st6)3

d (j100)2
h (qp30)10

6 Simplify.
a 50

b t0

c 2x0

d (5y)0

7 Expand the expressions.


a 5(2x + 3)
b (y 6)

c x(3 2x)

d 4m(2m + 1)

8 Expand and simplify each of the following.


a 3(x 2) + 9
c 4m(m 3) + 3m 5

b 2(5m 1) 3
d 2x(3 2x) (4x 3)

9 Solve the equations.


a 42 7b = 14
c 2(x + 5) = 3

b 12t 11 = 4t + 5
d 2(4p 3) = 2(3p 5)

10 Solve the equations.


a 5(x 2) + 3(x + 2) = 0
c 7(2x 5) 4(x + 20) = x 5

S ho rt
a nsw er

b 8(3x 2) + 4(4x 5) = 7x

11 In the formula P = 2l + 2b find l, when P = 78 and b = 24.


Chapter 10 Further algebraic skills and techniques 295

12 The formula C = 2r is used to find the circumference of a circle given the radius. Find the radius of

a circle with a circumference of 136 m. Give your answer correct to 1 decimal place.
13 In the formula A = 6s2, find s when A = 216.
1
14 The volume of a square-based pyramid can be found using the formula V = s2h, where s is the side
3
length of the square base and h is the height of the pyramid. Find the side length of a square-based
pyramid with a volume of 108.864 cm3 and a height of 6.3 cm.
Extend ed
R e spons e

1 Simplify

3
2
+
2x 3x
b Verify your answer to Part a by substituting x = 5 into both expressions.
c Jack claims that if you double the value of x you will double the value of the final expression.
Explain whether or not Jack is correct.
2 The time taken for an investment to double in value when invested at 7.5% p.a. can be found by
solving the equation (1.075)n = 2.
a Use the method of substitution to find the solution to this equation, correct to the nearest
wholenumber.
b Write an equation that can be used to find the amount of time that it will take for the value of an
item to halve in value if it depreciates at 15% p.a.
c Solve this equation, correct to 1 decimal place.
a

Digital doc
Test Yourself
doc-11094
Chapter 10

296 Maths Quest HSC Mathematics General 2

ICT activities
10B The index laws

10eEquations and forumulas

INTERACTIVITY
int-2360: Indices. (page 281)

Digital docs
SkillSHEET 10.1 (doc-11091): Raising a power to a power. (page 291)
SkillSHEET 10.2 (doc-11092): Solving linear equations. (page 291)
WorkSHEET 10.2 (doc-11093): Apply your knowledge of algebra to
problems. (page 293)

10CExpanding and simplifying


Digital doc
WorkSHEET 10.1 (doc-11090): Apply your understanding of algebra
to problems. (page 286)
INTERACTIVITY
int-0075: Expanding brackets. (page 285)

10d Solving problems

Chapter review
Test Yourself (doc-11094): Take the end-of-chapter test to test your
progress. (page 296)

To access eBookPLUS activities, log on to www.jacplus.com.au

INTERACTIVITies
int-2778: Solving equations. (page 286)
int-2373: Solving equations. (page 286)
int-2764: Solving equations. (page 286)
int-0077: Keeping equations balanced. (page 287)

Chapter 10 Further algebraic skills and techniques 297

Answers chapter 10
Further algebraic skills
and techniques

c 1 d
1
c 18a + 29b d
19c + 22d
9 a 1 b
1
e 4a + 3b f
4x y
c 1 d
1
g 5p 10q h
8c + 9d
10 a 3 b
12
i 13x 19y j
8c + d
c 4 d
17
7 a 2m 11n b
6x 11y
11 a 4r3 b
5 c
13
c 9x + 5y d
14p 8q
12 a 6a6e7 b
8p6h12
e 7p + 11q f
5a + 3b
8 a 2ab + 2a 3b b
2xy + 4x 2y
c 80m9 d
6g3h6
c 2cd + 3c d
2qp 2p
e 30p6q9 f
48u9w7
e 5cd 11c f
5ab 21a 3b
g 27d11y17 h
42b14c9
g 5m h
6cd 36c
i 24r16s18 j
60h38v20
2
i
6m
+
6m

10
j
9cd 8c
5
3p
8b5
5m0n6
2
2p2 + 7p 6
13 a
b
c k 12m 20m 4 l
2
3
6
m 7x2 + 41x 6
9x8y
4hk3
9 a 15a2 + 2b2 9ab
d
e
f
3j5f3
b 8c2 + 3d2 + 22cd
4
3
2
6
2
5
3
c 10y2 6y 12
4p rs
20f g
9a b c
g
h i

Exercise 10A Addition and subtraction


of algebraic expressions
1 a 5k + 11c b
15m + 16f
c 9d + 5c d
4f + 8h
e 12g + 13j f
15d + 13
g 12n + 11 h
7h2 + 14y
i 11nv + 10u
2 a 3m + 2c b
8a + 3f
c 2k + 2 d
4t + 2
e r + 5 f
2v 5
g 4p 14 h
4w + 4
i 7c 17 j
j + 3c
k 7k + 4m l
4d + 3c
m 7y2 4y n
2x3 6x4
2
o c + 12
3
2
Exercise 10D Solving equations
3h6
3 a C b
B c
A
8
20
15
9b
25h
8k
2
2
1 a a = 24 b
k = 121
4 a x + 7x + 6 b
d + 4d 10
14 a
b
c
c g = 2.9 d
r=3
d6
4j4
27t24
c v2 10v 6 d
a2 2ab + b2
e h = 0.26 f
i = 2
e u2 + u 12 f
5n4 6n2 25
49p18
125y21
256a12
d
e f
1
5y
3y
13x
q=
g t = 5 h
64q44
27z39
2401c20
5 a b
c
6
12
12
40
1
1
1
15 a b
c i x = 0
y
14x
3w
d
e
f

b4
x3
z6
2 a f = 12 b
i = 60
9
28
5
1
1
3
c z = 7 d
v=7
89y
d e
f
32x
7x + 17
1
g
h
i
q7
m4
m2
e w = 5 f
k = 10
10
35
15
3
5q
1
7x + 30
2x 11
19x + 7
5
g
h
j
k l
m = 16
g a = 0.425 h
p
6m
12
30
6
8
21g6
1
36
6 a 0 b
16a 14b
16 a b
c i y = 211
a
c 14b 16a d
56a 49b
h3
p5
2
4
7 a 4x 2z b
6x 2y + 6z
3 a t = 100 b
b = 17
7p
20
d e
s f c q = 6.25 d
f = 1.2
x2 + 1
x2 1
p
4
q
8 a

b
16
3
x
x
p=
e
h
=
f
15a
g 3r6 h
8
49
9x2 + 25y2
9
b4
14
225
15xy
j=
g g =
h
Exercise 10C Expanding and simplifying
31
484
Exercise 10B The index laws
2
1 a 5k + 5 b
7m + 28
i a = 1
1 a 25p11 b
15x8 c
63y10
c 4y + 28 d
8d 72
3
d 18p8 e
70t6 f
20q15
4 a a = 4 b
b=6
e 12h 60 f
2k 12
6
9
7
12
8
2 a 6a e b
8p h c
80m
c i = 3 d
f=9
g 20m 8 h
30t + 25
1
2
d 6g3h7 e
30p6q8
i 16k 88 j
5m + 5n
e q = 1 f
r=5
3 a 5 b
77 c
s4
8
k 32y 24f l
18v + 42w
5
d z5 e
i3 f
6
5
4
m bc bd n
ki + kef
g s = 4 h
t=9
g 104 h
c4 i
t33
o 12pj 18mp
6
5
j h77 k
b70 l
f 900
1
2 a 3c 3 b
5d 10
i
a
=
7
2
2
7
4 a 4x b
6y c
9w
c 6m 66 d
8c 8d
2
d 3q4 e
10f9 f
10h90
e 12k + 8m f
14 + 21x
5 a f = 40 b
g = 30
10
10
3
g 2j h
5p i
8g
g 50 + 10y h
k2 2k
c r = 10 d
m = 18
3b5
9m4
10n90
i x2 + 3x
e n = 28 f
p = 62.4
j
k
l
1
3 a 5c + 23 b
17k + 42
7
2
2

6 a
x
=
1
b
y=9
c 13m + 41 d
j 16
5 a 5p4 b
6r4
3
2
1
e 2t 11 f
13m + 39
c 5a3 d
3b6
k=1
c m = 4 d
g 16c 14 h
d8
e 10r4 f
9q
2
5
2
1
i 4w 112 j
18h + 22
6 a x6 b
f 80
e
n
=
5
f
c
=
1
k 10y 19 l
9x + 27
c s100 d
r144
3
3
m 7h 5 n
6c + 38
e a10b15 f
p5q15
7 a k = 25 b
m = 16
3
1
o 5m + 22
g g15h10 h
81w36q8
c p = 11 d
u = 4
4 a y2 4y 12 b
w2 4w 12
i 25e10r4q8
7
8
8
c 2x2 13x + 20 d
3h2 + 8h + 5
7 a p10q6 b
r15w9 c
x10n18
x
=
e
f
v
=
3
e 3f 2 22f 16 f
8a2 + 6a 9
d j18k12 e
q4r20 f
h24j16
11
5 a D b
B c
D
g f 16g21 h
t10u8 i
i15j12
8 a B b
D c
C
6 a 8x + y b
10p + 8q
8 a 1 b
1

298 Maths Quest HSC Mathematics General 2

9 a x = 5 b
d = 1
c p = 7 d
x = 11
e h = 2 f
t=5
g v = 20 h
r = 3
i g = 0.8
10 a x = 1 b
v=1
c l = 2 d
g = 2

1
3

e t = 3 f
e = 23

3
g j = 3 h
k = 36
8
1
i f = 12
4
11 a x = 2 b
b = 5 c
w=2
1
d f = 7 e
t = 3 f
r=2
3
1
1
h = 2 i
g g = 1 h
a=0
3
5
2
12 a x = 1 b
c = 2 c
r=2
3
1
y = 1 f
d k = 1 e
g=7
8
2
1
g w = 1 h
m = i
p=1
3
5
4
13 a x = 15 b
y = 4 c
t = 21
5
5
1
1
d u = 2 e
f = 12 f
r=7
7
2
2
g d = 6 h
h = 12 i
x=1

1
3

i x = 1.5 j
x = 4
k x = 3 l
x=1
18 a x = 12 b
x = 5
c x = 7 d
x=7
e x = 9 f
x = 117
59

19 a x = 3 b
x = 4
c x = 5 d
x=8
e x = 9 f
x = 7
Exercise 10E

Equations and formulas

1 12.7 cm
2 38 m
3 a 7.2 b
12.4 c
4.1
d 26 e
20.4
4 5
5 10.22 cm
6 a 0.75 b
2.20 c
3.19
d 4.70 e
0.6

y1
2
A
8 l =
b
2A
2A
h=
9 a a =
b b
h
a+b
7 x =

10 r =

A
,r > 0

11 a m =

c2

8
10
f
x=
11
43

3
4
11
j x = 2 k
x = 2 l
x = 6
12
5
16 a x =
b
x = 15
17
10
2
c x = 6 d
x=
9
19
1
e x = 1 f
x = 192
2
4
g x = h
x = 12
7
1
i x = 3 j
x=3
4
5
k x = 52 l
x=1
8
5
31
x=1
17 a x =
b
19
58
1
15
c x = 21 d
x = 3
2
17
20
10
e x = 5 f
x = 1
43
13
2
9
x4
g x = 1 h
61
26
g x = 5 h
x = 5 i
x = 2

15 a r =
c r =

E
m

c=
b

V #
, r > 0 b
u = "v2 as

gT2
3 3V
L=
d
4
42

e a = "c2 b2

16 a l = !A, 7.746

3v
, 6.204
4

b r = 3

mg F
, 59.161
k
vu
, 4.167
d a =
t
S
S r2
r or h =
, 17.108
e h =
r
r
2
T
f l = ga b , 3.976
2
c v =

g d = "e2 4ft, 10.247

HU
, 10.75
P
(1 )K
i c =
, 2622
2
h v =

F
m

a=
18 a 10 N b
c 10.769 m/s2

fv
uv
b
u=
u+v
vf
c 150 cm
19 a f =

Chapter Review
1 A
2 B
3 C
4 A
5 A

Short answer

1 a 17p b
10m2
2
c 5y 2y d
10ab + 3b2
e 3 s2t f
4x + 9xy
g 16c2d 2cd h
5x2y 6
i n2 4p2q + 6 j
15ab 3a2b2

7y
7x + 18
22
b
c
6
10
15x
3 a b10 b
m11 c
k8
d f14 e
h10 f
6q17
g 35w19 h
12e5p8
4 a a3 b
t4 c
r7
d 9q4 e
5a f
5m3
4x
g 4x h
3y
5 a b8 b
a24 c
k70
d j200 e
a15b6 f
m14n24
g 8s3t18 h
q10p300
6 a 1 b
1
c 2 d
1
7 a 10x + 15 b
y + 6
c 3x + 2x2 d
8m2 4m
8 a 3x + 3 b
10m 1
c 4m2 9m 5 d
4x2 10x + 3
9 a b = 4 b
t=2
c x = 6.5 d
p = 2
1
2
10 a x = 0.5 b
x = 1 c
x = 12
11
9
11 15
12 21.6 m
13 s = 6
14 7.2 cm
2 a

12 D
14 a A b
C c
B
20
29
5
13 B
15 a x =
b
x = 3 c
x=
14 D
31
8
36
d x = 7 e
x = 2

H ov
, 4.706
Hi
17 a 42 cm
P
P 21
b w = l or w =
2
2
c 40 mm
j u =

Extended response

1 a 13/6x
b Both answers equal 13/30.
c Incorrect. The result is halved.
2 a n = 10 b
(0.85)n = 0.5
c n = 4.3

Chapter 10 Further algebraic skills and techniques 299

Chapter 11

Modelling linear relationships


CHAPTER CONTENTS
11A Linear functions
11B Direct variation
11C Linear modelling
11D Problem solving
11E Simultaneous equations

11A Linear

functions

A linear function is a function in which the highest power of both independent and dependent variables
is 1. When graphed, these values form a straight line.
An example of a linear function is y = 2x 1. One method of graphing the function is to create a table
of values, plotting the pair of coordinates that are formed on a number plane, and joining them with a
straight line. The independent variable is x, and as such, values of x are substituted into the equation to
find the corresponding values of y.
If we recognise the function as linear, we need to plot only three points. Two points are sufficient to
fix a line and the third is a check. If all three points are not in a straight line, we know that an error has
been made.

Interactivities
int-2378
Graphs of linear
functions
int-1020
Drawing a graph
int-2399
Slope and
equation of a line

WORKED EXAMPLE 1

Graph the relationship y = 2x 1.


THINK

Method 1: Technology-free
1 Draw a table of values for x. (Choose three
values of x.)
2

Substitute each value of x into the equation to


find the corresponding values of y.
Plot each of the points formed on a number
plane.

WRITE

x
y

x
y

1
1

2
3

1
y
5
4
3
2
1

5 4 3 2 1 0
1
2
3
4
5

1 2 3 4 5

Chapter 11 Modelling linear relationships 301

Join the points formed with a straight line and


label the line with the equation.

y
5
4
3
2
1
5 4 3 2 1 0
1
2
3
4
5

y = 2x 1
1 2 3 4 5

Method 2: Technology-enabled
1

From the MENU select GRAPH.

Delete any existing equation and enter


Y1 = 2X 1.

Press !3 [V-Window]. This allows


you to set the lower and upper limits to draw
on both the x- and y-axes. Enter the setting
shown on the screen at right.

Press w to return to the previous screen,


and then press 6 (DRAW) to draw the
graph.

The straight line in Worked example 1 has the equation y = 2x 1, which is written in
gradientintercept form. Any equation in the form y = mx + b is said to be in gradientintercept
form,becausethe gradient of the straight line is represented by m and the y-intercept is
representedby b.
This can be used to sketch any straight line. Considering Worked example 1, we can begin by
plotting the point (0, 1) as the y-intercept. Other points can then be plotted using the gradient,
byplotting points 1 across and 2 up. That is, starting with (0, 1), we plot (1, 1), (2, 3), (3, 5)
andso on.
At this point it is worth remembering the gradient formula:
m=
Interactivity
Int-0804
Application of
linear modelling

vertical change in position


horizontal change in position

We use this formula when we know two points on the graph, and this is useful on many occasions to
help us find the equation of a straight line.
Many real-life situations can be modelled by a linear function and/or graph. Once the equation
orrulehas been established, it can be used to make predictions or calculate specific values as
required.

302 Maths Quest HSC Mathematics General 2

WORKED EXAMPLE 2

The Avanti car rental company charges $80 for the hire of a car plus 22 cents per kilometre
travelled.
a How much will it cost to travel 300 kilometres?
b Determine the cost ($C) equation for a distance of x kilometres.
c Graph the function for 0 x 1000.
d If the final cost was $245, what distance was covered during the hiring period?
THINK

a 1 Express money in the same units.

WRITE

a 22 cents = $0.22

Find the cost of travelling


300kilometres.

Cost of travelling 300 km at $0.22 per km:


0.22 300
= $66

Add the charge for the hire of a car to


find the total cost.

Total cost:
C = 80 + 66
= $146

b 1 Write the cost of travelling any


2

b Cost of travelling x km at $0.22 per km:

distancex.

0.22 x

Add the charge for the hire to find the


total cost.

Total cost:
C = 0.22x + 80

c To graph the function:

c For 0 x 1000:

Find the coordinates of any two points.


End points are usually convenient to use.

when x = 0,
C = 0.22 0 + 80

= 80
when x = 1000, C = 0.22 1000 + 80

= 300
So the points are (0, 80) and (1000, 300).

Draw the set of axes with distance on the


horizontal axis and cost on the vertical
axis, plot the two points and join them
with the straight line. Note that since
neither distance nor cost can be negative,
we only need the first quadrant.

C($)
350
300
250
200
150
100
50
0
0 200 400 600 800 1000 1200 x (km)

d Substitute $245 for C in the general equation

of the cost and solve for x.

d C = 0.22x + 80

When C = 245:
245 = 0.22x + 80
165 = 0.22x
165
x=
0.22
= 750
Hence, 750 kilometres were covered.

In Worked example 2 both the gradient and y-intercept were known, so it was a simple matter of
substituting given values into the general rule y = mx + b to establish the equation of the cost.
Sometimes, however, we know the gradient only, or the y intercept only, and sometimes neither of them
is given. In such cases there will always be some extra information describing the relation between the
variables which will enable you to find the equation.
Chapter 11 Modelling linear relationships 303

When two linear functions are graphed on the same pair of axes, the intersection of the two graphs
shows the point where both equations hold true. This can have applications in a practical context.
Graphing linear functions can be used to determine profit, loss or break-even points. If cost
andreceipts are graphed, the difference between the y-values at any point will determine the
profitorloss. The point where the graphs intersect will be the break-even point, where no profit or
loss is made.

WORKED EXAMPLE 3

Interactivity
int-2780
Simultaneous
linear equations

The cost of producing shoes in Asia is given by the equation C = 2000 + 15n, where n is the
number of pairs of shoes produced per day. The cost of producing shoes in Australia is given by
the equation C = 1000 + 20n.
a
On the same pair of axes, graph the cost equations for producing shoes in Asia and
Australia.
b When is it more cost efficient to produce the shoes in Asia?
THINK

WRITE

Method 1: Technology-free
a 1 Draw a table of values for each cost

equation.

a C = 2000 + 15n

0
2000

n
C

100
3500

200
5000

100
3000

200
5000

C = 1000 + 20n
0
1000

n
C
2

Plot a pair of points generated by each cost


equation.

C
10 000

Join each with a straight line labelling each


with its equation.

8000

C = 1000 + 20n
C = 2000 + 15n

6000
4000

b It will be more efficient to produce the shoes

in Asia after the point of intersection.

Method 2: Technology-enabled
1

From the MENU select GRAPH.

Delete any existing equations and enter


Y1 = 2000 + 15X and Y2 = 1000 + 20X.
Note that we replace C with Y1 and Y2
and n with X.

304 Maths Quest HSC Mathematics General 2

10

00

80

60

40

20

2000

b If more than 200 pairs of shoes are produced

per day, it will be cheaper to produce the shoes


in Asia. This is because if n > 200 the value of
C is less, if the shoes are produced in Asia.

Press !3 [V-Window]. This allows


you to set the lower and upper limits to
draw on both the x- and y-axes. Enter the
setting shown on the screen at right.

Press w to return to the previous screen,


and then press 6 (DRAW) to draw the
graphs.

Press !5 [G-Solv], followed by


5 [ISCT] (intersection). This will find
the point of intersection and display the
coordinates of this point. Be patient: this
may take a moment.

From this we can see that the intersection occurs at x = 200 and y = 5000. Interpreting this result in
terms of the question shows us that when 200 pairs of shoes are produced the cost will be $5000 in either
Australia or Asia. From that point on it will be cheaper to produce the shoes in Asia.

Exercise 11A

Linear functions

1 WE1 Graph the function y = x + 3.


2 Graph each of the following linear functions on separate axes.
a y = 2x
b y = 3x 2
1
d y = 5 2x
e y = 2 x + 3
3 Consider the linear function 3x + 2y 6 = 0.
a Copy and complete the table at right.
b Graph the function 3x + 2y 6 = 0.

x
y

4 WE2 The cost, C, of a taxi hire is given by the linear equation

C = 3 + 1.5d, where d is the distance travelled in kilometres.


a Copy and complete the following table.
b Graph the cost function for the taxi hire.
c Use the graph to determine the cost of a 20 km taxi journey.
d Katie has $24. How far can Katie afford to travel in a taxi?

c y = x
1
f y = 1 4x

Digital doc
SkillSHEET 11.1
doc-11095
Substitution into a
formula

d
C

10

30
Digital doc
SkillSHEET 11.2
doc-11096
Recognising linear
functions

5 A concert promoter finds that the profit made on a performance is given by the equation

P = 3n 24000, where n is the number of people who attend


the concert.
n
0
a Complete this table of values, and use it to graph the profit
p
0
equation.
b What profit will the promoter make if 20000 people attend the concert?
c What will be the financial outcome for the promoter if 5000 people attend the concert?
d How many people will need to attend the concert for the promoter to break even?

10000

6 It is found that the number of ice-creams that will be sold during a day at the beach decreases as the

price of the ice-creams increases. The number of ice-creams that will be sold can be determined by
the equation N = 1000 5P, where P is the price of the ice-creams in cents.
a Graph the function.
b How many ice-creams will be sold at $1 each?
c If the ice-cream salesman has only 100 ice-creams to sell, at what price should he sell them?
7 Two linear functions are represented by y = 4 x and y = 3x.
a Graph both linear functions on the same pair of axes.
b What is the point of intersection of the two graphs?

Digital docs
SkillSHEET 11.3
doc-11097
Gradient of a
straight line
SkillSHEET 11.4
doc-11098
Graphing linear
equations

Digital doc
EXCEL Spreadsheet
doc-1391
Plotting linear
graphs

Chapter 11 Modelling linear relationships 305

8 By graphing both functions on the same pair of axes, find the point of intersection of the graphs

y = 2x 6 and y = x 1.

9 Find the point of intersection of the graphs x + 2y 4 = 0 and y = 2x + 2.


10 WE3 A factory produces two types of computer games: game A and game B.
a The factory can produce a maximum of 120 games per week. This can be represented by the

linear equation A + B = 120. Graph this function.

b Sales research shows that twice as many copies of game A will sell as game B. This can be

represented by the equation 2A = B. On the same pair of axes, graph this function.

c Find the point of intersection of the two graphs and make a conclusion about the number of each

game that should be produced by the factory each week.


11 The cost of running an old refrigerator is $1.20 per day. This can be represented by the equation

C = 1.2d. A new refrigerator will cost $900 but the cost to run will be only 30c per day. This can be
represented by the equation C = 900 + 0.3d.
a Copy and complete the table below.
d
C (old)
C (new)

1000

2000

b Graph both linear functions on the same pair of axes.


c Find the point of intersection of the two graphs; hence, state after how many days it will become

more economical to purchase a new refrigerator.


12 The cost, in dollars, of producing calculators can be given by the equation C = 15n + 1500, where n

is the number of calculators produced. When selling the calculators the receipts can be given by the
equation C = 20n.
a Graph both linear functions on the same pair of axes.
b Determine the number of calculators that need to be sold in order for the manufacturer to
breakeven.

Further development
13 Amex Car Rentals charges $75 per day plus $15 per hundred kilometres.
a How much would it cost to rent a car for one day if the car travelled 265 km?
b The bill for one days rental came to $142.50. How many kilometres did the car travel?
c Sketch a graph of the cost of renting the car for one day (C) versus the number of kilometres

travelled (d).
14 An employee of a telecommunications company sells mobile-phone plans. She is offered two

different salary packages by her employer:


Plan A: $400 per week plus $25 for each plan sold
Plan B: $150 per week plus $45 for each plan sold.
a Copy and complete the following table.
Number of plans sold
5
10
15
20

Salary package A

Salary package B

b On the same set of axes sketch the graph of salary (S) versus number of plans (n) for both

Package A and Package B.


c How many plans need to be sold before salary package B is the better package?

Conversion of temperature
To convert a temperature from degrees Celsius to degrees Fahrenheit, you can use the formula
9C
+ 32. A simpler but less accurate way is to double degrees Celsius and add 30. This
F=
5
approximation written as a formula becomes F = 2C + 30.
1. Use a spreadsheet or graphics calculator to graph each function on the same set of axes.
2. Describe the accuracy of the simpler formula and state the value(s) for which it is accurate.
306 Maths Quest HSC Mathematics General 2

11B Direct

variation

A variation occurs when one quantity is proportional to another. Consider the following variation
problem.
The number of cars produced on an assembly line varies directly with the number of workers
employed on the line. Twenty workers can produce 30 cars per week.
From this information, we can determine that the number of cars produced each week will be 1.5 times
the number of workers employed on the assembly line. Using this, we can draw the table below.
No. of workers (N)

No. of cars produced (C)

10
20
30
40
50

15
30
45
60
75

These figures are plotted on the axes at right.


In any example where one quantity varies directly with another, the
graph that is drawn will be a linear function through the origin (0, 0). To
draw the function, we need to know only one other point on the graph.
This is known as a direct linear variation.

C
160
140
120
100
80
60
40
20
0

Interactivity
Int-2399
Slope and
equation of a line

20 40 60 80 100 N

WORKED EXAMPLE 4

The distance travelled by a car is directly proportional to the speed at which it is travelling. If
the car travels 225km in 3 hours, draw a graph of distance travelled against time.
THINK

Draw a set of axes showing time on the horizontal axis


and distance on the vertical axis.

Plot the points (0, 0) and (3, 225).

Join them with a straight line.


Distance

WRITE

800
700
600
500
400
300
200
100
0

0 1 2 3 4 5 6 7 8 9 10
Time

If we examine the gradient of a variation function, we see that the gradient is equal to the constant of
variation. For example in Worked example 11, the gradient is 75. This is the speed at which the car is
travelling.
Any variation can be graphed using the form y = ax where a, the gradient, is also the constant of
variation.
WORKED EXAMPLE 5

It is known that y is directly proportional to x. When x = 30, y = 12.


a Draw the graph of y against x.
b What is the gradient of the graph?
c Write an equation linking y and x.

Tutorial
Int-2317
Worked example 5

Chapter 11 Modelling linear relationships 307

THINK

WRITE

a Draw a straight line graph through (0, 0) and (30, 12).

y
40
35
30
25
20
15
10
5
0

vertical change in position


horizontal change in position

b 1 Gradient =
2

20 40 60 80 100

12

b Gradient = 30

= 0.4

Simplify.

c The equation is in the form y = ax, where a is the

c y = 0.4x

gradient.

Exercise 11B

Direct variation

1 WE4 The distance travelled by a car varies directly with the time that the car has been travelling.

Ifthe car travels 400km in 5 hours, draw the graph of distance against time.
Digital doc
SkillSHEET 11.5
doc-11099
Substitution

Digital doc
SkillSHEET 11.6
doc-11346
Graphing linear
equations

2 A team of 6 people can unload 9 containers from a wharf per day.


a Draw a graph showing the number of containers, n, that can be unloaded by a team of people, p.
b What is the gradient of the graph drawn?
c Write an equation linking n and p.
3 WE5 It is known that y varies directly with x. When x = 5, y = 40.
a Draw the graph of y against x.
b What is the gradient of the graph?
c Write an equation linking y and x.
4 The distance, D, travelled by a car in a certain period of time will be directly proportional to the cars

speed, s. A car moving at 40km/h travels 120km.


a Draw the graph of D against s.
b Write an equation linking D and s.
5 The wage, W, earned by a worker is directly proportional to the hours, h, worked. A person who
works 35 hours earns $306.25. Draw the graph of W against h.
6 The quantity of petrol, l, used by a car varies directly with the distance, d, travelled by the car. A car
that travels 100km uses 12.5L of fuel.
a Draw the graph of l against d.
b Use the graph to find the quantity of petrol needed to travel 240km.
7 The height of a tree, h, is directly proportional to the girth, g. A tree with a girth of 2.5m has a height
of 14m.
a Draw the graph of h against g.
b Use the graph to find the height of a tree with a girth of 3m.
c Use the graph to find the girth of a tree that is 9m tall, correct to 1 decimal place.
8 It is known that A$100 will buy US$67.50. Draw a conversion graph between Australian and
USdollars.

Further development
9 For each of the following, state if the graph could be a direct variation. For those which are not give

a reason.

a y

308 Maths Quest HSC Mathematics General 2

b y

c y

d y

f y

e y

10 The directions on a bottle of kitchen mould remover recommend that you dilute half a cup of the

concentrate in 5 litres of warm water.


a Complete the following table.
Volume of water (L)
Amount of cleaner (cups)

10

15

20

30

b When graphed what will be the gradient of the variation?


c How does this relate to the mixture of concentrate and water?
11 The perimeter of a certain shape is directly proportional to the side length. When P = 15, s = 3.
a Find the perimeter of the shape if the side length is 6.2cm.
b Find the length of the side when the perimeter is 67cm.
c Name the shape.

(Hint: You may or may not need to draw the graph to answer this question.)
12 Mika is going to polish all the floors in her unit, except for the kitchen, laundry and bathroom, where

she has tiles. The plan of Mikas unit is shown below.

Co

rri

do

Bathroom
Bedroom
2

Bedroom
1

2m

Laundry

Backyard
2m

3m

Living

3m

Kitchen

Dining

4m

3m

Entrance
10 m
a Find the area that is to be polished.
b A particular type of varnish is sold in 3 L cans. If one can covers 17.25 m2 of flooring, how many

cans of varnish will Mika need to purchase in order to do the floors twice?
c How much varnish will be wasted?
13 The following graph shows the relationship between acceleration (a) of a

certain body and force (F) acting on that body.


a Find the gradient of the graph.
b Write the equation of the relationship.
c Find the force necessary to produce an acceleration of 4 m/s2
d Find the acceleration required to produce a force of 1000 N.
14 If (2, a) and (a, 8) are points on a direct variation find:
a the value of a
b the constant of variation (the gradient of the graph).

F (N)
(2,1600)

1600

a (m/s2)

INVESTIGATE: Currency conversions

Find out the current rate of conversion for each of the following foreign currencies and draw a linear
function that will convert between Australian dollars and each currency.
1. US dollars
2. Euro
3. Pound Sterling
4. Japanese yen
5. New Zealand dollars

Chapter 11 Modelling linear relationships 309

Linear modelling

Distance

11C

In the previous section we studied direct variation. Direct variations are


a simple example of a linear model. They are simple because all direct
variations pass through the point (0, 0). Consider the variation in Worked
example 4.
In this case the car travelled 225 km in three hours
In the preliminary course we saw that the gradient (m) is defined as the
rate of change in the dependent variable for every one unit increase in the
independent variable.
m=

800
700
600
500
400
300
200
100
0

0 1 2 3 4 5 6 7 8 9 10
Time

vertical change in position


horizontal change in position

Applying this formula to the example above


225
3
= 75
The gradient for this graph of 75 corresponds to the average speed of the car, 75 km/hr.
In any linear representation of a direct variation the gradient will be equal to the constant of variation,
that is; the rate of change in the quantity.
m=

WORKED EXAMPLE 6

The graph below represents the exchange rate between the Australian
dollar and the Euro.
Australian dollar versus the Euro
70
60

Euro

50
40
30
20
10
0

20

40

60
80
Australian dollar

100

120

Find the gradient of the function. What does the gradient represent?
THINK

WRITE

Find the gradient by selecting two points on


the line and using the gradient formula.

m=

Explain the interpretation of the gradient in


the context of the graph.

A gradient of 0.6 means that $1 Australian


= 60 Euro cents.

vertical change in position


horizontal change in position
30
=
50
= 0.6

Many real life applications such as fees charged for services, cost of manufacturing or running a
business, patterns in nature, sporting records and so on follow linear relationships. These relationships
may take the form of a linear equation; for example, F = 50 + 30t may be used by a tradesperson to
calculate her fee for t hours of work.
310 Maths Quest HSC Mathematics General 2

Here, F is the fee in dollars, and t the time in hours. The 50 represents an initial fee of $50 for simply
turning up, while the 30t is the amount charged for the time spent on the job.
For example, if t = 2 hours, 30t = 60, so the total charged for the work would be $(50 + 60) = $110.
Equations like F = 50 + 30t are sometimes referred to as linear models, a common form which is
Total cost = Fixed cost + Cost per unit Number of units.
This is, of course, similar to y = mx + b, where m is the gradient and b is the vertical intercept.
WORKED EXAMPLE 7

Gemhaven hire-company charges a $200 delivery fee, and a rental fee of $1500 per week for a
power generator.
a What would it cost to rent the power generator for 4 weeks?
b
Write an expression relating total charge (C) to the number of weeks (w) for which the generator
is hired.
c Sketch a graph of the relationship.
THINK

WRITE

a The cost will be $200 plus a Cost = $200 + 4 $1500

4 lots of $1500.

= $6200

b 1 Write the rule for a


2

linearmodel.
The cost (C) of the
generator is $1500 per
week (m) while the
fixed cost is $200 (b).

c Draw the graph with a

b y = mx + b

C = 1500w + 200

vertical intercept at 200


and with a gradient
of 1500.

9000
8000
7000

Cost ($)

6000
5000
4000
3000
2000
1000
0

Exercise 11C

Linear modelling

1 WE7 The cost of hiring a tennis court consists of a

booking fee and an hourly rate.


a What would be the charge for 3 hours?
b Use the photo to write an equation for the total
hire in terms of the hourly rate.
c Sketch a graph of the relationship.

3
Number of weeks

Hire Ch

arges

Book
in
Hour g fee $5
ly rate
$10

Chapter 11 Modelling linear relationships 311

2 A singing telegram service charges a $60 appearance fee, and $8 per minute sung.
a Write an equation for the total cost of a singing telegram in terms of the number of minutes sung.
b Sketch a graph of the relationship.
c What would be the charge for a 5-minute singing telegram?
3 Colleen delivers junk mail. She is paid $32 to traverse a particular route and a further 10 cents per

leaflet delivered.
a Write an equation for the total payment she receives.
b Sketch a graph of the relationship expressed in (a).
c What would be Colleens pay if she delivers 1650 leaflets along the route?
4 Josh works weekends delivering pizza. Josh is paid $40 for each shift plus $5 per delivery.
a Write an equation for Joshs pay (P) each shift where he delivers p pizzas.
b Draw a graph to represent this equation.
c Use your graph to find
i Joshs pay for delivering 12 pizzas
ii The number of pizzas delivered when Josh earns $55.
d What is the meaning of the gradient and vertical intercept in this graph.
5 The Scarford High School Social committee is organising the Year 12 farewell. Venue hire will cost

$800, a band will cost $600, DJ will cost $400 and a cake will cost $250. The meal for the evening
will cost $35 per head.
a Complete the table of values for the number attending (n) and total cost (C).
n
C

50

100

150

200

250

b Write an equation for C in terms of n.


c What is the gradient and vertical intercept for this function?
d Explain the meaning of the vertical intercept and gradient in this context.
6 A new car costs $45 000 and depreciates by $7500 per year.
a Write an equation to link the salvage value of the car (S) and the age of the car in years (a).
b What is the vertical intercept and gradient of this function?
c Explain the meaning of the gradient and vertical intercept in this context.
7 A pay-TV salesperson receives $300 per week plus $20 for every household he signs up to have

pay-TV connected.
Write an equation and graph the salesmans potential earnings.
8 The weekly cost of running a car (C) is given by C = 0.8k + 80 where k is the number of kilometres

driven.
a Draw a graph of this function.
b What is the gradient and vertical intercept of this function?
c Explain the meaning of the gradient and vertical intercept in this context.

Further development
9 The cost (C) of a taxi fare is given by the equation C = 2.5 + 0.4d where d is the distance of the

journey.
a What is the gradient and vertical intercept of this linear model?
b Explain why negative values of d have no meaning in this case.

10 The salvage value of a computer (V) after n years can be found using the formula

S = 3000 500n.
a Find the value of S when n = 0.
b Explain the meaning of this value in this context.
c Find the value of S when n = 5.
d Explain why values of n > 6 have no meaning.

11 The cost of a taxi fare is C = 3 + 0.4d where d is the distance in kilometres. Explain why a 20 km

fare is cheaper than two 9 km fares.

Digital doc
Worksheet 11.1
doc-11100

12 The equation T = 10 0.5n is used to calculate the time (in days) taken to erect a garage wheren

isthe number of workers on the job. For what values of n would this linear model have no
meaning?

312 Maths Quest HSC Mathematics General 2

11D Problem

solving

In many cases a linear model can be used to solve practical problems.


WORKED EXAMPLE 8

A builder finds that the time taken to build a house decreases in a linear pattern as the number of
assistants increases.
A builder can build a house in 80 days without any assistants.

With five assistants the building time is reduced to 20 days. How many assistants would be required
to complete the building in 12 days?
1

Write the important information.

Define the pronumerals to be used.

Label a pair of coordinate axes and plot the


coordinate pairs (0, 80) and (5, 20). Rule a straight
line through both points.
Note: From the graph it seems that the builder
could complete the job in 0 days if he had 6 23 (or 7)
assistants. Naturally, this is impossible so the graph
is discontinued.

WRITE

0 assistants take 80 days


5 assistants take 20 days
Let a represent the number of assistants.
Let d represent the number of days.
90
80
Number of days (d)

THINK

70
60
50
40
30
20
10
0

Use the two points to form a right angled triangle


and find the gradient. The vertical intercept will be
the point (0, 80).

3
4
5
6
No of assistants (a)

3
4
5
6
No of assistants (a)

90
80
Number of days (d)

70
60
50
40
30
20
10
0

Use the linear rule y = mx + b to find the equation.


First determine the gradient, m.

Chapter 11 Modelling linear relationships 313

Then substitute the y-intercept, b.


Write the equation using the defined
pronumerals.

Substitute d = 12 and solve to find the number


of assistants required.

vertical change in position


horizontal change in position
60
m=
5
= 12,
b = 80
m=

y = mx + b
d = 12a + 80
12 = 12a + 80
12a = 68.
a = 5.6

Write the answer.

Exercise 11D

The builder will require 6 assistants if he is to


complete the project in 12 days.

Problem solving

1 Springfield High School needs to purchase textbooks for its very large Year 10 class. The supplier

quotes a price of $4400 for 110 books, and $2200 for 55 books.
a Find a linear equation relating cost to number of books.
b If it turns out that the school needs 89 books, what is the cost?
2 WE8 A builder can complete a building in 40 days without any assistants, but if he hires four

assistants he can build it in 30 days. How many assistants would be required to complete the building
in 25 days?
3 A printer quotes a price of $400 to print 700 brochures and a price of $500 to print 1000 brochures.
a Find a linear equation relating cost to number of brochures.
b Find the cost of printing 150 brochures.
4 A chef can cater for 100 people is she is given 1 hour in preparation time, while if she caters for 200

people she requires 1 hour and 30 minutes.

a Find a linear equation relating preparation time to the number of people.


b Predict the preparation time required to cater for 450 people.
5 The selling price of a mathematics textbook is related to the number of pages in the text. A 300 page

book sells for $35 and each additional 10 pages increases the price by $1.
a Find a linear equation relating the selling price to number of pages.
b Predict the cost of a 264 page book.
314 Maths Quest HSC Mathematics General 2

6 Angela Shyster, a barrister, charges $160 per hour for her legal services.
a Find the linear equation relating charge to time.
b Predict the cost for a 14-hour legal case.
7 A computer program can be written in 50 days if there are

two programmers working on the project, while if there are


five programmers it takes only 35 days. Predict the time
it would take six programmers to complete the project.
8 Nile.com, an internet seller of compact disks quotes a
shipping cost of $10 for 3 CDs and a cost of $15 for 5 CDs.
a Find a linear equation relating shipping cost to number
of CDs.
b What would be the shipping cost for 200 CDs?

Further development
9 Five students can clean the cafeteria in 15 minutes, while

three students would take 27 minutes.


How long should it take one student to complete the task?
10 A taxi charges a flagfall of $3.25 plus a distance rate of
$0.80 per kilometre.
a Find a linear equation relating cost to distance travelled.
b What would be the cost of a 15.3 kilometre journey?
c How far could you go for $17.40?

11E Simultaneous

equations

When a point belongs to more than one line, the coordinates of that point satisfy both equations.
Theequations of the lines are called simultaneous equations.
Simultaneous equations can be solved using a number of different methods:
Graphically by drawing a graph
Algebraically by substitution
Algebraically by elimination

Solving simultaneous equations


graphically
This method involves drawing a graph of each equation on a set of axes and then finding the point
ofintersection.

WORKED EXAMPLE 9

Solve the simultaneous equations 2x 3y = 6 and x + y = 3 graphically.


THINK
1

Determine the intercepts for each


equation.
The y-intercept occurs when
x = 0.
The x-intercept occurs when
y = 0.
State the coordinates of the
x- and y-intercepts.

WRITE

2x 3y = 6
y-intercept: let x = 0
2(0) 3y = 6
3y = 6
y = 2
x-intercept: let y = 0
2x 3(0) = 6
2x = 6
x = 3
(0, 2), (3, 0)

x+y=3
y-intercept: let x = 0
(0) + y = 3
y=3
x-intercept: let y = 0
x + (0) = 3
x=3
(0, 3), (3, 0)

Chapter 11 Modelling linear relationships 315

On grid paper, plot the x- and y-intercepts


for each line and join them to graph
eachline.

Carefully read the coordinates of the


point of intersection. The point of
intersection appears to be (3, 0), as shown
in green.

y
4
x+y=3
3
2
(3, 0)
1
110 1 2 3 4 5 x
2 2x 3y = 6
3

Write the solution.

The solution is x = 3 and y = 0, the


point (3, 0).

Check that the solution satisfies both


equations by substituting the values x = 3
and y = 0 into each equation.

2x 3y = 6
LHS = 2(3) 3(0)
=6
LHS = RHS
The solution is correct.

x+y=3
LHS = 3 + 0

=3
LHS = RHS

Solving simultaneous equations algebraically


bysubstitution
This method is useful when one equation is in a form where one of the variables is the subject. For
example, y is the subject in the equation y = 5x + 3.
WORKED EXAMPLE 10

Solve the system of equations y = x + 3 and y = 2x + 5.


THINK

WRITE

Write the two equations, labelling


oneasequation 1 and the other as
equation 2.

y = x + 3 [1]
y = 2x + 5 [2]

At the point of intersection the two


y-values are equal, so [1] = [2].

At point of intersection:
[1] = [2]
x + 3 = 2x + 5

Solve the equation for x:


x appears on each side of the equation,
so subtract x from both sides.
Subtract 5 from both sides.

x + 3 x = 2x + 5 x
3=x+5
35=x+55
2 = x

The x-coordinate of the point of


intersection is 2. To find the value of the
y-coordinate, substitute the value of x into
one of the equations (it does not matter
which).

Write the solution as the pair of


coordinates representing the intersection
point.

316 Maths Quest HSC Mathematics General 2

y1 = x + 3
= (2) + 3
=1

The solution is x = 2 and y = 1, or (2, 1).

Solving simultaneous equations algebraically


byelimination
This method involves adding two equations to create a third in which one of the original variables has been
eliminated, thus allowing substitution to take place in order to find the value of the remaining variable.
WORKED EXAMPLE 11

Solve the following pair of simultaneous equations.


x 5y = 17 2x + 3y = 5
THINK

WRITE

Write the equations, one under the other and


number them.

x 5y = 17 [1]
2x + 3y = 5
[2]

Look for a single multiplication that will


create the same coefficient and opposite sign
of either x or y. Multiply equation [1] by 2
and call the new equation [3].

[1] 2: 2x + 10y = 34 [3]

Add equations [2] and [3] in order to


eliminate x.

[2] + [3]: 13y = 39

Solve for y by dividing both sides of the


equation by 13.

Substitute the value of y into equation [2].

Solve for x.
(a)Subtract 9 from both sides of the equation.
(b)Divide both sides of the equation by 2.

Answer the question.

Solution: x = 2, y = 3 or (2, 3)

Check the answer by substituting into


equation [1].

Check: Substitute into x 5y = 17.


LHS = (2) 5(3)
= 2 15
= 17
LHS = RHS and therefore the solution is correct.

y=3
Substituting y = 3 into [2]:
2x + 3(3) = 5
2x + 9 = 5
2x = 4
x = 2

In many cases linear models and simultaneous equations can be used to solve problems.
WORKED EXAMPLE 12

Rent-a-Chef provides food cooked and served by a qualified chef at


parties. The company charges $120 as a booking fee, and an additional
$30 per hour.
Another company, Greased lightning, provides fast food served by two
students at a cost of $65 per hour, with no booking fee. Under what
conditions would it be cheaper to hire Greased lightning?

Chapter 11 Modelling linear relationships 317

THINK

WRITE

Define convenient pronumerals.

Let C = Cost (total) in dollars and t = time in hours.

Write an equation for the cost of


hiring both organisations.

Rent-a-Chef
C = 120 + 30t
[1]
Greased lightning C = 65t [2]

Draw the graph for the cost of both


companies.

c
500
400

C = 65t

300

C = 120 + 30t

200
100
0
4

From the graph read off the point of


intersection which will show when
both costs are the same.

Exercise 11E

It is cheaper to hire Greased lightning for period of up to


3.4 hours.

Simultaneous equations

1 WE9 Solve the following pairs of simultaneous equations by graphing the equations and identifying

the point of intersection.


a y = 2x + 3
b y + 2x = 8
y = 8 3x
y = 2x + 4
e 31 = y 2x
f x + 2y = 4
2x + 2y = 14
y = 2x 3

c 2y = 12x + 16

3y =

6x

24

d 2x y = 1

3x + y = 11

2 WE10 Solve the following pairs of equations simultaneously using the substitution method.
a 2x + y = 17
b y + 4x = 6
c 3x 2y = 5
d y = 1 2x

x = 2y + 1

e y = 2x 5

5x y = 1

y = 2x 3

f y = x + 4

y = 3 4x

y = 3 4x

5x = 3y

3 WE11 Use the elimination method to solve the following simultaneous equations.
a 3x + 2y = 13
b 4x 2y = 7
c y x = 5
d x + y = 20
x

+ 3y = 8
e 3x + 2y = 10
12x 5y = 14

3x + 2y = 14
f 2x + 5y = 4
7x + 15y = 9

3x 5y = 21

3x + 11y = 100

4 WE12 A computer firm, SuperComputers Inc., offers a back-up plan covering the ongoing service

and troubleshooting of its systems after sale. The cost of signing up for the service plan is $125, and
there is an hourly rate of $65 for the servicepersons time. Purchasers not signing up for the plan are
charged a flat rate of $150 per hour for service. Would it be advisable to sign up for the service plan
if you expected to need 3 hours of service assistance during the life of a computer purchased from
SuperComputers Inc?
5 A telephone company, Opus, offers calls to Great Britain for a connection fee of $14, and thereafter

$1 per minute. Its rival, Belecom, offers calls for $2 per minute (no connection fee) to the same
country.
a Compare the cost of a 10 minute call to Great Britain using each company.
b At what point would it be cheaper to use Opus?
318 Maths Quest HSC Mathematics General 2

6 It costs you $6 to get into a taxi (the flagfall), and $1.50 per kilometre if you use Pink Cabs, while

NoTop taxis charge $8 flagfall, and $1.20 per kilometre.


a How much would it cost with each company to travel 15 km in one of its cabs?
b When would it cost the same to use both companies?
7 Two amusement parks show the following information for school age tourists in a promotional

brochure.

$8.00 entry

$2.50 per ride

$12 entry
$1.50 per ride

After how many rides does an excursion to Fun World


become the cheaper option for the same number of rides?
8 Medirank, a health insurance company, charges

$860 per year (for a single person), and requires


customers to pay the first $100 of any hospital visit.
HAB, on the other hand, charges an annual fee of $560
and requires its members to pay the first $150 of any
hospital visit. Determine the number of hospital visits
in a year for which the cost of health services is the
same whichever company insures you.
9 Nifty is a car hire firm that charges insurance of $135,

and $50 per day car hire.


A competitor, Savus, simply charges $65 per day and
offers free insurance. You are planning a holiday and
would prefer to use Savus. Under what conditions
(days hired) could you justify this choice.

Further development
10 At the candy store Katrina buys 10 chocolates and a

8 sherbets for $4.30 while Vanessa buys 6 chocolates


and 9 sherbets for $4.05. By drawing appropriate
graphs find the cost of each sweet.
11 On a farm there are chickens and cattle.

In total there are 57 animals and 196 feet. How many chickens are there?
Chapter 11 Modelling linear relationships 319

12 From a sports store a school orders 16 cricket balls and 24 basketballs for $826.80.

Digital doc
Worksheet 11.2
doc-11101

A second school paid $633 for 12 basketballs and 32 cricket balls. Find the cost of each item.

320 Maths Quest HSC Mathematics General 2

Summary
Linear functions

Linear functions have powers of 1 for both the independent and dependent variables and are graphed
as straight lines.
To graph a linear function, a table of at least three values is drawn; the points generated are plotted
on a number plane and then joined in a straight line.
The intersection of two linear functions will give the point where both conditions hold true.

Direct variation

When two quantities vary directly with one another, the variation can be graphed as a linear function.
The variation will be in the form y = ax.
The graph is drawn from the point (0, 0) to one other point that is given.
The gradient of the function will be the constant of variation.

Linear modelling and


problem solving

Many situations can be represented using a linear model.


Use the rule y = mx + b where m is the gradient and b is the vertical intercept to model a linear
function.
Use the graph drawn to answer questions relating to the model

Simultaneous
equations

When there is two variables graph both linear models


The point of intersection of the two graphs will represent the point where both equations are true
simultaneously.

Chapter 11 Modelling linear relationships 321

Chapter review
mult ip le
ch oice

1 Which of the following is not an example of a linear function?


A y = 2x + 1

B y = 2x2

C 2y = x + 1

D x + 2y + 1 = 0

2 Beth lives 5 km from the nearest post office. At noon one day she rides from home at a speed of

20km/hr in a direct line away from the post office. Her distance at any time from the post office can
be found using the linear model
A D = 20t
B D = 5t + 20
C D = 20t + 5
D D = 20t 5

3 Which of the following points represents the solution to the pair 2x + 3y = 18 and 5x y = 11?
B (3, 4)

A (6, 2)

Sh ort
a nswer

1 Graph each of the following linear functions.


a y = 3x
b y = x + 3
e 2y = 4x 3
f 3x 2y + 6 = 0

C (3, 9)

D (3, 4)

c y = 2 x

d y = 5 3x

2 The cost, C, of a taxi fare is given by the formula C = 3 + 0.4d, where d is the distance travelled by

the car, in kilometres.


a Copy and complete the table below.
d

10

15

20

C
b Graph the cost function.
3 At a fete, 400 cans of soft drink are purchased for $320. The cans are then sold for $1.25 each.
a Write, as a linear function, an expression for the profit on the sale of the cans, where n is the

number of cans sold.


b Graph the profit function.
c What will be the financial outcome if:
i 300 cans are sold
ii 142 cans are sold?
d How many cans will need to be sold for the drink stall to break even?
4 The cost of a tank of petrol varies directly with the amount of petrol purchased. If 25L of petrol

costs $21.25, graph the variation.


5 It is known that p varies directly with q. When p = 5, q = 15.
a Draw the graph of q against p.
b What is the gradient of the graph?
c Write an equation linking p and q.
6 The washing machine company Washed Out charges $75 to come to your house, as well as an

hourly charge of $65, calculated to the nearest half hour.


a Write an equation that can be used to calculate the cost of a service call.
b Sketch the graph of the relationship between repair cost and time taken.
c What would be the cost of a repair job that takes 3 hours?
7 The cost of a parachuting course is $250 for hire of

equipment and tuition plus $55 per jump.


a Express the cost (C) as a function of the number
of jumps (j).
b How many jumps could a person afford to do
with a budget of $1000?
8 A rectangular house has a total perimeter of

34 metres and the length is 5 metres greater


than the width. What are the dimensions of
the house?
9 Calculate the cost of a carton of milk and a loaf of bread given that 3 cartons of milk and

2 loaves of bread costs $9.95 while 2 cartons of milk and a loaf of bread costs $5.85.
322 Maths Quest HSC Mathematics General 2

1 As a fundraising activity, a school hires a cinema to show the premiere of a

movie. The cost of hiring the cinema is $500. People are then charged $10 to
attend the movie.
a Write a function for the profit or loss made on the movie in terms of the
number of people attending.
b Graph the function.
c Use the graph to calculate the number of people who must attend the
movie for the school to break even.
d A rival cinema offers to waive the hire fee but the school will receive
only $5 per person attending. On the same axes graph the function P = 5n.
e The school chose to pay the $500 and receive $10 per person. How many
people must attend the premiere to make this the better of the two options?
2 A physics student conducts an experiment to find out how much a spring
stretches when various weights are hung from it. Her results are shown in the
table below.
Length of spring (cm)
4
7
12
16.5
20.5
25
a
b
c
d

Ex tended
R es p ons e

Force applied (N)


0
10
20
30
40
50

What is the natural or unstretched length of the spring?


Plot a graph of the students results.
Draw a straight line through the points that best describes the data.
Select two points on the line and use them to fit a linear equation to the line.
A second student conducts the same experiment on a similar spring. His results
are shown below.
Length of spring (cm)
5
10
16
21
24
28

Force applied (N)


0
10
20
30
40
50

e On the same set of axes you used in part b, plot the results of the second experiment and join the

points with a line of best fit.


f Write an equation that describes the relationship between the force applied and the length of the

second spring.
The gradients of graphs such as the ones you have drawn give an indication of the stiffness of a
spring. The greater the gradient, the harder it is to stretch the spring. The lower the gradient, the
easier it is to stretch the spring.
g Comment on the stiffness of the two springs investigated by the students.
h Is it likely that these two springs will ever be the same length at a given force, that is, is it likely
the intersection of the two graphs could ever really happen? Explain your answer.

Digital doc
Test Yourself
doc-11102
Chapter 11

Chapter 11 Modelling linear relationships 323

ICT activities
11A Linear functions

11C Linear modelling

INTERACTIVITies
int-2378: Graphs of linear functions. (page 301)
int-1020: Drawing a graph. (page 301)
int-2399: Slope and equation of a line. (page 301)
int-0804: Application of linear modelling. (page 302)
int-2780: Simultaneous linear equations. (page 304)

Digital doc
WorkSHEET 11.1 (doc-11100): Apply your knowledge of linear
relationships to problems. (page 312)

Digital docs
SkillSHEET 11.1 (doc-11095): Substitution into a formula. (page 305)
SkillSHEET 11.2 (doc-11096): Recognising linear functions. (page 305)
SkillSHEET 11.3 (doc-11097): Gradient of a straight line. (page 305)
SkillSHEET 11.4 (doc-11098): Graphing linear equations. (page 305)
EXCEL Spreadsheet (doc-1391): Plotting linear graphs. (page 305)

11B Direct variation


INTERACTIVITY
int-2399: Slope and equation of a line. (page 307)
TUTORIAL
WE5 int-2317: Learn about direct proportionality. (page 307)
Digital docs
SkillSHEET 11.5 (doc-11099): Substitution. (page 308)
SkillSHEET 11.6 (doc-11346): Graphing linear equations. (page 308)

324 Maths Quest HSC Mathematics General 2

11e Simultaneous equations


Digital doc
WorkSHEET 11.2 (doc-11101): Apply your knowledge of linear
relationships to problems. (page 320)

Chapter review
Digital doc
Test Yourself (doc-11102): Take the end-of-chapter test to test your
progress. (page 323)

To access eBookPLUS activities, log on to www.jacplus.com.au

Answers chapter 11
Modelling linear
relationships

1000
800
600
400
200

10
5 3 1
1 1

3 x

2 a

y
4
3
2 y = 2x
1
4 2 0 1 2 3 x
2
3

y y = 3x 2
4
3
2
10
4 2 1 1 2 3 4 x
2
3
4

y
4
3 y = x
2
1 0

3 1 1 2 3 x
2
3
4

f
y
5
4
3 y = 1 x + 3
2
2
10
1
3 1 1 2 3 4 x

3 a

C($)

120
100

80
A + B = 120

40
20
40

120 A

80

c Game A: 40; Game B: 80.


11 a

10

30

10.5

18

48

C = 3 + 1.5d

40

0 1000 2000

C (old)

0 1200 2400

C (new)

900 1200 1500

C = 1.2d

C
1500

C = 900 + 0.3d

1000
500
0

c $33
d 14 km
5 a
P P = 3n 24 000

C = 20n
C = 15n + 1500

6000

24000

6000

b 300
13 a $114.75
b 450 km

50

10000

30

8000

2000

$600

15

$775

$825

20

$900

$1050

Plan B
Plan A

5 10 15 20
Number of plans
sold (n)

Direct variation

900
800
700
600
500
400
300
200
100
0

0 1 2 3 4 5 6 7 8 910 t

2 a n
16
14
12
10
8
6
4
2
0

0 1 2 3 4 5 6 7 8 910 p

3 a y
90
80
70
60
50
40
30
20
10
0

0 1 2 3 4 5 6 7 8 910 x

b 8
c y = 8x
4 a D

4000

20

10

10 000

b $36 000

c After 1000 days


C
12 a
8000

000
000

50 000
40 000
30 000
20 000
10 000 0
10 000
20 000
30 000

20
00

40

10
00

20

10

$650

b 1.5
c n = 15p

2000

20

10

1 D
2A = B

$375

Exercise 11B

60

$525

c 13

Intersection (5, 4)
9 (0, 2)
10 a, b B

1200
1000
800
600
400
200

5 y=x1
4
3
2
1
3 1 0 1 2 3 4 5 x
y = 2x 6
3

200 400
d(km)

b ($)

b (1, 3)
8
y

5
y = 3x
4
3
2
1
3 1 01 2 3 4 5 x
y=4x
3

y
5
4 3x + 2y 6 = 0
3
2
10
x
3 1
1 1 2 3 4 5
2
3

4 a

y
5
4
3
1
2 y = 1 _x
4
1
0
x
3 11 1 2 3 4
2
3

Number of
Salary
Salary
plans sold package A package B

b 500
c $1.80
y
7 a

y
5 y = 5 2x
4
3
2
1 0
x
3 1
1 1 2 3 4

60

14 a

150
120
90

N = 1000 5P

20
0

Linear functions

y
5
3 y=x+3

12
0

40

Exercise 11A

c $9000 loss
d 8000 people
6 a
N

350
300
250
200
150
100
50
0

0 20 40 60 80 100 s

b D = 3s

Chapter 11 Modelling linear relationships 325

3 a P = 0.1n + 32

5 W
500
400
300
200
100
0

b Payment

0 10 20 30 40 50 h

6 a L (L)
70
60
50
40
30
20
10
0

10 20 Number
of leaflets

0 00 00 00 00 00 d (km)
1 2 3 4 5

c $197

b 30L
7 a h (m)
120
100
80
60
40
20
0

4 a P = 5p + 40
b 100

b 16.8m
8 $US
300
250
200
150
100
50
0

90
80
70
60
P 50
40
30
20
10
0

8 12 16 20 g (m)

c 1.6m

0 00 00 00 00 00 A$
1 2 3 4 5

9 a Not a direct variation, does not pass through (0, 0)


b Not a direct variation, the graph is not linear.
c This is a direct variation.
d Not a direct variation, the graph is not linear.
e Not a direct variation, does not pass through (0,0)
f Not a direct variation, does not pass through (0,0)
10 a

($)
34
33
32

Volume of water (L)

1 2 3 4 5 10 15 20 30

Amount of cleaner (cups)

1
10

1
5

3
10

2
5

1
2

1 1 1 2 3
2

1
10

c This is the ratio of concentrate to water.


11 a 31 cm
b 13.4 cm c Regular pentagon
12 a 46 m2
b 6 cans c 2 L
13 a 800
b F = 800a
c 3200 N
d 1.25 m/s2
14 a 4
b 2
Exercise 11C

1 a $35
c Cost ($)

Linear modelling
b C = 10t + 5

25
20
15
10
5

6
p

10

12

c i$100
ii 3
d The vertical intercept indicates that Josh will receive $40 even if

there areno deliveries. The gradient indicates that the gets


$5 per delivery.

5 a

n 50 100 150 200 250


C 3800 5550 7300 9050 10800

b C = 35n + 2050

c m = 35, b = 2050
d The gradient is the additional cost per person who attends and the

y-intercept is the fixed costs


6 a S = 45000 7500a
b m = 7500, b = 45000
c The gradient is the annual decrease in value
of the car and the vertical intercept is the cost
of the car new.
7 C = 20n + 300
600

500

1 2 Time (h)
2 a C = 8t + 60
b Cost ($)
76
68
60

400
C 300
200
100

1 2 Time (min)
c $100

326 Maths Quest HSC Mathematics General 2

6
n

10

12

8 a

600

500
Cost ($)

y
y + 2x = 8

300

Intersection point (3, 2)

200

y
10 2y = 12x + 16
5

100
0

100

200

300
400
Distance (km)

500

vertical intercept represents the fixed costs.


9 a m = 0.4, b = 2.5
b d is distance travelled and therefore negative values have no
meaning
10 a 3000
b The value of the computer when new.
c 500
d When n = 6 the computer has no value therefore if n > 6 the
computer cannot be worth less than 0.
11 The single journey is cheaper because with two separate journeys
you play the fixed cost of $3 twice.
12 n < 1 has no meaning as there must be at least one worker on the
job. Values of n 20 have no meaning as this would mean that the
building took no time or a negative time to build.
Exercise 11D

1 a P = 40n
b $3560
2 6

1 a

y = 2x + 3
(1, 5)
y = 8 3x
2

y
40 31 = y 2x
30
20
(8, 15)
10
0
20 10 10 20 x
2x + 2y = 14

Intersection point (8, 15)


y
4
x + 2y = 4 2

y = 2x 3
(2, 1)
2 4 6x

3
2

2 a (7, 3)

2 4x

Intersection point (2, 1)

(2, 5)
3x + y = 11

220
4

Simultaneous equations

2x y = 1

Intersection point (2, 5)

y
8

y
10
8
6
4
2
4 220

3 a C = n + 166.67

Exercise 11E

3y = 6x 24

Intersection point (2, 4)


d

Problem solving

1
3
b $216.67
4 a T = 0.005n + 0.5
b 2 hours 45 minutes
5 a P = 0.1p 5
b $21.40
6 a C = 160t
b $2240
7 30 days
8 a C = 2.5n + $2.50
b $502.50
9 39 minutes
10 a C = 0.8d + 3.25
b $15.49
c 17.69 km

2 4x

4 250
(2, 4)
10

600

b m = 0.8, b = 80
c The gradient is the cost per kilometre to run the car while the

1 2x

5 4 3 2 120
(3, 2)
4

400

y = 2x + 4

4
2

4x

Intersection point (1, 5)

c (1, 1)

b Q , 0R
4

23

d (1, 1)

e Q ,

3 a (5, 1)

b Q3, R

5
2

3 5
2 2

f Q , R

c (2, 3)

d (15, 5)
e (2, 2)
f (3, 2)
4 Yes cost under the plan $320 without the plan $450
5 a Opus $24, Belecom $20
b After 14 min
6 a Pink cabs $28.50, No Top $26.00
b 6.7 km
7 After 4 rides
8 6
9 Savus would be cheaper for less than 9days.
10 Chocolate 15c and Sherbert 35 c
11 16 chickens and 41 cattle
12 Basketballs $28.46, Cricket balls $9.11

Chapter Review
Multiple choice

1 B
2 C
3 D

Chapter 11 Modelling linear relationships 327

Short answer


d
y
4
3 y=2x
2
10
x
3 1
1 1 2 3 4
3
2
3
4

f
y
3
2
1 0 2y = 4x 3
x
3 1
1 1 2 3 4
3
4
5

2 a

10

15

20

11

y
5
4 y = 5 3x
3
2
10
11 1 3 4 x
2
3
y
5
4
3 3x 2y + 6 = 0

10
x
3 1
1 1 2 3 4
2
3

C = 3 + 0.4d

18
12
6
0

P = 1.25n 320

80
16
0
24
0
32
0
40
0

400

Cost ($)

c i $55 profit
ii $142.50 loss
d 256
4

5 a

0
1
2
3
c $302.50
7 a C = 250 + 55j
b 13
8 11 m 6 m
9 Milk $1.75, Bread $2.35

0 10 20 30 40 50
Amount of petrol (L)

P = 10n 500
P = 5n
n

c 50
e More than 100 people
2 a 4 cm
b, c, e
60
50
40
30
20

Student 1
Student 2

10
5

10 15 20 25 30
Length of spring (cm)

95
.
13
f Answers will vary. One possible answer is y = 2x 10.
g The first spring is stiffer than the second, as the gradient of its
graph is greater than that of the second spring.
h The graphs intersect at the point (8.75, 27.5). It is not possible
for the springs to have a negative length, so this point is not
achievable.
d Answers will vary. One possible answer is y =

Extended response

60
50
40
30
20
10
0

1000

200
300

45
40
35
30
25
20
15
10
5
0

200

0
500

3 a P = 1.25n 320
b P

0
100

400

2000
1500
1000
500

0 10 20 30 40 50 d

200
100

600

1 a P = 10n 500
b, d
P

b C
24

6 a C = 75 + 65t
b
c

50
10
0
15
0
20
0

y

b
y
4
4
3 y=x+3
3
2 y = 3x
10
1
x
5 3 1
3 1 01 2 3 4 x
1 1 2 3
2
3
3
4

Force applied to spring (N)

1 a

8 12 16 20 p

b 3
c q = 3p

328 Maths Quest HSC Mathematics General 2

30
x
13

Chapter 12

Modelling non-linear relationships


CHAPTER CONTENTS
12A Quadratic functions
12B Cubics, hyperbolas and exponential functions
12C Direct variation
12D Inverse variation
12E Graphing physical phenomena

12A

Quadratic functions

A quadratic function is a function in which the highest power of the independent variable (x) is2.
Thegraph of a quadratic function is a parabola, a curved line that comes to either a minimum or
maximum point.
The graph of a quadratic function can be drawn by creating a table of values and plotting the pairs of
coordinates generated. Because the graph is not a straight line, it is necessary to plot more than just three
points to show the shape of the curve.
The most basic quadratic function is y = x2. The table of values is drawn showing at least nine values
of x, to ensure there one enough points to accurately demonstrate the shape.
x

3 2 1

12

1
4

1
2
1
4

Interactivity
int-2785
Sketching
parabolas

y
9
8
7
6
5
4
3
2
1

Plotting these points gives the graph shown on the right. The points are joined
with a smooth curve.
This graph has a minimum at (0, 0) and forms the basic shape for all parabolas.
In general, the form of a quadratic function is y = ax2 + bx + c, and for now
we need consider only positive values of x.

y = x2

4 3 2 1 0 1 2 3 4
1

WORKED EXAMPLE 1

Consider the quadratic function y = x2 4x + 7.


a Complete the table of values below.
x
y

b Graph the function for x 0.


c State the minimum value of y = x2 4x + 7.
THINK

Method 1: Technology-free
a Substitute each value of x into the function.

WRITE

x
y

0
7

1
4

2
3

3
4

4
7

5
12

Chapter 12 Modelling non-linear relationships 329

b 1 Plot the points generated by the table of values.


2

Join the points plotted with a smooth curve.

y
12
11
10
9
8
7
6
5
4
3
2
1
1 0
1

c The minimum value is the y-value at the point where the

graph turns.

1 2 3 4 5

c For y = x2 4x + 7,

minimum value= 3.

Method 2: Technology-enabled
1

From the MENU select GRAPH.

Delete any existing equation and enter


Y1 = X2 4X + 7.

Press !3 [V-Window]. In this course you will


not need to consider negative values for x. Enter the
setting shown on the screen at right.

Press w to return to the previous screen, and then


press 6 (DRAW) to draw the graph.

Press !5 [G-Solv], followed by 3 (MIN).


This will find the minimum point and display the
coordinates of that point. Be patient: this may take a
moment.

Notes:
1. When setting the view window you do not have to get the limit right the first time. It may take a bit of
trial and error, especially with the y-values to make sure that you have the minimum (or maximum)
point in your display.
2. Any question that has a negative value of x2 (such as Worked example 2) will be concave downwards
and as such will have a maximum point and not a minimum point. In step 5 after pressing !5
[G-Solv] you will need to press 2 (MAX).

330 Maths Quest HSC Mathematics General 2

Shape of a parabola
For quadratic functions that have a positive x2 term, the parabola is concave up. This means that the
graph has a minimum point. When the x2 term is negative, the graph is concave down and the graph has
a maximum point.
WORKED EXAMPLE 2

Graph the function y = 1 + 4x x2.


THINK
1

Draw a table of values.

Substitute each value of x into the function.

WRITE
Tutorial
int-2432
Worked example 2

Plot the points formed by each pair of coordinates.

Join the points with a smooth curve.


Note: For this function, the maximum value is 5.

5
4
3
2
1
1 0 1 2 3 4 5
1
2
3
4
5

Quadratic models can be applied to solve several practical situations.


WORKED EXAMPLE 3

A ball is thrown in the air. Its height, h, after t seconds can be given by the formula h = 20t 5t2.
Graph the function to calculate the maximum height the ball will reach.
THINK
1

Draw a table of values.

Substitute the values of t to calculate the


corresponding values of h.

WRITE

15

20

15

25

Plot the points formed by each pair of coordinates.


Negative values of h can be ignored because height
must be positive.

20

Join the points formed with a smooth curve.

12

h
16

8
4
1 0
1

The maximum height reached by the ball will be the h


value at the turning point on the curve.

1 2 3 4 5

The maximum height reached by the


ball is 20m.

Chapter 12 Modelling non-linear relationships 331

Exercise 12A

Quadratic functions

1 WE1 For the quadratic function y = x2 2x + 3:


a copy and complete the table of values below
Digital doc
EXCEL Spreadsheet
doc-1392
Graphing
quadratics

x
y

b draw the graph of the function


c state the minimum value of x2 2x + 3.

2 For the quadratic function y = x2 4x 2, draw up a table of values and use the table to draw the

graph of the function for x 0.

3 Graph each of the following functions for x 0.


a y = x2 6x + 5
b y = x2 + x + 5

c y = (x 2)2

4 On the one set of axes, graph the following quadratic functions for x 0.
1
a y = x2
b y = 2x2
c y = 2 x2

5 On the one set of axes, graph each of the following quadratic functions for x 0.
a y = x2
b y = x2 + 2
c y = x2 3
6 Use your answers to questions 5 and 6 to answer the following.
a Describe the effect a coefficient of x2 has on the graph of a quadratic function.
b Describe the effect adding a constant term has on the graph of a quadratic function.
7 Graph the function y = (x 1)2 + 4. Compare this with the graph of y = x2 2x + 5. Explain why

this occurs.

8 Graph the function y = 2 + 2x x2 for x 0.

9 WE2 Graph each of the following functions for x 0.


a y = 4 + 6x x2
b y = 8 x2

c y = (2 x)2

10 MC Which of the following functions is not a quadratic function?


A y = x2 + 5x 4

B y = (x 4)2

C y = (x 2)(x + 2)

D y =

x2
x+2
11 MC The graph drawn on the right could have the equation:
A y = (x 2)2 + 3
B y = (x 2)2 3
2
C y = 4 (2 x)
D y = (2 x)2 3
12 MC Which of the following functions will produce the

same graph as y = (x 4)2 + 3?


A y = x2 4x 1
B y = x2 4x + 19
C y = x2 8x 1
D y = x2 8x + 19

13 Graph the quadratic function y = 2x2 4x + 8 for x 0.


14 WE3 An object dropped from a height falls to Earth

according to the equation d = 5t 2, where d is the distance


fallen in metres and t is the time in seconds since the
object was dropped.
a Draw the graph of d against t.
b How far will the object fall in 4 seconds?
c How long will it take for an object to fall a distance
of 500m?

15 The height of a ball in metres that is thrown vertically

upwardsis given by the equation h = 30t 5t2, where


tistimein seconds.
a Draw the graph of h against t.
b Find the maximum height reached by the ball.
c Find the length of time taken for the ball to
return toEarth.

332 Maths Quest HSC Mathematics General 2

16 A rectangular field is to be made out of 100m of fencing. If the length of the field is x metres:
a show that the width of the field is (50 x) metres
b show that the area is given by the quadratic function A = 50x x2
c draw the graph of the function
d find the maximum area of the field and what dimension the field must be to give the maximumarea.
17 Another rectangular field is to be built with 100m of fencing using a

river as one side of the field as shown on the right.


a Show that the area of the field can be given by the equation
A = 100x 2x2.
b Graph the function.
c Calculate the dimensions of the field so that the area of the field
is a maximum.

River
x
100 2x

Further development
18 State whether each of the following graphs is wider or narrower than the graph of y = x2, and state

the coordinates of the turning point of each one.


1
a y = 5x2
b y = 3 x2
2
5

f y = 0.25x2

e x2

c y = 7x2

d y = 10x2

g y = 1.3x2

h y = !3x2

19 In each of the following state whether the graph is wider or narrower than y = x2 and whether it has a

maximum or a minimum turning point.


a y = 3x2
b y = 3x2
e y = 4x2
i y =

f y = 0.25x2

4 2
x
3

j y =

1 2
x
5
0.16x2

d y =

k y =

l y = !11x2

g y = !3x2

200x2

c y = 2 x2

"5x2

h y =

20 The distance, d, of a rocket from a satellite is given by the equation d = 5t2 100t, where t is the

number of hours since the rocket was launched. At what value of t will the rocket reach the satellite?
21 Julie breeds sea monkeys. The number of sea monkeys, N, in Julies tank is found to follow the
equation N = 0.0751h2 + 0.69h + 200, where h is the number of hours since the tank was supplied
with food and stocked with sea monkeys.
a How many sea monkeys were there initially (i.e. at h = 0)?
b Copy and complete the table below.
h
N

10

15

20

30

50

c By drawing a graph estimate to the nearest hour,

how long after being fed could the colony survive


without further food before none were left?
22 Matilda is being pushed on a swing in her backyard.
The swing follows the path given by the formula
h = 14 (x2 3x + 4), where h metres is the
height of the swing above the ground, x metres
from the point where Matilda was pushed.
a Find the height above the ground at the point
where Matilda was first pushed.
b Find the lowest distance that Matilda comes to
the ground.

Maximising areas
1. Sketch ten rectangles that each have a perimeter of 40m.
2. Show the length, width and area of each rectangle in a table.
3. If the length of the rectangle is x:

(a) explain why the width of the rectangle will be 20 x


(b) write a quadratic equation for the area of the rectangle.
4. Use a spreadsheet or graphics calculator to graph your function.
5. What is the maximum area of the rectangle?
Chapter 12 Modelling non-linear relationships 333

Cubics, hyperbolas and


exponentialfunctions

12B
interactivity
int-1149
Exponential graphs

When considering modelling situations it is useful to be familiar with other non-linear graphs.

Cubic functions

A cubic function has the independent variable (x) raised to a power of 3. Its equation is of the form y = ax3.
WORKED EXAMPLE 4

Graph the function y = 2x3.


THINK
1

Draw a table of values.

Substitute values of x to find the


corresponding values of y.

Plot the points generated by the table.

Join the points with a smooth curve.

WRITE

x
y

0
0

1
2

2
16

3
54
y
100
80
60
40
20
0

y = 2x 3

1 2 3 4

Hyperbolas

a
The equation of a hyperbolic function is of the form y = , where a is a constant. For hyperbolas, x 0,
x
and so for now we graph only values of x > 0. As the value of x increases, the value of y will decrease and
vice-versa, and therefore we need to look at values close to x = 0 and y = 0 when creating our table of values.
As x becomes very large, the graph approaches the x-axis but never touches it. As x becomes very
small (approaches 0), the graph approaches the y-axis, but never touches it. The line x = 0 (the y-axis) is
a vertical asymptote, and the line y = 0 (the x-axis) is a horizontal asymptote.
(An asymptote is a line or a curve which a curve approaches but never touches.)
WORKED EXAMPLE 5

2
Graph the function y = .
x
Tutorial
int-2433
Worked example 5

THINK

WRITE

Draw a table of values.

1
4

1
2

Substitute the x-values into the equation to


find the corresponding y-values.

2
3

1
2

Plot each pair of coordinates


generated by the table.

Join each point with a smooth curve.

As y is never actually equal to zero


thex-axis is an asymptote. As x 0 the
y-axis is an asymptote.

y
9
8
7
6
5
4
3
2
1
0

334 Maths Quest HSC Mathematics General 2

x
y= 2

1 2 3 4

Exponential graphs
An exponential function is of the form y = ax or y = b(ax), where a and b are both constants. It is called
an exponential function because the independent variable (x) is in the exponent. An exponential graph
can increase rapidly.
WORKED EXAMPLE 6

Graph the function y = 2x.


THINK
1

Draw a table of values.

Substitute values of x to find the


corresponding valuesofy.

Plot the points generated by the table.

Join the points with a smooth curve.

The x axis is an asymptote since y is


never actually equalto zero.

WRITE

x
y

0
1

1
2

2
4

3
8

4
16
y
20
16
12
8
4
0

y = 2x

1 2 3 4 5

An exponential function of the form y = b(ax) represents an example of exponential growth. These
functions may, for example, show the growth of an investment over a period of time. In examples where
the value of a is between 0 and 1, the function could model exponential decay. An example of this is the
depreciation of an asset over time.
WORKED EXAMPLE 7

Glenn invests $10000 at 8% p.a. with interest compounded annually. The growth of this investment
can be given by the exponential function A = 10000(1.08)n, where n is the number of years of the
investment and A is the amount to which the investment grows. Graph the growth of this investment.

Draw a table of values.

Substitute values of n to find the


corresponding values of A.

Plot the points generated by the table.

Join the points with a smooth curve.

WRITE

n
A
n
A

1
2
3
4
5
10800 11664 12597 13605 14693
6
7
8
9
10
15869 17138 18509 19990 21589

35 000

A = 10 000 (1.08)n

30 000
Investment ($)

THINK

25 000
20 000
15 000
10 000
5000
0
0

10
15
Number of years

20

Chapter 12 Modelling non-linear relationships 335

Exercise 12B

functions

Cubics, hyperbolas and exponential

1 WE4 Graph the cubic function y = x3 for x 0.

2 Graph the following functions for x 0.


Digital doc
EXCEL Spreadsheet
doc-1394
Function grapher

a y = 3x3

b y = 2x3

c y = x3

4
for x > 0.
x
4 WE5 Graph each of the following functions for x > 0.
10
1
a y =
b y =

x
x
x
5 Graph the function y = 3 .
3 Graph the hyperbolic function y =

c y =

6 WE6 Graph each of the following functions.


a y = 4x

7 Graph the function y = 5(2x).

1
x

1 x

b y = 10x

c y = Q 2 R

8 MC The equation of the graph shown on the right could be:


A y = x3

B y = 3x

C y = 3x

D y =

3
x

2
x

9 MC Which of the graphs shown below could be the graph of y = ?


A

x
C

10 WE7 Ming Lai invests $1000 at 10% p.a. interest with interest compounded annually. This

investment can be represented by the function A = 1000(1.1)n, where A is the amount to which the
investment grows and n is the number of years of the investment. Draw the graph of the function.

11 Kevin invests $50000 at 12% p.a. interest, compounded annually.


a Write an equation for the amount, A, to which the investment will grow in terms of the number of

years of the investment, n.


b Graph the function.
c Use the graph to estimate the amount of time that it will take for the investment to reach $70000.
12 A new car is purchased for $40000. The car depreciates at the rate of 15% p.a. The value, V, of the

car after a number of years, n, can be given by the equation V = 40000(0.85)n. Graph this function.

Further development
13 Sketch the graph of each of the following for x 0.
a y = x3 + 4
b y = x3 1

c y = 1 8x3

14 Sketch the graph of each of the following for x 0. Show the horizontal asymptote on your sketch

by drawing a broken line in red. Label this line with its equation.
1
1
a y = + 1
b y =
x
x+1
15 Find the equation of the asymptote and the y-intercept for each of the following. Hence, sketch the
graph of each and state its domain and range.
a y = 2x 1
b y = 3x + 2
c y = 2x + 3
336 Maths Quest HSC Mathematics General 2

Compound interest
The amount to which an investment will grow under compound interest can be found using the following
formula:
A = P(1 + r)n
Consider an investment of $10000 at an interest rate of 8% p.a.
1. If interest is compounded annually, the amount to which the investment will grow can be given by
the function A = 10000(1.08)n, where n is the number of years. Graph this function using graphing
software or a graphics calculator.
2. If interest is compounded six-monthly, the function becomes A = 10000(1.04)2n. On the same set of
axes graph this function.
3. Write a function that will show the amount to which the investment will grow if interest is
compounded quarterly, and graph this function on the same set of axes.
4. Use the graphs drawn to describe the overall effect of shortening the compounding period.

12C

Direct variation

In the previous chapter we studied direct linear variations. In this section we will study direct
non-linearvariations.
From our work on measurement we know that the area of a circle is given by the formula A = r2,
whereA is the area and r is the radius of the circle.
This is an example of a quantity (area) that varies in proportion with the power of another quantity
(radius). This can be written as A r2. The symbol means in proportion to. In this example is the
constant of variation, that is, the amount by which r2 must be multiplied to calculate the area.
An equation of the form y = ax2 or y = ax3 can be used to model several variations. In such cases we
may need to calculate the constant of variation from some known or given information.
WORKED EXAMPLE 8

It is known that y varies directly with the cube of x. It is known that y = 24 when x = 2. Write an
equation connecting the variables x and y.
THINK

WRITE

Write a proportion statement.

y x3

Insert a constant of variation (k) to form an equation.

y = kx3

Substitute the known values of x and y to find the


value of k.

When x = 2, y = 24.
24 = k 23
= 8k
k=3

Replace the known value of k in the equation.

y = 3x3

Once we have calculated the constant of variation, we are able to calculate one quantity given the other.
WORKED EXAMPLE 9

The surface area of a cube varies directly with the square of the length of the cubes edge.
a A cube of edge length 5.5cm has a surface area of 181.5cm2. Find the constant of variation.
b Find the surface area of a cube with an edge length of 7.2cm.
THINK

a 1 Write a proportion statement choosing

pronumerals s and e.

WRITE

a s e2

Insert the constant of variation, k, to form an equation.

s = ke2

Substitute known information.

When e = 5.5, s = 181.5


181.5 = k 5.52
Chapter 12 Modelling non-linear relationships 337

Calculate 5.52.

Solve the equation (divide by 30.25).

b 1 Rewrite the proportion statement with k = 6.

181.5 = k 30.25
k=6
b s = 6e2

Substitute e = 7.2.

When e = 7.2,
s = 6 7.22

Calculate s.

s = 311.04

Give a written answer.

The surface area of a cube with an edge


of 7.2cm is 311.04cm2.

WORKED EXAMPLE 10

Research conducted by a physiotherapist has determined that the height-to-mass rate (in cm/kg)
of adult males is 2.26. Use this information to predict:
a the height of a 70-kg adult male
b the mass of a 180-cm adult male. (Round answers to 1 decimal place.)
THINK

a 1 Define the variables.

WRITE

a Let h cm be the height of an adult male.

Let w kg be the mass of an adult male.


h
= 2.26
w
h = 2.26w

Use the given rate to find the height.

Use the rule to predict the height.

If w = 70,
h = 2.26 70
= 158.2

Answer the question in a sentence.

The height of an adult male with a


mass of 70kg is about 158.2cm.

b 1 Use the rule to predict the mass.

Answer the question in a sentence.

b If h = 180,

180 = 2.26w
w = 79.6
The mass of an adult male who is
180cm tall is about 79.6kg.

WORKED EXAMPLE 11

A new car has a fuel consumption of 7.2 L/100km (this means it requires 7.2 L of petrol to
travel100km).
a How much fuel is required for a journey of 1134km?
bThe previous model of the same car had a fuel consumption of 7.8 L/100km. Which model
ismore economical to run? (Round answers to 2 decimal places.)
THINK

a 1 Define the variables.

WRITE

a Let L be the number of litres of fuel.

Let d km be the distance travelled.


2

Find the rate relating the variables.

338 Maths Quest HSC Mathematics General 2

L 7.2
=
d 100
L
= 0.072
d

Use the rate to find the rule relating the number of


litres and the distance travelled.

L = 0.072d

Use the rules to find the number of litres required


for a journey of 1134km.

If d = 1134,
L = 0.072 1134
= 81.648

Answer the question in a sentence.

It takes about 81.65L of fuel to travel


1134km.

b Select the car which uses less fuel per 100km.

b 7.2 L < 7.8 L

The newer model is more economical


to run.

WORKED EXAMPLE 12

If the distance, d km, travelled by a person varies directly as the time, t hours, and it is known
that the person travelled 12km while walking for 2.5 hours, find:
a how far he will travel in 3 hours
b how long he must walk in order to travel 6.72km.
THINK

WRITE

a 1 Write the rule for k. Since d t, then d = kt and

d
hence k = .
t

Substitute the given values for d and t into the


equation and solve for k.

Substitute t = 3 into the equation

Transpose the equation to make d the subject.

Answer the question and include the appropriate unit.

d
= 4.8.
t

b 1 Substitute d = 6.72 into the equation obtained in

d
part a; that is, = 4.8.
t

k=

Tutorial
int-1057
Worked
example12

d
t

12
2.5
= 4.8
The constant of variation is 4.8. Therefore,
d
= 4.8
t
d
When t = 3, = 4.8
3
k=

d = 4.8 3
= 14.4
He will travel 14.4km in three hours.
d
t
6.72
When d = 6.72,
= 4.8
t

b From part a: = 4.8

6.72 = 4.8 t
4.8t = 6.72
6.72
t=
4.8
= 1.4

Transpose the equation to make t the subject.

Convert the answer to hours and minutes by


multiplying the decimal part of the answer by 60.

1.4 hours = 1 h + (0.4 60) min



= 1 h 24 min

Answer the question.

In order to travel 6.72km, he must


walk for 1 h 24 min.

Chapter 12 Modelling non-linear relationships 339

Exercise 12C

Direct variation

1 WE8 It is known that y varies directly with the square of x. If y = 88 when x = 4, write an equation

connecting y with x.

2 It is known that b varies directly with the cube of a. When a = 6, b = 108. Write an equation

connecting b with a.

3 It is known that the distance, d, an object will fall varies directly with the square of the time, t, it has

been falling. An object that has been falling for 2 seconds falls a distance of 19.6 metres.
a Write an equation connecting d with t.
b Graph the relationship between d and t.
4 WE9 The surface area of a cube varies directly with the square of its side length.
a A cube of side length 15cm has a surface area of 1350cm2. Find the constant of variation.
b What is the surface area of a cube that has a side length of 6.2cm?
5 The area of a circle varies directly with the square of its radius.
a If the area of a circle with radius 6cm is 113.1cm2, find the constant of variation. (Give your

answer correct to 2 decimal places.)


b What is the area of a circle with a radius of 12cm?
6 The mass of an egg varies directly as the cube of

the eggs length.


a An egg of length 5cm has a mass of 31.25g.
Find the constant of variation.
b What will be the mass of an egg with a length
of 6cm?
c If an egg has a mass of 70g, what would be
the eggs length? (Give your answer correct to
1 decimal place.)
7 a 
WE10 In a study of a group of adult women, it was

found that the height-to-mass rate (in cm/kg) is 2.48.


Use this information to predict:
i the height of a 60kg woman in this group. (Give your answer correct to 1 decimal place.)
ii the mass of a 170cm woman in this group. (Give your answer correct to 1 decimal place.)
b Find the height-to-mass rate if the height is measured in metres.
8 WE11 A new sports car has a fuel consumption of 10.5 L/100km (it requires 10.5 L of petrol to

travel 100km).
a How much fuel is required for a journey of 5430km?
b A Nissan Pulsar has a fuel consumption of 9 L/100. Which of the cars is more economical
to run?
9 In Pear Fisher Bay, land can be purchased for a price of $5.50 per square metre.
a How much land can be purchased for $10000?
b What would be the cost of a block of land of 6500 square metres?

Further development
10 A large computer company can hire graduate computer programmers for a salary of $40000 per

year, or experienced professional programmers at $55000 per year.


a If there is a budget of $480000, how many:
i graduates could be hired
ii professionals could be hired?
b How many professional programmers are equivalent (in salary) to 23 graduate programmers?
11 An architect determines that all the windows in a new building will have a height-to-width ratio

of10 : 7 or 10
.
7
a Determine a rule relating height and width.
b If a window is 60cm wide, how high is it?
c If a window is 100cm high, how wide is it?

12 The top gear ratio on a bicycle is 7 to 2. If the larger sprocket contains 140 teeth, how many teeth

does the smaller sprocket contain?


340 Maths Quest HSC Mathematics General 2

13 WE12 An experienced cyclist can travel at an average speed of 26km/hour.


a How far can she travel in 24 hours?
b How long will it take her to travel 1000km? (Give your answer correct to the nearest 10 minutes.)
c If she rests 1 hour after every 4 hours of travel, how long will it take her now to travel 1000km?
14 A large four-wheel drive has a fuel consumption of 12.15 L/100km, while for a small car it is

5.7L/100km.
a How many litres of fuel will be used by each vehicle for a 674km journey? Give your answers to
1 decimal place.
b How far could the small car go on the same fuel that the four-wheel drive used to travel 1000km?
Give your answer correct to the nearest kilometre.
15 An aeroplane uses 600 L of fuel (its full tank) for a journey of 1250km.
a Find the fuel consumption ratio.
b If an additional 800 L can be stored in an extra tank, what is the farthest distance that the

Digital doc
doc-11103
WorkSHEET 12.1

aeroplane can travel? Give your answer correct to the nearest 10km.

12D

Inverse variation

Consider the following example. Stan used to collect basketball cards. Eventually he became bored
with this hobby and decided to give all of his 120 cards to his classmates. If Stan distributed the whole
collection between his 2 best friends, Mark and Eugene, they would each receive 60 cards. If he included
another friend, Ashley, they would each receive 40 cards and so on. The more people who shared Stans
collection, the fewer cards each person received. There are 25 people in Stans class, including himself.
So if he were to distribute 120 cards between all of his classmates, each student would receive 5 cards.
This information can be represented graphically or as shown in the table.
n

10

12

15

20

24

120

60

40

30

24

20

15

12

10

(Note that only factors of 120 are included in order to avoid fractional
C
answers).
120
It is obvious that as the number of students, n, who are to share the
collection increases, the number of cards, C, that each student
receives, decreases.
The product of the two variables is constant for each pair and equal
60
to 120 the size of the collection. That is: 1 120 = 2 60 =
40
3 40 = 4 30 = 5 24 = 6 20 = 8 15 = 10 12 = 120 and
30
20
so on.
10
Hence, the relationship between two variables can be written as:
123456 8 10 12 15 20 24 n

C n = 120, or
120

C=
.
n
The graph of the relation is a hyperbola which has the C and n axes as its asymptotes.
Summarising our observations, we can say that the following is true for the given information:
1. An increase in one variable causes a decrease in the other.
2. The product of the two corresponding variables is constant.
3. Neither variable is equal to 0.
4. The graph which represents the data is a hyperbola.
1
If we calculate the values of for each of the values in our table, we will then be able to draw a graph
n
1
of C against .
n
n

10

12

15

20

24

1
n

1
2

1
3

1
4

1
5

1
6

1
8

1
10

1
12

1
15

1
20

1
24

120

60

40

30

24

20

15

12

10

Chapter 12 Modelling non-linear relationships 341

1
C
is a straight line directed
n
120
from, but not passing through, the origin. (Note that we exclude
the origin itself, hence the open circle at (0, 0), since the number
of cards per person when shared between 0 students is
undefined.)
60
1
Hence, we can deduce that C varies directly as , that is, as the
n
reciprocal of n.
30
In cases like this, we say that one variable varies inversely as
5
(or is inversely proportional to) the other. The product of any two
1
1
1
1
0
1 n
24
4
2
corresponding variables is constant and is called a constant of
proportionality, k.
Hence, C is inversely proportional to n (or C varies inversely as n, or directly as the reciprocal
1
ofn).Itis written as C .
n
Here, the product of any two corresponding values of C and n is constant and equal to 120, that is
Cn = 120. Therefore the constant of proportionality k = 120.
120
Therefore, the relationship between the two variables can be written as C =
.
n
1
Generally, for any two variables x and y, where y varies inversely as x, that is, y , there exists
x
k
a relationship between them such that y = or yx = k, where k is a constant, called the constant of
x
proportionality (or the constant of variation). The graph of the relationship is a hyperbola whereas
1
the graph of y against is a straight line directed from, but not passing through, the origin, and
x
having thegradient k (where x 0). As with direct variation, the existence of inverse variation canbe
established either numerically, or graphically.
Summarising this:
As you can see, the graph of C versus

If y

1
x

k
x
where k is the constant of proportionality and x R\{0}.
then y =

WORKED EXAMPLE 13

y varies inversely with x, and y = 10 when x = 2.


a Find the constant of proportionality, k, and hence the rule relating x and y.
bPlot a graph of the relationship between x and y, for values of x which are positive factors of
kless than 11.
THINK

a 1 Write the relationship between the variables

using the symbol .

Rewrite as an equation using k, the constant of


variation.

Substitute the given values of the variables and


find the value of k.

Write the rule relating the variables.

b 1 Set up a table of values for x and y, taking

values for x that are positive factors of k so that


only whole number values of y are obtained.

342 Maths Quest HSC Mathematics General 2

WRITE

a y

1
x

y=

k
x

k
2
k = 20
20
y=
x
10 =

10

20

10

Plot the points on a clearly labelled set of axes


and join the points with a smooth curve.

y
25
20
15
10
5
0

10

WORKED EXAMPLE 14

When a force is applied to a certain object, its acceleration varies inversely as its mass. When the
acceleration of an object 12 m/s2, the corresponding mass is 3kg.
a Find the constant of proportionality.
b Find the rule relating acceleration and mass.
c Find the acceleration of a 1.5kg object.
d Find the acceleration of a 6kg object.
THINK

WRITE

a 1 Define the variables.

a Let the mass of the object be m kg.

Let the acceleration be a m/s2.


1
a
m

Write the relationship between the variables


using .

Rewrite as an equation using k, the constant of


proportionality.

a=

Substitute the given values of the variables and


find the value of k.

12 =

b Write the rule by substituting the value of k into

the equation.
c 1 Substitute the value of the mass into the

equation to find the acceleration.

Write the answer in a sentence.

d 1 Substitute the value of the mass into the

equation to find the acceleration.

Write the answer in a sentence.

Exercise 12D

Tutorial
int-2434
Worked
example14

k
m

k
3
k = 36
The constant of proportionality is 36.
b a=

36
m

c If m = 1.5,

36
1.5
= 24
a=

The acceleration is 24 m/s2.


d If m = 6,

36
6
=6
a=

The acceleration is 6 m/s2.

Inverse variation

1 WE13 y is inversely proportional to x, and y = 100 when x = 10.


a Find the constant of proportionality, k, and hence the rule relating x and y.
b Plot a graph of the relationship between x and y for values of x that are positive factors of k less

than 21.
Chapter 12 Modelling non-linear relationships 343

2 p varies inversely as q, and p = 12 when q = 4.


a Find the constant of proportionality, k, and hence the rule relating p and q.
b Plot a graph of the relationship between q and p for values of q which are positive factors of

k less than 11.


3 y varies inversely as x and y = 42 when x = 1.
a Find the constant of proportionality, k, and hence the rule relating x and y.
b Plot a graph of the relationship between x and y for values of x from 1 to 10.
4 It is known that y varies inversely with x. When y = 10, x = 5; write an equation connecting y

with x.

5 It is known that m varies inversely with n. When m = 0.5, n = 2; write an equation connecting

m and n.

6 The time taken, t, to travel between two points varies inversely with the average speed, s, for the

trip.If the journey takes 2.5 hours at 60km/h:


a write an equation that connects t with s
b graph the relationship between t and s.
7 The time, t, taken to dig a trench varies inversely with the number of workers, n, digging. It takes

four workers 5 hours to dig the trench.


a Find the constant of variation.
b How long would it take 10 workers to dig the same trench?
8 The fuel economy, f, of a car varies inversely

with the speed, s, at which it is driven. A car


that averages 40km/h has a fuel economy of
15km/L. What will be the fuel economy of
a car that averages 50km/h?
9 In an electricity circuit, the current (measured

in amps, a) is inversely proportional to the


resistance (measured in ohms, r). When the
resistance is 40 ohms, the current is measured
at 3 amps. What will be the current when the
resistance is 15 ohms?
10 WE14 When a force is applied to a certain object, its acceleration varies inversely as its mass.

Whenthe acceleration of an object is 40 m/s2, the corresponding mass is 100kg.


a Find the constant of variation.
b Find the rule relating mass and acceleration.
c Find the acceleration of a 200-kg object.
d Find the acceleration of a 1000-kg object.

Further development
11 The number of colouring pencils sold varies inversely

with the price of each pencil. Two thousand pencils are


sold at the price of $0.25 each.
a Find the constant of proportionality.
b Find the number of pencils that could be sold for
$0.20 each.
c Find the number of pencils that could be sold for
$0.50 each.
12 The time taken to complete a journey is inversely

proportional to the speed travelled. A trip is completed in


4.5 hours when travelling at 75km per hour.
a Find the constant of variation.
b Find how long (to the nearest minute) the trip would
take if the speed was 85km per hour.
c Find the speed required to complete the journey in 3.5 hours.
d Find the distance travelled in each case.
344 Maths Quest HSC Mathematics General 2

13 The cost per person travelling in a charter plane varies

inversely with the number of people in the charter group.


It costs $350 per person when 50 people are travelling.
a Find the constant of variation.
b Find the cost per person if there are 75 people travelling.
c Find how many people are required to reduce the cost to
$250 per person.
d Find the total cost of hiring the charter plane.
14 The electrical current in a wire varies inversely with the

resistance of the wire to that current. There is a current of


10 amps when the resistance of the wire is 20 ohms.
a Find the constant of variation.
b Find the current when the resistance is 200 ohms.
c Find the resistance of the wire when the current is 15 amps.
15 The time taken to complete a long computer program is

inversely proportional to the speed of the computers processor.


If the program can be completed in 1 minute when the
processor speed is 200mHz, find:
a the constant of variation
b the time taken when the processor speed is halved
c the processor speed required to complete the program in
20 seconds.
16 The time taken to complete a large building project varies

inversely with the number of workers. If the building can be


completed in 140 days with 75 workers, find:
a the constant of variation
b the time taken to complete the building with 50 workers
c the number of workers required to complete the building
within 100 days.

12E

Graphing physical phenomena

In many cases, an algebraic function can be used to describe a physical


situation. Consider the case of a sphere of radius r. The volume of a sphere
can be given by the formula V = 43r3. We can create a table of values that
allows us to graph the function for volume.
0

10

4.19

33.51

113.10

268.08

523.60

4189

3500
3000
Volume

4000

2500
2000
1500
1000

We can then plot each pair of points from the table and join the points
with a smooth curve. The graph shown at right shows the relationship
between volume for a sphere and the length of its radius.

4 r3
V =
3

500
0

5 10 15
Length of the radius

WORKED EXAMPLE 15

The surface area of a sphere is given by the formula A = 4r2.


a Complete the table below.
r

10

A
b Graph the surface area function.

Chapter 12 Modelling non-linear relationships 345

THINK

WRITE

a Substitute each value

of r into the formula.

b 1 Plot each pair of

0 12.57 50.27 113.10 201.06 314.16 452.39 615.75 804.25 1017.88 1256.64

10

points generated
by the table.

3000

Join the points


with a smooth
curve and
extrapolate the
graph.

2000

A = 4r 2

2500
Area

1500
1000
500
0
0

10
Radius

15

Many graphs have physical restrictions placed on them. Consider the case
h
h = 15t 5t 2
of a ball that is thrown vertically upwards. The height, h, of the ball at any
12
time, t, can be given by the equation h = 15t 5t2. The height of the ball
9
must always be positive, and when the ball returns to Earth we can consider
6
the height to be zero and so the graph stops as shown on the right.
When we graph several points, we try to estimate other values by
3
interpolating (estimating values between given points by drawing the
0
t
1
2
3
4
graph joining the points) or extrapolating (estimating values by extending
the graph beyond the points given).
Other graphs need to have restrictions placed upon them when we try to interpolate or extrapolate.
There may be a limit placed upon one or both of the variables, and this will indicate a change in the graph.
WORKED EXAMPLE 16

Tutorial
int-2435
Worked
example16

A cinema owner believes that more people will attend the movies on cold days and so believes the
number of people attending each session of a movie varies inversely with the temperature of the day.
When the temperature is 15 C, 80 people attend a movie. The cinema has a maximum of 120 seats,
and the cinema owner believes that a minimum of 40 people will attend, regardless of temperature.
aWrite an equation connecting the number of people attending the movie, N, with the
temperature, T.
b Graph the relationship between attendance and temperature.
THINK

a 1 Write an inverse proportion statement.

WRITE

1
T
k
N=
T

a N

Insert a constant of variation, k, to form an


equation.

Substitute the known values of N and T to


find the value of k.

When T = 15, N = 80
k
80 =
15
k = 1200

Replace the known value of k in the


equation.

N=

346 Maths Quest HSC Mathematics General 2

1200
T

b 1 Create a table of values.

Substitute each value of T into the


equation.

If the value of N > 120, then we enter 120


for N (maximum attendance); if N < 40,
enter 40 for N (minimum attendance).

T
N

5
120

10
120

15
80

20
60

25
48

30
40

35
40

N
120
100
80
60
40
20
0

10

20

30

40

Plot the points and join with a smooth


curve. The minimum and maximum
attendance is drawn with a straight line.

Exercise 12E

Graphing physical phenomena

1 WE15 The surface area of a cube is given by the formula A = 6s2.


a Complete the table of values below.

A
b Draw the graph to represent the surface area of a cube of a given side length.
2 The distance that an object will fall when dropped from a height can be given by the formula d = 5t2,

where d is in metres and t is in seconds. Draw a graph of the function.

3 A car is travelling at v km/h and the driver needs to brake. It takes 2.5 seconds to react and in that

time the car will travel a distance of 0.7v m. The total stopping distance, d, can be given by the
function d = 0.01v2 + 0.7v.
a Copy and complete the table below.
v

10

20

30

40

d
b Draw the graph of the stopping distance of a vehicle.
4 WE16 Lorraine organises a lottery syndicate at her work. If they win a prize of $100000, the

amount is shared equally between the members of the syndicate. There must be at least one member
of the syndicate and a maximum of 10.
a Write an equation putting the amount, A, each person receives in terms of the number of
members, n.
b Graph the function.
5 A car is purchased new for $40000. After one year the depreciated value of the car is $30000. After

the first year the car depreciates at a rate of 20% p.a.


a Copy and complete the table below.
Age (years)

Value
b The car will always be worth a minimum of $2000 in scrap metal and accessories. Graph the

value of the car against the age of the car.


Chapter 12 Modelling non-linear relationships 347

6 The mass of a newborn baby increases by

20% per month for the first four months of life.


If the average mass of a newborn baby is 3.3kg,
graph the mass function up to n = 4.
7 A square piece of sheet metal has a side length

of 12 m. A square of side length x m is to be cut


from each corner of the sheet metal and the
sides bent up to form an open rectangular prism.
12 m
xm

a What is the maximum possible value of x?


b Show that the volume of the prism formed can

be given by the function V = x(12 2x)2.


c Graph the volume function.

8 The population of a city is growing at a rate of

5% p.a. If the population in 2007 is 1.5 million,


the population function can be given by the
function P = 1.5(1.05)n, where P is the population,
in millions. The city cannot sustain a population
greater than 4000000.
a Complete the table below.
Year

2007

2008

2009

2010

2011

2020

2027

Population (million)
b Plot the points given and extrapolate to graph the population function.
c Use your graph to state when the population will reach its maximum sustainable level.
d What will happen to the graph when it reaches this level?

Further development
9 The following points represent a variation of the form y = kx2.

12

27

48

75

Find the value of k.


10 The Safety Council conducted research on the breaking distance of vehicles and its relationship to

the speed of the vehicle. The following data were obtained.


Speed (s) (km/h)

30

45

60

80

100

Breaking distance (d) (metres)

7.5

16.9

30

53.3

83.3

Find the equation relating d and s?


11 In an electrical circuit the current (I) flowing through a resistor for different resistance is shown in

the table below.


Resistance (R) (ohms)

100

200

1000

1500

Current (I) (milliamps)

300

150

30

20

Deduce a relationship between I and R.


348 Maths Quest HSC Mathematics General 2

a
or
x
y = a!x. For each set of data, plot the values of y against x, and hence select the most appropriate
rule and state the value of a.

12 The data in each of the tables below exactly fit one of these rules: y = ax2, y = ax3, y =

0.3

2.4

8.1

8.1 2.4 0.3

24

x
y

0
0

0.5
1.13

1
1.6

x
y

1
5

2
2.5

4
1.25

5
1

10
0.5

3
40.5

2
12

1
1.5

24 54

1.5
2
1.96 2.26

1.5 12

13 For her Science assignment, Rachel had to find the relationship between the intensity of the light, I,

and the distance between the observer and the source of light, d. From the experiments she obtained
the following results.
d
I

1
270

1.5
120

2
67.5

2.5
43.2

3
30

3.5
22.04

4
16.88

a Plot the values of I against d. What form of relationship does the graph suggest?
b Nathan (Rachels older brother) is a Physics student. He tells Rachel that from his studies he

a
is certain that the relationship is of the type I = . Use this formula to help Rachel to find the
2
d
model for the required relationship.
14 Joseph is a financial adviser. He is studying the prices of shares of a particular company over the last
10 months.
Months
Price ($)

1
6.00

2
6.80

3
7.45

4
8.00

5
8.50

6
8.90

7
9.30

8
9.65

9
10.00

10
10.30

a Represent the information graphically.


b Establish a suitable mathematical model, which relates the share price, P, and the number of

themonth, m.
c Use your model to help Joseph predict the share price for the next 2 months.

Digital doc
WorkSHEET 12.2
doc-11104

Force of gravity
When an object is dropped, the distance that it will fall in
t seconds can be approximated by the formula d = 5t2.
The coefficient of t2 is half the force of gravity (10 m/s2)
and so will change if an object were to be dropped on another
planet. For example, on the moon this equation would
become d = 0.8t2.
1. Use a graphics calculator or graphing software to graph
the equations for both the Earth and the moon.
2. Find out the force of gravity on other planets and compare
the graphs formed with that for the Earth.

Chapter 12 Modelling non-linear relationships 349

Summary
Quadratic functions

A quadratic function is a function where the independent variable is raised to the power of 2.
The graph of a quadratic function is a parabola, a curved graph with either a minimum
(positive x2 term) or a maximum (negative x2 term).
A quadratic function is graphed by plotting the points formed from a table of at least seven values.

Cubes, hyperbolas
and exponential
functions

A cubic function uses a power of 3 for the independent variable. It is of the form y = ax3.
a
A hyperbola is a function of the form y = . In a hyperbolic function, as one variable increases the
x
other decreases.
An exponential function is of the form y = ax. When a > 1, an exponential function models
exponential growth, while if 0 < a < 1, the function models exponential decay.
Each of these functions is graphed by plotting points from a table of values.

Direct variation

A variation occurs when one quantity changes in proportion with another.


If one quantity varies directly with another, as one increases so does the other.
If the quantity varies directly with the square of the other, it can be expressed as a function in the
form y = ax2. If it varies with the cube of another, it can be expressed in the form y = ax3.

Inverse variation

An inverse variation occurs when one quantity decreases, while the other increases. An inverse
a
variation can be expressed in the form y = .
x
The constant of variation, a, is calculated by using a known quantity of each variable. Once this
hasbeen calculated, if we know one quantity we can calculate the other.

Graphing physical
phenomena

Algebraic models can be used to represent many physical situations.


When graphing physical phenomena, we need to consider any restrictions that may exist on one or
both of the variables.

350 Maths Quest HSC Mathematics General 2

Chapter review
1 Which of the following equations is an example of a quadratic function?

m u lti p l e
c hoic e

2

C 2y = x3 + 1
D x + 2y + 1 = 0
x
2
2 Which of the following quadratic equations is equivalent to y = (x 3) + 7?
A y = x2 3x 2
B y = x2 3x + 16
C y = x2 6x 2
D y = x2 6x + 16
A y = 2x2 + 1

B y =

3 The graph shown below could have the equation:

A y = x2

2
x
C y = 2x
B y =

1 x

D y = Q R
2

4 It is known that y varies inversely with x. The variation can be modelled by the equation:
A y = ax

B y = ax2

C y = ax3

D y =

a
x

S ho rt
a nsw er

1 For the quadratic function y = x2 4x + 5:


a copy and complete the table of values below

x
y

b draw the graph of the function for x 0


c state the minimum value of the function y = x2 4x + 5.
2 For the quadratic function y = x2 2x 2, draw a table of values and use the table to sketch the

graph for x 0.

3 Sketch each of the following quadratic functions for x 0.


a y = (x 4)2
b y = 5 x2

c y = 4 + 2x x2

4 An object is dropped from a height of 500 m. Its height above the ground at any time, t, is given by

the function h = 500 5t2.


a Draw the graph of the function.
b How many seconds does it take for the object to fall to Earth?

5 A team of workers are digging a mine shaft. The number of kilograms of earth moved each hour

bythe team is given by the function E = 24n n2, where n is the number of workers digging
theshaft.
a Graph the function.
b What is the maximum amount of earth that can be moved by the team of workers in one hour?
How many workers are needed to move this amount of earth?
c Explain possible reasons why the amount of earth moved each hour then begins to decrease as
more workers are used.

6 Graph each of the following cubic functions for x 0.


a y = x3

b y = 2 x3

7 Graph each of the following hyperbolic functions for x > 0.

1
2
b y =
x
x
8 Graph each of the following exponential functions.
a y =
a y = 2x

1 x

b y = Q 2 R

9 The average inflation rate is 4% p.a. In 2006 it cost the average family $500 per week in living

expenses. The future cost of living, C, can be estimated using the function C = 500(1.04)n where n is
the number of years since 2006.
a Graph the cost of living function.
b Use the graph to estimate the cost of living in 2016.
c When will the cost of living first reach $1000 per week?
Chapter 12 Modelling non-linear relationships 351

10 If the value of a computer purchased for $5000 depreciates by 20% p.a., the future value of the

computer, V, can be given by the equation V = 5000(0.8)n, where n is the age of the computer, in years.
a Graph the function.
b Find when the value of the computer is approximately $1000.
11 It is known that y varies directly with the square of x. When x = 4, y = 80. Write an equation
connecting x with y.
12 The mass, m, of an egg varies directly with the cube of its length, l. An egg of length 5.5cm, has a
mass of 75 g.
a Write an equation connecting m with l.
b Find the mass of an egg with a length of 5cm.
c Find the length of a 50 g egg.
13 It is known that y varies inversely with x. When x = 8, y = 8; write an equation connecting y with x.
14 The amount of food in a camp varies inversely with the number of people to feed. There is enough
food to feed 100 campers for 10 days.
a Write an equation connecting the amount of food, A, with the number of campers, n.
b Calculate how long the food would last 125 campers.
c If the food lasts for four days, calculate the number of campers.
15 The area of a circle is given by the formula A = r2.
a Complete the table of values below.
r

A
b Draw the graph of A against r.
16 A ball is thrown directly up in the air. The height, h, of the ball at any time, t, can be found using the

equation h = 20t 5t2.


a Draw a graph of the height equation.
b Use the graph to find:
i the maximum height of the ball
ii the time taken for the ball to fall back to earth.
17 An investment of $10000 at 6% p.a. can be modelled using the equation A = 10000(1.06)n, where n
is the number of years of the investment.
a Graph the function.
b Use your graph to estimate the value of the investment after 8 years.
c Use your graph to find the amount of time that it will take for the investment to grow to $15000.
Ext end ed
R espons e

1 As a fundraising activity, a school hires a cinema to show the premiere of a movie. The cost of hiring

the cinema is $500. People are then charged $10 to attend the movie.
a Write a function for the profit or loss made on the movie in terms of the number of people attending.
b Graph the function.
c Use the graph to calculate the number of people who must attend the movie for the school to
break even.
d A rival cinema offers to waive the hire fee but the school will receive only $5 per person
attending. On the same axes graph the function P = 5n.
e The school chose to pay the $500 and receive $10 per person. How many people must attend the
premiere to make this the better of the two options?
2 A rock is thrown from a cliff 20 m above ground level. The height of the rock at any time is given by
the quadratic function h = 20 + 15t 5t2.
a Copy and complete the table below.
t

h
b Graph the function and use your graph to find the maximum height reached by the ball.
352 Maths Quest HSC Mathematics General 2

2
x

3 a On the one set of coordinate axes, sketch the graphs of y = 2x3 and y = .

2
x
4 The growth of an investment made at 8% p.a. can be modelled by the equation y = 1.08x.
a Graph the function.
b Use your graph to determine the amount of time that it will take for the investment to double
invalue.
c The depreciation of an item at 8% p.a. can be modelled by the equation y = 0.92x. Graph this
function.
d Use your graph to determine the amount of time that it will take for the item to halve in value.
b Use your graphs to find the point of intersection of the graphs y = 2x3 and y = .

Digital doc
Test Yourself
doc-11105
Chapter 12

Chapter 12 Modelling non-linear relationships 353

Activities
12A Quadratic functions

12D Inverse variation

Interactivity
int-2785: Sketching parabolas. (page 329)

Tutorial
WE14 int-2434: Calculating using inverse variation. (page 343)

Tutorial
WE2 int-2432: Graphing a quadratic function. (page 331)

12EGraphing physical phenomena

Digital doc
Spreadsheet (doc-1392): Graphing quadratics. (page 332)

Tutorial
WE16 int-2435: Graphing a physical situation. (page 346)

12B Cubics, hyperbolas and exponential functions

Digital doc
WorkSHEET 12.2 (doc-11104): Apply your knowledge of graphing
physical situations. (page 349)

Interactivity
int-1149: Exponential graphs. (page 334)
Tutorial
WE5 int-2433: Graphing hyperbolic functions. (page 334)
Digital doc
Spreadsheet (doc-1394): Function grapher. (page 336)

12C Direct variation


Tutorial
WE12 int-1057: Calculating using variation. (page 339)
Digital doc
WorkSHEET 12.1 (doc-11103): Apply your knowledge of non-linear
relationships to problems. (page 341)

354 Maths Quest HSC Mathematics General 2

Chapter review
Digital doc
Test Yourself (doc-11105): Take the end-of-chapter test to test your
progress. (page 353)

To access eBookPLUS activities, log on to www.jacplus.com.au

Answers chapter 12
Modelling non-linear
relationships
Exercise 12A

1 a

6 a The coefficient will make the graph

Quadratic functions

11

18

steeper if it is greater than 1 and


flattenthe graph if it lies between
0and1.
b Adding a constant will lift the graph
while subtracting a constant will lower
the graph.
7 y

y y = x2 2x + 3
18
15
12
9
6
3
10 1 2 3 4 5 6 x
1

16
12

5 x

9 a 16y
12

15
10
5

y
10

3 4 x

2
1 2

4 y

3 4

20
y = 1_ x2
2
(c)

15
10
5
1

3 4

5 x

5 y y = x2+ 3 y = x2

y = x2 3

9
6

(b)

(a)

(c)
2

3 4

A
600

0 1 2 3 4 5x
4
8

200

y
4
2

A = 50x x2

20

40

d 625 m2 when the field is 25 m 25 m


17 a A = l b

= x(100 2x)
= 100x 2x2

400
0

2 4x

10 D
11 C
12 D
13 y y = 2x2 4x + 8
25
20
15
10
5

A = 100x 2x2

1200
800

y = (2 x)2

0
2
4

C
5

= x(50 x)
= 50x x2

400

5 x
y = x2
(a)

y = 2x2
(b)

25

8
4

b 80 m c
10 s
15 a h
h = 30t 5t2

y = 4 + 6x x2

0 1 2 4 5x
3
6
9
y = 8 x2

5 t

Sum of adjacent sides = 50 m


b A = l b

3 4

50 x

b 9y
6

y = (x 2)2

5 x

0 12 45 x
2
4
6
y = 2 + 2x x2

y=x +x+5

b 45 m c
6s
x
16 a

3 4

2 0 1 2 3 4 5 x
2
4

1 2

8 y

y
25
20

25

y
2
6 y = x 6x + 5
4

50

20

3 a

75


They are the same function.

d = 5t2

40

y y = x2 4x 2
6

3 0 1
3
6

d
125
100

c Min. value = 2
2

y = x2 2x + 5

20

14 a

5 x

20

40

c 25 m 50 m
18 a Narrower, TP (0, 0)
b Wider, (0, 0)
c Narrower, TP (0, 0)
d Narrower, TP (0, 0)
e Wider, TP (0, 0)
f Wider, TP (0, 0)
g Narrower, TP (0, 0)
h Narrower, TP (0, 0)
19 a Narrower, minimum
b Narrower, maximum
c Wider, minimum
d Wider, maximum
e Narrower, maximum
f Wider, minimum
g Narrower, minimum
h Wider, maximum
i Narrower, minimum

Chapter 12 Modelling non-linear relationships 355

j Narrower, maximum
k Narrower, minimum
l Narrower, maximum
20 20 h
21 a 200
b

30 50

N 200 201 199 193 183 153 46

10

0
1

2
3
4

12

15

20

V
40 000
30 000

13 a

0 1

2 3

4 5

y
(2, 12)

y = 3x

20
10

1 y

V = 40 000(0.85)n

20 000

10 000
0

30

Cubics, hyperbolas and


exponential functions

y = 1x

5 y
c 56.4 h
40
22 a 1 m b
0.4375 m
Exercise 12B

3 4

(1, 5)

8
7
6
5
4
3
2
1
0

0 1 2 3 4 x
6 a y

y=x

0 1
2 a y
50

40
30

2 3 4 x

40
30
20
10
0

3 4 x

1 2

y = 1 x3
2

2
1
0

2 3 4 x

2 3 4 x

0
2
4

y = x3

3 y
4

0
1

4 x

2 3

0 1

y = (12 )

2 3

4 5 x

14 a

0 1
4 a y
4
3
2
1
0

3 4

40
30
20
10
0

(1, 2)
y = 5(2 )

1 2 3 4

(1, 12)
x

y=0

A = 1000(1.1)n

2 3

0 2 4 6 8 n

15 a

y = 2x 1

11 a A = 50000(1.12)n
A
b

4 x

65 000

b y
8

2
0

1000
1

6
4

y=1

1
x

1400
1200

y = x

1
2

7 y

1600

y
1

8 C
9 B
10
A

4
y = x

c y

4
0

y = 10 x

0
1

40
20

10

y
(2, 7)

2 3 4 x

0 1
b y
100
80
60

b y
8

y = 4x

20
10
0

y = 3x3

(0, 0.5)
A = 50 000(1.12)n

55 000

10

y=0

y=
x
0
0

2 4

6 8

0 1 2 3

c 3 years

356 Maths Quest HSC Mathematics General 2

4 5 n

Asymptote: y = 0; y-intercept: (0, 0.5);


domain: (, ); range: (0, )

Exercise 12D

Inverse variation
1000
1000
1 a k = 1000, x =
or y =
x
y

y = 3x + 2
(0, 9)

y=0

Asymptote: y = 0; y-intercept: (0, 9);


domain: (, ); range: (0, )
y = 2x + 3

y
1000
800
600
400
200
0
0 5 10 15 20 25 x

2 a k = 48, p =

48
q

b p
(0, 4)
x

y=0

Asymptote: y = 0; y-intercept: (0, 4);


domain: (, ); range: (0, )
Exercise 12C

1 y =
2 b = 0.5a3
3 a d = 4.9t2
b d

5.5x2

Direct variation

50
40
30
20
10
0

d = 4.9t
0 1

2 3

4 t

50
40
30
20
10
0

Exercise 12E

1 a

0 2 4 6 8 10 q

b y

42
x

24

54

96

150

A
400
320
240
160
0

0 2 4 6 8 10 x

50
4 y =
x
1
5 m =
n
150
6 a t =
s

A = 6s2

80

4 a 6 b
230.64cm2
5 a 3.14 b
452.16cm2
b t
6 a 0.25 b
54kg c
6.5cm
50
7 a i 148.8cm
40
ii
68.5kg
30
b 0.0248
20
8 a 570.15 L
10
b The Nissan Pulsar as it uses less petrol

to cover 100km.
9 a 1818 m2
b $35750
10 a i 12
ii
8
b 16.73
10w
11 a h =
7
5
b 85 cm
7
c 70cm
12 40
13 a 624km
b 38 h 30 min
c 47 h 30 min
14 a 4WD = 81.9 L, Small car = 38.4 L
b 2132km
15 a 48 L/100km or 0.48 L/km
b 2920km

Graphing physical

phenomena

3 a k = 42, y =
50
40
30
20
10
0

13 a 17500
b $233.33
c 70
d $17500
14 a 200
b 1 amp
c 13.3 ohms
15 a 200
b 2 min
c 600 mHz
16 a 10500
b 210 days
c 105 workers

2 d
500
400
300
200
100
0

30
20
10
0

1
3

10 a 4000

4000
m
c 20 m/s2
d 4 m/s2
500
11 a n =
c
b 2500
c 2000
12 a 337.5
b 3 h 58 min
c 96.42km/h
d 337.5km
b a =

10

20

30

40

18

30

44

d = 0.01v 2 + 0.7v

0 10 20 30 40 50 v

4 a A =
b

8 10 t

40

8 12km/L or 8 L/100km
9 8 amps

2 4 6

b d

0 10 20 30 40 50 s

7 a 20
b 2 hours

d = 5t2

3 a

150
t=
s

6 8 10 s

2 4

100 000
n

A
100 000
80 000
60 000
40 000
20 000
0

000
A = 100
n

0 2

4 6

8 10 n

Chapter 12 Modelling non-linear relationships 357

5 a

Age (years)

Value
b

14 a P

$30000 $24000 $19200 $15360 $12288

(4, 8)

V
40 000

(0, 4)

30 000

V = 30 000(0.8)n1

20 000
10 000
0

6 W

2 4 6

8 10

b p = 2!m + 4
c m = 11,

p = 2!11 + 4
= 10.63
m = 12,
p = 2!12 + 4
= $10.93

W = 3.3(1.2)n

6
4
2
0

0 1

2 3

Chapter Review

Multiple choice

7 a Just less than 6


b V = l b h = (12 2x)(12 2x)x = x(12 2x)2
c
V
125
100

1 B
2 D
3 B
4 D

V = x(12 2x)2

Short answer

75
50
25
0

8 a

1 a

0 1 2

3 4 5

Year

1.5

1.58

1.65

1.74

1.82

2.83

4
2
0

2
0

10

c
1

30000
R
12 a y = ax3, a = 0.3
b y = ax2, a = 6
c y = a!x, a = 1.6
a
d y = , a = 5
x
e y = ax3, a = 1.5
11 I =

13 a I

(15, 1.2)
d

270
d2

358 Maths Quest HSC Mathematics General 2

0 1 2

4 5 x

y = x2 2x 2

4
2
0
2

2 3

4 x

3 a y

y
b

20

y = (x 4)2

16

1 0
4
8
12

12
8
4
0

c
(1, 270)

y = x2 4x + 5

y
6

8 12 16 20 n

c 2027
d The graph will become a straight, horizontal line.
9 k = 3
10 d = 0.008 33s2

b I =

8
6

3.98

1
0

10

P = 1.5(1.05)n

4
3
2

y
b y

2007 2008 2009 2010 2011 2020 2027

Population
(million)
b P

6 x

0 2

y
4

1 0
4
8
12

6 8x
y = 4 + 2x x2

2 3

4 5 x

y = 5 x2

1 2

4 x

9 a

h
500
400
300
200
100
0

h = 500 5t2

0
2

8 10 t

4 6

90
60
30
0 4

8 12 16 20 24

b 144kg, 12 workers
c Too many people getting in each others

way etc.

6 a y
8

y = x3

6
4

0 1 2

b y
10
8
6
4
2

y = 2 x3

1 2 3 4 x

r 0

40
20
0

0 1 2 3 4 x

2
y = x

16 a

4
0

0 1 2 3 4 x

16
12

y = 2x

2 3

4 x

b y
1 x

y = ( 2)

0 1 2 3 4 x

20

30

30

20

h = 20 + 15t 5t2

10
8

6
4
2
0

3 4

y = 2x3
y = 2x
2 3 4 5 x

b (1, 2)
4 a y
4
3

y = 1.08x

2
1
0

h = 20t 5t2

0 2

4 6 8 10 x

b 10 years
c y
1

y = 0.92x

0.75
0.5
1 2 3 4 t

b i 20 m
ii
4s
17 a
A

8 a y

Max. height = 31.25 m when t = 1.5


3 a y

12

10

1 2 3 4 5 r

h
20
16

20

A = r2

30

b A
80
60

b h

c 250 campers
15 a

y = x

b y

2
1
0

6 8 10 n

b 8 days

3
2

8
4
0

1000
14 a A =
n

b 7 years
11 y = 5x2
12 a m = 0.45l3
b 56.25 g
c 4.8cm

7 a y

1
0

c 50
e More than 100 people
2 a

A 0 3.14 12.57 28.27 50.27 78.54


0

1000

64
13 y =
x

3 4

P = 5n

0
500

V = 5000(0.8)n

2000
1000
0

P = 10n 500

2000
1500
1000
500

8 12 16 20 n

0 4

5000
4000
3000

120

C = 500(1.04)n

b $740
c 2024
10 a V

E = 24n n2

150

2
0

1 a P = 10n 500
b, d
P

500
250

b 10 s
5 a E

Extended response

C
1000
750

50
10
0
15
0
20
0

4 a

18 000
16 000
14 000
12 000
10 000
0

0.25
0

0 2 4 6 8 x

d 8 years
A = 10 000(1.06)n

2 4

6 8 10 n

b $16000
c 7 years

Chapter 12 Modelling non-linear relationships 359

Chapter 13

Mathematics and health


CHAPTER CONTENTS
Body measurement
13A Drawing scatterplots
13B Correlation
13C Fitting a regression line
Medication
13D Units of measurement
13E Dosage rate
Life expectancy
13F Life expectancy

Body measurements
13A Drawing scatterplots
Do you know your height and weight? It is reasonable to assume that a taller person of average build
is heavier than a shorter person of similar build. In this section we are going to compare various
measurements and try and develop a few rules that will allow us to make an estimate of one
measurement when given another.
The table below shows the height and weight (mass) of a group of football players in the 2013
St GeorgeIllawarra Dragons squad.
Player

Height

Mass

Player

Height

Mass

Matt Cooper
Nathan Fien
Brett Morris
Jason Nightingale

187 cm
182 cm
183 cm
183 cm

95 kg
84 kg
93 kg
91 kg

Michael Weyman
Jack de Belin
Ben Creagh
Dan Hunt

185 cm
188 cm
191 cm
188 cm

113 kg
107 kg
103 kg
106 kg

Interactivity
int-2789
Scatterplots

The figure below shows this information in the form of a scatterplot.


115
110

Mass (kg)

105
100
95
90
85
80
180

182

184

186
188
Height (cm)

190

192

Chapter 13 Mathematics and health 361

When drawing a scatterplot, it is important to choose the correct variable to assign to each of the
axes. The convention is to place the independent variable on the x-axis and the dependent variable on
the y-axis. The independent variable in an experiment or investigation is the variable that is deliberately
controlled or adjusted by the investigator. The dependent variable is the variable that responds to changes
in the independent variable.
Neither of the variables involved in the football example is controlled in any way, but we might normally
expect that a persons weight may change according to their height more than the other way around. As
Weight is the dependent variable, we graph it on the vertical axis and the Height on the horizontal axis.
WORKED EXAMPLE 1

The table below shows the height and mass of ten Year 12 students.
Height (cm)
Mass (kg)

120
45

124
50

130
54

135
59

142
60

148
65

160
70

164
78

170
75

175
80

Display the data on a scatterplot.


THINK

WRITE

Show the height on the horizontal axis and the


mass on the vertical axis.

Plot the point given by each pair.

Mass (kg)

Method 1: Technology-free
80
70
60
50
40
30
0

100 110 120 130 140 150 160 170 180


Height (cm)

Method 2: Technology-enabled
1

From the MENU select STAT.

Delete any existing data, and store the data for


height in List 1 and mass in List 2.

Press 1 (GRPH) (you may have to press 6


for more options first); then press 6 (SET). Set
the graph type to Scatter by arrowing down to
graph type and pressing 1 (Scat) (again you
may have to press 6 for more options first).
Ensure that XList is List 1, YList is List 2 and
Frequency is 1 as shown at right.

Press J to return to the previous screen, and


then press 1 (GPH1). The scatterplot will then
be drawn.

Note that the graphics calculator sets the values on the x- and y-axes automatically. You can press
! 3 (V-Window) to set the scale as you see fit.

362 Maths Quest HSC Mathematics General 2

InvestigatE: Height versus mass

Obtain the heights and weights (or masses) of a group of 20 people and display the data on a scatterplot.
Compare the differences between this group and the football players.
Once we have obtained a set of data and put the data onto a scatterplot it is useful to draw a line of
best fit, if possible. A line of best fit is a line that we draw on the scatterplot as close as possible to as
many points on the scatterplot.
When fitting a line of best fit there should be an equal number of points above and below the line.
Forexample, if there are 12 points in the data set there should be 6 above the line and 6 below the
line.This may appear logical or even obvious but fitting a line by eye involves a considerable margin
oferror.

WORKED EXAMPLE 2

Fit a straight line to the data in the figure using the


equal-number-of-points method.

x
THINK
1

Note that the number of points (n) is 8.

Fit a line where 4 points are above the line. Using a clear plastic
ruler, try to fit the best line.

DRAW

x
3

The first attempt has only 3 points below the line where there
should be 4. Make refinements.

x
4

The second attempt is an improvement, but the line is too close


to the points above it. Improve the position of the line until a
better balance between upper and lower points is achieved.

Once the scatterplot has been drawn, we can determine if any pattern is evident. Worked example 2
shows how, as a general rule, as height increases so does mass.
We can also look to see if the pattern is linear. In Worked example 2, although the points are not in a
perfect straight line, the data approximate a straight line. The figures below show examples of linear and
non-linear relationships.

Chapter 13 Mathematics and health 363

Linear relationships
y

Non-linear relationships
y

In other cases it may be that there is no relationship at all between the two variables. Such a scatterplot
would look like the one below.
y

The following worked example shows an example of a non-linear relationship.


WORKED EXAMPLE 3

The table below shows the length and mass of a dozen eggs.

Length (cm)
Mass (g)

6.2
60

3.9
15

4.5
25

5.8
50

7.2
95

7.6
110

6.1
55

6.7
75

7.3
95

5.1
35

6.0
54

7.3
96

a Display this information in a scatterplot.


bDetermine if there is any relationship between the length and mass of the eggs and state if the

relationship is linear.

364 Maths Quest HSC Mathematics General 2

THINK

WRITE

a 1 Display length on the

120

x-axis and mass on the


y-axis.
Mass (kg)

100

Plot the point given by


each pair.

80
60
40
20
0

b 1 Study the scatterplot to

3 4 5 6
Length (cm)

b As length increases, so does the mass of the egg.

see if mass increases as


length increases.
2

Study the scatterplot to


see if the points seem to
approximate a straight line.

The points do not approximate a straight line, and so the


relationship is not linear.

Once a line of best fit has been drawn we are able to use our knowledge of linear functions to find the
equation of that line.

WORKED EXAMPLE 4

The table below shows the waist measurements of nine


people and their weights.
Waist (cm)

102 90 85 92 99 76 84 82 90

Weight (kg)

80

77 75 77 81 65 68 71 79

a
Represent the data in a scatterplot, and draw a line

of best fit by eye.

b
Use your line to find an equation to relate the weight

to waist measurement.

THINK

number of sections on
the horizontal axis and
the fare on the vertical
axis.
2

Plot the data and


draw your line on the
scatterplot.

90
80
70
Weight (kg)

a 1 Draw the axis with the

WRITE

60
50
40
30
20
10
0

20

40

60
Waist (cm)

80

100

120

Chapter 13 Mathematics and health 365

b b = 20

b 1 Find the y-intercept.


2

Choose two points on


the line and use the
gradient formula. It
is easiest to make the
y-intercept one of your
points.

Take (0, 20) and (102, 80)


vertical change in position
m=
horizontal change in position
80 20
=
102 0
= 0.59

Use the gradient


intercept form of a
straight line to write
your equation.

W = 0.59x + 20

Exercise 13A

Drawing scatterplots

1 WE1 The list below is a group of famous boxers.

The table shows their heights and their reach


with outstretched arms.
Name
George Foreman
Tyson Fury
Muhammad Ali
Michael Tyson
Floyd Mayweather
Jeff Fenech
Kostya Tszyu
Manny Pacquia
Vick Darchinian
Billy Dib
Danny Green
Daniel Geale
Anthony Mundine

Height (cm)
193
206
191
178
173
172
170
167
166
172
185
178
180

Reach (cm)
208
216
203
180
183
171
170
170
164
171
185
180
178

Display this data on a scatterplot.


2 The data below shows some famous models, their height and waist

measurements.
Name
Miranda Kerr
Gisele Bundchen
Elle McPherson
Megan Gale
Jennifer Hawkins
Kate Moss
Heidi Klum
Naomi Campbell
Brooke Burke
Brooklyn Decker
Cindy Crawford
Lara Bingle

Height
175 cm
180 cm
183 cm
180 cm
180 cm
173 cm
185 cm
175 cm
170 cm
172 cm
177 cm
168 cm

Display this data on a scatterplot.


366 Maths Quest HSC Mathematics General 2

Waist
61 cm
63 cm
62 cm
64 cm
67 cm
61 cm
60 cm
65 cm
58 cm
60 cm
66 cm
60 cm

3 WE2,3 The table below shows the foot lengths F, of 20 people

and the width of their hand spans, H. Display this information in


a scatterplot.
F

22

21

24

23

22

19

21

24

20

21

17

17

18

18

19

16

20

21

17

18

25

26

26

27

22

22

19

23

21

25

21

22

23

22

19

18

17

18

20

19

a Display the results in a scatterplot.


b Determine if there is any relationship between foot length and hand span. If so, state whether the

relationship is linear.
4 The table below shows the heights of a group of people (in metres) and the surface area of skin on
their bodies (in square metres).
Height (m)

1.6

1.66

1.69

1.7

1.74

1.77

1.77

1.8

1.82

Skin area (m2)

1.6

1.64

1.7

1.71

1.82

1.9

1.95

1.95

2.2

a Display the information on a scatterplot.


b Is there any relationship between height and skin area? If there does appear to be a relationship,

isthe relationship linear?


5 The table below shows forearm length and hair length (both in cm)
of 10 girls.
Forearm length (cm)

40 35 30 33 35 32 33 39 37 36

Hair length (cm)

12 5 32 15 18 9 17 16 27 30

a Display the information on a scatterplot.


b Determine if there appears to be any relationship between the

two variables and if the relationship appears to be linear.

6 MC Which of the following scatterplots does not display a linear relationship?


A y

B y

x
C y

D y

x
Chapter 13 Mathematics and health 367

7 MC In which of the following is no relationship evident between the variables?


A y

B y

x
x
C y

D y

8 Give an example of a situation where the scatterplot may look like the ones below.
a y

b y

Further development
9 If a relationship appears to exist and if one quantity increases and the other also increases, then the

relationship is said to be a positive one. If as one quantity increases the other decreases, then the
relationship is said to be negative.
For each of the following state whether you would expect a relationship to exist and if so whether
it would be positive or negative.
a Time spent studying and the marks achieved
b The number of hours spent training for a cricket team and the number of runs scored
c Age of a person and income level
d Amount spent each week on groceries and the number of hours television watched
e The amount spent on petrol each week and the distance driven
10 For each of the scatterplots drawn below state:
i if a linear relationship exists
ii if a relationship exists, whether that relationship is positive or negative.

b y

c y
a y

d y

e y

f y

The questions below represent data collected by groups of students conducting different
environmental projects. The students have to fit a straight line to their data sets.
Note: For many of these questions your answers may differ somewhat from those provided at the end
of the chapter. The answers are provided as a guide but there are likely to be individual differences
when fitting straight lines by eye.
368 Maths Quest HSC Mathematics General 2

11 Fit a straight line to the data in the scatterplots using the equal-number-of-points

method.
a y

b y

x
d y

e y

x
f y

x
g y

c y

h y

x
i y

12 For each of the following draw a line of best fit and for each find:
i the gradient
ii the vertical intercept.
y
a y

b
70
70
60
60
50
50
40
40
30
30
20
20
10
10
0
0
x
x
0 2 4 6 8 10
0 20 40 60 80 100 120

y
3000
2500
2000
1500
1000
500
0

10 15 20 25

13 WE4 The table below shows the length of an elastic when stretched by a force expressed in

newtons.
Force
Length

440

450

462

470

484

492

500

508

518

528

a Represent the data in a scatterplot, and draw a line of best fit by eye.
b Use your line to find an equation to relate the length (L) to the force (f ).
c Explain the meaning of the vertical intercept and gradient of the line in this context.
Chapter 13 Mathematics and health 369

13B

Correlation

Correlation is a description of the relationship that exists between two variables. When one variable
increases with another, it is said that there is a positive correlation between the variables. In such a case,
the median regression line will have a positive gradient. Similarly, if one variable decreases while the
other increases, the median regression line will have a negative gradient and the correlation is negative.
Consider the following examples in which ten Year 11 students were surveyed to find the amount of
time in hours that they spend doing exercise each week. This was compared with their blood cholesterol
level.
6

12

16

12

Blood cholesterol level

Blood cholesterol level

Period of exercise (h)

10
8
6
4
2
0

4 6 8 10 12 14 16
Period of exercise (h)

In this example there seems to be a general downward trend, and the line of best fit therefore has a
negative gradient. As the amount of exercise increases, the level of blood cholesterol decreases.
Notice that in this case the points are not as closely aligned as in the previous examples. We can say
that the relationship (or correlation) between the variables is only weak. In general terms, the closer that
the points are to forming a straight line, the stronger the relationship is between the variables.
Sometimes we find that there is no relationship between the variables. In the scatterplot below,
a researcher was looking for a link between peoples heights and their IQs. The points appear to be
randomly dispersed across the scatterplot. In cases like this, it can be concluded that there is no clear
relationship between the variables.

IQ

140
120
100
80
60
120 140 160 180 200
Height (m)
WORKED EXAMPLE 5

Weight (kg)

In the figure on the right, describe the correlation as being positive or negative.
100
90
80
70
60
50
40
30
20
10
0

10 20 30 40 50 60 70 80 90 100 110 120 130 140 150 160 170 180 190
Height (cm)

370 Maths Quest HSC Mathematics General 2

THINK

Add a line of best fit to the


scatterplot.
The gradient of the regression
line is positive.

Weight (kg)

WRITE

100
90
80
70
60
50
40
30
20
10
0

Describe the correlation.

10 20 30 40 50 60 70 80 90 100 110 120 130 140 150 160 170 180 190
Height (cm)

There is a positive correlation.

The strength of a correlation is measured using the Pearsons Moment Correlation Coefficient. The
correlation coefficient is a measure the relationship between two variables.
The correlation coefficient is a number that lies between 1 and +1. It measures how close the points
on a scatterplot are to being in a straight line. Your calculator will give you this value and it is assigned
the pronumeral r.
A correlation coefficient of:
1 means that there is a perfect negative correlation. On the scatterplot the points will be in a perfect
straight line with a negative gradient.
+1 means that there is a perfect positive correlation. On the scatterplot the points will be in a perfect
straight line with a positive gradient.
0 means there is no relationship at all between the two variables.
Any other correlation coefficient will indicate whether there is a positive or negative relationship
between the variables and how strong that relationship is. The closer the correlation coefficient is to 1
the stronger the negative relationship is, the closer to +1 it is the stronger the positive relationship. The
diagram below demonstrates this a bit further.

Correlation coefficient

Description

Perfect positive correlation

Between 0.75 and 1

Strong positive correlation

Between 0.5 and 0.75

Moderate positive correlation

Between 0.25 and 0.5

Weak positive correlation

Interactivity
int-0183
Pearsons
product moment
correlation
coefficient

Scatterplot

(continued)
Chapter 13 Mathematics and health 371

(continued)

Correlation coefficient

Description

Scatterplot

Between 0.25 and 0.25

No correlation

Between 0.5 and 0.25

Weak negative correlation

Between 0.75 and 0.5

Moderate negative correlation

Between 1 and 0.75

Strong negative correlation

Perfect negative correlation

The correlation coefficient is found using a calculator once the data has been entered. Check with your
teacher on how your calculator calculates the value of r.
Once we have calculated the correlation coefficient we are then able to comment on the strength of
that correlation.
Again consider the data used earlier in the chapter about the football team.
WORKED EXAMPLE 6

The data below shows the heights (cm) and mass (kg) of the players in a football squad.

Player
Matt Cooper
Nathan Fien
Brett Morris
Jason Nightingale

Height
187 cm
182 cm
183 cm
183 cm

Mass
95 kg
84 kg
93 kg
91 kg

Player
Michael Weyman
Jack de Belin
Ben Creagh
Dan Hunt

Height
185 cm
188 cm
191 cm
188 cm

Mass
113 kg
107 kg
103 kg
106 kg

Use a calculator to find the correlation coefficient and interpret the strength of the correlation.
372 Maths Quest HSC Mathematics General 2

THINK

WRITE

Enter the data into your calculator as two


separate data sets one for height and
one for weight.

Find the correlation coefficient.

r = 0.62

Refer to the table on pages 3712 to


interpret the strength of the correlation.

This represents moderate, positive correlation, which


means that taller football players tend to be heavier.

WORKED EXAMPLE 7

Researchers looking at a cure for whooping cough test a


new drug.
The researchers calculate a correlation coefficient for the
quantity of the drug taken and the number of bouts of
coughing as being 0.65. Describe the correlation
between the dosage of the drug and the number of bouts
of coughing.
THINK

The correlation coefficient is between 0.5


and 0.75 and so it is a moderate negative
correlation.

WRITE

There is a moderate negative correlation between


the dosage given and the number of coughing bouts
patients have.
This means that the higher the dosage of the drug the
fewer coughing bouts experienced by patients.

Causality
Causality refers to one variable causing another. For example,
there is a high correlation between a persons shoe size and shirt
size. However, one does not cause the other. Similarly, there is a
high correlation between number of cigarettes smoked and lung
cancer but, in this case, there is plenty of scientific evidence
to show smoking does cause lung cancer.
Explain whether a positive or negative relationship exists and
discuss causality in each of the following.
1. Hours of study and exam marks
2. Hours of exercise and resting pulse rate
3. Weight and shirt size
4. The number of hotels and churches in country towns
5. The number of motels in a town and the number of flights
landing at the nearest airport
It is possible to make a qualitative judgement as to the type
of correlation that is involved in a relationship by the general
appearance of the graph. Care must be taken before making a
statement about one variable causing the other.
Just because there is a strong relationship between two variables, it does not mean that one variable
causes the other. For example, there is a very strong positivecorrelation in people between their shoe
size and their shirt size, but one does not cause theother.
Similarly, there is a very strong correlation between the amount of study done for an exam and the
result achieved on the exam. In this case it can be argued that the study causes the high exam mark.
Eachcase needs to be considered on its merit.
Chapter 13 Mathematics and health 373

WORKED EXAMPLE 8

A manufacturer who is interested in minimising the cost of training gives 15 of his plant
operators different amounts of training and then measures the number of errors made by each
of these operators. The results of the experiment are placed on a scatterplot and the correlation
between the number of hours of training and the number of errors made is measured to have a
correlation coefficient of 0.69.
a What can be said of the correlation between training and errors?
b What conclusion could the manufacturer make about causality in this case?
THINK

WRITE

a 1 The correlation coefficient is between 0.75

and 0.5.

A correlation coefficient in this range


indicates a moderate negative correlation.

b In this case it would seem logical that those

that have undertaken more training would make


fewererrors.

There is a moderate negative correlation


between the amount of training and the
number of errors made.
b The manufacturer could reasonably presume that

the more training a person is given, the less likely


they are to make errors with the machinery.

Correlation

Exercise 13B

1 WE5 For each of the following, state whether a positive or negative correlation exists.
a

b

c

2 A sample of 10 drivers was taken. Each driver was asked

their age and the number of speeding offences they had


committed in the past five years.
The results are in the table below.
Age
22 36 48 40 58 64 23 25 30 45
Speeding 4 2 1 1 2 0 3 7 1 0
offences
a Display the information on a scatterplot.
b State if there is a positive or a negative correlation between the age and speeding offences.
3 Match each of the following scatterplots with the correlation that it shows.
a

b

c

d

374 Maths Quest HSC Mathematics General 2

Strong positive correlation


Weak positive correlation
Weak negative correlation
Strong negative correlation

Moderate positive correlation


No correlation
Moderate negative correlation

4 WE6 A pie seller at a football notices that there seems to be a relationship between the

number of pies that he sells and the temperature of the day.


He collects the following data.
Daily temperature (C)

12

Number of pies sold

620 315 295 632 660 487 512 530 546 492

22

26

11

18

14

16

15

16

a Draw a scatterplot of the data.


b State the type of correlation that the scatterplot shows and draw a conclusion from

the graph.
5 A researcher is investigating the effect of living in air-conditioned buildings upon general health.

Sherecords the following data.


Hours spent each week in air
conditioned buildings

Number of days sick due to


fluand colds

13

48

15

40

13

10

14

16

18

10

a Plot the data on a scatterplot.


b State the type of correlation the graph shows and draw a conclusion from it.
c The researcher finishes her experimental report by concluding that air-conditioning is the cause of

poor health. Is she correct to say this? What other factors could have influenced the relationship
shown by the scatterplot?
6 The data below shows the population and area of the Australian states and territories.
State

Area ( 1000 km2)

Population ( 1000)

Vic.

228

5092

NSW

802

6828

ACT

329

Qld

1727

4053

NT

1346

207

WA

2526

2051

SA

984

1555

Tas.

68

489

a Plot the data on a scatterplot.


b State the type of correlation the graph shows and draw a conclusion from it.
Chapter 13 Mathematics and health 375

7 In an experiment, 12 people were administered different doses of a drug. When the drug had taken

effect, the time taken for each person to react to a set stimulus was measured. The results are
detailedbelow.
Amount of drug (mg)

Reaction time (s)

0.1

0.030

0.2

0.025

0.3

0.028

0.4

0.036

0.5

0.040

0.6

0.052

0.7

0.046

0.8

0.068

0.9

0.085

1.0

0.092

1.1

0.084

1.2

0.096

a Plot the data on a scatterplot.


b State the type of correlation the graph shows, and draw a conclusion from it.
8 WE7 What type of correlation would be represented by scatterplots that had the following

correlation coefficients?
a 1.0
b 0.4
e 0.35
f 0.21
i 0.25
j 1.0

c 0.8
g 0.75

d 0.7
h 0.50

9 A researcher investigating the proposition that tall mothers have tall sons measures the heights

of 12 mothers and the heights of their adult sons. The correlation coefficient is found to be 0.67.
Describe the correlation between tall mothers and tall sons.
10 A teacher who is interested in the amount of time students spend doing homework asks

15studentsto record the amount of time that they spend on homework and on watching
television.
The correlation coefficient is found to be 0.45. Interpret the correlation between homework and
television watching.
11 A psychologist asked 20 people to rate their level of

contentment on a scale of 0 to 10 (10 representing


perfectly content).
This rating is compared to annual income.
a The correlation coefficient is found to be 0.18.
Describe the correlation between income and level
of contentment.
b The researcher then intends to write an essay entitled
Money cant buy happiness. Do the results confirm
this statement?
12 WE8 An experimenter who is investigating the relationship between exercise and obesity measures

the weights of 30 boys (of equal height) and also documents the amount of physical exercise that the
boys completed each week. The correlation coefficient is found to be 0.47.
a What can be said of the correlation between obesity and exercise?
b What conclusion could be made about causality in this case?
376 Maths Quest HSC Mathematics General 2

13 MC A researcher is interested in the association between the work rate of production workers and

the level of incentive that they are offered under a certain scheme. After drawing a scatterplot, she
calculates the correlation between the two variables at 0.82. The researcher can conclude that:
A There is a strong positive correlation between the variables; the greater the incentive, the lower
the work rate.
B There is a strong positive correlation between the variables; the greater the incentive, the greater
the work force.
C There is a strong negative correlation between the variables; the greater the incentive, the lower
the work rate.
D There is a strong negative correlation; incentives cause an increase in the work rate.

13C

Fitting a regression line

Fitting lines by eye is useful, but it is not the most accurate of methods. Greater accuracy is achieved
through closer analysis of the data. Upon closer analysis it is possible to find the equation of a line
ofbest fit of the form y = mx + c, where m is the gradient and c is the y-intercept. Several mathematical
methods provide a line with a more accurate fit.
One of these methods is called the least squares regression line of best fit. It is used when data show
a linear relationship. It can even by used when the data contain outliers.
The least squares regression equation minimises the average deviation of the points in the data
set from the line of best fit. The following summary data are required to determine the least squares
regression equation.
Summary data required
x the mean of the independent variable (x-variable)
y the mean of the dependent variable (y-variable)
x the standard deviation of the independent variable
y the standard deviation of the dependent variable
r the correlation coefficient.
The equation of the least squares line of best fit is then found using the rule:
y = mx + c
where
the slope of the regression line is m = r

y
x

the y-intercept of the regression line is c = y mx


WORKED EXAMPLE 9

Find the equation of the least squares regression line for the data in the table below.
x

THINK

WRITE

Calculate the mean and standard deviation of the


xscores.

For the x scores


x = 3.67, x = 1.97

Calculate the mean and standard deviation of the


yscores.

For the y scores


y = 3.83, y = 1.95

Calculate the correlation coefficient.

Calculate the gradient.

r = 0.85
y
m=r
x
= 0.85
= 0.84

1.95
1.97

Chapter 13 Mathematics and health 377

Calculate the y-intercept.

c = y mx
= 3.83 (0.84 3.67)
= 0.67

Write the equation.

Equation : y = 0.86x + 0.67

WORKED EXAMPLE 10

Weight (kg)

The scatterplot below shows a comparison between the heights and weights of 12 boys.

100
90
80
70
60
50
40
30
20
10
0

10 20 30 40 50 60 70 80 90 100 110 120 130 140 150 160 170 180 190
Height (cm)

a Calculate the mean and standard deviation of the boys heights.


b Calculate the mean and standard deviation of the boys weights.
c Find the gradient and y-intercept of the least squares line of best fit, and hence its equation.
THINK

a Enter the heights into your calculator to find the

meanand standard deviation. Let x represent the


heights
b Enter the weights into your calculator to find the

meanand standard deviation. Let y represent the


weights
c 1 Find the correlation coefficient.
2

Use the formula to find the gradient.

WRITE

a x = 170.4, x = 13.9

b y = 75.4, y = 9.4

c r = 0.94

= 0.94
= 0.64

9.4
13.9

Use the formula to find the y-intercept.

c = y mx

Write the equation.

= 75.4 (0.64 170.4)


= 33.7
y = 0.64x 33.7

Once the regression line has been found, we are able to use the equation to make predictions about
other pieces of data. If the point for which we are making a prediction is within the boundaries
of known data this is called interpolation, while if it is outside known boundaries it is called
extrapolation.

378 Maths Quest HSC Mathematics General 2

WORKED EXAMPLE 11

In the previous worked example it was found that the height and weight of boys can be estimated
using the least squares line of best fit which had equation y = 0.64x 33.7
a Use the equation to estimate the weight (to the nearest kg) of a boy of height 172 cm.
b Use the equation to estimate the height (to the nearest cm) of a boy who weighed 72 kg.
THINK

WRITE

a y = 0.64x 33.7

a 1 Write the equation of the line of best fit.


2

Substitute 172 for x.

Calculate y.

Give a written answer.

y = 0.64 172 33.7


= 76.38
The estimated weight is 76 kg.

b 1 Write the equation of the regression line.

y = 0.64x 33.7

72 = 0.64x 33.7

Substitute 72 for y.

Solve the equation.

105.7 = 0.64x
x = 165.2

Give a written answer.

The estimated height is 165 cm.

Exercise 13C

Fitting a regression line

1 WE9 Find the equation of the least squares line of best fit for the data in the table below.

2 Find the equation of the least squares line of best fit for the data shown in the table below.

10

20

40

45

55

60

20

18

12

10

3 WE10 The table below shows the heights and weights of a group of boys.

Height (cm)

164

170

181

163

152

156

147

159

173

164

Weight (kg)

76

62

89

56

54

62

57

72

80

55

a Calculate the mean and standard deviation of the boys heights.


b Calculate the mean and standard deviation of the boys weights.
c Find the gradient and y-intercept of the least squares line of best fit, and hence its equation.
4 In an experiment, a student measures the length of a spring when different masses are attached to it.

Her results are shown below.


Mass (g)
Length of spring (mm)

100

200

300

400

500

600

700

800

900

220

225

231

235

242

246

250

254

259

264

a Draw a scatterplot of the data.


b Find the gradient and y-intercept of the regression line, and hence find the equation of the least

squares line of best fit.


c On your scatterplot draw the line of best fit.
Chapter 13 Mathematics and health 379

5 A sports scientist is interested in the importance of muscle bulk

to strength.
He measures the biceps circumference of ten people and tests their
strength by asking them to complete a lift test. His results are
given in the following table.
Circumference of biceps (cm)

Lift test (kg)

25
25
27
28
30
30
31
33
34
36

50
52
58
51
60
62
53
62
61
66

a Draw a scatterplot of the data.


b Find a rule for determining the ability of a person to complete a lift test, S, from the

circumference of their biceps, B.


6 WE11 Detectives can use the equation H = 6.1f 5 to estimate the

height of a burglar who leaves footprints behind. (H is the height


of the burglar, in cm, and f is the length of the footprint.)
a Find the height of a burglar whose footprint is 27 cm in length.
b Find the height of a burglar whose footprint is 30 cm in length.
c Find the footprint length of a burglar of height 185 cm. (Give your
answer correct to 2 decimal places.)
d Find the footprint length of a burglar of height 152 cm. (Give your
answer correct to 2 decimal places.)

7 A pie seller at a football match finds that the number of pies sold is related to the temperature of the

day. The situation could be modelled by the equation N = 870 23t, where N is the number of pies
sold and t is the temperature of the day.
a Find the number of pies sold if the temperature was 5 degrees.
b Find the number of pies sold if the temperature was 25 degrees.
c Find the likely temperature if 400 pies were sold.
d How hot would the day have to be before the pie seller sold no pies at all?

8 A market researcher finds that the number of people who would purchase Wise-up (the thinking

mans deodorant) is related to its price. He provides the table of values below.
Price ($)

Weekly sales ( 1000)

1.40
1.60
1.80
2.00
2.20
2.40
2.60
2.80
3.00
3.20
3.40

105
101
97
93
89
85
81
77
73
69
65

380 Maths Quest HSC Mathematics General 2

a Draw a scatterplot of the data.


b Draw in the line of best fit.
c Find an equation that represents the relationship between the number of cans of Wise-up sold,

N (in thousands), and its price, p.


d Use the equation to predict the number of cans sold each week if:
i the price was $3.10
ii the price was $4.60.
e At what price should Wise-up be sold if the manufacturers wished to sell 80 000 cans?
f Given that the manufacturers of Wise-up can produce only 100 000 cans each week, at what

price should it be sold to maximise production?

Further development
9 Rock lobsters (crayfish) are sized according to the length of

their carapace (main body shell).


The table below gives the age and carapace length of 16 male
rock lobsters.
Age (years)

Length of carapace (mm)

65

2.5

59

4.5

80

4.5

80

3.25

68

7.75

130

150

6.5

112

12

200

14

210

4.5

82

3.5

74

2.25

51

1.76

48

10
9.5

171
160

Display this information on a scatterplot.


Find the equation of the least squares line of best fit.
Use the equation to find the likely size of a 5-year-old male rock lobster.
Use the equation to find the likely size of a 16-year-old male rock lobster.
Rock lobsters reach sexual maturity when their carapace length is approximately 65 mm. Use the
equation to find the age of the rock lobster at this stage.
f The fisheries department wants to set minimum size restrictions so that the rock lobsters have
three full years from the time of sexual maturity in which to breed before they can be legally
caught. What size should govern the taking of a male rock lobster?
Note: Answers for this exercise are approximate and may vary due to the precise location of the
line of best fit.
a
b
c
d
e

10 MC When finding a least squares line of best fit, which of the following measurements is not

needed?
A The mean of both data sets.
C The standard deviation of both data sets.

B The median of both data sets.


D The correlation coefficient.

DIGITAL DOC
WorkSHEET 13.1
doc-11106

Chapter 13 Mathematics and health 381

Medication
13D

Units of measurement

From your earlier work with measurement you will have learned about
common units of mass such as tonne, kilogram and gram. In this section
where we are studying measurements that relate to medication which are
often small quantities which need a unit of measurement that is smaller
than a gram.
The unit that is most frequently used in medication is the milligram (mg).
As we have seen with other units of metric measurement the prefix milli
means 1000 times smaller.

1000

mg

1000

1000

kg

1000

WORKED EXAMPLE 12

Complete each of the following conversions


a 2.4 g = ___ mg
b 75 mg = ___ g

c 0.9 kg = ___ mg

THINK

a A gram is larger than a milligram so multiply the number

ofgrams by 1000.
b A milligram is smaller than a gram so divide the number

ofmilligrams by 1000.
c 1 Convert kilograms to grams by multiplying by 1000.
2

WRITE

a 2.4 g = 2.4 1000

= 2400 mg

b 75 mg = 75 1000

= 0.075 g

c 0.9 kg = 0.9 1000

= 900 g

= 900 1000
= 900 000 mg

Convert grams to milligrams by multiplying by 1000.

In many cases some drugs are given as part of a mixture. For example a cough mixture will contain
other ingredients other than what is specifically needed. This may be to make it easier to swallow or
make it tastier. In such cases the key ingredient (called the active ingredient) will be expressed in the
form ofa rate.
WORKED EXAMPLE 13

A natural cough mixture comes in a 200 mL bottle.


The active ingredient is Olea europaea and the label states
the presence of the active ingredient is 22mg / 5 mL dose.
a How many doses of the cough mixture are in each bottle?
bCalculate the number of grams of the active ingredient in
each bottle.
THINK

a Each dose is 5 mL. Divide the bottle capacity by the

dose size.
b 1 Multiply the amount of active ingredient per dose

by the number of doses in the bottle.


2

Convert this quantity to grams.

382 Maths Quest HSC Mathematics General 2

WRITE

a Number of doses = 200 5

= 40

b Total = 22 mg 40

= 880 mg

= 0.88 g

Exercise 13D

Units of measurement

1 WE12 Complete each of the following conversions.


a 3000 g = ..... kg
b 0.2 kg = .... g
d 0.25 g = .... mg
e 20 g = .... mg

c 8000g = .... mg
f 0.4 kg = .... mg

2 Convert the following amounts to grams.


a 4000 mg
b 600 mg

c 12.5 mg

d 5 mg

3 Convert the following amounts to milligrams.


a 5 g
b 0.4 g

c 1.2 kg

d 0.025 g

4 WE13 Jeremy has a prescription for a course or antibiotics.

There are 28 tablets in each box and each tablet has a mass of 4 mg.
a Calculate the total mass of the tablets in milligrams.
b What is the total mass in grams?
5 Panadol tablets have 500 mg of paracetamol in each tablet.
a The recommended dosage advice on the box says to take 1 to

2tablets every 4 to 6 hours as necessary. Henry takes two tablets for


a headache. How much paracetamol does he consume?
b The maximum recommended dosage of paracetamol is 4000 mg per
day. How many tablets is this equivalent to?
c If a person takes the maximum amount suggested on the labelling by
how much will they exceed the recommended maximum daily intake
of paracetamol?
d Children aged between 7 and 12 can take a maximum 4 tablets in any
24-hour period. Francis is 7years old takes one tablet and then another half tablet every 4 hours
over a 24-hour period. Isthis in excess of the maximum allowed, and if so by how much?
6 The actual volume of cough medicine in a cough bottle is 250 mL.
a Jim is prescribed the medicine with instructions to take 15 mL four times a day. How many days
will it take Jim to take the entire bottle?
b The cough medicine contains 20% Noscapine. How many milligrams of Noscapine are in each
dose of the medicine?
7 Streptomycin is a drug used to treat tuberculosis.

How many 50 mg injections can a doctor make from a 1-gram container of the drug?

Further development
8 Voltaren and Fenac are two tablets used for the relief of muscle and back pain, each containing

diclofenac potassium. Voltaren contains 12.5 mg of diclofenac potassium while Fenac contains 50 mg.
a How many times stronger are the Fenac tablets than the Voltaren tablets?
b The Voltaren tablets indicate that a maximum six tablets should be taken in any 24-hour period.
How many mg of diclofenac potassium is this?
c The instructions on the Voltaren packet say to take 1 to 2 tablets every four to six hours as
necessary. Jackie has back pain and wakes up at 7.00 am and goes to bed at 10.00 pm. Suggest
when and how many Voltaren tablets Jackie should take throughout the day.
d The Fenac tables are available only on prescription from a doctor. The instructions on the
prescription say to take one table three times a day immediately after food. Why would the
instruction after food be important?
e Compare the dosage of diclofenac potassium by following the Fenac prescription to the maximum
dosage suggested on the Voltaren packet. Can you offer an explanation for your findings?
Chapter 13 Mathematics and health 383

9 Zyrtec is an antihistamine with an active ingredient

of cetirizine hydrochloride.
There is 10 mg of the active ingredient in each
tablet. How many grams of cetirizine hydrochloride
are in a packet of 200 Zyrtec tablets?
10 Eno is a powder dissolved in water for the relief
of upset stomachs. Each dose of 5 g contains 2.16 g
of Citric Acid Anhydrous.
a Calculate the number of 5 g doses in a 300 g jar of Eno.
b Calculate the amount of Citric Acid Anhydrous
in one bottle of Eno.

13E

Dosage rate

All medications have a prescribed dosage rate. Some


medicines are known as over-the-counter medications,
which means that they can be bought without a
prescription, usually from a pharmacy but in some
cases from a supermarket.
Other medications need a prescription from a doctor
and can only be purchased from a pharmacy. Regardless
of whether the medication is a prescription drug or an
over-the-counter medicine, there will be a recommended
dosage rate.
For over-the-counter medicines the recommended dosage is usually printed on the label but for
prescription drugs the doctor sets the dosage rate. Many medicines have side effects, that is; they may
affect something other than what is being treated. For example, antihistamines are used to treat allergies
but have the potential to cause drowsiness.
Most medications have a maximum daily dosage, which should not be exceeded. This maximum dosage
is usually less for children and many medications should not be given to children below a certainage.
For example, the Panadol label says that the adult dosage is 12 tablets every 46 hours but children
between 7 and 12 should take 12 1 tablet in the same time period.
In many cases a formula may be used to calculate the appropriate dosage for a child. The three most
common formulas are as follows:
Frieds formula, which is used for children 12 years of age and uses the age of the child in months to
calculate the correct dosage.
age(of child in months) adult dosage
Dosage for children 12 years =
150
As children get older Youngs rule is more appropriate and uses the age of the child in years. Youngs
formula is for children aged 112 years
age (of child in years) adult dosage
Dosage for children 112 years =
age (of child in years) + 12
Clarks formula is a more general formula and ignores age basing its calculation on the weight of the
child in kilograms. This formula can be used for a child of any age.
weight in kg adult dosage
Dosage =
70
WORKED EXAMPLE 14

Bella is 15 months old and weighs 10 kg.


She needs to take a medicine for which the adult dose
is 30 mL. Calculate the dosage that Bella should take.

384 Maths Quest HSC Mathematics General 2

THINK

WRITE

Bella is aged between 1 and 2 years old so


Friedsformula should be used. Write the
formula.

Substitute the known values into the formula.

Calculate the dosage for Bella.

age(of child in months) adult dosage


150
15 30
=
150

Dosage =

= 3 mL
Bella should be given 3 mL of the medicine.

Your understanding of measurement is also important when considering the rate at which intravenous
drips work.
WORKED EXAMPLE 15

Nancy is in hospital patient and is placed on an IV drip.


She is required to take 1000 mg of the medication, which comes in a
solution form. The medication is present in the solution at a rate of
50mg / 5 mL. There are 5 drips per mL and drips are delivered at
30per minute. Calculate
a
The amount (in L) of the IV drip that Nancy must be given.
b
The time it will take for Nancy to receive the required amount of
medication.

THINK

WRITE

a Calculate the amount of solution needed to

deliver1000 mg of the medication.


b 1 Calculate the number of drips needed.
2

Calculate the time taken.

Exercise 13E

a Solution

= 20 5 mL
= 100 mL

b No. of drips = 100 5

= 500

Time = 500 30
= 16.7 min

Dosage rate

1 WE14 Frank is 18 months old.

He needs to take medication for which the adult dose is 40 mg.


Calculate the dose that should be given to Frank using Frieds
formula.
2 Use Frieds formula to calculate the dose of medicine that should
be given to each of the following children.
a Andrea is 12 months old and the adult dose of the medicine
is30 mL.
b Phillip is 14 months old and the adult dose of the medicine
is50 mL.
c Glen is 16 months old and the adult dose of the medicine
is20mL.
d Michonne is 2 years old and the adult dose of the medicine
is35 mL.
Chapter 13 Mathematics and health 385

3 Daryl is 4 years old. Calculate the dosage for Daryl of a medicine for which the adult dose is

50mLusing Youngs Formula.


4 Amoxil is an antibiotic for which the daily dosage for an adult is 750 mg. Calculate the daily dosage for

each of the following children using Youngs Formula. Give each answer correct to thenearest 5mg.
a Rick is 2 years old and weighs 12 kg.
b Maggie is 5 years old and weighs 15 kg.
c Milton is 9 years old and weighs 42 kg.
d Merle is 3 years old and weighs 15.5 kg.
5 The recommended dose of Brondecon cough mixture

for an adult is 20 mg.


Ron is 4 years old and weighs 25 kg. Calculate Rons
dosage of Brondecon using Clarks formula.
6 Using Clarks formula, calculate the dosage of a

medicine for which the adult dosage is 75 mg for each


of the children listed in question 4.
7 WE15 A patient is placed on a IV drip. The medication contains 50 mg in every 5 mL of the

solution. The patient is prescribed 1500 mg of the medication. Given that the drip is administered at
a rate of 20 drips per minute and there are 5 drips per mL, calculate:
a The amount of solution the patient will require
b The length of time it will take for this to be administered.
8 Francis is to be administered 3000 mg of medication through an IV drip. The solution through

which it can be administered comes in two forms, 50 mg per 5 mL or 8 mg per 5 mL. In either
case the drip with be delivered at a rate of 20 drips per minute and there are 5 drips per mL.
Calculate how much quicker the stronger solution will take to deliver the 3000 mg of
medication.

Further development
9 Diane is prescribed 750 mg of a medication, twice a day. Tablets come in 500 mg form and in a

packet of 20.
a How many tablets should Diane take at each dose?
b How many milligrams of the medication are in each pack?
c How many days will it take for Diane to take the whole packet?
10 A patient is brought into hospital suffering from

dehydration.
He is placed on a saline drip that is administered
intravenously. The patient must receive 2 litres of fluid
over an 8-hour period. Calculate the flow rate
a in mL per hour
b in drips per minute given that 5 drips make 1 mL of
fluid.

L ife

expectancy
13F Life expectancy
Life expectancy refers to the mean age up to which people
can expect to live.
Life expectancy is generally considered to be
a measure of the affluence of a country and is an
important statistic that is measured on a regular basis by
organisations such as the United Nations and the World
Health Organization.
Earlier in this chapter we studied scatterplots, correlation
and lines of best fit. In this section we are going to compare
important pieces of data about a number of countries.
386 Maths Quest HSC Mathematics General 2

Exercise 13F

Life expectancy

1 The graph below shows the average life expectancy of Australian men and women since 1900.
Average life expectancy of Australian
men and women since 1900
90
80

Age in years

70
60
50

Males

40

Females

30
20
10
0

1900 1910 1920 1930 1940 1950 1960 1970 1980 1990 2000 2010
Year

a What was the average life expectancy of an Australian man in 1900?


b What was the average life expectancy of an Australian woman in 2010?
c Life expectancy has clearly increased over this period of time for both men and women. Has it

increased equally for both sexes?


d The growth in life expectancy for men appeared to flatten out for men in the middle of the 1900s.

What possible explanation could there be for this?


e Does the growth in life expectancy approximate a linear pattern? Would you expect this pattern to

continue in the years to come?


2 The graph below shows the life expectancy of Australians in each state.

86
National
average
(Female)

84
82
80

National
average
(Male)

78
76
74

Male
Female

72
70

a
b
c
d

NSW

Vic.

Qld

WA

SA

Tas.

ACT

NT

Which states are above the national average life expectancy for both men and women?
Which state (or territory) has the highest life expectancy?
Which state (or territory) has the lowest life expectancy?
Offer some possible reasons for the answers to parts b and c.
Chapter 13 Mathematics and health 387

3 The table below shows the life expectancy, in terms of additional year of life, for people over a

period of time in Australia.


Life expectancy (additional years of life) for people at selected years of age

18811890
18911900
19011910
19201922
19321934
19461948
19531955
19651967
19751977
19851987
19951997
20042006
20072009

at 0
47.2
51.1
55.2
59.2
63.5
66.1
67.1
67.6
69.6
72.7
75.6
78.7
79.3

Males
at 25
at 45
37.1
23.0
38.9
24.0
40.6
24.8
42.7
26.0
44.4
26.9
45.0
26.8
45.5
27.2
45.4
27.0
46.9
28.3
49.5
30.8
51.8
33.1
54.7
35.7
55.2
36.3

at 65
11.1
11.3
11.3
12.0
12.4
12.3
12.3
12.2
13.1
14.6
16.1
18.3
18.7

at 0
50.8
54.8
58.8
63.3
67.1
70.6
72.8
74.2
76.6
79.2
81.3
83.5
83.9

Females
at 25
at 45
39.7
25.6
41.7
26.7
43.4
27.6
45.7
29.0
47.2
29.7
48.7
30.5
50.2
31.4
51.2
32.3
53.1
34.0
55.4
36.1
57.1
37.7
59.2
39.7
59.5
40.1

at 65
12.3
12.8
12.9
13.6
14.2
14.4
15.0
15.7
17.1
18.6
19.8
21.5
21.8

a Jenny was born in 1953. What was her life expectancy?


b What was Jennys additional life expectancy:
i in 1977 when she turned 25
ii in 1997 when she turned 45?
c Explain why each of the answers above does not give the same final age for life expectancy.
d Use an Excel spreadsheet to graph the above life expectancy data for men.
4 GDP (Gross domestic product) is the total value of all production in a country and is a measure of

the economic strength of a country. The importance of health care in a country can be measured by
total spending on health care as a percentage of GDP. The table below shows a selection of countries
and compares their health care spending as a percentage of GDP with life expectancy.
Country
Australia
USA
Japan
China
Singapore
United Kingdom
Germany
Italy
Switzerland
France
New Zealand
Finland
Brazil
Argentina
Papua New Guinea
Central African Republic
India

388 Maths Quest HSC Mathematics General 2

Life expectancy

Spending on health care


(as a percentage of GDP)

81.44
77.97
82.73
72.71
80.60
79.91
79.85
81.37
81.81
80.95
80.13
79.34
72.24
75.30
61.50
45.91
64.19

9.1
17.6
9.5
5.1
4.1
9.6
11.6
9.3
11.4
11.6
10.1
8.9
8.8
9.5
3.7
4.0
4.2

Country
Zimbabwe
Israel
Iraq
Iran
Afghanistan
Ethiopia

Life expectancy
46.59
80.69
67.32
72.06
47.32
57.21

Spending on health care


(as a percentage of GDP)
8.1
7.6
4.1
5.7
7.6
4.4

a Complete a scatterplot of life expectancy against health care spending.


b Calculate the correlation coefficient and comment on any relationship that may exist between

health care spending and life expectancy.


c Find the equation of the least squares line of best fit.
d The government in Papua New Guinea plans to double its spending on health care as a percentage

of GDP. What impact could you expect this to have on life expectancy in this country? Would you
expect this impact to be immediate?
5 Another important measure of health care is infant mortality. Infant mortality is measured in the

number of deaths per 1000 births or children before they reach their first birthday.
The table below compares life expectancy with infant mortality.
Country
Australia
America
Japan
China
Singapore
United Kingdom
Germany
Italy
Switzerland
France
New Zealand
Finland
Brazil
Argentina
Papua New Guinea
Central African Republic
India
Zimbabwe
Israel
Iraq
Iran
Afghanistan
Ethiopia
a
b
c
d
e

Life expectancy

Infant mortality rate

81.44
77.97
82.73
72.71
80.60
79.91
79.85
81.37
81.81
80.95
80.13
79.34
72.24
75.30
61.50
45.91
64.19
46.59
80.69
67.32
72.06
47.32
57.21

4.55
5.98
2.21
15.62
2.65
4.56
3.51
3.36
4.03
3.37
4.72
3.4
20.05
10.52
42.05
97.17
46.07
28.23
4.07
40.25
41.11
121.63
75.29

Plot the data on a scatterplot.


Calculate the correlation coefficient for the data and comment on the strength of the relationship.
Find the equation of the least squares line of best fit.
Identify any pieces of data that are irregular, considering the overall trend.
A country is known to have an average life expectancy of 70 years. Based on these results what
can we expect the infant mortality rate to be?
Chapter 13 Mathematics and health 389

6 The graph below shows the results of a New Zealand study that examined the percentage of males

who smoke in each area and the life expectancy in that area.

Male life expectancy by smoking propensity


80

Male life expectancy at birth (years), 20052007


Territorial authority areas (R2 = 0.77)
Regional council areas (R2 = 0.90)
New Zealand

79
78
77
76
75
74
73

18

20
22
24
26
Percentage of 15+ male population smoking regularly, 2006 Census

28

a Add a line of best fit to the data. This can be done by eye and not by using the least squares

Digital doc
WorkSHEET 13.2
doc-11107

b
c
d
e

method.
Describe the correlation between life expectancy and smoking.
What would be the vertical intercept of your linear function? What is its meaning in this context?
Is your answer to c realistic in this context? What limits should be placed on this linear function.
What would be the gradient of your linear function? What is the meaning of the gradient in this
context?

390 Maths Quest HSC Mathematics General 2

Summary
Body measurements
Drawing
scatterplots

A scatterplot is used to compare two variables.


The one variable is drawn on the horizontal axis and the other variable on the vertical axis.
After a scatterplot has been drawn look to see if a pattern is evident. A pattern may be linear which
may be linear, if the pattern approximates a straight line, or non-linear.
A line of best fit can be drawn on a scatterplot to demonstrate any evident trend.

Correlation

Fitting a
regression line

A regression line is a line of best fit on a scatterplot to indicate any trends.


A regression line can be drawn by eye ensuring that there are equal numbers of points on each side
of the regression line but this is not a very accurate method.
The equation of the least squares line of best fit is then found using the rule:
y = mx + c
where
y
the slope of the regression line is m = r
x
the y-intercept of the regression line is c = y mx
x the mean of the independent variable (x-variable)
y the mean of the dependent variable (y-variable)
x the standard deviation of the independent variable
y the standard deviation of the dependent variable
r the correlation coefficient.

Correlation is a description of the relationship between two variables.


A positive correlation means that as one quantity increases so does the other.
A negative correlation means that as one quantity increases the other decreases.
Causality is when the change in one variable causes the change in the other. A high correlation does
not necessarily imply causality.

Medication
Units of
measurement

Medication uses very small quantities and is often measured in milligrams (mg).
There are 1000 mg in 1 g.
The active ingredient in a medication is that which performs the task intended. In liquid form the
quantity of the active ingredient will be measured as a rate.

Dosage rate

Most medicines have a standard adult dose. Children will take a smaller dose of the medicine.
To calculate the amount of a childs dose one of the following formulas should be used:
Frieds formula, which is used for children 12 years of age and uses the age of the child in months
to calculate the correct dosage.
age (of child in months) adult dosage
Dosage for children 12 years =
150
As children get older Youngs rule is more appropriate and uses the age of the child in years. Youngs
formula is for children aged 112 years.
age (of child in years) adult dosage
Dosage for children 112 years =
age (of child in years) + 12
Clarks formula is a more general formula and ignores age basing its calculation on the weight of the
child in kilograms. This formula can be used for a child of any age.
weight in kg adult dosage
Dosage =
70
An intravenous (IV) drip is a method used to provide medication slowly and constantly over a period of
time. Measurements and flow rates are used to make calculations associated with IV drips.

Chapter 13 Mathematics and health 391

Life expectancy
Life expectancy

Life expectancy is a measure of how old the average person in society can expect to live.
There are many factors that affect life expectancy. These include social, economic and political factors.
Life expectancy is a measure of how successful a society is.
We can measure the impact of factors on life expectancy by using scatterplots and looking at
correlation coefficients.

392 Maths Quest HSC Mathematics General 2

Chapter review
1 Which of the following graphs best depicts a strong negative correlation between variables?
A y

B y

C y

D y

2 What type of correlation is shown by the graph on the right?


A
B
C
D

Strong positive correlation


Moderate positive correlation
Moderate negative correlation
Strong negative correlation

multi p l e
c hoic e

x
y

3 A researcher finds that there is a correlation coefficient of 0.62 between the number of pedestrian

crossings in a town and the number of pedestrian accidents. The researcher can conclude that:
A Pedestrian crossings cause pedestrian accidents.
B Pedestrian crossings save lives.
C There is evidence to show that pedestrian crossings cause accidents.
D There is evidence to show that the greater the number of pedestrian crossings, the smaller the
number of pedestrian accidents.

1 The table below shows the maximum and minimum temperature on 10 days chosen at random

throughout the year. Display this information on a scatterplot.


Maximum temperature (C)
25
36
21
40
24
26
30
18
20
25

S ho rt
a nsw er

Minimum temperature (C)


12
21
11
23
12
15
19
10
8
13

2 The table below shows the number of sick days taken by ten employees and relates this to the

number of children that they have.


No. of children

No. of sick days

1
0
3
2
2
4
6
0
1
2

5
3
10
8
4
12
12
0
1
2

a Show this information on a scatterplot.


b Does a relationship appear to exist between the number of sick days taken and the number of

children they have? If so, is the relationship linear?


Chapter 13 Mathematics and health 393

3 The table below shows the number of cars and number of televisions in each household.

No. of cars

No. of televisions

a Show this information on a scatterplot.


b Does a relationship appear to exist between the number of televisions in each household and the

number of cars they have? If so, is the relationship linear?


4 The table below shows the relationship between two variables, x and y.

18

12

11

10

16

103

75

20

66

70

50

95

40

27

42

30

a Prepare a scatterplot of the data.


b On the scatterplot, fit a regression line by eye.
c By measuring the gradient and the y-intercept of the median regression line, find its approximate

equation.
5 For each of the following scatterplots, state whether the correlation is positive or negative.
a

b

c

6 For each of the following, state the type of correlation if the correlation coefficient is:
a 0
b 1
c 0.5
d 0.84
e 0.3.
7 A survey in which people were asked to state their age and the age of their car revealed a correlation

coefficient of 0.65.
a What type of correlation exists in this case?
b What can be said about causality in this case?

8 Consider the data in the table below.

10

11

15

23

21

20

14

16

12

a Find the mean and standard deviation of the x and y values.


b Find the correlation coefficient.
c Calculate the equation of the least squares line of best fit.
394 Maths Quest HSC Mathematics General 2

9 A drug company wishes to test the effectiveness of a

drug to increase red blood cell counts in people.


The following data were collected:
Day of experiment
4
Red blood cell count 210
a
b
c
d

5
240

6
230

7
260

8
260

9
290

Find the correlation coefficient of this data.


Comment of the strength of any relationship present.
Find the equation of the least squares line of best fit.
Explain the meaning of the gradient and vertical intercept
in this context.

10 Complete each of the following conversions.


a 3.7 g = ___ mg
b 850 mg = ___ g

c 1.1 kg = ___ mg

11 A cough medicine comes in a 250 mL bottle. The active ingredient DXM is present at the rate of

30mg / 5 mL. The recommended adult dose is 40 mL.


a How much DXM is in each dose?
b How many doses of cough medicine are in each bottle?
c How much DXM is in each bottle?
12 Brad is a 2-year-old boy who weighs 16 kg.

He needs to take a medicine for which the adult dose is


40 mL. Calculate the dose that Brad should take using.
(Give your answers correct to the nearest mL.)
a Frieds formula
b Youngs formula
c Clarks formula
1 A medication is to be delivered by way of an IV drip. The active ingredient in the solution is present

in 5 mg per 100 mL. The patient is prescribed 500 mg of the active ingredient.
a Calculate the amount of solution required.
b Calculate the length of time that it will take to administer the solution at 40 drips per minute given
that 5 drips is equal to 1 mL.

Ex tended
R es p ons e

2 The life expectancy of a child born in a certain country has been recorded every 20 years since 1800.

The results are shown in the table below.


Years since 1800
0
20
40
60
80
100
120
140
160
180
200

Life expectancy
52
52
53
53
55
60
63
63
67
72
82

a Plot the data on a scatterplot and fit a least squares line of best fit to the data.
b Find the equation of the least squares line of best fit and draw this on your scatterplot.
c Is the data linear? Explain your answer.

Digital doc
doc-11108
Test Yourself
Chapter 13

Chapter 13 Mathematics and health 395

ICT activities
13A Drawing scatterplots

13F Life expectancy

INTERACTIVITY
int-2789: Scatterplots. (page 361)

DIGITAL DOC
WorkSHEET 13.2 (doc-11107): Apply your knowledge of health
statistics to problems. (page 390)

13BCorrelation
INTERACTIVITY
int-0183: Pearsons product moment correlation coefficient.
(page 371)

13C Fitting a regression line


DIGITAL DOC
WorkSHEET 13.1 (doc-11106): Apply your knowledge of health
statistics to problems. (page 381)

396 Maths Quest HSC Mathematics General 2

Chapter review
DIGITAL DOC
Test Yourself (doc-11108): Take the end-of-chapter test to test your
progress. (page 395)

To access eBookPLUS activities, log on to www.jacplus.com.au

Answers chapter 13
Mathematics and health
Exercise 13A

5 a

Drawing scatterplots

30
Hair length (cm)

250

Reach (cm)

200
150

20
15
10
0

Waist (cm)

25

100
50

35

50

68
67
66
65
64
63
62
61
60
59
58
57
165

170

150
100
Height (cm)

180
175
Height (cm)

200

185

250

50
40
30
20
Forearm length (cm)
b There is no relationship between the length of a persons forearm
and the length of their hair.
6 D
7 A
8 a Examination results against hours studied.
b Weight loss (%) against time dieting.
9 a Positive relationship
b Positive relationship
c Positive relationship
d No relationship
e Positive relationship
10 a Positive linear relationship
b Positive linear relationship
c Negative linear relationship
d Negative linear relationship
e No relationship
f Negative non-linear
relationship
11 a y
b y
10

190

25
Hand span (cm)

3 a

20

y

d

c y

15
10
x

5
0

30
25
20
15
10
Foot length (cm)
b There appears to be a positive linear relationship. People with
longer feet tend to have wider hand spans.
0

y

f

e y

2.5

4 a

x
x

Skin area (m2)

y

h

g y

1.5
1
x

0.5
0

i y

2
1.8
1.6
1.4
Height (m)
b There appears to be a positive linear relationship. That means that
taller people tend to have a larger skin surface area.
1

1.2

12 a m = 5, b = 12

b m = 0.4, b = 70 c m = 100, b = 750

Chapter 13 Mathematics and health 397

6 a

L
550
500
450
400
350

Population ( 1000)

Length

13 a

0 123456789 f
Force

b L = 10f + 440
c The vertical intercept is the length of the elastic with no force,

8000
7000
6000
5000
4000
3000
2000
1000
0

and the gradient is the amount by which the elastic is stretched by


each unit of force.

Exercise 13B

Correlation
b Positive

7
6
5
4
3
2
1
0

c Positive

1000

1500

2000

2500

3000

b There is no correlation between area and population hence no


7 a

conclusion can be made.


0.12

0.1

10

70
60
50
40
30
Age (years)
b There is a negative correlation which means that as people get
older the number of speeding offences they commit reduces.
3 a Weak positive correlation
b Moderate positive correlation
c Strong negative correlation
d Weak negative correlation
e No correlation
f Moderate negative correlation
g Strong positive correlation
4 a
700
Number of pies sold

500

Area ( 1000 km2)

Reaction time (s)

No. of speeding offences

1 a Negative
2 a
8

20

600
500
400
300
200
100
0

10
30
20
15
25
Temperature (C)
b There is a negative correlation. The warmer the temperature the
fewer pies will be sold.
5 a
18
16
14
12
10
8
6
4
2
0
60
50
40
30
10
20
0
Hours spent each week in air-conditioned buildings
b There is a weak positive correlation. Conclusion that the more
time spent in air-conditioned buildings the more likely a person is
to have time off work.
c There can be no conclusion here about causality. Check your
reasoning with your teacher.
5

Number of sick days

398 Maths Quest HSC Mathematics General 2

0.08
0.06
0.04
0.02
0

0.5
1
Amount of drug (mg)

b The graph shows a strong positive correlation. We can

1.5

concludethe higher the dose of the drug the longer the reaction
time will be.
8 a Perfect positive
b Weak positive
c Strong positive
d Moderate negative
e Weak positive
f No correlation
g Strong negative
h Moderate negative
i Weak negative
j Perfect negative
9 There is a moderate positive correlation.
10 There is a weak negative correlation.
11 a No correlation
b There is no evidence to connect money and happiness which
confirms the statement.
12 a There is a weak negative correlation between obesity and
exercise.
b There is little evidence that connects the amount of exercise and
obesity.
13 B
Exercise 13C

Fitting a regression line

1 y = 0.73x + 1.08
2 y = 0.28x + 23.06
3 a Mean = 162.9, SD = 9.58
b Mean = 66.3, SD = 11.57
c Weight = 0.88 height 76.96

Weekly sales ( 1000)

8 a

Length of spring (mm)

2.5

3.5

2
1.5
Price ($)
c N = 20.08p + 133.19 d
 i 71 000
ii 41 000
e $2.65
f
$1.65
9 a
250

2.5

3.5

0.5

140
120

1000
800
600
400
Mass (g)
b m = 0.05, b = 220.1 L = 0.05m + 220.1
c
270
265
260
255
250
245
240
235
230
225
220
215
1000
800
600
200
400
0
Mass (g)
5 a
70
60
50
40
30
20
10
0
10
20
5
0
40
35
30
25
15
Circumference of biceps (cm)
b S = 1.2B + 21.62
6 a 159.7 cm
b 178 cm
c 31.15 cm
d 25.74 cm
7 a 755
b 295
c 20.43C
d 37.82C
Lift test (kg)

140
120
100
80
60
40
20
0

2
1.5
Price ($)

Weekly sales ( 1000)

270
265
260
255
250
245
240
235
230
225
220
215

200

100
80
60
40
20
0

length of carapace (mm)

Length of spring (mm)

4 a

0.5

200
150
100
50
0

b L = 14.38A + 21.18
d 251.16 cm
f 107 cm
10 B
Exercise 13D

10
8
Age (years)
c
93 cm
e
3 years

12

14

16

Units of measurement
b
200 g
d
250 mg
f
400 000 mg
b
0.6 g
d
0.005 g
b
400 mg
d
25 mg
b
0.112g
b
8 tablets
d
No, this equals the daily allowance.
b
3 mL

1 a 3kg
c 8 000 000 mg
e 20 000 mg
2 a 4 g
c 0.0125 g
3 a 5000 mg
c 1200000 mg
4 a 112 mg
5 a 1000 mg
c 2000 mg
6 a 5 days
7 20

Chapter 13 Mathematics and health 399

4 a

slightly.

d On an empty stomach the side effects of the drugs will be

stronger.

e 2 tablets will exceed the amount suggested on Voltaren. Hence

only a doctor can prescribe the higher dosage

9 2 grams
10 a 60 b
129.6 g

c 6.7 days
10 a 250 mL / h b
21 drips/min

Life expectancy

1 a 51 years
b 83 years
c Yes (approximately)
d This period coincided with the Great Depression and World War II
e It is approximately linear; however, this cannot continue

indefinitely as there must be a limit on lifespan.

2 a Vic., WA, ACT


b ACT
c NT
d In the ACT there are a lot of public servant with

comfortablelifestyles and of a high standard of living. The


NT has a high indigenous population and a lower standard of
living.This is an issue that Australian governments need to
address.
3 a 72.8
b  i53.1 ii37.7
c Life expectancy is an average and includes those who die at a
young age. Therefore as you get older the average will become
higher.

90
80
70
60
50
40
30
20
10
0

18
8
18 11
9 8
19 11 90
0 9
19 11 00
2 9
19 01 10
3 9
19 21 22
4 9
19 61 34
5 9
19 31 48
6 9
19 51 55
7 9
19 51 67
8 9
19 51 77
9 9
20 51 87
0 9
20 42 97
07 00
2 6
00
9

Male life expectancy

5
10
15
20
Spending on health (% of GDP)
b r = 0.49 There is a weak (almost moderate) positive correlation
c L = 1.79S + 57.25
d It will hopefully increase life expectancy by about 10 years but
the impact of this we would not expect to be immediate.
5 a
90
80
70
60
50
40
30
20
10
0
150
100
50
0
Infant mortality rate
b r = 0.88. There is a strong negative correlation
c L = 0.32M + 79.83
d The Zimbabwe data is irregular as it has a low life expectancy but
low infant mortality.
e 30.9
6 a Check with your teacher
b There is a moderate negative correlation
c Check with your teacher
d Your answer is likely to be unrealistic as it would be too high if
no one smoked.
e The gradient is the decrease in life expectancy for every 1%
increase in smoking

Chapter Review
Multiple Choice

at 0
at 25
at 45
at 65

1 D

2 C 3D

Short answer

Minimum
temperature (C)

Exercise 13F

90
80
70
60
50
40
30
20
10
0

Life expectancy (years)

Dosage rate

1 4.8 mg
2 a 2.4 mL b
4.7 mL
c 2.1 mL d
5.6 mL
3 12.5 mL
4 a 105 mg b
220 mg
c 320 mg d
170 mg
5 7 mL
6 a 13 mg b
16 mg
c 45 mg d
17 mg
7 a 150 mL b
37.5 min
8 6 hours and 34 min
1
9 a 1 b
10 000 mg

2 a

Number of sick days

Exercise 13E

Life expectancy (years)

8 a 4 times stronger b
75 mg
c 2 tablets at 7.00 am, 2.00 pm and 10.00 pm but answers will vary

20
10
0
0 20 40
Maximum
temperature (C)

12
8
4
0

0 2 4 6
Number of children

b There appears to be a positive relationship which is linear.

400 Maths Quest HSC Mathematics General 2

4
2
0

0
2
Number of cars

b There is no apparent relationship.


4 a, b y

Extended response

80
40
0

10 a 3700 mg
b 0.85 g
c 1 100 000 mg
11 a 240 mg
b 6
c 1500 mg
12 a 6 mL
b 6 mL
c 9 mL

16

c y = 99 5x
5 a Negative
b Positive
c Negative
6 a None
b Perfect positive
c Weak negative
d Strong negative
e Weak positive
7 a Moderate negative
b Older people have newer cars.
8 a x values Mean = 7.5, SD = 4.5 : y values Mean = 15,

SD = 5.8

b r = 0.93
c y = 1.2x + 24
9 a r = 0.93
b Strong positive correlation
c R = 14D + 157
d Gradient is the daily increase in red blood cell count, vertical

1 a 10 L
b 20 hours 50 min
2 a
90

Life expectancy

Number of
televisions

3 a

80
70
60
50
40
30
20
10
0

50
100
150
200
Number of years since 1800

250

b L = 0.14Y + 47
c The data does not appear linear as there is a curved upward trend.

intercept is the initial red blood cell count.

Chapter 13 Mathematics and health 401

ICT activities
Life expectancy
SEARCHLIGHT ID: PRO-0152

Scenario
You are asked to delve into the issue of life expectancy
and examine the factors that have an impact upon
it. The focus of your research is Australia and the
subgroups within our country. You will need to focus on
where Australia fits on a global scale.
You will need to analyse factors that impact upon life
expectancy. Many of these are economic factors, but
you will also look at health and social factors, and the
prevalence of political issues such as war.
Also evident are some key differences in life
expectancy data. Explore these further by considering
the following questions.
1. What is the current life expectancy of a child born in
Australia today?

402 Maths Quest HSC Mathematics General 2

2. Research life expectancy among the following


subgroups.
(a) Men versus women
(b) Smokers versus non-smokers
(c) Indigenous peoples versus non-indigenous
peoples
(d) Immigrants versus those born in Australia
(e) State-by-state differences
3. Find out where Australia currently ranks in life
expectancy and list the top ten and bottom ten
nations for life expectancy.
4. There are many factors that have an impact on life
expectancy. Research the following factors to find
correlations between life expectancy and each factor.
(a) health spending and facilities
(b) education
(c) GDP
(d) birth rate.
5. Find out the birth rate in a group of 20 countries and
compare this to infant mortality in the selected
countries. (Infant mortality is defined to be the number
of children per 1000 who die before their first birthday.)

Task
1. Watch the introductory video.
2. Identify the key sources of data on life expectancy.
This will include sources from within Australia but
also from outside. It will be important to compare the
results found to verify accuracy.
3. Identify the key factors in society that will have
an impact on life expectancy. This will include
geographical location, the prevalence of smoking, the
level of education and government spending on health.
4. Identify the key subgroups in Australian society and
look for differences in their life expectancies. Try to
identify what the cause of any differences is.
5. Compile your findings on an Excel spreadsheet.
Use the spreadsheet functions to draw graphs
and perform statistical analysis such as finding the
correlation coefficient between data sets.
6. Present a report in the form of your choice that shows
trends and differences both within Australia and with
the rest of the world.

Process
Open the ProjectsPLUS application for this chapter in
your eBookPLUS. Watch the introductory video, click
the Start Project button and then set up your project
team. You will invite another member of your class
to form a partnership. Save your settings and the
project will be launched.
Your teacher will have access to a number of
websites that will get your research started. Use
these to analyse the available data and find the
answers to the key questions.
You will need to access the details of your home
internet connection. This may be available in the
contract or on your internet providers website.
Import the data into an Excel spreadsheet.
Use the spreadsheet to create suitable graphs to
display the information.
Create a summary of your findings to present to an
audience in an appropriate format (e.g. a PowerPoint
presentation or a movie).
Write a report that includes the results of all your
findings. Submit this to your teacher
forassessment.

Suggested software



ProjectsPLUS
Microsoft Word
Microsoft Excel
Microsoft PowerPoint

ICT activities projectsplus 403

Chapter 14

Mathematics and resources


CHAPTER CONTENTS
14A Interpreting information about water usage
14B Collecting and using water
14C Dams, land and catchment areas
14D Energy and sustainability

Interpreting information about


water usage
14A

Water availability and usage


Have you ever thought about your water use?

Elesson
eles-1615
Water: A vital
source
eles-1616
A world of water

You may have seen advertisements on television asking for us all to conserve water. Unless you pay
the water bill it is unlikely that you actually know how much water that you, or members of your
household,use.
Chapter 14 Mathematics and resources 405

Consider the water bill shown below.

Water account
ACCOUNT DETAILS
Date of issue:

22/11/2013

Amount due:

$268.00
21/12/2013

Please pay by:

YOUR ACCOUNT SUMMARY


Property details

YOUR HOUSEHOLD WATER USE COMPARED

(Res) Single Dwelling/Vac:Land

No. of people
in household

Garden
size
none
small
medium
large
none
small
medium
large
none
small
medium
large
none
small
medium
large
none
small
medium
large
none
small
medium
large

Supply period
From: 14/08/2013
To:

14/11/2013

Balance brought forward:


Current charge:

$0.00
$268.80

Amount due:

$268.80

Your average daily water consumption comparison


700

Your daily average

Typical
water use
120
220
200
344
322
390
457
509
417
550
828
674
520
723
706
854
858
949
1004
1107
784
1254
1437
1582

554 L

Efficient
water use
100
147
100
176
206
250
263
279
309
353
366
382
412
456
469
485
515
559
572
588
616
662
675
691

Your personal target*

0L

Litres per day

600
500
400

Description

This account Previous acct.

Change

300
200

Total usage

100

Days in period

Current
period

Same
period
last year

Previous
account

Your
Your local
water
area
consumption average

Your
personal
target

Average daily usage

51 L

36 KL

92

90

554 L

400 L

154 KL

1000 litres (L) = 1 kilolitre (kL)

Amount due:

$268.80

This bill contains a lot of information about:


the amount of water used in this billing period
the amount of water used in the corresponding billing period last year
the amount of water used in the previous account
the amount of water that is used by typical and efficient households of different sizes.
WORKED EXAMPLE 1

Look at the water bill shown above.


a What is the amount of this bill?
b How many litres of water are being used per day?
c How does this compare with the same period last year?
THINK

a Identify the amount due. You will find this

information at the top right-hand corner of the


bill under Account details.

406 Maths Quest HSC Mathematics General 2

15 KL

WRITE

a Amount due = $268.80

b This information can be seen from the graph or

from the table.


c The graph shows this information. The

b Daily usage = 554L


c There has been an increase of about

amount used this year is 554L compared to


approximately 390L at the same time last year.

164Lperday.

To manage water usage it is important to have an understanding of the amount of water that is used for
various tasks.
The table below shows the amount of water typically used in undertaking various household activities.
Water usage
Shower

18L per minute

Toilet

12L per flush

Garden hose

20L per minute

Dishwasher

40 litres per load

Bath

100L per bath

Tap

10L per minute

Washing machine
Top loader
Front loader

170 litres per load


80 litres per load

Sprinkler

20L per minute

Fill a swimming pool

55000L

WORKED EXAMPLE 2

The Warren household has four members and have eight showers a day, in total.

Given that each shower takes an average seven minutes calculate:


a
the number of litres of water used each day for showers.
b
The amount of water that can be saved each year by cutting back to 5-minute showers.
THINK

a 1 Calculate the total number of minutes the

shower is running each day.


2

Multiply the number of minutes by 18L


perminute to find the total water usage.

b 1 Calculate the number of minutes of

showering saved per day.

WRITE

a Minutes of shower = 8 7

= 56

Amount of water = 56 18
= 1008L
b Time saved = 2 8

= 16 minutes

Chapter 14 Mathematics and resources 407

Calculate the water saved each day by


multiplying the number of minutes by the
number of showers taken daily.

Water saved daily = 16 18


= 288L

Calculate the water saving each year by


multiplying the number of litres used daily by
the number of days in a year.

Water saved yearly = 288 365


= 105120L

Rainfall is the source of fresh water that we all use. Hence, in


periods of drought, the government will impose restrictions to
conserve water.
Rainfall is measured in millimetres. This is the depth to which
a container would be filled if it were left out in the rain. Accurate
measurements of this are made using a device called a rain gauge.
In this exercise you will also have to interpret information about
rainfall from a rainfall graph such as that shown in the following
example.

WORKED EXAMPLE 3

Dec.

Nov.

Oct.

Sept.

Aug.

July

June

May

Apr.

Mar.

Feb.

160
140
120
100
80
60
40
20
0

Jan.

Precipitation (mm)

The graph below shows the average rainfall in Sydney, each month of the year.

Use the graph to find:


a the wettest month of the year
b the amount of rain in June.
THINK

a The month with the highest rainfall is the

WRITE

a March has the highest rainfall.

monthwith the tallest bar on the graph.


b Select the bar on the graph that represents

the month of June and then read the height


ofthatbar.

408 Maths Quest HSC Mathematics General 2

b Average June rainfall = 125mm

Interpreting information about


waterusage
Exercise 14A

1 WE1 Use the water bill on page 406 to answer the following.
a How much more water is used for this account than the previous account?
b How does water consumption for this household compare to the local average?
c What is the cost per kilolitre of water?
2 WE2 The Tran household has five members and have altogether, six showers a day. Given that each

shower is an average six minutes and uses 18L of water per minute, calculate:
a the number of litres of water used each day for showers
b the amount of water that can be saved each year by cutting back to 4-minute showers.
3 A family has 4 members who have an average four showers each day, each using 18L of water per

minute. Each shower lasts an average of 10 minutes.


a Calculate the amount of water used in showers each day.
b If a water-saving showerhead is installed the shower will only use 12L per minute.

Calculate the amount of water saved if a water-saving showerhead is installed and the length of
showers are cut back to 6 minutes.
4 The Chang family have a dishwasher. After dinner each night the dishwasher is only half full but is

used anyway.

a If a tap is run for 3 minutes to rinse all the plates before loading and the dishwasher is then run,

calculate the amount of water used.


b Calculate the water saving per year if the plates are left in the dishwasher after rinsing and the

machine is only used when full.


Chapter 14 Mathematics and resources 409

5 Kevin waters his garden for half an hour every day.

Calculate the amount of water that he uses every year.


6 MC The water used in a bath is closest to that used in a shower of length:

A 4 minutes

B 5 minutes

C 6 minutes

D 8 minutes

7 Tracey does the washing in her house twice a week. Calculate the annual water saving in her

household if she replaces her top-loading washing machine with a front-loading machine.
8 Brett and Samantha install a swimming pool in their backyard.

a Calculate the cost of filling the swimming pool at a cost of $1.55/kL.


b Every week the pool loses water that is splashed out. The pool is topped up by running the garden

hose for 15 minutes. Calculate the annual cost of keeping the water in the pool topped up.
410 Maths Quest HSC Mathematics General 2

9 WE3 The graph below shows the rainfall in Sydney for a particular year and compares it to average

rainfall.
200405 monthly rainfall

180

Long-term monthly average rainfall

Rainfall/evaporation (mm)

160
140
120
100
80
60
40
20
0

July

Aug.

Sept.

Oct.

Nov.

Dec.
Jan.
Month

Feb.

Mar.

Apr.

May

June

What was the wettest month of the year?


What is normally the wettest month of the year?
In what two months was the citys longest dry spell?
What was the rainfall in January?
How much higher or lower was rainfall in January than average?
In which month would you say the amount of rain differed most from normal? Explain your
answer.
g List the months in which rainfall was above average.
a
b
c
d
e
f

Further development
10 The table below shows state-by state and industry-by-industry water usage. (Note: 1 GL = 1000000L)

ACT NSW
Water consumption in 200405 (GL)
Agriculture
1 4,133
Forestry and fishing
11
<1
Mining
63
<1
Manufacturing
126
<1
Electricity and gas supply
76
Water supply
5
631
Other
17
310
House hold
31
573
Total
56 5,922

NT

State or Territory
Qld
SA
Tas. Vic.

47
<1
17
6
1
8
30
31
141

2,916 1,020
3
<1
83
19
158
55
81
3
426
71
201
52
493
144
4,361 1365

258
4
16
49
<1
20
18
69
434

WA

3,281 535
8
25
32 183
114
81
99
13
793 128
262 168
405 362
4,993 1495

Australia
12,191
52
413
589
271
2,083
1,059
2,108
18,767

Which state uses the most water?


What industry uses the most water?
What percentage of all water used is used in NSW?
What percentage of all water used is used by households?
In which state is the most water used in the supply of electricity and gas? Give a possible reason
for this.
f What percentage of all water used by the mining industry is used in Western Australia?
a
b
c
d
e

Chapter 14 Mathematics and resources 411

11 The table below shows information about the number of households connected to the water supply

and the amount of water used in most major Australian cities.

Inflows
(GL)

City

Canberra, incl.
236
Queanbeyan
Brisbane
1,614
Hobart
4,632
Melbourne
1,752
Darwin
795
Adelaide
214
Sydney
1,446
Perth
1,496
Total
12,185

Extraction/
diversions
(GL)
68
325
40
543
62
250
861
411
2,560

Total urban
water
supplied
(GL)
52
255
41
431*
35
166
526
237
1,743

Population
Water
receiving
Total
supplied per
water supply connected total connected
services
properties properties (kL/
(000s)
(000s)
property)
363

136

355

975
188
3,583
101
1,095
4,228
1,484
12,017

420
83
1,533
43
492
1,685
649
5,041

419
499
281
799
336
312
347
419

a Which city has the greatest inflow of water?


b In which city is the greatest amount of water per household used? What might be an explanation

for this?
c Which is the most water-wise city in Australia?

14B
INTERACTIVITY
int-3077
Waterworks: Learn
how the water
cycle works
int-3078
Water benefits us
int-3079
Thurtila Pula: Learn
how Indigenous
Australians use
water

Collecting and using water

One way in which we can all conserve water is by


collecting rainwater. All new homes in NSW must
now have the facilities to allow rainwater to be
collected. Rain is collected from the roofs of houses.
We need to be able to calculate the amount of
water that can be collected from various roofs in
order to determine what size tank would be suitable.
The ability to analyse water collection and usage
is important for all of us, as this precious resource
must be conserved. It is especially important for
people who live in country areas, not close to towns,
who rely on rainwater for all their household water.
Consider a household that is collecting water from their rectangular roof that measures 20 metres by
10 metres.
Now suppose that 10mm of rain falls, how much water is collected?
We consider that if all this water remained on the roof it would fill like a rectangular prism.

412 Maths Quest HSC Mathematics General 2

Using the formula for the volume of a


rectangular prism
V = lbh
= 20 10 0.01 (10mm = 0.01m)
= 0.2m3
= 200L (using 1m3 = 1000L)
20 m
The importance of water collection is
demonstrated here when you consider the water bill for the household looked at in the previous section.
This is not even half of their daily average use.

10 mm
10 m

WORKED EXAMPLE 4

A house collects rainwater from the roof of the house.


The roof is 18 metres long by 12 metres wide. Calculate the
amount of water collected from 24mm of rain.

THINK

WRITE

Convert 24mm to metres.

24mm = 0.024m

Find the volume of the rectangular prism.

V = lbh
= 18 12 0.024
= 5.184m3

Convert to litres (using 1m3 = 1000L).

= 5184L

This water that is collected needs to be stored in a tank.


Most large tanks are cylindrical but many are now being designed to
look good especially as water collection is compulsory with all new
homes built.

WORKED EXAMPLE 5

A cylindrical tank has a diameter of 2.6 metres and a height of 2.1 metres. Calculate the capacity
of the tank in litres.
THINK
1

Write the formula for the volume of a cylinder.

Substitute the values for r and h.

Calculate the volume.

Convert to litres (using 1m3 = 1000L).

WRITE

V = r2h
= 1.32 2.1
= 11.15m3
Capacity = 11150L

Chapter 14 Mathematics and resources 413

Exercise 14B

Collecting and using water

1 WE 4 A homeowner collects water from the roof of her house. The roof is 16 metres long by

14metres wide. Calculate the amount of water collected from 25mm of rain.
2 Rainwater is collected from a flat rectangular carport roof.

The dimensions of the carport are 8 metres by 5 metres. Calculate the amount of water collected
from 32mm of rain.
3 The figure below shows the dimensions of a roof.

12 m

10 m

10 m

20 m
a Calculate the area of the roof in square metres.
b Calculate the amount of water that would be collected if 60mm of rain falls.
4 WE 5 A cylindrical tank has a diameter of 2.6 metres and a height of 2.1 metres. Calculate the

capacity of the tank in litres.


5 Find the capacity in litres of the water tank below.
m

2540 m

2200 mm

414 Maths Quest HSC Mathematics General 2

6 Find the capacity in litres of the water tank below. Answer correct to the nearest 10L.
2510 mm 780 mm 1910 mm

7 The Justice family live on a property and rely

on tank water completely.


They use on average about 400kL of water
per year.
a Calculate their average daily water usage
in litres.
b To store water they have three cylindrical tanks
3 metres in diameter and 2 metres in height each.
Calculate the capacity of each tank.
c Calculate the number of days that the tank
water will last the family when the tanks
are all full, and if there is no rain.
8 A cylindrical water tank has a diameter of 1400mm and a height of 1500mm. Calculate the amount
of water in the tank when it is half full.
9 A cylindrical water tank has a diameter of 2 metres and a capacity of 10000 litres. Calculate the
height of the tank.

Further development
10 A house has a rectangular roof of length 20 metres and width 12 metres.
a Calculate the amount of water collected after 75mm of rain.
b The water collected is funnelled into a cylindrical water tank 2 metres in diameter. Calculate the

amount by which the depth of water in the tank will rise.


11 A rectangular roof that is 15 metres wide and 18 metres long collects rainwater which is funnelled
into a cylindrical tank that is 1.8 metres in diameter and has a height of 2 metres.
a Calculate the capacity of the tank in litres.
b When the tank is installed it is empty. Calculate the number of millimetres of rain that must fall in
order to fill the tank.

14C

DIGITAL DOC
doc-11109
WorkSHEET 14.1

Dams, land and catchment areas

Dams or reservoirs are the main source of water for


cities and most large communities.
Rainwater and rivers supply water to the dams,
which in turn have water piped from them into
homes.
The exact size of most dams is difficult to
measure so we need to use a variety of methods
such as scale drawings, and Simpsons rule to
estimate the size of dams, which are almost always
irregular in shape.

Chapter 14 Mathematics and resources 415

WORKED EXAMPLE 6

The figure below shows an image of an Olympic sized


swimming pool.
It is known that the length of the pool in reality is
50 metres. Calculate the scale used.
THINK

WRITE

2 cm

Measure the length of the pool.

Pool length = 2cm

Write the map length and actual


length as a ratio.

Scale = 2cm : 50m

Simplify the ratio.

= 2 : 5000
= 1 : 2500

The ProjectsPLUS activity on pages 4323 will help you to explore issues in relation to water
catchments.
Earlier we considered rainfall and how it is collected for rainwater tanks. On a larger scale we can
consider dams and the amount of water they collect. While roofs generally have a regular shape, dams
do not and so their area needs to be approximated using Simpsons rule.

Simpsons rule
Simpsons rule is a method used to approximate the area of an irregular
figure. Simpsons rule approximates an area by taking a straight boundary
and dividing the area into two strips. The height of each strip (h) is measured.
Three measurements are then taken perpendicular to the straight boundary,
as shown in the figure on the right. The formula for Simpsons rule is:
h
A = (df + 4dm + dl)
3
where h = distance between successive measurements
df = first measurement
dm = middle measurement
dl = last measurement

dm

dl

df
h

WORKED EXAMPLE 7

The figure below shows a small dam that is used to irrigate a golf course.

36 m
5m

1m
40 m

40 m
28 m

8m

36 m

aBy considering the top half and the bottom half of the diagram separately use Simpsons rule

to estimate the surface area of the dam.

b Then, estimate the amount of water that is collected in a week where 250mm of rain falls.
THINK

a 1 Use Simpsons rule to find the area as shown in the top half

of the diagram.

416 Maths Quest HSC Mathematics General 2

WRITE

h
[df + 4dm + dl]
3
40
= [5 + 4 36 + 1]
3
= 2000m2

a A

Use Simpsons rule to find the area as shown in the bottom


half of the diagram.

h
A [df + 4dm + dl]
3
40
= [28 + 4 36 + 8]
3
= 2400m2

Add the two areas together.

Area = 2000 + 2400


= 4400m2
b 250mm = 0.25m

b 1 Convert 250mm to metres.


2

Use the volume formula V = A h to calculate the volume.

Use 1m3 = 1000L to calculate the amount of water.

Exercise 14C

V=Ah
= 4400 0.25
= 1100m3
= 1100 1000L
= 1100000L

Dams, land and catchment areas

100 m

1 WE6 The photo below is an aerial shot of the Sydney Football stadium.

The distance between the try lines is 100 metres. Find the scale on the photo.
2 The photo below is an aerial shot of the Sydney Cricket Ground.

20.2 m

Chapter 14 Mathematics and resources 417

The cricket pitch is 20.2 metres long.


a Calculate the scale on the photo.
b Use the scale to calculate the length and width of the entire ground.
c By averaging the length and width of the ground consider the ground to be a circle and estimate
the circumference of a ground.
d Use the area of an ellipse formula to find the area of the ground.
3 The photo below is of Warragamba Dam, which is the major water storage dam in Sydney.

The shape of the dam is almost a trapezium.


a Use the area of a trapezium together with the scale to estimate the surface area of the dam.
b Calculate the amount of water that would be collected by 25mm of rainfall.
418 Maths Quest HSC Mathematics General 2

4 The satellite image below shows Porters Creek Dam, located in the Morton National Park.

The scale is 1cm : 200m. Rule up a page of transparent paper into squares of side
length 1cm.
a What is the area of 1cm2 on your paper equivalent to on the photo?
b Use your transparent paper to estimate the surface area of Porters Creek Dam.
c If the average depth of Porters Creek Dam is 20 metres, estimate the capacity of the dam in
megalitres.

Chapter 14 Mathematics and resources 419

5 WE7 The figure below is of a dam.

38 m
6m
35 m

10 m
60 m

60 m

28 m

26 m

a By considering the top half and the bottom half of the diagram separately use Simpsons rule to

estimate the surface area of the dam.


b Then, estimate the amount of water that is collected in a week in which 150mm of rain falls.

Further development
6 The photo below is of Mollymook Beach in NSW.

70 m

1000 m
110 m

1000 m
40 m

a Use Simpsons rule to estimate the area of water enclosed by the two headlands as shown.
b To improve the estimate of the area two more offsets are taken. The length of these offsets is

80metres (taken halfway between the 40m and 110m offset) and 120 metres (taken halfway
between the 100m and 70m offset).
Calculate a new estimate for the area by applying Simpsons rule twice.

420 Maths Quest HSC Mathematics General 2

14D

Energy and sustainability

Earlier in the chapter we looked at water saving and information that is on a water bill that can help us
to conserve water. In this section we will be looking at power bills and completing a similar task about
conserving electricity.
Consider the electricity bill below.

Energy account
Account summary: 10 Aug. 13 to 09 Nov. 13

Account no.:

Opening balance

$610.09

Payments received thank you

$610.09CR

Balance carried forward

$0.00

385 098 456 328

Total amount due:


Debited on:

$374.29
30 Nov. 13

New charges
Total electricity charges incl. discounts

$374.29

Total amount due incl. overdue amount

$374.29

New charges incl. GST charges of

$34.03

Payments received after the due date may incur a fee of $12.00
Average cost per day:

$4.07

Average daily usage:

12.66 kWh

Same time last year:

12.36 kWh

Indicative greenhouse gas emissions (tonnes)


Generated this account:

0.9 t

Same time last year:

0.9 t

Saved with a Green Product:

0.3 t

The bill shows not only the amount owing but also the amount of electricity used, the average daily
usage and makes comparisons with the same period last year.
Investigate: Electricity bills

Answer the following questions relating to the sample electricity bill above
1. What is the amount of the bill?
2. What was the average cost per day?
3. What was the total number of kWh used?
4. Research what off peak energy is. Explain why it is cheaper?
5. Research what Green Power or Green Energy is.
6. Collect a sample of electricity bills among the members of your class. Complete a set of summary
statistics for power usage and as a group discuss your findings. How do households in the sample
compare?
As can be seen on the bill, power is measured in kilowatt hours (kWh).
As we know from previous work on measurement theprefix
kilo means 1000 times larger and so 1 kilowatt = 1000 watts.
1 kilowatt hour is a measure of the amount of power used
in running an appliance of 1 kilowatt for one hour.
Each electrical appliance has a wattage which is the power
taken to run the device. Consider for example
1. A 50-watt light bulb
2. A 2400-watt kettle
Chapter 14 Mathematics and resources 421

The kettle is a heating device and hence uses a lot more power then the light bulb.
Time taken to use 1 kWh of power.
50 watt light bulb = 1000 502400 watt kettle = 1000 2400

= 20 hours
= 0.4167 hours

= 25 minutes
So as can be seen here boiling water in the kettle for 25 minutes uses the same amount of power as
having a 50-watt light bulb burn for 20 hours.
WORKED EXAMPLE 8

A television runs on 220 watts of power.


On average the television is watched for 5 hours per day.
a Calculate the number of kWh used.
b Calculate the cost of running the television at 15c/kWh.

THINK

WRITE

a 1 Calculate the total number of watts

used.
2

a Watts = 220 5

= 1100
= 1.1 kWh

Divide by 1000 to find the number of


kWh used.
Calculate the cost by multiplying
by0.15.

a Cost = 1.1 0.15

= $0.165
Note: We can leave this as a decimal with three decimal
places. All electricity costs are metered and only
rounded to the nearest cent in the final calculation.

The table below shows the approximate wattage of a number of household devices.
Device

Wattage

Incandescent light blub

80 watts (but varies greatly)

Compact fluorescent light blub

25 watt

Microwave

850 watts

Television

200 watts

Fan

50 watts

Refrigerator

250 watts

Heater

2000 watts

Washing machine

800 watts

Computer

120 watts

Iron

500 watts

Music system

30 watts

When a major appliance is purchased in store it will generally have an energy-rating sticker on it. The
more stars the appliance has the more energy efficient the appliance is. The key figure on this sticker is
the number in the middle. This represents the number of kilowatt hours.
422 Maths Quest HSC Mathematics General 2

WORKED EXAMPLE 9

The following energy sticker shows a


refrigerator energy consumption.
Calculate the cost of running the refrigerator
per year at an average 18c/kWh.

THINK

505
kWh
per year

WRITE

Multiply the annual kWh by the costperkWh.

Exercise 14D

Energy
consumption

Cost = 505 0.18


= $90.90

Energy and sustainability

1 Complete each of the following conversions.


a 1000 W = ___ kW
b 2400 W = ___ kW
d 1.2 kW = ___ W
e 60 W = ___ kW

c 0.8 kW = ___ W
f 2100 W = ___ kW

2 One megawatt (MW) is equal to 1 million watts or 1000 kilowatts. Complete each of the following

conversions giving your answers in scientific notation.


a 2.5 MW = ___ W
b 12 MW = ___ W
3 One gigawatt (GW) =

c 3600 MW = ___ kW

109

watts. In the movie Back to the future Marty McFly is trapped in 1955. He
needs to get back to 1985 and needs 1.21 GW of energy to power the time machine. The only source
of this amount of energy in 1955 is a lightning bolt. Write this amount of energy in:
a MW
b kW
c W.

4 Complete each of the following giving your answer in scientific notation.


a 2 GW = ___ W
b 3.2 GW = ___ kW
c 0.4 GW = ___ W
5 Below is a list of wattages. Calculate the length of time that each will take to use 1 kWh of power.
a 40 watts
b 200 watts
c 450 watts
d 800 watts
e 1600 watts
f 2500 watts
6 Below is the time taken for two appliances to use 1 kWh of energy. Calculate the wattage of the

appliance. (Give your answer to the nearest 10 W.)


a 40 minutes
b 24 hours
7 WE8 A microwave oven of 850 watts is used for 40 minutes to cook a meal.
a Calculate the number of kWh used.
b Calculate the cost of operating the microwave at 15c/kWh.
Chapter 14 Mathematics and resources 423

8 A household has 100-watt light globes in each room.

The lights are on for a total of 15 hours per day.


a Calculate the number of kWh used by the lights in a 3-month
billing period (3 months = 91 days).
b Calculate the cost of the lighting at 15c per kWh.
c Calculate the saving on the power bill if they are all replaced
with energy efficient 40-watt globes.

9 WE9 The figure below shows the energy rating for a television.

Energy
consumption

216
kWh
per year

Calculate the cost of running the television for one year at a cost of 21c/kWh.
10 The figure below is the energy rating label for a dishwasher.

Energy
consumption

317
kWh
per year

a Calculate the cost of running the dishwasher for one year at 18c/kWh.
b The above figure is based on two loads in the dishwasher each day. Calculate the cost of each load

in the dishwasher.
c In a household of two people who only eat breakfast and dinner at home it takes three meals

before the dishwasher is full. Calculate the savingin power if the dishwasher is only used
whenfull.
424 Maths Quest HSC Mathematics General 2

11 The energy sticker below shows a clothes dryers energy rating.

Energy
consumption

206
kWh
per year

a Calculate the cost of running the clothes dryer per year at 18c/kWh.
b The above figure is based on doing one load of drying per week. Calculate the cost of each load in

the dryer.
DIGITAL DOC
doc-11110
WorkSHEET 14.2

Further development
12 The energy sticker on a refrigerator says that it will use 505 kWh of

energy per year.


Given that a refrigerator will operate 24 hours a day 7 days a week.
a Calculate the length of time (in hours as a decimal) for the
refrigerator to use 1 kWh.
b Calculate the wattage of the refrigerator.
c Joan is thinking of purchasing this refrigerator for $1500. She
has the alternative or a refrigerator that has an energy rating of
420 kWh per year. However, this refrigerator costs $2000. Given
that electricity is charged at 21c / kWh calculate the annual
saving in electricity costs of the more energy efficient
refrigerator.
d Calculate the length of time that it will take for the more energy
efficient refrigerator to become the more economical purchase.

InvestigatE: BASIX certificate

It is important that we all try to conserve energy. For this reason when new homes are built they need to
be sustainable. We have already seen that all new homes must collect rainwater but there are other things
that can be done to save energy such as the installation of solar panels.
1. Visit the website of your local council and determine what energy efficient requirements are set down
for new dwellings.
2. What is a Building Sustainability Index (BASIX) certificate?
3. What are the sustainability requirements of obtaining a BASIX certificate?
4. What are the key measures and issues that are looked at by the BASIX?
5. How is the BASIX calculated.

Chapter 14 Mathematics and resources 425

Summary
Interpreting
information about
water usage

Personal water usage is shown by your water bill that is usually sent quarterly.
The amount of water used is measured in and charged by the kilolitre (kL).
1kL = 1000L
Being water-wise will allow you to save water and hence save money.
To do this you need to be aware of the amount of water used in completing certain tasks.
Some examples of the approximate amount of water used by certain tasks are shown on the table
on page 407.

Collecting and using


water

Rainwater can be collected from roof tops and stored in tanks. This method is very important to
many people who live on properties away from major centres and rely on rain water as their main
source of water.
Rainfall is measured in millimetres and is measured by a rain gauge. This is the depth of water that
the water from the rain would fill if it were not absorbed, run off or evaporated.
The amount of water collected by rain can be calculated using the formula V = Ah, where A is the
area of the catchment and h is the amount of rainfall.
Many water tanks are cylindrical. The volume of a cylinder is found using the formula V = r2h.

Dams, land and


catchment areas

Dams are the main supply of water for cities and other major population centres.
Dams are usually irregular in shape and so approximate methods need to be used to estimate
theirarea.
An area can be estimated by placing an overlay grid on the diagram and estimating the area using
the scale.
h
Simpsons rule A [df + 4dm + dl] , can also be used to estimate many areas.
3

Energy and
sustainability

Energy is measured in watts, kilowatts, megawatts and gigawatts.


1 kW = 1000 watts
1 MW = 1000 kW = 1000000 watts
1 GW = 1000 MW = 106 kW = 109 watts
The amount of electricity used is measured in kilowatt hours. This is the amount of energy
needed to run a 1000-watt appliance for one hour.
Most major appliances have a rating label. The more stars the label has the more energy
efficient it is.

426 Maths Quest HSC Mathematics General 2

Chapter review
1 A toilet has a dual flush system. The smaller flush uses 12 litres of water but the larger flush

uses 18 litres of water. In a family of four, each person uses one large flush and two small
flushesperday.Given that water is charged at $1.60 per kilolitre, the cost of toilet flushes
peryearisclosestto:
A $50
B $100
C $150
D $200
2 A rectangular roof measures 16 metres by 11 metres. Rain falls onto this roof and is
funnelled intoa cylindrical water tank 2 metres in diameter. The amount by which the height
ofwater in the tank increases during 25mm of rain is closest to:
A 0.35m
B 0.7m
C 1.4m
D 2.8m
3 An aerial photo is taken and a length that is known to be 150m measures 1cm on the photo. The
scale on the map is:
A 1 : 150
B 1 : 1500
C 1 : 15000
D 1 : 150000
4 2.4 GW equals
A 2.4 106 watts
B 2.4 107 watts
C 2.4 108 watts
D 2.4 109 watts
1 Look at the water bill on page 406.
a What is the amount of this bill?
b How many litres of water are being used per day?
c How does this compare with the last billing period?
d Use the above answers to estimate the amount of the previous bill.

m u lti p l e
c hoic e

S ho rt
a nsw er

2 A tank collects water off the roof of the house. The roof is 16 metres long by 16metres wide.

Calculate the amount of water collected from 18mm of rain in litres.


3 A cylindrical water tank has a diameter of 1.5 metres and a height of 2 metres. Calculate
thecapacityof the tank correct to the nearest 100 litres.
4 A farm house has a rectangular roof of length 18 metres and width 12 metres.
a Calculate the amount of water collected after 45mm of rain.
b The water collected is funnelled into a cylindrical water tank of diameter 3 metres.
Calculatetheamount by which the depth of water in the tank will rise.

Chapter 14 Mathematics and resources 427

5 The photo below shows Flemington Racecourse, where the Melbourne Cup is run.

120

0m

The long straight on the race course is 1200 metres long. Calculate the scale on the photo.
6 The photo below shows Double Bay, which is a bay on Sydney Harbour.

200 m

200 m

300 m
400 m

500 m

Use Simpsons rule to estimate the area enclosed by Double Bay.


428 Maths Quest HSC Mathematics General 2

7 Complete the following conversions.


a 3400 watts = ___ kW
b 0.75 kW = ___ watts

c 0.5 MW = ___ watts

8 A lamp has a 120-watt light globe. In total the lamp is on for 3 hours per day.
a Calculate the kilowatt hours used by the lamp in a 3 month billing period (3 months = 91 days).
b Calculate the savings on the power bill if the bulb is replaced with an energy efficient 30-watt

globe given that electricity is charged at 22c/kWh.


9 The figure below shows the energy label for a clothes dryer.

Energy
consumption

206
kWh
per year

a Calculate the cost of running the clothes dryer for one year at 23c/kWh.
b Given that this is based on the clothes dryer being run for three hours per week calculate the cost

per hour of running the clothes dryer.


1 An air conditioner uses 1200 watts of power.

Jim lives in northern Queensland and has to


run his air conditioner for 6 hours per day
for 4 months (120 days) of the year.
a How many kilowatt hours of energy does
Jim use?
b What is the cost of running the air
conditioner per year at 21c per kWh?
c An electric fan uses 150 watts of power but
he will need to run the fan for 12 hours per
day over the same 4-month period. How
much can Jim save by using his fan instead
of the air conditioner?
2 A roof has an area of 280m2. Water from the roof funnels into a cylindrical tank that has a diameter
of 2 metres and a height of 2.5 metres.
a Calculate the capacity of the tank in litres.
b Calculate the amount of rain needed (in mm) to fill the tank.

Ex tended
R es p ons e

Digital doc
doc-11111
Test yourself
Chapter 14

Chapter 14 Mathematics and resources 429

ICT activities
14A Interpreting information about water usage

14DEnergy and sustainability

ELESSON
eles-1615: Water: A vital source. (page 405)
eles-1616: A world of water. (page 405)

DIGITAL DOC
WorkSHEET 14.2 (doc-11110): Apply your knowledge of the
mathematics of resources to problems. (page 425)

14B Collecting and using water

Chapter review

INTERACTIVITY
int-3077: Waterworks: Learn how the water cycle works. (page 412)
int-3078: Water benefits us. (page 412)
int-3079: Thurtila Pula: Learn how Indigenous Australians use water.
(page 412)
DIGITAL DOC
WorkSHEET 14.1 (doc-11109): Apply your knowledge of the
mathematics of resources to problems. (page 415)

430 Maths Quest HSC Mathematics General 2

Test Yourself (doc-11111). Take the end-of-chapter test to test your


progress. (page 429)

To access eBookPLUS activities, log on to www.jacplus.com.au

Answers chapter 14
Mathematics and
resources
Exercise 14A Interpreting information
about water usage
1 a 15kL
b about 250kL higher
c $5.27
2 a 108L b
13140L saved.
3 a 720L b
432L/day
4 a 70L b
12740L
5 219000L
6 C
7 9360L
8 a $85.25 b
$24.18
9 a October
b March
c April and May
d 80mm
e 10mm lower
f October had 141mm of rain more
than double the average of 70mm.
g October, November, December,
February andJune
10 a NSW b
Agriculture
c 31.6% d
11.2%
e Victoria electricity and gas plants are
located in this state.
f 44.3%
11 a Hobart
b Darwin extremely hot. Lots of people
have pools, use air conditioners etc.
c Melbourne
Exercise 14B

Collecting and using water

1 5600L
2 1280L
3 a 320m2 b
19200L
4 11150L

5 11147L
6 3740L
7 a 1096L b
14137L
c 38 days
8 1154L
9 3.18m
10 a 18000L b
5.7m
11 a 5089.3L b
2mm
Exercise 14C

9 $45.36
10 a $57.06 b
7.8c
c $38.09
11 a $37.08 b
71c per week.
12 a 17.35 hr b
57 W
c $17.85 d
5.47 years

Dams, land and catchment

areas
1 1 : 5000
2 a 1 : 2020 b
447.68m
c 15355m2
3 a 795000m2 b
198750kL
4 a 40000m2
b 360000m2 (answers may vary)
c 7200 ML
5 a 6700m2 b
1005kL
6 a 183315m2 b
188337m2
Exercise 14D

Multiple Choice

1 B
2 C
3 C
4 D
Short Answer

Energy and sustainability

1 a 1 kW b
2.4 kW
c 800 W d
1200 W
e 0.06 kW f
2.1 kW
2 a 2500000 W
b 12000000 W
c 3600000 kW
3 a 1210 MW
b 1210000 kW
c 1210000000 W
4 a 2 109 W b
3.2 106 kW
c 4 108 W
5 a 25 h b
5h
c 2.2 h d
1 h 15 min
e 37.5 min f
24 min
6 a 1500 W b
40 W
7 a 0.57 kWh b
8.5c
8 a 136.5 kWh b
$20.48
c $12.29

Chapter Review

1 a $268.80
b 554L
c 154 L/day higher
d $195.07
2 4608L
3 3500L
4 a 9720L b
1.375m
5 1 : 20000
6 180000L
7 a 3.4 kW
b 750 W
c 500000 W
8 a 32.76
b $5.41
9 a $47.38
b 30c/hr
Extended response

1 a 864 kWh
b $181.44
c $136.08
2 a 7854. L
b 28mm

Chapter 14 Mathematics and resources 431

ICT activities
Water catchment
SEARCHLIGHT ID: PRO-0152

Scenario
Living in Australia we consider ourselves to be The Lucky
Country. However, as a nation we regularly have to face
the effects of natural disasters. Bushfires, floods and
tropical cyclones often make the news for the sudden
impact that they have on peoples lives.
One such natural disaster that Australia faces often is
drought.

Unlike other natural disasters, drought impacts slowly


rather than with sudden force, yet the results can be just
as devastating. People on farms feel the impact first
as crops and stock die through lack of water, meaning
the farmers will struggle to make a living. Due to short
supply, prices for vital commodities rise, and in this way
drought affects all our lives.

432 Maths Quest HSC Mathematics General 2

For this reason we need to be aware of how


important water conservation is, be aware of dam
capacity, and before we are beset by drought to ensure
that plenty of water is kept in reserve.
In this activity you will be researching Sydneys water
supply and how water is managed in times of drought.
1. What is the main dam that supplies Sydneys water?
2. Obtain an aerial photo of this dam and estimate the
area of the dams surface.
3. Research the amount of rain that falls annually in area
in which the dam is located.
4. Calculate the number of kilolitres of rain water that
this dam collects annually.
5. Research:
(a) the number of households in Sydney that are
serviced by water from this dam
(b) the average daily water usage of Sydney
households.
6. Use you results to draw a conclusion about the
amount of water collected in Sydney and the amount
used.

7. Other than rain, where does water for this dam flow
from?
8. In the most recent drought, what water restrictions
were imposed? Use your knowledge of water use to
calculate the amount by which water usage can be
reduced by water restrictions.

Create a summary of your findings to present to an


audience in an appropriate format (e.g. a PowerPoint
presentation or a movie).
Write a report that includes the results of all
yourfindings. Submit this to your teacher for
assessment.

Task
1. Watch the introductory video.
2. Identify the impact of drought on Sydney people and
why people in wider NSW are usually affected before
city people.
3. Identify the main source of Sydneys water and get a
aerial photo with a scale so that measurements can
be made.
4. Use the scale together with Simpsons rule to estimate
the surface area of the dam.
5. Visit the Bureau of Meteorology website
(www.bom.gov.au) to research the amount of rain in
the catchment area each year.
6. Within your class obtain as many water bills as
possible, preferably from different parts of the
Sydneyarea. Use the sampling techniques you
havelearned to make calculations about typical
wateruse.
7. Research the most recent water restrictions in Sydney.
You will have learned how much water is used in
completing household tasks. Use this information to
set out calculations that show the water that will be
saved by the restrictions.
8. Present a report in the form of your choice that shows
water use and make a conclusion about Sydneys
capacity to cope with drought at a time when the
citys population is growing.

Process
Open the ProjectsPLUS application for this chapter in
your eBookPLUS. Watch the introductory video, click
the Start Project button and then set up your project
team. You will invite another member of your class
to form a partnership. Save your settings and the
project will be launched.
Your teacher will have access to a number of
websites that will get your research started. Use
these to analyse the available data and find the
answers to the key questions.
You will need to access the details of your home
internet connection. This may be available in the
contract or on your internet providers website.
Import the data into an Excel spreadsheet.
Use the spreadsheet to create suitable graphs to
display the information.

Suggested software




ProjectsPLUS
Microsoft Word
Microsoft Excel
Microsoft PowerPoint
Google Maps

ICT activities projectsplus 433

Glossary
Angle of depression: The angle through which you must look down
from the horizontal to sight an object.
Angle of elevation: The angle through which you must look up from
the horizontal to sight an object.
Annuity: A form of investment involving regular periodical
contributions to an account. On such an investment, interest
compounds at the end of each period and the next contribution to the
account is made.
Annulus: The area between two circles that have the same centre
(concentric).
Area chart: When line graphs are stacked on top of each other so
thatthe area between each of the lines can be used to compare each
data set.
Asset: An item that is of value to its owner.
Bell shape: A normal distribution where scores are distributed evenly
about the mean is said to be a bell shaped distribution.
Box-and-whisker-plot: A method of graphically displaying a fivenumber summary. The plot is drawn to scale with the box representing
the interquartile range and the whiskers representing the range. Within
the box, the median is also shown.
Building Sustainability Index (BASIX): A measure of how
efficiently a home uses water, electricity and gas resources. It is a
requirement under NSW State Government legislation that new homes
must be given a BASIX certificate. This certificate will only be given
once certain standards are reached.
Capture-recapture technique: A method of estimating a population.
A sample is captured and tagged before being released into the wider
population. A second sample is ten recaptured. The percentage of
that second sample that is tagged allows or an estimate of entire
population.
Causality: When the occurrence of one variable causes another. For
example there is a strong positive correlation between a persons shirt
size and shoe size but one does not cause the other. On the other hand,
there is a strong positive correlation between the amount of a Lottery
jackpot and the number of tickets sold. In this case, it would seem that
one does cause the other.
Compass radial survey: A survey taken from a central point in a
field. Critical points are sighted from this central point and radial lines
are drawn. The distance from the central point to each critical point is
then measured and the compass bearing of each radial line is noted on
the survey.
Correlation coefficient: A figure between 1 and 1 which indicates
the strength of a correlation. The closer to 1 the coefficient is, the
stronger the positive correlation; the closer to 1 the coefficient is, the
stronger the negative correlation. A coefficient close to zero indicates
no correlation or, at best, a very weak correlation.
Correlation: A statement reflecting the relationship between two
variables. A positive correlation indicates that as one variable
increases so does another, while a negative correlation indicates that
as one variable increases, the other decreases.
Cosine rule: A trigonometric formula used in non-right- angled
triangles that allows:
a side length to be found when given the two other side lengths and
the size of the included angle
an angle to be found when given the three side lengths of the
triangle.
Cross tabulation: Data is collected in terms of two variables and
presented in the form of a two way table.
Cubic function: A function where the independent variable is raised
to the power of 3. A cubic function is in the form y = x3.
Cumulative frequency: A progressive total of the frequencies.

Depreciation: The reduction in value of an asset.


Ellipse: An oval shaped figure. The shortest distance from the centre
of an ellipse to the circumference is called the semi-minor axis and
the longest distance from the centre to the circumference is called the
semi-major axis.
Energy rating: A rating given to household appliances which
indicates the amount of power that the appliance uses.
Expected outcome: The number of times that an outcome can be
expected to occur in a given number of trials.
Exponential function: A function where the independent variable
isin the exponent (index). An exponential function is of the form
y = ax or y = b(ax).
Extrapolate: To extend a graph so as to make predictions about future
trends.
Flat rate loan: A loan where the interest is calculated using only the
initial amount borrowed.
Fundamental counting principle: The number of elements of the
sample space for a multi-stage probability experiment is found by
multiplying the number of ways each stage can occur. This is the
fundamental counting principle.
Future value (FV): The value of a compound interest investment on
maturity (the completion of its term).
Future value of an annuity: The value of an annuity at the end of
agiven period of time.
Great circle: A circle of the greatest possible diameter that can be
drawn on the surface of a sphere.
Greenwich Mean Time (GMT): The standard time in Greenwich
which is used as the basis for calculating the time in all other parts
ofthe world.
Greenwich Meridian: The meridian of longitude from which angular
distances in the eastwest direction are measured. Using the longitude
calculated from the Greenwich Meridian, time in different places on
the Earths surface is calculated.
Grouped data: A data set tabulated in small groups rather than as
individual scores.
Histogram: A column graph that displays the frequency for a set
ofscores.
Hyperbolic function: A function where the independent variable is
inthe denominator of the function. A hyperbolic function is of the
form y = x.
Interest factor: The value of $1 under particular investment
conditions.
International Date Line: The meridian of longitude opposite to the
Greenwich Meridian. The International Date Line is, however, bent
for convenience. When crossing the International Date Line, the
datechanges.
Interpolate: Drawing a graph using data found at the end points.
Interquartile range: A number that represents the spread of a data
set. The interquartile range is calculated by subtracting the lower
quartile from the upper quartile.
Kilowatt: A measure of power used to run an appliance.
1 kilowatt = 1000 watts.
Kilowatt hour: The amount of power used to maintain one kilowatt
of power for one hour.
Latitude: The angular distance of a point on the Earths surface either
north or south of the equator.
Least squares regression line of best fit: A line drawn on a scatter
plot that is drawn such that the shortest distance of each point on the
scatterplot, when squared is a minimum.
Life expectancy: The average life span of a particular group of people.

Glossary 435

Line of best fit: A line drawn on a scatterplot that passes through or


is close to as many points as possible.
Longitude: The angular distance East or West of the Greenwich
Meridian.
Lower quartile: The lowest 25% of scores in a data set.
Measure of location: A general term for the typical score in a data
set. The typical score can be measured as the mean, median or mode.
Measure of spread: A general term for the distribution of data.
Measures of spread are the range, interquartile range and standard
deviation.
Meridian of longitude: A line on the Earths surface that runs from
the North Pole to the South Pole. Each meridian of longitude is
measured by the number of degrees east or west it is of the Greenwich
Meridian.
Nautical mile (M): A distance on the surface of the Earth that is
equal to an angular distance of 1 minute on a great circle.
1 M = 1.852 km.
Normal distribution: Occurs when a data set of scores is
symmetrically distributed about the mean.
Offset survey: A survey taken by taking offsets to key points from a
traverse line.
Offset: In a traverse study, an offset is a line perpendicular to the
transversal. It is drawn from the transversal to a vertex on the area
being surveyed.
Ordered selection: A selection of outcomes where the order each
item is selected is important
Outlier: A score in a data set which is an extreme value. The outlier
can be much greater or much less than all other scores in the data set.
The effect of an outlier is to greatly increase or decrease the mean
such that the mean is no longer a reliable indicator of a typical score
in the data set.
Parallel of latitude: A line on the Earths surface parallel to the
equator. Each parallel of latitude is measured in terms of the angular
distance either north or south of the equator.
Percentage error: The maximum error in a measurement as a
percentage of the measurement given.
Plane table radial survey: A survey taken from a central point in
a field where critical points are sighted from this central point and
radial lines are drawn. The distance from the plane table to each
critical point is then measured and the angle between the radial lines
is measured with a protractor.
Present value (PV): The initial amount that is to be invested under
compound interest.
Present value of an annuity: The single sum of money which, if
invested at the present time, will produce the same financial outcome
as an annuity.
Probability tree: A tree diagram where each outcome is not equally
likely. The probability of each selection is written on the branches and
the probability of a sequence of selections is calculated by multiplying
along the branches of each individual selection.
Quadrant: A quarter of a circle.
Quadratic function: A function where the independent variable is
raised to the power of 2. A quadratic function will take the form
y = ax2 or y = ax2 + bx + c.
Quartile: 25% of the data set. The upper quartile is the top 25% of
the data set and the lower quartile is the bottom 25% of the data set.

436 Glossary

Radar chart: A type of line graph drawn around a central point. The
categories are labelled in a circle and data points marked on each line
emanating from the centre. The points are then joined. A radar chart
is suitable to show a pattern that is likely to repeat. For example, sales
made during each month of the year.
Range: A number that represents the spread of a data set. The range is
calculated by subtracting the smallest score from the largest score.
Reducing balance loan: A loan where the interest is calculated
periodically on the outstanding balance at that point in time.
Scatterplot: A graph that shows two variables, one on each axis,
andtheir relationship by plotting the points generated by each
datapair.
Sector: The area between any two radii of a circle.
Simpsons rule: A method used to approximate the area of an
irregular field.
Simulation: An experiment where a computer or other device
replicates the results that would be obtained if an experiment were
performed in reality.
Sine rule: A trigonometric formula used in non-right-angled triangles
that allows:
a side length to be found when given one other side length and the
size of two angles
an angle to be found when given two side lengths and one
non-included angle.
Skewness: The distribution of a set of scores in a data set.
a. Positively skewed more scores are gathered at the lower end of
the distribution.
b. Negatively skewed more scores are distributed at the upper end
of the distribution.
Small circle: A circle that is drawn on the surface of a sphere that is
of a smaller diameter than a great circle.
Standard deviation: A measure of the spread of a data set. The
standard deviation is found on a calculator using either the population
standard deviation or the sample standard deviation.
Standardised score: (also called z-score) A figure which
represents the distance of a score from the mean in terms of the
standard deviation. The standardised score is the number of standard
deviations that a member of the data set is from the mean.
Stem-and-leaf plot: A method of displaying a data set where the
first part of a number is written in the stem and the second part of the
number is written in the leaves.
Term of the loan: The length of time that a loan is repaid over.
Tree diagram: A method of listing the sample space for a multi-stage
probability experiment. The diagram branches once for each stage
ofthe experiment at each level showing all possible outcomes to
eachstage.
Two-way table: A table that compares a population in terms of two
variables. One variable is displayed in rows the second variable in
columns.
Ungrouped data: Data for which each score is individually tabulated.
Unordered selection: A selection of outcomes where the same items
selected in a different order is considered the same selection.
Upper quartile: The highest 25% of scores in a data set.
Watt: A measure of power usage.
z-score: see standardised score.

Index
addition
algebraic expressions 27981, 294
algebraic fractions 27981, 294
algebra
index form 281
index laws 2815, 294
algebraic expressions
addition 27981, 294
division 2812, 294
expanding 2856, 294
explained279
like terms 279
multiplication 2812, 294
raising a power to another power 282
simplifying 2856, 294
subtraction 27981, 294
zero index 2823
algebraic fractions
addition 27981, 294
subtraction 27981, 294
algebraic functions
force of gravity 349
graphing physical phenomena 3459, 350
representations of physical situations 345
angles of depression 181
angles of elevation 181
annuities
future value 2732, 43
present value (PVA) 326, 43
annulus, area 134, 167
arcs, lengths 22933, 248
area
annulus 134, 167
circles133
composite shapes 13641, 167
ellipses167
irregular figures 1416
parts of circles 1336, 167
quadrants133
sectors 1334, 167
Simpsons rule 1416, 167
triangles 2026, 219
see also surface area
area charts
analysis of data sets 65
comparison of data sets 69
bearings 20613, 219
compass bearings 2067, 219
true bearings 2079, 219
types206
body measurements 36182, 391
correlation 3707, 391
drawing scatterplots 3619, 391
fitting a regression line 37781, 391
box-and-whisker plots
analysing data sets 624
comparison of data sets 69
calculators, TVM mode 3
causality 3734, 391
circles
area133
area of parts 1336, 167
great circles 233, 2346, 248
small circles 233, 2346, 248
compass bearings 2067, 219
compass radial surveys 21516
composite shapes, area 13641, 167

composite solids, volume 15763, 167


compound interest
formula 16, 27
present and future value 1619, 20
cones, volume 154, 167
constant of proportionality 342
correlation 3707, 391
causality 3734, 391
correlation coefficient 3713
explained 370, 391
Pearsons Moment Correlation
Coefficient371
positive or negative 3702, 391
strength of 3713
cosine rule
derivation of 1934
finding angles 197202, 219
finding side lengths 1937, 219
counting techiques 25761, 272
committee selections 259
fundamental counting principle 257
ordered arrangements 2578, 272
and probability 2657, 272
tree diagrams and ordered
arrangements2589
unordered selections 259, 272
credit cards 16
annual rates 1
minimum payments 15
overview 1, 20
cross tabulation, comparison of data sets
712
cubes, surface area 146, 167
cubic functions 334, 336, 350
cylinders, surface area 14953, 167
data
grouped501
organising4951
ungrouped50
data sets
analysis5969
area charts 65
box-and-whisker plots 624
comparison 6976, 81
cross tabulation 712
displaying statistical data 68
grouped data 4953, 81
measures of location and spread 549, 83
radar charts 545
skewness 7680, 81
stem-and-leaf plots 62
direct linear variation 3079, 321
direct non-linear variation 33741, 350
distances
on Earths surface 2403, 248
nautical miles 241
division, algebraic expressions 282, 294
dosage rate, medication 3846
Earths surface, distances 2403, 248
electrical appliances, energy and
sustainability, 4215, 426
ellipses, area 167
energy and sustainability 4215, 426
equations
and formulas 2903, 294
simultaneous equations 31520, 321
solving 2869, 294

exponential functions
form335
graphs 335, 336, 350
extrapolation346
flat rate interest 711, 20
force of gravity 349
formulas, and equations 2903, 294
frequency tables, histograms and polygons,
grouped data 49, 51
future value (FV)
annuities 2732, 43
compound interest investments 1619, 20
creating table in Excel 289
gradient formula 302
graphs, loan repayments 402, 43
gravity, calculating force of 349
great circles 233, 2346, 248
Greenwich Mean Time (GMT) 2434
Greenwich Meridian 237, 243
home loans
home loan calculators 1214
repayments1112
hyperbolas 334, 336, 350
hyperbolic functions, equation 334, 350
index form 281
index laws 2815, 294
First Index law 281, 294
Fourth Index Law 2823, 294
Second Index law 281, 294
Third Index Law 282, 294
interest
compound interest 1617, 27
flat rate interest 711
simple interest 2, 34, 7
International Date Line 237, 244
interpolation346
interquartile range (IQR) 55
inverse operations, to solve linear
equations286
inverse variation 3415, 350
investments, present and future value 1619, 20
irregular figures, area 1416, 41617
latitude 23740, 248
important parallels 240
parallels of latitude 237
life expectancy 38690, 392
like terms 279
linear equations
explained286
solving using inverse operations 286
linear functions 3016, 321
conversion of temperature 306
linear modelling
linear relationships 31012, 321
problem solving 31315, 321
loan repayments 1116, 20, 3640, 43
calculating each repayment 368
graphs 402, 43
home loans 1114
present value interest factors (PVIF) 368
reducing balance loans 11, 36, 43
location
comparing data sets 701
measures of 545, 56, 57, 701, 83

Index 437

longitude 23740, 248


Greenwich Meridian 237, 243
International Date Line 237, 244
meridians237
means 54, 11819
measurements
body measurements 36182, 391
electrical power 421
error in 1636, 167
gauging accuracy of 130, 167
location 545, 56, 57, 701, 83
maximum error 130, 163, 167
medication3824
percentage error 130, 167
relative error 12933, 167
spread 55, 57, 701, 83
medians54
medication
Clarks formula for children
(112 yrs) 384, 391
dosage rate 3846
Frieds formula for children
(12 yrs) 384, 391
units of measurements 3824, 391
Youngs formula for children
(112 yrs) 384, 391
meridians of longitude 237
milligrams (mg) 382
modes57
multiplication, algebraic expressions
2812, 294
nautical miles 241
normal distribution
comparison of scores 1003, 108
distribution of scores 1037, 108
explained95
terminology1045
z-scores 95100, 108
obtuse angles, trigonometric ratios 1834
offset surveys 213
ogives 49, 512
ordered arrangements
counting techiques 2578, 272
and tree diagrams 2589
outliers602
parabolas331
parallels of latitude 237
Pearsons Moment Correlation Coefficient 371
plane table radial surveys 21315, 220
present value (PV)
annuities (PVA) 326, 43
creating table in Excel 335
interest factors (PVIF)368
investments 1619, 20
prisms, surface area 1469, 167
probability
and counting techiques 2657, 272
expected outcomes 26871, 272
formula265
popular gaming 266
rolling a die 268
pyramids, volume 1534, 167
Pythagoras theorem 137
quadrants, area 133
quadratic functions 32934, 350
explained329
graphs329

438 Index

maximising areas 333


shape of parabolas 331
radar charts
analysis of data sets 545
comparison of data sets 69
radial surveys 21318, 220
compass radial surveys 21516, 220
plane table radial surveys 21315, 220
random number generators 11516
random samples 115, 123
range55
rectangular prisms, surface area 1467, 167
reducing balance loans 11, 36, 43
calculating each repayment 368
graphs402
home loans 1114
regression lines
fitting 37781, 391
least squares regression line of best fit
377
line of best fit 377
relative error 12933, 167
right-angled triangles, review 17783, 219
sampling
estimating populations 1202, 123
explained115
and means 11819
random samples 115, 123
stratified samples 115, 123
systematic sampling 115, 123
types 11517, 123
scatterplots
drawing 3619, 391
investigation3636
sectors, area 1334, 167
simple interest
calculating2
formula 2, 7
using calculator 34
Simpsons rule 1416, 167, 41617
simultaneous equations 31520, 321
explained315
solving algebraically by elimination
317
solving algebraically by substitution
316
solving graphically 31516
sine rule
ambiguous case 190
derivation of 1846
finding angles 18993, 219
finding side lengths 1838, 219
skewness, data sets 7680, 81
small circles 233, 2346, 248
spheres, surface area 14953, 167
spherical geometry
arc lengths 22933, 248
distances on Earths surface 2403, 248
great circles 233, 2346, 248
latitude and longitude 23740, 248
small circles 233, 2346, 248
time zones 2437, 248
spread
comparing data sets 701
measures of 55, 57, 701, 83
standard deviation 55
standardised score see z-scores
stem-and-leaf plots
analysis of data sets 62
comparison of data sets 69

stratified samples 115, 123


subtraction
algebraic expressions 27981, 294
algebraic fractions 27981, 294
superannuation see annuities
surface area
cubes 146, 167
cylinders 14953, 167
minimising153
prisms 1469, 167
rectangular prisms 1467, 167
spheres 14953, 167
surveying
compass radial surveys 21516
offset survey 213
plane table radial surveys 21315, 220
radial surveys 21318, 220
sustainability and energy 4215, 426
systematic sampling 115, 123
temperature, conversion from Celsius to
Fahrenheit306
time zones 2437, 248
in Australia 2445
daylight saving 245
Greenwich Mean Time (GMT) 2434
tree diagrams
multi-stage events 2614
and ordered arrangements 2589
triangles, area 2026, 219
trigonometric ratios
formulas177
obtuse angles 1834
trigonometry
area of triangles 2026, 219
bearings 20613, 219
cosine rule to find angles 197202, 219
cosine rule to find side lengths 1937, 219
derivation of sine rule 1846
formulas for trigonometric ratios 177
radial surveys 21318, 220
right-angled triangles 17783, 219
sine rule to find angles 18993, 219
sine rule to find side lengths 1838, 219
true bearings 2079, 219
two-way tables, comparison of data
sets69
variation307
constant of variation 307
direct linear variation 3079, 321
direct non-linear variation 33741, 350
inverse variation 3415, 350
volume
composite solids 15763, 167
cones 154, 167
pyramids 1534, 167
spheres 154, 167
water usage
collecting and using water
41215, 426
dams, land and catchment areas
41520, 426
interpreting information about
40512, 426
and water availability 4058
z-scores 95100, 108
distribution of scores 1037, 108
terminology1045
using to compare scores 1003, 108

Das könnte Ihnen auch gefallen